You are on page 1of 237

CLASSIFICATION OF PROPERTY

Immovable and Movable Property

A HOUSE IS AN IMMOVABLE EVEN IF THE OWNER OF THE HOUSE IS DIFFERENT

1. Ladera v. Hodges
Vol. 48, No. 12, O.G. 5374, September 23, 1952
Reyes, J.

FACTS:
This is an appeal to the decision of the Court of Appeals which held that the public auction was invalid for
non-compliance with Rule 39.

Respondent Hodges entered into a contract of sale with petitioner Ladera. Ladera built a house on the lot.
However, Ladera failed to pay so Hodges rescinded and filed an action for ejection, The MTC ruled in favor
of Hodges and issued an alia writ of execution. Notices of sale were posted, but were not published in a
newspaper of general circulation.

Hodges sold the house in a public auction to Magno. Ladera was not able to attend. Villa, later on purchased
the house from Magno. Ladera fild an action against Hodges for the nullification of the sale for non-
compliance with Rule 39 of the Rules of Court regarding the sales of real property.

Hodges contends that the house, which was built on a lot owned by another, should be regarded as a
movable or personal property.

ISSUE:
Is a house built on a lot owned by another a movable property?

HELD:
No, a house built on a lot owned by another is not a movable property. As enumerated in the Civil Code,
immovable property includes lands, buildings, roads and constructions of all kinds adhered to the soil. The
law does not make any distinction whether or not the owner of the lot was the one who built the construction.
Also, since the principles of accession regard buildings and constructions as mere accessories to the land
on which it is built, it is logical that said accessories should partake the nature of the principal thing.

1
CLASSIFICATION OF PROPERTY
Immovable and Movable Property

BUILDING IS A SEPARATE IMMOVABLE PROPERTY FROM THE LAND IT IS ADHERED TO

2. Lopez v. Orosa, Jr.


G.R. Nos. L-10817-18, February 28, 1958
Felix, J.

FACTS:
Petitioner Lopez is trying to secure a modification of the decision of the CA which held Defendants Orosa,
Jr, and Plaza Theatre are jointly liable and that the lien did not extend to the land.

Petitioner Lopez entered in to a contract with respondent Orosa, Jr. Lopez agreed to supply the lumber
necessary for the construction of the proposed theatre. The Plaza Theatre was erected on a piece of land
formerly owned by Orosa, Jr. and was acquired by Plaza Theatre, Inc. Lopez demanded payment from
Orosa, Jr. and Plaza Theatre, Inc. Plaza Theatre, Inc. mortaged its properties. It also obtained a loan with
Luzon Surety Company. In the registration of the land, the mortaged was not revealed.

Lopez filed a case against Orosa, Jr. and Plaza Theatre, Inc for the unpaid balance. Lopez prayed that the
land and building be sold in public auction with the proceeds applied to the balance.

ISSUE:
Whether a materialman's lien for the value of the materials used in the construction of a building attaches
to said structure alone and does not extend to the land on which the building is adhered to

HELD:
No, the lien does not extend to the land. While it is true that generally, real estate connotes the land and the
building constructed thereon, it is obvious that the inclusion of the building, separate and distinct from the
land, in the enumeration of what may constitute real properties1 could mean only one thing—that a building
is by itself an immovable property.

Moreover, and in view of the absence of any specific provision of law to the contrary, a building is an
immovable property, irrespective of whether or not said structure and the land on which it is adhered to
belong to the same owner.

The law gives preference to unregistered refectionary credits only with respect to the real estate upon which
the refection or work was made. This being so, the inevitable conclusion must be that the lien so created
attaches merely to the immovable property for the construction or repair of which the obligation was incurred.
Evidently, therefore, the lien in favor of appellant for the unpaid value of the lumber used in the construction
of the building attaches only to said structure and to no other property of the obligors.

2
CLASSIFICATION OF PROPERTY
Immovable and Movable Property

PROPERTY THAT CAN BE REMOVED WITHOUT BREAKING THE MATERIAL OR DETERIORATION


IS PERSONAL PROPERTY

3. Yap v. Tañada
G.R. No. L-32917, July 18, 1988
Narvasa, J.

FACTS:
In a petition for review on certiorari, petitioner Yap seeks to reverse the decision of respondent Tañada
denying Yap’s application to prove ad recover damages resulting from alleged irregularities in the process
of execution.

Gould Pumps International (Phil.), Inc. (GPI) filed a complaint against Yap seeking to recover the balance
of the price and installation cost of a water pump in Yap’s residences. Respondent Tañada rendered
judgment requiring Yap to pay for the unpaid balance. Tañada granted GPI’s Motion for Issuance of Writ of
Execution. The Sheriff levied on the water pump. Tañada ordered the suspension of the sale, but a copy
was not transmitted to the Sheriff. The water pump was sold to GPI as the highest bidder.

Yap assails the validity of the execution sale for being made without the required notice by Sec. 18, Rule
39 of the Rules of Court, notice by publication in case of execution sale of real property. The pump and its
accessories being immovable because it is attached to the ground with character of permanency in
accordance with Article 425 of the Civil Code.

ISSUE:
Is the water pump in question an immovable property?

HELD:
No, the water pump is not an immovable property. The Civil Code considers as immovable property, among
others, anything "attached to an immovable in a fixed manner, in such a way that it cannot be separated
therefrom without breaking the material or deterioration of the object." The pump does not fit this description.
It could be, and was in fact separated from Yap's premises without being broken or suffering deterioration.
Obviously the separation or removal of the pump involved nothing more complicated than the loosening of
bolts or dismantling of other fasteners.

3
CLASSIFICATION OF PROPERTY
Immovable and Movable Property

PROPERTIES REMOVED MUST BE IN THE SAME CONDITION

4. Machinery & Engineering Supplies, Inc. v. Court of Appeals


G.R. No. L-7057, October 29, 1954
Concepcion, J.

FACTS:
"On March 13, 1953, the herein petitioner filed a complaint for replevin in the Court of First Instance of
Manila, Civil Case No. 19067, entitled 'Machinery & Engineering Supplies, Inc., Plaintiff, vs. Ipo Limestone
Co,, Inc., and Dr. Antonio Villarama, defendants', for the recovery of the machineries and equipment sold
and delivered to said defendants at their factory in barrio Bigti, Norzagaray, Bulacan. Upon application ex-
parte of the petitioner, accompanied by the affidavit of Ramon S. Eoco, president of petitioner company,
and upon approval of petitioner's bond in the sum of P15,769.00, on March 13, 1953, respondent judge
issued an order, commanding the Provincial Sheriff of Bulacan to seize and take immediate possession of
the properties specified in the order. On March 23, 1953, the defendants' counsel asked the Provincial
Sheriff if the machineries and equipments, dumped on the road would be re-installed to their former position
and condition

ISSUE:
Whether or not the property removed was done within right by the Sheriff

HELD:
No. While the seizure of the equipments and personal properties was ordered by the respondent Court, it
is, however, logical to presume that said court did not authorize the petitioner or its agents to destroy, as
they did, said machineries and equipments, by dis mantling and unbolting the same from their concrete
basements, and cutting and sawing their wooden supports, thereby rendering them unserviceable and
beyond repair, unless those parts removed, cut and sawed be replaced. he Provincial Sheriff's tortuous act,
in obedience to the insistent proceedings of the president of the petitioner, Ramon S. Roco, has no
justification in law, notwithstanding the Sheriff's claim that his duty was ministerial. It was the bounden duty
of the respondent Judge to give redress to the respondent Company, for the unlawful and wrongful acts
committed by the petitioner and its agents. And as this was the true object of the order of March 30, 1953,
we can not but hold that same was within its jurisdiction to issue. The ministerial duty of the Sheriff should
have its limitations.

We have not found, any authority squarely in point obviously because real property are not subject to
replevin it is well settled that, when the restitution of what has been ordered, the goods in question shall be
returned in substantially the same condition as when taken. In as much as the machinery and equipment
involved in this case were duly installed and affixed in the premises of respondent company when
petitioner's representative caused said property to be dismantled and then removed, it follows that petitioner
must also do everything necessary to the re-installation of said property in conformity with its original
condition.

4
CLASSIFICATION OF PROPERTY
Immovable and Movable Property

DETERMINATION OF IMMOVABLE TAXABLE REAL PROPERTY

5. Mindanao Bus Co. v. City Assessor and Treasurer


G.R. No. L-17870, September 29, 1962
Labrador, J.

FACTS:
Petitioner is a public utility solely engaged in transporting passengers and cargoes by motor trucks. It owns
a land where it maintains and operates a garage for its TPU motor trucks; a repair shop; blacksmith and
carpentry shops, and with machineries placed therein, its TPU trucks are made; body constructed; and
same are repaired in a condition to be serviceable in the TPU land transportation business it operates.

The machineries have never been or were never used as industrial equipment to produce finished products
for sale, nor to repair machineries, parts and the like offered to the general public indiscriminately for
business or commercial purposes.

Respondent City Assessor of Cagayan de Oro City assessed at P4,400 petitioner’s above-mentioned
equipment. Petitioner appealed the assessment to the respondent Board of Tax Appeals on the ground that
the same are not realty. The Board of Tax Appeals of the City sustained the city assessor, so petitioner
herein filed with the Court of Tax Appeals a petition for the review of the assessment.

The CTA held the petitioner liable to the payment of the realty tax on its maintenance and repair equipment
mentioned above. Hence, this petition.

ISSUE:
Should the tools and equipment in the petitioner company’s repair shop be considered immovable taxable
real properties?

HELD:
No. Movable equipment to be immobilized in contemplation of the law must first be “essential and principal
elements” of an industry or works without which such industry or works would be “unable to function or carry
on the industrial purpose for which it was established.” The tools and equipment are not essential and
principle municipal elements of petitioner’s business of transporting passengers and cargoes by motor
trucks. They are merely incidentals — acquired as movables and used only for expediency to facilitate
and/or improve its service. The transportation business could be carried on without the repair or service
shop if its rolling equipment is repaired or serviced in another shop belonging to another.

5
CLASSIFICATION OF PROPERTY
Immovable and Movable Property

SEARCH WARRANTS MUST BE SPECIFIC AND PARTICULAR

6. Burgos, Sr. v. Chief of Staff


G.R. No. 64261, December 26, 1984
Escolin, J.

FACTS:
Assailed in this petition for certiorari prohibition and mandamus with preliminary mandatory and prohibitory
injunction is the validity of two [2] search warrants issued on December 7, 1982 by respondent Judge Ernani
Cruz-Pano, Executive Judge of the then Court of First Instance of Rizal [Quezon City], under which the
premises known as No. 19, Road 3, Project 6, Quezon City, and 784 Units C & D, RMS Building, Quezon
Avenue, Quezon City, business addresses of the “Metropolitan Mail” and “We Forum” newspapers,
respectively, were searched, and office and printing machines, equipment, paraphernalia, motor vehicles
and other articles used in the printing, publication and distribution of the said newspapers, as well as
numerous papers, documents, books and other written literature alleged to be in the possession and control
of petitioner Jose Burgos, Jr. publisher-editor of the “We Forum” newspaper, were seized. Petitioners fault
respondent judge for his alleged failure to conduct an examination under oath or affirmation of the applicant
and his witnesses, as mandated by the above-quoted constitutional provision as wen as Sec. 4, Rule 126
of the Rules of Court .6 This objection, however, may properly be considered moot and academic, as
petitioners themselves conceded during the hearing on August 9, 1983, that an examination had indeed
been conducted by respondent judge of Col. Abadilla and his witnesses. In the determination of whether a
search warrant describes the premises to be searched with sufficient particularity, it has been held “that the
executing officer’s prior knowledge as to the place intended in the warrant is relevant. This would seem to
be especially true where the executing officer is the affiant on whose affidavit the warrant had issued, and
when he knows that the judge who issued the warrant intended the building described in the affidavit, And
it has also been said that the executing officer may look to the affidavit in the official court file to resolve an
ambiguity in the warrant as to the place to be searched.”

It is contended by petitioners, however, that the abovementioned documents could not have provided
sufficient basis for the finding of a probable cause upon which a warrant may validly issue in accordance
with Section 3, Article IV of the 1973 Constitution.

ISSUE:
Whether general description of the things to be seized is enough to constitute probable cause to validate
the issuance of a search warrant and seizure

HELD:
No, Probable cause for a search is defined as such facts and circumstances which would lead a reasonably
discreet and prudent man to believe that an offense has been committed and that the objects sought in
connection with the offense are in the place sought to be searched. And when the search warrant applied
for is directed against a newspaper publisher or editor in connection with the publication of subversive
materials, the application and/or its supporting affidavits must contain a specification, stating with
particularity the alleged subversive material he has published or is intending to publish. Mere generalization
will not suffice. Thus, the broad statement in of the respondent’s application that petitioner “is in possession
or has in his control printing equipment and other paraphernalia, news publications and other documents
which were used and are all continuously being used as a means of committing the offense of subversion
punishable under Presidential Decree 885, as amended …” is a mere conclusion of law and does not satisfy
the requirements of probable cause. Bereft of such particulars as would justify a finding of the existence of
probable cause, said allegation cannot serve as basis for the issuance of a search warrant and it was a
grave error for respondent judge to have done so. Equally insufficient as basis for the determination of
probable cause is the statement contained in the joint affidavit of Alejandro M. Gutierrez and Pedro U.
Tango, “that the evidence gathered and collated by our unit clearly shows that the premises above-
mentioned and the articles and things above-described were used and are continuously being used for

6
subversive activities in conspiracy with, and to promote the objective of, illegal organizations such as the
Light-a-Fire Movement, Movement for Free Philippines, and April 6 Movement.”

In mandating that “no warrant shall issue except upon probable cause to be determined by the judge, …
after examination under oath or affirmation of the complainant and the witnesses he may produce; the
Constitution requires no less than personal knowledge by the complainant or his witnesses of the facts upon
which the issuance of a search warrant may be justified. this Court ruled that “the oath required must refer
to the truth of the facts within the personal knowledge of the petitioner or his witnesses, because the purpose
thereof is to convince the committing magistrate, not the individual making the affidavit and seeking the
issuance of the warrant, of the existence of probable cause.” As couched, the quoted averment in said joint
affidavit filed before respondent judge hardly meets the test of sufficiency established by this Court.

7
CLASSIFICATION OF PROPERTY
Immovable and Movable Property

TRANSFORMERS, ELECTRIC POSTS, TRANSMISSION LINES, INSULATORS, AND ELECTRIC


METERS QUALIFY AS “MACHINERIES” UNDER THE LGC, WHICH ARE SUBJECT TO REAL
PROPERTY TAX

7. Manila Electric Co. v. City Assessor


G.R. No. 166102, August 5, 2015
Leonardo-De Castro, J.

FACTS:
The present case is a petition for review on certiorari under Rule 45 of the Rules of Court seeking the
reversal of the decision of the CA. The CA affirmed the decision of the Central Board of Assessment Appeals
(CBAA), which upheld with modification the decision of the Local Board of Assessment Appeals (LBAA),
ruling that Manila Electric Company (MERALCO) is liable for real property tax on its electric facilities
(transformers, electric posts, transmission lines, insulators, and electric meters).

Petitioner, MERALCO, is a private corporation operating as a public utility engaged in electric distribution.
It has been granted franchises to operate in Lucena City since 1922. In 1989, MERALCO received from the
City Assessor of Lucena a tax declaration subjecting its electric facilities to real property tax. MERALCO
appealed the said declaration to the LBAA, which ruled in its favor. In 1997, six years after the said decision
of the LBAA became final and executory, MERALCO received a letter, coupled with a notice of assessment
and tax declarations, from the City Treasurer of Lucena stating that the company was being assessed on
its real property tax delinquency on its machineries beginning 1990. MERALCO appealed the said tax
declarations to the LBAA, which ruled that MERALCO is not exempted from the payment of real property
tax. On appeal, the CBAA affirmed the decision of the LBAA. When MERALCO elevated the matter to the
CA, the CA held that MERALCO’s electric facilities are considered as machineries under the LGC, which
are subject to real property tax.

MERALCO contended that its electric facilities are not subject to real property tax given that the definition
of "machinery" under Section 199 (o) of the Local Government Code, on which real property tax is imposed,
must still be within the contemplation of real or immovable property under Article 415 of the Civil Code.

On the other hand, the respondents argued that under the LGC, which expressly repealed the Real Property
Tax Code, the transformers, electric posts, transmission lines, insulators, and electric meters of MERALCO
fall within the new definition of "machineries," deemed as real properties subject to real property tax.

ISSUE:
Do the transformers, electric posts, transmission lines, insulators, and electric meters owned by MERALCO
qualify as “machineries” subject to real property tax?

HELD:
Yes, the transformers, electric posts, transmission lines, insulators, and electric meters qualify as
machineries subject to real property tax.

While the LGC does not provide for a specific definition of "real property," Sections 199 (o) and 232 of the
said Code, respectively, gives an extensive definition of what constitutes "machinery" and unequivocally
subjects such machinery to real property tax. Under Section 199 (o) of the Local Government Code,
machinery, to be deemed real property subject to real property tax, need no longer be annexed to the land
or building as these "may or may not be attached, permanently or temporarily to the real property," and in
fact, such machinery may even be "mobile." The same provision though requires that to be machinery
subject to real property tax, the physical facilities for production, installations, and appurtenant service
facilities, those which are mobile, self-powered or self-propelled, or not permanently attached to the real
property (a) must be actually, directly, and exclusively used to meet the needs of the particular industry,
business, or activity; and (2) by their very nature and purpose, are designed for, or necessary for
manufacturing, mining, logging, commercial, industrial, or agricultural purposes.

In this case, MERALCO is a public utility engaged in electric distribution, and its transformers, electric posts,
transmission lines, insulators, and electric meters constitute the physical facilities through which MERALCO
delivers electricity to its consumers. Although the said electric facilities are not annexed to the land, said

8
facilities are actually, directly, and exclusively used to meet the business of MERALCO; and the same are
necessary for its industrial purposes.

Therefore, MERALCO’s transformers, electric posts, transmission lines, insulators, and electric meters
qualify as machineries subject to real property tax.

9
CLASSIFICATION OF PROPERTY
Immovable and Movable Property

SUBMARINE OR UNDERSEA COMMUNICATION CABLES ARE REAL PROPERTIES SUBJECT TO


REAL PROPERTY TAX

8. Capitol Wireless, Inc. v. Provincial Treasurer of Batangas


G.R. No. 180110, May 30, 2016
Peralta, J.

FACTS:
The present case is a petition for review on certiorari under Rule 45 of the Rules of Court seeking to annul
and set aside the decision and resolution of the CA, both denying the appeal of petitioner, Capitol Wireless
Inc., (Capwire) against the decision of the RTC, which denied Capwire’s petition for prohibition and
declaration of nullity of warrant of levy, notice of auction sale and/or auction sale.

Petitioner, Capwire, is a Philippine corporation which provides international telecommunications services. It


has signed agreements with other local and foreign telecommunications companies covering an
international network of submarine cable system such as the Asia Pacific Cable Network System (APCN).
Capwire claims that it co-owns a cable system together with PLDT and thus engaged an appraiser to assess
the market value of the said cable system. Capwire also avers that its cable system "interconnects at the
PLDT Landing Station in Nasugbu, Batangas," which is covered by a transfer certificate of title and tax
declarations in the name of PLDT. As such, the Provincial Assessor of Nasugbu, Batangas had determined
that the submarine cable systems described in Capwire's Sworn Statement of True Value of Real Properties
are taxable real property and thus, subsequently issued an assessment of real properties against Capwire.

In the present petition, Capwire holds that the cable system is not a subject to real property tax and the
same lies outside the Philippine territory. Respondent, however, disagrees with Capwire and avers that the
submarine cable system is a taxable real property.

ISSUE:
Is a submarine cable system considered as a real property which can be subjected to real property tax?

HELD:
Yes, submarine cable systems are considered as real properties subject to real property tax.

Submarine or undersea communications cables are akin to electric transmission lines which this Court has
recently declared in Manila Electric Company v. City Assessor and City Treasurer of Lucena City, as "no
longer exempted from real property tax" and may qualify as "machinery" subject to real property tax under
the Local Government Code. To the extent that the equipment's location is determinable to be within the
taxing authority's jurisdiction, the Court sees no reason to distinguish between submarine cables used for
communications and aerial or underground wires or lines used for electric transmission, so that both pieces
of property do not merit a different treatment in the aspect of real property taxation. Both electric lines and
communications cables, in the strictest sense, are not directly adhered to the soil but pass through posts,
relays or landing stations, but both may be classified under the term "machinery" as real property under
Article 415 (5) of the Civil Code for the simple reason that such pieces of equipment serve the owner's
business or tend to meet the needs of his industry or works that are on real estate.

In this case, the submarine cable systems owned by Capwire are deemed akin to electric transmission lines
and are no longer exempted from real property tax" and may qualify as "machinery" subject to real property
tax under the Local Government Code.

10
CLASSIFICATION OF PROPERTY
Immovable and Movable Property

POWER BARGES INSTALLED BY ITS OWNER IN POWER FACILITIES ARE REAL PROPERTY
SUBJECT TO REAL PROPERTY TAX

9. FELS Energy, Inc. v. Province of Batangas


G.R. Nos. 168557 & 170628, February 6, 2007
Callejo, Sr., J.

FACTS:
These are two consolidated petitions for review on certiorari, filed by FELS Energy Inc., (FELS) and the
National Power Corporation (NPC), assailing the decision of the CA and its resolution which dismissed both
petitions on the ground of prescription.

In 1993, NPC entered into a lease contract, denominated as an Energy Conversion Agreement, with Polar
Energy Inc., (Polar) over diesel power barges moored at Balayan, Bay in Calaca, Batangas. Subsequently,
Polar assigned its rights under the agreement to FELS. In 1995, FELS received an assessment on real
property taxes on the power barges from the Provincial Assessor of Batangas City. It then referred the
matter to the NPC to represent it in any conference regarding the said assessment. NPC sought for the
reconsideration of the assignment but as the same was denied, it filed a petition with the LBAA to set aside
the assessment and the declaration of the power barges as non-taxable items. LBAA denied the said petition
on the ground that the power plant facilities, although movable properties, are considered as real properties
for taxation purposes because they are considered at a specific location with a character of permanency. In
its appeal to the CBAA, it first ruled that the power facilities are exempt from real property taxes but later
affirmed the decision of the LBAA. FELS and NPC appealed the decision to the CA but the CA dismissed
their respective petitions.

Petitioners maintained that the power barges are exempt from real property taxes under Section 234 of the
LGC because they are actually, directly and exclusively used by petitioner NPC, a government- owned and
controlled corporation engaged in the supply, generation, and transmission of electric power. On the other
hand, the Provincial Assessor averred that the barges were real property for purposes of taxation under
Section 199 (c) of the LGC.

ISSUE:
Are power barges in the case at bar considered as real properties subject to real property tax?

HELD:
Yes, the power barges owned by FELS are considered as real properties subject to real property tax.

Article 415 (9) of the New Civil Code provides that "docks and structures which, though floating, are intended
by their nature and object to remain at a fixed place on a river, lake, or coast" are considered immovable
property. Thus, power barges are categorized as immovable property by destination, being in the nature of
machinery and other implements intended by the owner for an industry or work which may be carried on in
a building or on a piece of land and which tend directly to meet the needs of said industry or work.

In this case, the power barges in the power facilities of NPC and FELS are considered as real properties for
taxation purposes as they are installed at a specific location with a character of permanency, and are being
used by the owner thereof in an industry which tend directly to meet the needs of the said industry.

Thus, being categorized as real properties, the said power barges are subject to real property tax.

11
CLASSIFICATION OF PROPERTY
Immovable and Movable Property

HOUSE IS NOT PERSONAL BUT REAL PROPERTY FOR PURPOSES OF ATTACHMENT

10. Evangelista v. Alto Surety & Ins. Co., Inc.


G.R. No. L-11139, April 23, 1958
Concepcion, J.

FACTS:
In 1949, Santos Evangelista instituted Civil Case No. 8235 of the CFI Manila (Santos Evangelista vs.
Ricardo Rivera) for a sum of money. On the same date, he obtained a writ of attachment, which was levied
upon a house, built by Rivera on a land situated in Manila and leased to him, by filing copy of said writ and
the corresponding notice of attachment with the Office of the Register of Deeds of Manila. In due course,
judgment was rendered in favor of Evangelista, who bought the house at public auction held in compliance
with the writ of execution issued in said case on 8 October 1951. The corresponding definite deed of sale
was issued to him on 22 October 1952, upon expiration of the period of redemption. When Evangelista
sought to take possession of the house, Rivera refused to surrender it, upon the ground that he had leased
the property from the Alto Surety & Insurance Co., Inc. and that the latter is now the true owner of said
property. It appears that on 10 May 1952, a definite deed of sale of the same house had been issued to Alto
Surety, as the highest bidder at an auction sale held, on 29 September 1950, in compliance with a writ of
execution issued in Civil Case 6268 of the same court (Alto Surety & Insurance vs. Maximo Quiambao,
Rosario Guevara and Ricardo Rivera)" in which judgment for the sum of money, had been rendered in favor
of Alto Surety. Hence, on 13 June 1953, Evangelista instituted an action against Alto Surety and Ricardo
Rivera, for the purpose of establishing his title over said house, and securing possession thereof, apart from
recovering damages. After due trial, the CFI Manila rendered judgment for Evangelista, sentencing Rivera
and Alto Surety to deliver the house in question to Evangelista and to pay him, jointly and severally, P40.00
a month from October 1952, until said delivery. The decision was however reversed by the Court of Appeals,
which absolved Alto Surety from the complaint on account that although the writ of attachment in favor of
Evangelista had been filed with the Register of Deeds of Manila prior to the sale in favor of Alto Surety,
Evangelista did not acquire thereby a preferential lien, the attachment having been levied as if the house in
question were immovable property.

ISSUE:
Whether or not a house constructed by the lessee of the land on which it is built, should be dealt with, for
purpose of attachment, as immovable property?

HELD:
The court ruled that the house is not personal property, much less a debt, credit or other personal property
not capable of manual delivery, but immovable property. As held in Laddera vs. Hodges (48 OG 5374), "a
true building is immovable or real property, whether it is erected by the owner of the land or by a usufructuary
or lessee.” The opinion that the house of Rivera should have been attached, as "personal property capable
of manual delivery, by taking and safely keeping in his custody", for it declared that "Evangelista could not
have validly purchased Ricardo Rivera's house from the sheriff as the latter was not in possession thereof
at the time he sold it at a public auction” is untenable. Parties to a deed of chattel mortgage may agree to
consider a house as personal property for purposes of said contract. However, this view is good only insofar
as the contracting parties are concerned. It is based, partly, upon the principle of estoppel. Neither this
principle, nor said view, is applicable to strangers to said contract. The rules on execution do not allow, and
should not be interpreted as to allow, the special consideration that parties to a contract may have desired
to impart to real estate as personal property, when they are not ordinarily so. Sales on execution affect the
public and third persons. The regulation governing sales on execution are for public officials to follow. The
form of proceedings prescribed for each kind of property is suited to its character, not to the character which
the parties have given to it or desire to give it. The regulations were never intended to suit the consideration
that parties, may have privately given to the property levied upon. The court therefore affirms the decision
of the CA with cost against Alto Surety.

12
CLASSIFICATION OF PROPERTY
Immovable and Movable Property

A CHATTEL MORTGAGE CONSTITUTED PERMANENTLY ON THE GROUND IS CONSIDERED AS


PERSONAL PROPERTY

11. Makati Leasing and Finance Corp. v. Wearever Textile Mills, Inc.
G.R. No. L-58469, May 16, 1983
De Castro, J.

FACTS:
To obtain financial accommodations from Makati Leasing, Wearever Textile discounted and assigned
several receivables under a Receivable Purchase Agreement with Makati Leasing. To secure the collection
of receivables, it executed a chattel mortgage over several raw materials and a machinery – Artos Aero
Dryer Stentering Range (Dryer).

Wearever defaulted thus the properties mortgaged were extrajudicially foreclosed. The sheriff, after the
restraining order was lifted, was able to enter the premises of Wearever and removed the drive motor of the
Dryer. The CA reversed the order of the CFI, ordering the return of the drive motor since it cannot be the
subject of a replevin suit being an immovable bolted to the ground. Thus the case at bar.

ISSUE:
Whether the dryer is an immovable property

HELD:
If a house of strong materials, like what was involved in the above Tumalad case, may be considered as
personal property for purposes of executing a chattel mortgage thereon as long as the parties to the contract
so agree and no innocent third party will be prejudiced thereby, there is absolutely no reason why a
machinery, which is movable in its nature and becomes immobilized only by destination or purpose, may
not be likewise treated as such. This is really because one who has so agreed is estopped from denying
the existence of the chattel mortgage.

13
CLASSIFICATION OF PROPERTY
Immovable and Movable Property

THE NATURE OF THE MACHINERIES, THE FACT THAT THEY WERE BOLTED, HEAVY OR
CEMENTEDOES NOT MAKE THEM IPSO FACTO IMMOVABLE UNDER ARTICLE 415 PARAGRAPHS
(3) AND (5) OF THE NEW CIVIL CODE BECAUSE THE INTENT OF THE PARTIES MUST BE LOOKED
INTO

12. Tsai v. Court of Appeals


G.R. Nos. 12009 & 120109, October 2, 2001
Quisumbing, J.

FACTS:
Ever Textile Mills, Inc. obtained two loans from Philippine Bank of Communications (PBCom), secured by
a deed of Real and Chattel Mortgage over the lot where its factory stands.

Upon EVERTEX’s failure to meet its obligation to PBCom, PBCom commenced extrajudicial foreclosure
proceedings vs. EVERTEX under Act 3135 and Act 1506 or “The Chattel Mortgage Law”. PBCom
consolidated its ownership over the lot and all the properties in it. It leased and eventually sold the entire
factory premises to Ruby Tsai, including the contested machineries.

EVERTEX filed a complaint for annulment of sale, reconveyance, and damages against PBCom, alleging
inter alia that the extrajudicial foreclosure of subject mortgage was not valid and that PBCom, without any
legal or factual basis, appropriated the contested properties which were not included in the Real and Chattel
Mortgage of the first mortgage contract nor in the second contract which is a Chattel Mortgage, and neither
were those properties included in the Notice of Sheriff’s Sale.

ISSUE:
Are the contested properties personal or movable properties?

HELD:
Petitioners contend that the nature of the disputed machineries, i.e., that they were heavy, bolted or
cemented on the real property mortgaged by EVERTEX to PBCom, make them ipso facto immovable under
Article 415 (3) and (5) of the New Civil Code. This assertion, however, does not settle the issue. Mere nuts
and bolts do not foreclose the controversy. We have to look at the parties’ intent. While it is true that the
controverted properties appear to be immobile, a perusal of the contract of Real and Chattel Mortgage
executed by the parties herein gives us a contrary indication. In the case at bar, both the trial and the
appellate courts reached the same finding that the true intention of PBCom and the owner, EVERTEX, is to
treat machinery and equipment as chattels.

14
CLASSIFICATION OF PROPERTY
Immovable and Movable Property

CONTRACTING PARTIES MAY VALIDLY STIPULATE THAT REAL PROPERTY BE CONSIDERED AS


PERSONAL

13. Serg's Products, Inc. v. PCI Leasing & Finance, Inc.


G.R. No. 137705, August 22, 2000
Panganiban, J.

FACTS:
This is a Petition for Review on Certiorari assailing the January 6, 1999 Decision of the Court of Appeals
(CA) in affirming the decision of the RTC in issuing the Writ of Seizure of several chocolate-making
equipment after the application by the respondent, PCI Leasing and Finance, Inc.

Petitioner is engaged in chocolate making business. It entered into a lease agreement with the respondent
which provides that the machine used in chocolate-making “shall at all times be and remain, personal
property notwithstanding that the PROPERTY or any part thereof may now be, or hereafter become, in any
manner affixed or attached to or embedded in, or permanently resting upon, real property or any building
thereon, or attached in any manner to what is permanent." Respondent then filed with the RTC-QC a
complaint for a sum of money with an application for a writ of replevin covering the subject machines.
Petitioner contends that the machines that were the subjects of the Writ of Seizure were placed by
petitioners in the factory built on their own land and that they were essential and principal elements of their
chocolate-making industry. Hence, although each of them was movable or personal property on its own, all
of them have become "immobilized by destination because they are essential and principal elements in the
industry."

ISSUE:
Should the machines be considered as a personal property and subject to a writ of replevin?

HELD:
Yes, the machines in this case should be considered as a personal property and are subject to a writ of
replevin

Article 415 of the Civil Code enumerates the immovable or real property. Paragraph 5 of Art 415 provides
“Machinery, receptacles, instruments or implements intended by the owner of the tenement for an industry
or works which may be carried on in a building or on a piece of land, and which tend directly to meet the
needs of the said industry or works” However, the Court has held that contracting parties may validly
stipulate that a real property be considered as personal. After agreeing to such stipulation, they are
consequently estopped from claiming otherwise.

The machines that were the subjects of the Writ of Seizure were placed by petitioners in the factory built on
their own land. Indisputably, they were essential and principal elements of their chocolate-making industry.
Hence, although each of them was movable or personal property on its own, all of them have become
"immobilized by destination because they are essential and principal elements in the industry." In the
present case, the Lease Agreement clearly provides that the machines in question are to be considered as
personal property. Hence petitioner is estopped from denying the said agreement.

Hence, the subject machines are personal property and are subject to a writ of replevin.

15
CLASSIFICATION OF PROPERTY
Property in Relation to Whom It Belongs

A PROPERTY CONTINUES TO BE PART OF THE PUBLIC DOMAIN “UNTIL THERE IS A FORMAL


DECLARATION ON THE PART OF THE GOVERNMENT TO WITHDRAW IT FROM BEING SUCH”

14. Laurel v. Garcia


G.R. No. 92013, July 15, 1990
Gutierrez, J.

FACTS:
These are two petitions for prohibition seeking to enjoin respondents (Executive Secretary and Foreign
Affairs Secretary), their representatives and agents from proceeding with the bidding for the sale of the
3,179 square meters of land at 306 Ropponggi, 5-Chome Minato-ku, Tokyo, Japan

The subject property is one of the four (4) properties in Japan acquired by the Philippine government under
the Reparations Agreement entered into with Japan on May 9, 1956. The Roponggi property consists of the
land and building "for the Chancery of the Philippine Embassy". As intended, it became the site of the
Philippine Embassy until the latter was transferred to Nampeidai on July 22, 1976 when the Roppongi
building needed major repairs. Due to the failure of our government to provide necessary funds, the
Roppongi property has remained undeveloped since that time. The President issued Executive Order No.
296 entitling non- Filipino citizens or entities to avail of reparations' capital goods and services, including
the Roponggi property, in the event of sale, lease or disposition.

Petitioner contends that that the Roppongi property is classified as one of public dominion, and not of private
ownership under Article 420 of the Civil Code hence, it cannot be appropriated, and is outside the commerce
of man. Respondents alleged that it has become patrimonial property because it has not been used for
public service or for diplomatic purposes for over thirteen (13) years and because the intention by the
Executive Department and the Congress to convert it to private use has been manifested by overt acts

ISSUE:
Was the non-use of the Roponggi property converted it into a patrimonial property?

HELD:
No. The non-use for 13 years of the Roponggi property did not convert it into a patrimonial property.

Article 420 of the Civil Code provides that property of public dominion shall include (2) Those which belong
to the State, without being for public use, and are intended for some public service or for the development
of the national wealth. The Roppongi property is correctly classified under paragraph 2 of Article 420 ofthe
Civil Code as property belonging to the State and intended for some public service.

The fact that the Roppongi site has not been used for a long time for actual Embassy service does not
automatically convert it to patrimonial property. A property continues to be part of the public domain, not
available for private appropriation or ownership "until there is a formal declaration on the part of the
government to withdraw it from being such. Abandonment cannot be inferred from the non-use alone
specially if the non-use was attributable not to the government's own deliberate and indubitable will but to a
lack of financial support to repair and improve the property. Any such conveyance must be authorized and
approved by a law enacted by the Congress. It requires executive and legislative concurrence.

Hence the Roponggi property continued to be property of public dominion.

16
CLASSIFICATION OF PROPERTY
Property in Relation to Whom It Belongs

A PRIVATE PROPERTY WHICH SUBSEQUENTLY BECAME A FORESHORE LAND SHOULD BE


RETURNED TO PUBLIC DOMINION

15. Republic v. Court of Appeals and Morato


G.R. No. 107709, November 14, 1997
Panganiban, J.

FACTS:
This is a petition assailing the Court of Appeals' Decision affirming the decision of the Regional Trial Court
of dismissing petitioner's complaint for cancellation of the Torrens Certificate of Title of private respondent
Morato and for reversion of the parcel of land subject thereof to the public domain.

Private respondent Morato is the grantee of a Free Patent Title of a lot situated in Pinagtalleran Calauag,
Quezon. He filed for such grant in December 1972 and was issued on February 4, 1974. While at the time
of the grant of free patent to respondent Morato, the land was not reached by the water, however, due to
gradual sinking of the land caused by natural calamities, the sea advances had permanently invaded a
portion of subject land. As disclosed at the trial, the land was under water during high tide in the month of
August 1978. The water margin covers half of the property, but during low tide, the water is about a
kilometer. Also, in 1974, after the grant of the patent, the land was covered with vegetation, but it
disappeared in 1978 when the land was reached by the tides. During high tide, at least half of the land
(632.5 square meters) is 6 feet deep under water and three (3) feet deep during low tide. Petitioner avers
that the land should be considered as a foreshore land and should be returned to public dominion.

ISSUE:
Should the said land be considered foreshore lands and be returned to public dominion?

HELD:
Yes, the land should be considered as foreshore land and should be returned to public domain.

Art. 420. Of the Civil Code provides:


The following things are property of public dominion:
(1) Those intended for public use, such as roads, canals, rivers, torrents, ports and bridges constructed by
the State, banks, shores, roadsteads, and others of similar character;

Foreshore lands have been defined as the strip of land that lies between the high and low water marks and
that is alternatively wet and dry according to the low of the tide.

In this case a portion of the subject land was invaded by the waves and sea advances. During high tide, at
least half of the land (632.5 square meters) is 6 feet deep under water and three (3) feet deep during low
tide. When the sea moved towards the estate and the tide invaded it, the invaded property became foreshore
land and passed to the realm of the public domain. In fact, the Court in Government vs. Cabañgis annulled
the registration of land subject of cadastral proceedings when the parcel subsequently became foreshore
land. The subject land in this case, being foreshore land, should therefore be returned to the public domain.

Hence, the land should be returned to public domain

17
CLASSIFICATION OF PROPERTY
Property in Relation to Whom It Belongs

AS A PROPERTY OF PUBLIC DOMINION AKIN TO A PUBLIC THOROUGHFARE, A ROAD RIGHT-OF-


WAY (RROW) CANNOT BE REGISTERED IN THE NAME OF PRIVATE PERSONS UNDER THE LAND
REGISTRATION LAW AND BE THE SUBJECT OF A TORRENS TITLE; AND IF ERRONEOUSLY
INCLUDED IN A TORRENS TITLE, THE LAND INVOLVED REMAINS AS SUCH A PROPERTY OF
PUBLIC DOMINION

16. Hi-Lon Manufacturing, Inc. v. Commission on Audit


G.R. No. 210669, August 1, 2017
Peralta, J.

FACTS:
This is a Petition for Certiorari wherein HI-LON Manufacturing, Inc. (HI-LON) argues that the Commission
on Audit (COA) committed grave abuse of discretion when it held that HI-LON is not entitled to payment of
just compensation for the 29,690 sq. m. portion (used as road right-of-way (RROW) by the government) of
the subject property registered under its name.

Sometime in 1978, the government, through the then Ministry of Public Works and Highways (now DPWH),
converted to a RROW a 29,690 sq. m. portion of the 89,070 sq. m. parcel of land (subject property) located
in Calamba, Laguna, for the Manila South Expressway Extension Project. The subject property was
registered in the name of Commercial and Industrial Real Estate Corporation (CIREC) which was later on
transferred to Philippine Polymide Industrial Corporation (PPIC). PPIC then mortgaged the subject property
with the Development Bank of the Philippines (DBP) which later acquired the property in a foreclosure
proceeding in 1985.

In 1987, the DBP submitted all its acquired assets, including the subject property, to the Asset Privatization
Trust (APT) for disposal. APT disposed the subject property in a public bidding and subsequently executed
a Deed of Sale with TG Property, Inc. (TGPI) for the subject property. A Transfer Certificate of Title (TCT)
was issued in the name of TGPI, covering the entire 89,070 sq. m. subject property, including the 29,690
sq. m. RROW.

In 1995, TGPI executed a Deed of Absolute Sale in favor of HI-LON over the entire 89,070 sq. m. subject
property. HI-LON registered the Deed with the Register of Deeds which issued in its name a TCT.

Sometime in 1998, HI-LON requested assistance from DPWH for payment of just compensation for the
29,690 sq. m. portion of the subject property converted to a RROW. The DPWH created an Ad Hoc
Committee which valued the RROW at ₱2,500/sq. m. However, on post audit, the Supervising Auditor of
the DPWH arrived at the amount of ₱19.40/sq. m. as reasonable compensation.

Aggrieved, HI-LON filed a petition for review before the COA. In its assailed Decision, the COA declared
HI-LON not entitled to just compensation. The COA concluded that the proper valuation of the claim for just
compensation is irrelevant as HI-LON is not entitled thereto in the first place because HI-LON or its
predecessor-in-interest TGPI does not own the RROW in question, as it has been the property of the
Republic of the Philippines since its acquisition by the DBP up to the present.

ISSUE:
Whether or not HI-LON owns the RROW which is covered by the TCT registered under its name.

HELD:
No.

Article 420 of the New Civil Code considers as property of public dominion those intended for public use,
such as roads, canals, torrents, ports and bridges constructed by the state, banks, shores, roadsteads, and
others of similar character.

Being of similar character as roads for public use, a RROW can be considered as a property of public
dominion, which is outside the commerce of man, and cannot be leased, donated, sold, or be the object of
a contract, except insofar as they may be the object of repairs or improvements and other incidental matters.

18
As a property of public dominion akin to a public thoroughfare, a RROW cannot be registered in the name
of private persons under the Land Registration Law and be the subject of a Torrens Title; and if erroneously
included in a Torrens Title, the land involved remains as such a property of public dominion. In Manila
International Airport Authority v. Court of Appeals, the Court declared that properties of public dominion,
being for public use, are not subject to levy, encumbrance or disposition through public or private sale. "Any
encumbrance, levy on execution or auction sale of any property of public dominion is void for being contrary
to public policy. Essential public services will stop if properties of public dominion are subject to
encumbrances, foreclosures and auction sale."

19
CLASSIFICATION OF PROPERTY
Property in Relation to Whom It Belongs

RIVERS AND THEIR NATURAL BEDS ARE OF PUBLIC DOMINION

17. Republic v. Santos III


G.R. No. 160453, November 12, 2012
Bersamin, J.

FACTS:
Respondent Arcadio Ivan A. Santos III (Arcadio Ivan) applied for the registration of Lot 4998-B (the property)
in the RTC in Paranaque City. Arcadio Ivan amended his application for land registration to include Arcadio,
Jr. as his co-applicant because of the latter’s co-ownership of the property. He alleged that the property had
been formed through accretion and had been in their joint open, notorious, public, continuous and adverse
possession for more than 30 years.

The City of Parañaque (the City) opposed the application for land registration, stating that title to the property
could not be registered in favor of the applicants for the reason that the property was an orchard that had
dried up and had not resulted from accretion.

The RTC granted the application for land registration, which was later on affirmed by CA. The Republic filed
a MR but was denied. Hence, this appeal, in which the Republic urges that assuming that the land sought
to be registered was previously a part of the Paranaque River which became an orchard after it dried up,
the respondents could not claim the property by virtue of acquisitive prescription.

ISSUE:
Whether or not a dried-up river bed forms part of public dominion and therefore, cannot be acquired by
acquisitive prescription.

HELD:
Yes. Rivers and their natural beds are of public dominion.

Article 419 of the Civil Code distinguishes property as being either of public dominion or of private
ownership. Article 420 of the Civil Code lists the properties considered as part of public dominion, namely:
(a) those intended for public use, such as roads, canals, rivers, torrents, ports and bridges constructed by
the State, banks, shores, roadsteads, and others of similar character; and (b) those which belong to the
State, without being for public use, and are intended for some public service or for the development of the
national wealth. As earlier mentioned, Article 502 of the Civil Code declares that rivers and their natural
beds are of public dominion.

Indeed, under the Regalian doctrine, all lands not otherwise appearing to be clearly within private ownership
are presumed to belong to the State. No public land can be acquired by private persons without any grant,
express or implied, from the Government. It is indispensable, therefore, that there is a showing of a title
from the State. Occupation of public land in the concept of owner, no matter how long, cannot ripen into
ownership and be registered as a title.

Subject to the exceptions defined in Article 461 of the Civil Code (which declares river beds that are
abandoned through the natural change in the course of the waters as ipso facto belonging to the owners of
the land occupied by the new course, and which gives to the owners of the adjoining lots the right to acquire
only the abandoned river beds not ipso facto belonging to the owners of the land affected by the natural
change of course of the waters only after paying their value), all river beds remain property of public
dominion and cannot be acquired by acquisitive prescription unless previously declared by the Government
to be alienable and disposable. Considering that Lot 4998-B was not shown to be already declared to be
alienable and disposable, respondents could not be deemed to have acquired the property through
prescription.

20
CLASSIFICATION OF PROPERTY
Property in Relation to Whom It Belongs

LANDS RECLAIMED FROM FORESHORE AND SUBMERGED AREAS ALSO FORM PART OF THE
PUBLIC DOMAIN AND ARE ALSO INALIENABLE, UNLESS CONVERTED PURSUANT TO LAW INTO
INALIENABLE OR DISPOSABLE LANDS OF THE PUBLIC DOMAIN

18. Chavez v. Public Estates Authority


G.R. No. 133250, July 9, 2002
Carpio, J.

FACTS:
In 1973, the government, through the Commissioner of Public Highways, signed a contract with the
Construction and Development Corporation of the Philippines (CDCP) to reclaim certain foreshore and
offshore areas of Manila Bay.

In 1977, then President Marcos issued Presidential Decree No. 1084 creating PEA. PD No. 1084 tasked
PEA "to reclaim land, including foreshore and submerged areas," and "to develop, improve, acquire, x x x
lease and sell any and all kinds of lands." On the same date, then President Marcos issued PD No. 1085
transferring to PEA the "lands reclaimed in the foreshore and offshore of the Manila Bay" under the Manila-
Cavite Coastal Road and Reclamation Project (MCCRRP).

In 1988, then President Aquino issued Special Patent No. 3517, granting and transferring to PEA "the
parcels of land so reclaimed under the MCCRRP containing a total area of one million nine hundred fifteen
thousand eight hundred ninety-four (1,915,894) square meters.

In 1995, PEA entered into a Joint Venture Agreement with AMARI, a private corporation, to develop the
Freedom Islands.

In 1998, petitioner Frank I. Chavez, as a taxpayer, filed the instant Petition for Mandamus with Prayer for
the Issuance of a Writ of Preliminary Injunction and Temporary Restraining Order. Petitioner assails the
sale to AMARI of lands of the public domain as a blatant violation of Section 3, Article XII of the 1987
Constitution prohibiting the sale of alienable lands of the public domain to private corporations.

In a Resolution dated March 23, 1999, the Court gave due course to the petition and required the parties to
file their respective memoranda.

PEA and AMARI signed the Amended Joint Venture Agreement (Amended JVA) which was approved by
then President Joseph E. Estrada in 1999. The Amended JVA is not an ordinary commercial contract but
one which seeks to transfer title and ownership to 367.5 hectares of reclaimed lands and submerged areas
of Manila Bay to a single private corporation.

ISSUE:
Whether or not reclaimed lands are of public domain

HELD:
Yes. Lands reclaimed from foreshore and submerged areas also form part of the public domain and are
also inalienable, unless converted pursuant to law into alienable or disposable lands of the public domain.

The ownership of lands reclaimed from foreshore and submerged areas is rooted in the Regalian doctrine
which holds that the State owns all lands and waters of the public domain.

The Regalian doctrine is deeply implanted in our legal system. Foreshore and submerged areas form part
of the public domain and are inalienable. Lands reclaimed from foreshore and submerged areas also form
part of the public domain and are also inalienable, unless converted pursuant to law into alienable or
disposable lands of the public domain. Historically, lands reclaimed by the government are sui generis, not
available for sale to private parties unlike other alienable public lands. Reclaimed lands retain their inherent
potential as areas for public use or public service.

The mere fact that alienable lands of the public domain like the Freedom Islands are transferred to PEA and
issued land patents or certificates of title in PEA's name does not automatically make such lands private.

21
PD No. 1085, coupled with President Aquino’s actual issuance of a special patent covering the Freedom
Islands, is equivalent to an official proclamation classifying the Freedom Islands as alienable or disposable
lands of the public domain, open to disposition or concession to qualified parties.

While PEA may sell its alienable or disposable lands of the public domain to private individuals, it cannot
sell any of its alienable or disposable lands of the public domain to private corporations.

22
CLASSIFICATION OF PROPERTY
Property in Relation to Whom It Belongs

THERE MUST BE A LAW OR PRESIDENTIAL PROCLAMATION OFFICIALLY CLASSIFYING THE


RECLAIMED LANDS AS ALIENABLE OR DISPOSABLE AND OPEN TO DISPOSITION OR
CONCESSION

19. Chavez v. National Housing Authority


G.R. No. 164527, August 15, 2007
Velasco, Jr., J.

FACTS:
A Petition for Prohibition and Mandamus with Prayer for Temporary Restraining Order and/or Writ of
Preliminary Injunction under Rule 65.

Francisco Chavez, in his capacity as taxpayer, seeks to declare null and void the Joint Venture Agreement
(JVA) between the National Housing Authority (NHA) and R-II Builders, Inc. (RBI) Pursuant to MO 161
issued by Pres. Corazon Aquino, approving and directing the implementation of the Comprehensive and
Integrated Metropolitan Manila Waste Management Plan, NHA was ordered to "conduct feasibility studies
and develop low-cost housing projects at the dumpsite..”. NHA prepared the feasibility studies of the
Smokey Mountain low-cost housing project which resulted in the formulation of the "Smokey Mountain
Development Plan and Reclamation Project" (SMDPRP).

In 1992, President Ramos authorized NHA to enter into between NHA and RBI (one of the top two
contractors). He issued Proclamation No. 465 increasing the proposed area for reclamation. In 1995,
pursuant to Proclamation No. 39, the DENR issued Special Patent No. 3591 conveying in favor of NHA an
area of 211,975 square meters covering the Smokey Mountain Dumpsite. In 1996, the DENR issued Special
Patent No. 3598 conveying in favor of NHA an additional 390,000 square meter area.

ISSUES:
1. Are reclaimed lands classified as alienable and disposable lands of the public domain?
2. Must there be a law or presidential proclamation officially classifying these reclaimed lands as alienable
and disposable and open to disposition or concession for the reclaimed lands to be alienable?

HELD:
1. YES. When Proclamations Nos. 39 and 465 were issued, the inalienable lands covered by said
proclamations were converted to alienable and disposable lands of public domain. Furthermore, when the
titles to such reclaimed lands were transferred to the NHA, said alienable and disposable lands of public
domain were automatically classified as lands of the private domain or patrimonial properties of the State.
The reason is obvious: if the reclaimed land is not converted to patrimonial land once transferred to NHA,
then it would be useless to transfer it to the NHA since it will not be able to transfer such lands to qualified
entities and thus, it will not achieve its purpose.

2. YES. The reclaimed lands across R-10 were classified alienable and disposable lands of public domain
of the State for the following reasons: First, there were three (3) presidential proclamations classifying the
reclaimed lands across R-10 as alienable or disposable hence open to disposition or concession. Secondly,
Special Patents Nos. 3591, 3592, and 3598 issued by the DENR anchored on Proclamations Nos. 39 and
465 issued by President Ramos, without doubt, classified the reclaimed areas as alienable and disposable.
Admittedly, it cannot be said that MO 415, Proclamations Nos. 39 and 465 are explicit declarations that the
lands to be reclaimed are classified as alienable and disposable. We find however that such conclusion is
derived and implicit from the authority given to the NHA to transfer the reclaimed lands to qualified
beneficiaries. MO 415 and Proclamations Nos. 39 and 465 cumulatively and jointly taken together with
Special Patent Nos. 3591, 3592, and 3598 more than satisfy the requirement in Public Estates Authority
that "[t]here must be a law or presidential proclamation officially classifying these reclaimed lands as
alienable or disposable and open to disposition or concession.”

23
CLASSIFICATION OF PROPERTY
Property in Relation to Whom It Belongs

PROPERTIES OF PUBLIC DOMINION, EVEN IF TITLED IN THE NAME OF AN INSTRUMENTALITY,


REMAIN OWNED BY THE REPUBLIC OF THE PHILIPPINES

20. City of Lapu-Lapu v. Philippine Economic Zone Authority


G.R. No. 184203, November 26, 2014
Leonen, J.

FACTS:
These are consolidated petitions for review on certiorari the City of Lapu-Lapu and the Province of Bataan
separately filed against the PEZA.

In the exercise of his legislative powers, President Ferdinand E. Marcos issued Presidential Decree No. 66
in 1972, declaring as government policy the establishment of export processing zones in strategic locations
in the Philippines. Export Processing Zone Authority (EPZA) was created to operate, administer, and
manage the export processing zones established in the Port of Mariveles, Bataan and such other export
processing zones that may be created by virtue of the decree. The decree declared the EPZA non-profit in
character with all its revenues devoted to its development, improvement, and maintenance. To maintain this
non-profit character, the EPZA was declared exempt from all taxes.

In 1979, President Marcos issued Proclamation No. 1811, establishing the Mactan Export Processing Zone.
Certain parcels of land of the public domain located in the City of Lapu-Lapu in Mactan, Cebu were reserved
to serve as site of the Mactan Export Processing Zone. In 1995, the PEZA was created by virtue of Republic
Act No. 7916 or "the Special Economic Zone Act of 1995" to operate, administer, manage, and develop
economic zones in the country. President Ramos issued Executive Order No. 282, directing the PEZA to
assume and exercise all of the EPZA’s powers, functions, and responsibilities.

FIRST PETITION
In 1998 the City of Lapu-Lapu demanded from the PEZA 32,912,350.08 in real property taxes for the period
from 1992 to 1998 on the PEZA’s properties located in the Mactan Economic Zone, which granted the same.
The MR was denied. The CA dismissed the appeal for the ground of wrong mode of appeal.

SECOND PETITION
After the City of Lapu-Lapu had demanded payment of real property taxes from the PEZA, the Province of
Bataan followed suit. PEZA filed a petition for injunction before the Regional Trial Court of Pasay City,
arguing that it is exempt from payment of real property taxes. The trial court denied the PEZA’s petition for
injunction. The PEZA filed before the Court of Appeals a petition for certiorari but the CA dismissed. Hence,
this petition.

ISSUE:
Is PEZA exempt from payment of real property taxes?

HELD:
YES. The PEZA’s predecessor, the EPZA, was declared non-profit in character with all its revenues devoted
for its development, improvement, and maintenance. Consistent with this non-profit character, the EPZA
was explicitly declared exempt from real property taxes under its charter. The Special Economic Zone Act
of 1995, on the other hand, does not specifically exempt the PEZA from payment of real property taxes.
Nevertheless, we rule that the PEZA is exempt from real property taxes by virtue of its charter. A provision
in the Special Economic Zone Act of 1995 explicitly exempting the PEZA is unnecessary. The PEZA
assumed the real property exemption of the EPZA under PD No. 66.

Under Section 234(a) of the Local Government Code, real properties owned by the Republic of the
Philippines are exempt from real property taxes. Properties owned by the state are either property of public
dominion or patrimonial property. Article 420 of the Civil Code of the Philippines enumerates property of
public dominion. Properties of public dominion, being for public use, are not subject to levy, encumbrance
or disposition through public or private sale. Any encumbrance, levy on execution or auction sale of any
property of public dominion is void for being contrary to public policy. Essential public services will stop if
properties of public dominion are subject to encumbrances, foreclosures and auction sale.

24
In this case, the properties sought to be taxed are located in publicly owned economic zones. These
economic zones are property of public dominion. The City seeks to tax properties located within the Mactan
Economic Zone, the site of which was reserved by President Marcos under Proclamation No. 1811.
Reserved lands are inalienable and outside the commerce of man, and remain property of the Republic until
withdrawn from public use either by law or presidential proclamation.

Properties of public dominion, even if titled in the name of an instrumentality as in this case, remain owned
by the Republic of the Philippines. All told, the PEZA is an instrumentality of the national government.
Furthermore, the lands owned by the PEZA are real properties owned by the Republic of the Philippines.
The City of Lapu Lapu and the Province of Bataan thus cannot collect real property taxes from the PEZA.
Petitions denied.

25
CLASSIFICATION OF PROPERTY
Property in Relation to Whom It Belongs

A PROPERTY OF PUBLIC DOMINION CANNOT BE BURDENED BY A VOLUNTARY EASEMENT OF


RIGHT OF WAY IN FAVOR OF A PERSON, AND THE LATTER CANNOT CLAIM ANY RIGHT OF
POSSESSION OVER IT IF ITS USE BY THE PUBLIC IS BY MERE TOLERANCE OF THE
GOVERNMENT

21. Villarico v. Sarmiento


G.R. No. 136438, November 11, 2004
Sandoval-Gutierrez, J.

FACTS:
Teofilo C. Villarico is the owner of a lot in La Huerta, Parañaque City with an area of 66 square meters and
covered by TCT No. 95453. It is separated from the Ninoy Aquino Avenue (highway) by a strip of land
belonging to the government. As this highway was elevated by four (4) meters and therefore higher than
the adjoining areas, the DPWH constructed stairways at several portions of this strip of public land to enable
the people to have access to the highway.

Petitioner filed a complaint for accion publiciana against respondents whose structures on the government
land allegedly closed his "right of way" to the Ninoy Aquino Avenue. Respondents claim that petitioner has
no right over the subject property as it belongs to the government. RTC found that petitioner has never been
in possession of any portion of the public land in question. CA affirmed.

ISSUE:
Does the existence of petitioner’s right of way carry possession over the same?

HELD:
No. The of petitioner’s right of way carry possession over the same.

It is not disputed that the lot on which petitioner’s alleged "right of way" exists belongs to the state or property
of public dominion.

Public use is "use that is not confined to privileged individuals, but is open to the indefinite public." Records
show that the lot on which the stairways were built is for the use of the people as passageway to the highway.
Consequently, it is a property of public dominion.

Property of public dominion is outside the commerce of man and hence it: (1) cannot be alienated or leased
or otherwise be the subject matter of contracts; (2) cannot be acquired by prescription against the State; (3)
is not subject to attachment and execution; and (4) cannot be burdened by any voluntary easement.

Considering that the lot on which the stairways were constructed is a property of public dominion, it can not
be burdened by a voluntary easement of right of way in favor of herein petitioner. In fact, its use by the
public is by mere tolerance of the government through the DPWH. Petitioner cannot appropriate it for
himself. Verily, he can not claim any right of possession over it.

26
CLASSIFICATION OF PROPERTY
Property in Relation to Whom It Belongs

LAND CANNOT BE CONSIDERED IPSO JURE CONVERTED TO PRIVATE PROPERTY EVEN UPON
THE SUBSEQUENT DECLARATION OF IT AS ALIENABLE AND DISPOSABLE

22. Heirs of Malabanan v. Republic


G.R. No. 179987, September 3, 2013
Bersamin, J.

FACTS:
This is a motion for reconsideration of both the parties who assail the decision of the SC affirming the
decision of CA which denied the application of the petitioners for the registration of a parcel of land on the
ground that they had not established by sufficient evidence their right to the registration in accordance with
either Section 14 (1) or Section 14 (2) of PD 1529 (Property Registration Decree)

Mario Malabanan purchased a property from Eduardo Velazco and filed an application for land registration
in the RTC, claiming that the property formed part of the alienable and disposable land of the public domain,
and that he and his predecessors-in-interest had been in open, continuous, uninterrupted, public and
adverse possession and occupation of the land for more than 30 years, thereby entitling him to the judicial
confirmation of his title. As proof, Malabanan presented a certification issued by CENRO-DENR, declaring
the land as alienable or disposable since March 15, 1982. RTC granted Malabanan's application, however
CA reversed the decision. SC denied the petition for review on certiorari on the ground that they had not
established by sufficient evidence their right to the registration in accordance with either Section 14 (1) or
Section 14 (2) of PD 1529. Hence, both parties filed a motion for consideration.

The petitioners submit that the mere classification of the land as alienable or disposable should be deemed
sufficient to convert it into patrimonial property of the State. The Republic reiterates its view that an applicant
is entitled to registration only when the land subject of the application had been declared alienable and
disposable since June 12, 1945 or earlier.

ISSUE:
Whether or not the mere classification of the land as alienable or disposable should be deemed sufficient
to convert it into patrimonial property of the state

HELD:
No. Mere classification of the land as alienable or disposable is not sufficient to convert it to patrimonial
property.

To sum up, we now observe the following rules relative to the disposition of public land or lands of the public
domain, namely:
(1) As a general rule and pursuant to the Regalian Doctrine, all lands of the public domain belong to the
State and are inalienable. Lands that are not clearly under private ownership are also presumed to belong
to the State and, therefore, may not be alienated or disposed;
(2) The following are excepted from the general rule, to wit:
(a) Agricultural lands of the public domain are rendered alienable and disposable through any of the
exclusive modes enumerated under Section 11 of the Public Land Act. If the mode is judicial confirmation
of imperfect title under Section 48 (b) of the Public Land Act, the agricultural land subject of the application
needs only to be classified as alienable and disposable as of the time of the application, provided the
applicant's possession and occupation of the land dated back to June 12, 1945, or earlier. Thereby, a
conclusive presumption that the applicant has performed all the conditions essential to a government grant
arises, and the applicant becomes the owner of the land by virtue of an imperfect or incomplete title. By
legal ‫ﰁ‬fiction, the land has already ceased to be part of the public domain and has become private property.
(b)Lands of the public domain subsequently classified or declared as no longer intended for public use or
for the development of national wealth are removed from the sphere of public dominion and are considered
converted into patrimonial lands or lands of private ownership that may be alienated or disposed through
any of the modes of acquiring ownership under the Civil Code. If the mode of acquisition is prescription,
whether ordinary or extraordinary, proof that the land has been already converted to private ownership prior
to the requisite acquisitive prescriptive period is a condition sine qua non in observance of the law (Article
1113, Civil Code) that property of the State not patrimonial in character shall not be the object of prescription.
Ha

27
To reiterate, then, the petitioners failed to present sufficient evidence to establish that they and their
predecessors-in-interest had been in possession of the land since June 12, 1945. Without satisfying the
requisite character and period of possession — possession and occupation that is open, continuous,
exclusive, and notorious since June 12, 1945, or earlier — the land cannot be considered ipso jure converted
to private property even upon the subsequent declaration of it as alienable and disposable. Prescription
never began to run against the State, such that the land has remained ineligible for registration under
Section 14 (1) of the Property Registration Decree. Likewise, the land continues to be ineligible for land
registration under Section 14 (2) of the Property Registration Decree unless Congress enacts a law or the
President issues a proclamation declaring the land as no longer intended for public service or for the
development of the national wealth.

Hence, the Motion for Reconsideration is denied for lack of merit.

28
CLASSIFICATION OF PROPERTY
Property in Relation to Whom It Belongs

ACQUISITIVE PRESCRIPTION DOES NOT START TO RUN AGAINST PROPERTY WHICH IS STILL
CLASSIFIED AS PART OF PUBLIC DOMINION

23. Republic v. Aboitiz


G.R. No. 174626, October 23, 2013
Mendoza, J.

FACTS:
Aboitiz filed an Application for Registration of a parcel of land (1254 sqm) in Cebu. Hearing on the case
ensued to which Aboitiz submitted the necessary documents and presented witnesses. The RTC granted
Aboitiz’s application for registration.

The Republic appealed to the CA which reversed the ruling of the trial court for the reason that it was only
from the date of declaration of such lands as alienable and disposable that the period for counting the
statutory requirement of possession since June 12, 1945 or earlier would commence. Possession prior to
the date of declaration of the lands alienability was not included. The CA observed that the subject property
was declared as alienable and disposable only in 1957, and so the application clearly did not meet the
requirements of possession needed under the first requisite of Section 14 (1) of Presidential
Decree (P.D.) No. 1529 which must be since June 12, 1945, or earlier.

The CA later reversed itself and granted the registration upon a Motion for Reconsideration filed by Aboitiz.
In granting the application for registration of land title, the CA relied on Section 14(2) of P.D. No. 1529. It
stated that although the application for registration of Aboitiz could not be granted pursuant to Section 14(1)
of P.D. No. 1529 because the possession of his predecessor-in-interest commenced in 1963 (beyond June
12, 1945), it could prosper by virtue of acquisitive prescription under Section 14(2) of P.D. No. 1529 upon
the lapse of thirty (30) years.

ISSUE:
Is the subject land part of a classification that is susceptible to registration under Section 14(2) of PD No.
1529?

HELD:

No, the subject land is still part of the public domain and hence not subject to registration under a claim of
title arising from prescription.

In the case of Heirs of Mario Malabanan v. Republic, the Court clarified the import of Section 14(1) as
distinguished from Section 14(2) of P.D. No. 1529, viz: “(2) In complying with Section 14(2) of the Property
Registration Decree, consider that under the Civil Code, prescription is recognized as a mode of acquiring
ownership of patrimonial property. However, public domain lands become only patrimonial property not only
with a declaration that these are alienable or disposable. There must also be an express government
manifestation that the property is already patrimonial or no longer retained for public service or the
development of national wealth, under Article 422 of the Civil Code. And only when the property has become
patrimonial can the prescriptive period for the acquisition of property of the public dominion begin to run.”

Thus, under Section 14(2) of P.D. No. 1529, for acquisitive prescription to commence and operate against
the State, the classification of land as alienable and disposable alone is not sufficient. The applicant must
be able to show that the State, in addition to the said classification, expressly declared through either a law
enacted by Congress or a proclamation issued by the President that the subject land is no longer retained
for public service or the development of the national wealth or that the property has been converted into
patrimonial. Consequently, without an express declaration by the State, the land remains to be a property
of public dominion and, hence, not susceptible to acquisition by virtue of prescription.

In fine, the Court holds that the ruling of the CA lacks sufficient factual or legal justification. Hence, the Court
is constrained to reverse the assailed CA Amended Decision and Resolution and to deny the application for
registration of land title of Aboitiz.

29
CLASSIFICATION OF PROPERTY
Property in Relation to Whom It Belongs

UNLESS THE ALIENABLE AND DISPOSABLE LAND OF THE PUBLIC DOMAIN IS EXPRESSLY
CONVERTED INTO PATRIMONIAL PROPERTY, THERE IS NO WAY FOR ACQUISITIVE
PRESCRIPTION TO SET IN UNDER ARTICLE 1113 OF THE CIVIL CODE

24. Dumo v. Republic


G.R. No. 218269, June 6, 2018
Carpio, J.

FACTS:
Severa Espinas, Erlinda Espinas, Aurora Espinas, and Virginia Espinas filed a Complaint for Recovery of
Ownership, Possession and Damages with Prayer for Writ of Preliminary Injunction against the heirs of
Bernarda M. Trinidad. The Subject Property was purchased by Espinas from Carlos Calica through a Deed
of Absolute Sale dated 19 October 1943. Espinas exercised acts of dominion over the Subject Property by
appointing a caretaker to oversee and administer the property. In 1963, Espinas executed an affidavit stating
his claim of ownership over the Subject Property. Espinas had also been paying realty taxes on the Subject
Property.

Suprema Dumo filed an application for registration of two parcels of land with a total area of 1,273 square
meters. Dumo alleged that the lots belonged to her mother and that she and her siblings inherited them
upon their mother's death. She further alleged that through a Deed of Partition with Absolute Sale dated 6
February 1987, she acquired the subject lots from her siblings. Dumo traces her title from her mother,
Trinidad, who purchased the lots from Florencio Mabalay in August 1951. Mabalay was Dumo's maternal
grandfather. Mabalay, on the other hand, purchased the properties from Carlos Calica.

The heirs of Espinas opposed Dumo's application for land registration on the ground that the properties
sought to be registered by Dumo are involved in the accion reivindicatoria case. Thus, the RTC consolidated
the land registration case with the Complaint for Recovery of Ownership, Possession and Damages.

The RTC found that the heirs of Espinas had a better right to the property and rejected the application for
registration of Dumo. The Court of Appeals affirmed the RTC’s decision dismissing the application for
registration of Dumo but held instead that the property still belonged to the public domain, and the heirs of
Espinas were not able to establish open, continuous, exclusive and notorious possession and occupation
of the land under a bona fide claim of ownership since June 12, 1945 or earlier. Hence, Dumo filed this
petition before the Supreme Court raising, among others, that she and her predecessors-in-interest had
possessed the land in question for 56 years.

ISSUE:
Is the subject land part of the public domain hence not susceptible to registration under PD No. 1529?

HELD:
Yes. The land in question cannot be registered - it being a property of the public domain.

It is true that under Section 14 of PD No. 1529, one may acquire ownership of the land by prescription.
Particularly, paragraph 2 of Section 14 provides that "those who have acquired ownership of private lands
by prescription under the provision of existing laws" may file an application for registration of title to land.
The existing law mentioned in PD No. 1529 is the Civil Code of the Philippines.

Section 14(2) of PD No. 1529 puts into operation the entire regime of prescription under the Civil Code,
particularly Article 1113 in relation to Article 1137. A Indeed, Section 14(2) of PD No. 1529 provides that
one may acquire ownership of private lands by prescription.

Land of the public domain is converted into patrimonial property when there is an express declaration by
the State that the public dominion property is no longer intended for public service or the development of
the national wealth. Without such declaration, acquisitive prescription does not start to run, even if such
land is alienable and disposable and the applicant is in possession and occupation thereof.

30
Mere classification of agricultural land as alienable and disposable does not make such land patrimonial
property of the State – an express declaration by the State that such land is no longer intended for public
use, public service or the development of national wealth is imperative. This is because even with such
classification, the land remains to be part of the lands of the public domain.

Under CA No. 141, the power given to the President to classify lands as alienable and disposable extends
only to lands of the public domain. Lands of the public domain are public lands intended for public use,
or without being for public use, are intended for some public service or for the development of national
wealth.

Classifying lands as alienable and disposable does not take away from the fact that these lands still belong
to the public domain. These lands belonged to the public domain before they were classified as alienable
and disposable and they still remain to be lands of the public domain after such classification.

The alienable and disposable character of public agricultural land does not convert the land to patrimonial
property. It merely gives the State the authority to alienate or dispose the agricultural land, in accordance
with law. It is only when (1) there is an express government manifestation that the land is already patrimonial
or no longer intended for public use, public service or the development of national wealth, or (2) land which
has been classified as alienable and disposable land is actually alienated and disposed of by the State, that
such land becomes patrimonial.

In the present case, Dumo not only failed to prove that the land sought to be registered is alienable and
disposable, but also utterly failed to submit any evidence to establish that such land has been converted
into patrimonial property by an express declaration by the State. To repeat, acquisitive prescription only
applies to private lands as expressly provided in Article 1113 of the Civil Code. To register land acquired by
prescription under PD No. 1529 (in relation to the Civil Code of the Philippines), the applicant must prove
that the land is not merely alienable and disposable, but that it has also been converted into patrimonial
property of the State. Prescription will start to run only from the time the land has become
patrimonial.61 Unless the alienable and disposable land of the public domain is expressly converted into
patrimonial property, there is no way for acquisitive prescription to set in under Article 1113 of the Civil
Code.

31
CLASSIFICATION OF PROPERTY
Property in Relation to Whom It Belongs

PUBLIC LANDS NOT SHOWN HAVE BEEN CLASSIFIED, RECLASSIFIED, OR RELEASED AS


ALIENABLE AGRICULTURAL LAND OR ALIENATED TO A PRIVATE PERSON REMAIN PART OF THE
PUBLIC DOMAIN

25. Republic v. Spouses Alejandre


G.R. No. 217336, October 17, 2018
Caguioa, J.

FACTS:
The Spouses Alejandre filed an application for the registration of a lot under PD No 1529 (256 sqm). They
alleged that they are own the subject property by virtue of a deed of sale or conveyance; that the property
was sold to them by Angustia Lizardo Taleon through a Deed of Absolute Sale executed on June 20, 1990;
that said land is presently occupied by them.

The Land Registration Authority submitted a report noting discrepancies in the plan submitted by the
spouses which were referred to the Lands Management Sector or verification and correction. Eventually,
corrections were made and the trial court issued a notice setting the case for initial hearing.

The Republic opposed the application raising among others that the subject property applied for is a portion
of the public domain belonging to the Republic of the Philippines which is not subject to private appropriation.

The trial court granted the application which prompted the Republic to appeal. The CA denied the appeal
and found that the spouses were entitled to register the land having acquired ownership through the deed
of absolute sale from Taleon. The Republic appealed to the Supreme Court.

ISSUE:
Is the subject land still part of the public domain belonging to the Republic and thus not subject to private
appropriation?

HELD:
Yes, the land is still part of the public domain and cannot be privately appropriated.

Pursuant to Article 419 of the Civil Code, property, in relation to the person to whom it belongs, is either of
public dominion or of private ownership. As such, properties are owned either in a public capacity (dominio
publico) or in a private capacity (propiedad privado).

There are three kinds of property of public dominion: (1) those intended for public use; (2) those intended
for some public service; and (3) those intended for the development of national wealth.

As noted by Justice Edgardo L. Paras:

It is believed that forest and mining lands are properties of public dominion of the third class, i.e., properties
for the development of the national wealth. Upon the other hand, the public agricultural lands before being
made available to the general public should also be properties of public dominion for the development of
the national wealth (and as such may not be acquired by prescription); but after being made so available,
they become patrimonial property of the State, and therefore subject to prescription. Moreover, once already
acquired by private individuals, they become private property. x x x (Emphasis and underscoring supplied)

Thus, it can be gathered from the foregoing that the subject of the land registration application under Section
14 of PD 1529 is either alienable and disposable land of public domain or private land. While Section 14(4)
does not describe or identify the kind of land unlike in (1), which refer to "alienable and disposable lands of
the public domain;" (2), which refer to "private lands"; and (3) "private lands or abandoned river beds," the
land covered by (4) cannot be other than alienable and disposable land of public domain, i.e., public
agricultural lands and private lands or lands of private ownership in the context of Article 435.

This premise proceeds from the well-entrenched rule that all lands not appearing to be clearly of private
dominion or ownership presumptively belong to the State. Accordingly, public lands not shown to have been
classified, reclassified or released as alienable agricultural land or alienated to a private person by the State

32
remain part of the inalienable lands of public domain. Therefore, the onus to overturn, by incontrovertible
evidence, the presumption that the land subject of an application for registration is alienable and disposable
rests with the applicant.

Respondents, based on the evidence that they adduced, are apparently claiming ownership over the land
subject of their application for registration by virtue of tradition, as a consequence of the contract of sale,
and by succession in so far as their predecessors-in-interest are concerned. Both modes are derivative
modes of acquiring ownership. Yet, they failed to prove the nature or classification of the land. The fact that
they acquired the same by sale and their transferor by succession is not incontrovertible proof that it is of
private dominion or ownership. In the absence of such incontrovertible proof of private ownership, the well¬-
entrenched presumption arising from the Regalian doctrine that the subject land is of public domain or
dominion must be overcome. Respondents failed to do this.

33
CLASSIFICATION OF PROPERTY
Property in Relation to Whom It Belongs

LANDS WHICH ARE INTENDED FOR PUBLIC USE OR PUBLIC SERVICE SUCH AS RESERVATIONS
FOR PUBLIC OR QUASI-PUBLIC USES ARE PROPERTY OF THE PUBLIC DOMINION AND REMAIN
TO BE SO AS LONG AS THEY REMAIN RESERVED

26. Navy Officers’ Village Association, Inc. v. Republic


G.R. No. 177168, August 3, 2015
Brion, J.

FACTS:
This is a petition for review on certiorari assailing the December 28, 2006 decision and March 28, 2007
resolution of the Court of Appeals (CA) in CA-G.R. CV No. 85179. NOVAI alleges that the CA erred, in
declaring, among others that the property is inalienable land of the public domain.

The case stemmed from a Transfer Certificate Title (TCT) issued in Navy Officers’ Village Association, Inc
(NOVAI)’s name which covers a land situated inside the former Fort Andres Bonifacio Military Reservation
in Taguig. This property was previously a part of a larger parcel of land which TCT’s under the name of the
Republic of the Philippines.

The then President Garcia issued a Proclamation No. 423 which reserves for military purposes certain
parcels of the public domain situated in Pasig, Taguig, Paranaque, Rizal and Pasay City. Thereafter, then
President Macapagal issued Proclamation No. 461 which excluded Fort McKinley a certain portion of land
situated in the provinces abovementioned and declared them as AFP Officers’ Village to be disposed of
under the provisions of certain laws. However, this area was subsequently reserved for veterans’
rehabilitation, medicare and training center sites.

The property was the subject of deed of sale between the Republic and NOVAI to which the TCT was
registered in favour of the latter. The Republic then sought to cancel NOVAI’s title on the ground that the
property was still part of the military reservation thus inalienable land of the public domain and cannot be
the subject of sale. The RTC ruled that the property was alienable and disposable in character. The Court
of Appeals reversed RTC’s decision.

ISSUE:
Whether or not the property in question under NOVAI’s name is an inalienable land of public domain and
therefore cannot be the subject of sale

HELD:
Yes, the property is an inalienable land which remains as a part of the public domain and could not have
been validly disposed of in NOVAI’s favor.

Thus, from the perspective of the general Civil Code provisions on Property, lands which are intended for
public use or public service such as reservations for public or quasi-public uses are property of the public
dominion and remain to be so as long as they remain reserved.

As property of the public dominion, public lands reserved for public or quasi-public uses are outside the
commerce of man. They cannot be subject to sale, disposition or encumbrance; any sale, disposition or
encumbrance of such property of the public dominion is void for being contrary to law and public policy
To be subject to sale, occupation or other disposition, lands of the public domain designated as reservations
must first be withdrawn, by act of Congress or by proclamation of the President, from the public or quasi-
public use for which it has been reserved or otherwise positively declared to have been converted to
patrimonial property, pursuant to Sections 8 and 88 of C.A. No. 141 and Article 422 of the Civil Code.

NOVAI failed to discharge its burden of proving that the property was not intended for public or quasi-public
use or purpose.

34
CLASSIFICATION OF PROPERTY
Property in Relation to Whom It Belongs

REGARDLESS OF THE SOURCE OR CLASSIFICATION OF LAND IN THE POSSESSION OF A


MUNICIPALITY, EXCEPTING THOSE ACQUIRED WITH ITS OWN FUNDS IN ITS PRIVATE OR
CORPORATE CAPACITY, SUCH PROPERTY IS HELD IN TRUST FOR THE STATE FOR THE BENEFIT
OF ITS INHABITANTS, WHETHER IT BE FOR GOVERNMENTAL OR PROPRIETARY PURPOSES

27. Rabucov. Villegas


G.R. Nos. L-24661, L-24915, & L-24916, February 28, 1974
Teehankee, J.

FACTS:
The case at bar is a consolidated case involving the conversion of the lots in Malate area into disposable
and alienable lands of the state and placing its administration and disposal to the Land Tenure
Administration to be subdivided into lots and selling it to bona fide occupants thereof in installments
constitutes a deprivation of the City of Manila of its property by providing for its sale without the payment of
just compensation under Republic Act 3120.

Respondents city officials' contended that the Act must be stricken down as unconstitutional for depriving
the city of Manila of the lots in question and providing for their sale in subdivided small lots to bona fide
occupants or tenants without payment of just compensation.

ISSUE:
Whether or not the converted lots owned by the City of Manila be disposed by the Congress even without
just compensation to such city.

HELD:
Yes, the lots in question are manifestly owned by the city in its public and governmental capacity and are
therefore public property over which Congress had absolute control as distinguished from patrimonial
property owned by it in its private or proprietary capacity of which it could not be deprived without due
process and without just compensation.

Republic Act 3120 expressly declared that the properties were "reserved as communal property" and
ordered their conversion into "disposable and alienable lands of the State" for sale in small lots to the bona
fide occupants thereof. It is established doctrine that the act of classifying State property calls for the
exercise of wide discretionary legislative power which will not be interfered with by the courts.

The Court reaffirmed the established general rule that "regardless of the source or classification of land in
the possession of a municipality, excepting those acquired with its own funds in its private or corporate
capacity, such property is held in trust for the State for the benefit of its inhabitants, whether it be for
governmental or proprietary purposes. It holds such lands subject to the paramount power of the
legislature to dispose of the same, for after all it owes its creation to it as an agent for the performance of a
part of its public work, the municipality being but a subdivision or instrumentality thereof for purposes of
local administration. Accordingly, the legal situation is the same as if the State itself holds the property and
puts it to a different use" and stressed that "the property, as has been previously shown, was not acquired
by the City of Manila with its own funds in its private or proprietary capacity. That it has in its name a
registered title is not questioned, but this title should be deemed to be held in trust for the State as the land
covered thereby was part of the territory of the City of Manila granted by the sovereign upon its creation."

35
CLASSIFICATION OF PROPERTY
Property in Relation to Whom It Belongs

PROPERTIES OF PUBLIC DOMINION DEVOTED TO PUBLIC USE AND MADE AVAILABLE TO THE
PUBLIC IN GENERAL ARE OUTSIDE THE COMMERCE OF MAN AND CANNOT BE DISPOSED OF OR
LEASED BY THE LOCAL GOVERNMENT UNIT TO PRIVATE PERSONS

28. Macasiano v. Diokno


G.R. No. 97764, August 10, 1992
Medialdea, J.

FACTS:
This is a petition for certiorari under Rule 65 of the Rules of Court seeking the annulment of the decision of
the Regional Trial Court of Makati, Branch 62, which granted the writ of preliminary injunction applied for by
respondents Municipality of Parañaque and Palanyag Kilusang Bayan for Service (Palanyag for brevity)
against petitioner herein. This is in connection with Ordinance No. 86, Series of 1990 passed by respondent
Municipality of Paranque that authorized the closure of J. Gabriel, G.G. Cruz, Bayanihan, Lt. Garcia
Extension and Opena Streets located at Baclaran, Paranaque Metro Manila and the establishment of a flea
market thereon. The municipal council of Paranaque thereafter issued a resolution authorizing Paranaque
Mayor Walfrido N. Ferrer to enter into a contract with any service cooperative for the establishment,
operation, maintenance and management of flea markets and/or vending areas. By virtue of this,
respondent municipality and respondent Palanyag, a service cooperative, entered into an agreement
whereby the latter shall operate, maintain and manage the flea market in the aforementioned streets with
the obligation to remit dues to the treasury of the municipal government of Paranaque. Consequently,
market stalls were put up by Palanyag on the said streets. Petitioner Macasiano, PNP Superintendent of
the Metropolitan Traffic Command, then ordered the destruction and confiscation of the stalls along the
abovementioned streets.

The respondents filed with the trial court a joint petition for prohibition and mandamus with damages and
prayer for preliminary injunction, to which the petitioner filed his opposition to the issuance of the writ of
preliminary injunction. The trial court upheld the validity of the ordinance in question.

ISSUE:
Whether or not a road or street, being used for public service and therefore public properties, can be subject
to private appropriation or private contract even by the Municipality of Parañaque.

HELD:
No, it is still a public property therefore cannot be subject of private appropriation. The property of provinces,
cities and municipalities is divided into property for public use and patrimonial property (Art. 423, Civil Code).
As to what consists of property for public use, Article 424 of Civil Code states:

Art. 424. Property for public use, in the provinces, cities and municipalities, consists of the provincial roads,
city streets, the squares, fountains, public waters, promenades, and public works for public service paid for
by said provinces, cities or municipalities.

All other property possessed by any of them is patrimonial and shall be governed by this Code, without
prejudice to the provisions of special laws.

Article 424 of the Civil Code lays down the basic principle that properties of public dominion devoted to
public use and made available to the public in general are outside the commerce of man and cannot be
disposed of or leased by the local government unit to private persons. Aside from the requirement of due
process which should be complied with before closing a road, street or park, the closure should be for the
sole purpose of withdrawing the road or other public property from public use when circumstances show
that such property is no longer intended or necessary for public use or public service. When it is already
withdrawn from public use, the property then becomes patrimonial property of the local government unit
concerned.

It is only then that the respondent municipality can "use or convey them for any purpose for which other real
property belonging to the local unit concerned might be lawfully used or conveyed" in accordance with the
last sentence of Section 10, Chapter II of Blg. 337, known as Local Government Code.

36
Those roads and streets which are available to the public in general and ordinarily used for vehicular traffic
are still considered public property devoted to public use. In such case, the local government has no power
to use it for another purpose or to dispose of or lease it to private persons.

37
CLASSIFICATION OF PROPERTY
Property in Relation to Whom It Belongs

THE CLASSIFICATION OF MUNICIPAL PROPERTY DEVOTED FOR DISTINCTLY GOVERNMENTAL


PURPOSES AS PUBLIC SHOULD PREVAIL OVER THE CIVIL CODE CLASSIFICATION

29. Province of Zamboanga Del Norte v. City of Zamboanga


G.R. No. L-24440, March 28, 1968
Bengzon, J.P., J.

FACTS:
Prior to its incorporation as a chartered city, the Municipality of Zamboanga used to be the provincial capital
of the then Zamboanga Province. On October 12, 1936, Commonwealth Act 39 was approved converting
the Municipality of Zamboanga into Zamboanga City. Sec. 50 of the Act also provided that “buildings and
properties which the province shall abandon upon the transfer of the capital to another place will be acquired
and paid for by the City of Zamboanga at a price to be fixed by the Auditor Genera”l. The properties and
buildings referred to consisted of 50 lots and some buildings constructed thereon, located in the City of
Zamboanga and covered individually by Torrens certificates of title in the name of Zamboanga Province. In
1952, Republic Act 711 was approved dividing the province of Zamboanga into two (2): Zamboanga del
Norte and Zamboanga del Sur. Sec. 6 of that law provided that funds of the assets and properties and the
obligations of the province of Zamboanga shall be divided equitably between the Province of Zamboanga
del Norte and the Province of Zamboanga del Sur by the President of the Philippines. The Executive
Secretary, by order of the President, issued a ruling holding that Zamboanga del Norte had a vested right
as owner of the properties mentioned in Sec. 50 of Commonwealth Act 39, and is entitled to the price
thereof, payable by Zamboanga City. However, RA 3039 was approved providing that all buildings,
properties, and assets belonging to former province f Zamboanga located at the City of Zamboanga are
transferred to the latter free of charge. Consequently, the Secretary of Finance ordered the Commissioner
of Internal Revenue to stop from effecting further payments to Zamboanga del Norte and to return to
Zamboanga City the sum of P57,373.46 taken from it out of the internal revenue allotment of Zamboanga
del Norte. Plaintiff-appellee Zamboanga del Norte filed a complaint entitled "Declaratory Relief with
Preliminary Mandatory Injunction"

ISSUE:
What is the nature of the 50 lots and buildings, patrimonial or properties for public use?

HELD:
If the property is owned by the municipality or municipal corporation, in its public and governmental capacity,
the property is public and Congress has absolute control over it. But if the property is owned in its private
or proprietary capacity, then it is patrimonial and Congress has no absolute control. The municipality cannot
be deprived of it without due process and payment of just compensation.

Applying the Civil Code, all the properties in question, except the two (2) lots used as High School
playgrounds, could be considered as patrimonial properties of the former Zamboanga province. Even the
capital site, the hospital and leprosarium sites, and the school sites will be considered patrimonial for they
are not for public use. They would fall under the phrase "public works for public service" for it has been held
that under the ejusdem generis rule, such public works must be for free and indiscriminate use by anyone,
just like the preceding enumerated properties in the first paragraph of Art 424. 7 The playgrounds, however,
would fit into this category.

On the other hand, applying the norm obtaining under the principles constituting the law of Municipal
Corporations, all those of the 50 properties in question which are devoted to public service are deemed
public; the rest remain patrimonial. Under this norm, to be considered public, it is enough that the property
be held and, devoted for governmental purposes like local administration, public education, public health,
etc.

We are more inclined to uphold this latter view. Moreover, this Court is not inclined to hold that municipal
property held and devoted to public service is in the same category as ordinary private property. The
consequences are dire. As ordinary private properties, they can be levied upon and attached. They can
even be acquired thru adverse possession — all these to the detriment of the local community. Lastly, the
classification of properties other than those for public use in the municipalities as patrimonial under Art. 424
of the Civil Code — is without prejudice to the provisions of special laws." For purpose of this article, the

38
principles, obtaining under the Law of Municipal Corporations can be considered as "special laws". Hence,
the classification of municipal property devoted for distinctly governmental purposes as public should prevail
over the Civil Code classification in this particular case.

39
CLASSIFICATION OF PROPERTY
Property in Relation to Whom It Belongs

TO CONVERT A BARRIO ROAD INTO PATRIMONIAL PROPERTY, THE LAW REQUIRES THE LGU
TO ENACT AN ORDINANCE, APPROVED BY AT LEAST TWO-THIRDS (2/3) OF THE SANGGUNIAN
MEMBERS

30. Alolino v. Flores


G.R. No. 198774, April 4, 2016
Brion, J.

FACTS:
This is a petition for review on certiorari filed from the decision of the CA which reversed the Regional RTC
decision which dismissed petitioner Teofilo Alolino's complaint respondents Fortunato Flores and Anastacia
Marie Flores for the removal of their illegally constructed structure.

Alolino is the registered owner of two contiguous parcels of land situated at No. 47 Gen. Luna Street,
Barangay Tuktukan, Taguig. Alolino initially constructed a bungalow-type house on the property. Later, he
added a second floor to the structure and extended his two-storey house up to the edge of his property. The
respondent spouses Fortunato and Anastacia (Marie) Flores constructed their house/sari sari store on the
vacant municipal/barrio road immediately adjoining the rear perimeter wall of Alolino's house. Since they
were constructing on a municipal road, the respondents could not secure a building permit. The
respondents' construction deprived Alolino of the light and ventilation he had previously enjoyed and
prevented his ingress and egress to the municipal road through the rear door of his house. Alolino
demanded that the respondent spouses remove their structure but the latter refused. Thus, he complained
about the illegal construction to the Building Official of the Municipality of Taguig and another with the
Barangay of Tuktukan. The Building Official issued a Notice of Illegal Construction against the respondents.
Sometime, the respondents began constructing a second floor to their structure, again without securing a
building permit. Alolino filed another complaint with the Building Official of Taguig. The building official issued
a second Notice of Illegal Construction. The respondents did not comply with the directive from the building
official. Alolino sent them a demand letter. Despite receipt of the demand letter, the respondents refused to
comply. Thus, Alolino filed a complaint against the respondents with the RTC praying for: (1) the removal
of the encroaching structure; (2) the enforcement of his right to easement of light and view; and (3) the
payment of damages.

The respondents admitted to them that they did not secure a building permit because the property was
constructed on a municipal/barrio road. They claimed, however, that on March 1, 2004, the Sangguniang
Bayan of Taguig (the Sanggunian) reclassified the property as a residential lot from its prior classification
as a barrio/municipal road.

ISSUE:
Can a barrio road be converted to a patrimonial property by an LGU resolution?

HELD:
No.

The Local Government Code authorizes an LGU to withdraw a local road from public use. To convert
a barrio road into patrimonial property, the law requires the LGU to enact an ordinance, approved by at least
two-thirds (2/3) of the Sanggunian members, permanently closing the road.

In this case, the Sanggunian did not enact an ordinance but merely passed a resolution. The difference
between an ordinance and a resolution is settled in jurisprudence: an ordinance is a law but a resolution is
only a declaration of sentiment or opinion of the legislative body. Properties of the local government that are
devoted to public service are deemed public and are under the absolute control of Congress. Hence, LGUs
cannot control or regulate the use of these properties unless specifically authorized by Congress, as is the
case with Section 21 of the LGC. In exercising this authority, the LGU must comply with the conditions and
observe the limitations prescribed by Congress. The Sanggunian's failure to comply with Section 21 renders
ineffective its reclassification of the barrio road.

As a barrio road, the subject lot's purpose is to serve the benefit of the collective citizenry. It is outside the
commerce of man and as a consequence: (1) it is not alienable or disposable;(2) it is not subject to

40
registration under Presidential Decree No. 1529 and cannot be the subject of a Torrens title; (3) it is not
susceptible to prescription; (4) it cannot be leased, sold, or otherwise be the object of a contract; (5) it is not
subject to attachment and execution; and (6) it cannot be burdened by any voluntary easements.

41
OWNERSHIP
Concept of Ownership

RES JUDICATA WILL NOT APPLY BETWEEN AN ACTION FOR FORCIBLE ENTRY AND AN ACCION
REINVINDICATORIA

31. Javier v. Veridiano II


G.R. No. L-48050, October 10, 1994
Bellosillo, J.

FACTS:
In this petition for review on certiorari, petitioner Felicidad Javier questions the order of a regional trial court
citing the final decision of the city court previously dismissing her complaint for forcible entry agaist
respondent Reino Rosete, and on the basis thereof, dismissed her petition to quiet title on the ground of res
judicata.

Petitioner Javier filed a Miscellaneous Sales Application for Lot No. 1641 of the Olongapo Townsite
Subdivision, Lower Kalaklan, Olongapo City, with the District Land Officer, Bureau of Lands, Olongapo City.
Sometime in December 1970, she instituted a complaint for forcible entry docketed as Civil Case 926 before
the City Court of Olongapo City, alleging that she was forcibly dispossessed of a portion of the land by a
certain Ben Babol. The City Court of Olongapo Citydismissed the case on the ground that the Bureau of
Lands has considered the area in question to be outside Lot 1641 of the plaintiff. Subsequently, petitioner
Javier was granted Miscellaneous Sales Patent No. 5548 and issued Original Certificate of Title covering
Lot No. 1641. Meanwhile, Ben Babol who was the defendant and appellee in the complaint for forcible entry
had sold the property he was occupying, including the portion of about 200 square meters in question, to a
certain Reino Rosete. Thus Javier demanded the surrender of the same area in dispute from Reino Rosete
who repeatedly refused to comply with the demand. After about four (4) years from the finality of the
dismissal of civil case, petitioner instituted a complaint for quieting of title and recovery of possession with
damages against Ben Babol and Reino Rosete docketed as Civil Case No. 2203-0. .

Petitioner contends that res judicata cannot apply in the instant case since there is no identity of parties and
causes of action between her complaint for forcible entry, which had long become final and executory, and
her subsequent petition for quieting of title. With regard to the cause of action, she maintains that there is
no identity of causes of action since the first case was for forcible entry, which is merely concerned with the
possession of the property, whereas the subsequent case was for quieting of title, which looks into the
ownership of the disputed land.

Private respondent however submits that there is identity of parties in the two cases since he is a successor
in interest by title of the defendant in the first case after the commencement of the first action.

ISSUE:
Does res judicata apply between an action for forcible entry which has been final and executory, and an
action for quieting of title?

HELD:
No, res judicata will not apply. There is merit in petitioner's argument that there is no identity of causes of
action between Civil Case No. 926 and Civil Case No. 2203-0.

Civil Case No. 926 is a complaint for forcible entry, where what is at issue is prior possession, regardless
of who has lawful title over the disputed property. The only issue in an action for forcible entry is the physical
or material possession of real property, that is, possession de facto and not possession de jure. A judgment
rendered in a case for recovery of possession is conclusive only on the question of possession and not on
the ownership. It does not in any way bind the title or affect the ownership of the land or building.

On the other hand, Civil Case No. 2203-0 while inaccurately captioned as an action for "Quieting of Title
and Recovery of Possession with Damages" is in reality an action to recover a parcel of land or an accion
reivindicatoria under Art. 434 of the Civil Code, and should be distinguished from Civil Case No. 926, which
is an accion interdictal. Accion reivindicatoria or accion de reivindicacion is thus an action whereby plaintiff
alleges ownership over a parcel of land and seeks recovery of its full possession. It is different from accion
interdictal or accion publiciana where plaintiff merely alleges proof of a better right to possess without claim
of title.

42
In Civil Case No. 926 petitioner merely claimed a better right or prior possession over the disputed area
without asserting title thereto. It should be distinguished from Civil Case No. 2203-0 where she expressly
alleged ownership, specifically praying that she be declared the rightful owner and given possession of the
disputed portion. Hence, in Civil Case No. 926 petitioner merely alleged that she was "the true, lawful
(possessor) and in actual, prior physical possession" of the subject parcel of land, whereas in Civil Case
No. 2203-0 she asserted that she was "the absolute owner in fee simple" of the parcel of land "covered by
Original Transfer Certificate of Title No. P-3259." The complaint in Civil Case No. 2203-0 definitely raises
the question of ownership and clearly gives defendants therein notice of plaintiff's claim of exclusive and
absolute ownership, including the right to possess which is an elemental attribute of such ownership. Thus,
this Court has ruled that a judgment in forcible entry or detainer case disposes of no other issue than
possession and declares only who has the right of possession, but by no means constitutes a bar to an
action for determination of who has the right or title of ownership.

Consequently, there being no identity of causes of action between Civil Case No. 926 and Civil Case No.
2203-0, the prior complaint for ejectment cannot bar the subsequent action for recovery, or petition to quiet
title.

43
OWNERSHIP
Concept of Ownership

POSSESSION IS AN ESSENTIAL ATTRIBUTE OF OWNERSHIP

32. Spouses Bustos v. Court of Appeals


G.R. Nos. 120784-85, January 24, 2001
Pardo, J.

FACTS:
The case before the Court is an appeal via certiorari seeking to set aside the decision of the Court of
Appeals modifying that of the RTC the resolution denying reconsideration

Paulino Fajardo died intestate. He had four (4) children, namely: Manuela, Trinidad, Beatriz and Marcial.
The heirs executed an extra-judicial partition of the estate of Paulino Fajardo. Manuela sold her share to
Moses G. Mendoza, husband of Beatriz. Trinidad was in physical possession of the land. She refused to
surrender the land to her brother-in-law Moses G. Mendoza, despite several demands. Moses filed with the
Court of First Instance, Pampanga a complaint for partition claiming the one fourth share of Manuela which
was sold to him. During the pendency of the case for partition, Trinidad died. The heirs executed an extra-
judicial partition of the estate of Trinidad . Lucio Fajardo Ignacio, son of Trinidad sold Lot 284-B to spouses
Venancio Viray and Cecilia Nunga-Viray. The RTC rendered a decision in favor of Moses G. Mendoza.
Moses sold the subject land to spouses Warlito Bustos and Herminia Reyes-Bustos. In the meantime,
spouses Venancio Viray and Cecilia Nunga-Viray, buyers of Lucio Ignacios share of the property, filed with
the MTC an action for unlawful detainer against spouses Bustos, the buyers of Moses G. Mendoza, who
were in actual possession as lessees of the husband of Trinidad, Francisco Ignacio, of the subject land.
The MTC decided the case in favor of spouses Viray. Subsequently, the trial court issued writs of execution
and demolition, but were stayed when spouses Bustos filed with the RTC a petition for certiorari, prohibition
and injunction which was dismissed. Spouses Bustos appealed the decision to the Court of Appeals. The
CA dismissed the appeal and declared Mendoza as the true owner.

ISSUE:
Can petitioners could be ejected from what is now their own land?

HELD:
No, the petitioners cannot be ejected from their own land.

In this case, the issue of possession is intertwined with the issue of ownership. In the unlawful detainer
case, the Court of Appeals affirmed the decision of the trial court as to possession on the ground that the
decision has become final and executory. This means that the petitioners may be evicted. In the accion
reinvindicatoria, the Court of Appeals affirmed the ownership of petitioners over the subject land. Hence,
the court declared petitioners as the lawful owners of the land.

In the present case, the stay of execution is warranted by the fact that petitioners are now legal owners of
the land in question and are occupants thereof. To execute the judgment by ejecting petitioners from the
land that they owned would certainly result in grave injustice. Besides, the issue of possession was rendered
moot when the court adjudicated ownership to the spouses Bustos by virtue of a valid deed of sale.

Placing petitioners in possession of the land in question is the necessary and logical consequence of the
decision declaring them as the rightful owners of the property. One of the essential attributes of ownership
is possession. It follows that as owners of the subject property, petitioners are entitled to possession of the
same. An owner who cannot exercise the seven (7) juses or attributes of ownership--the right to possess,
to use and enjoy, to abuse or consume, to accessories, to dispose or alienate, to recover or vindicate and
to the fruits--is a crippled owner.

44
OWNERSHIP
Concept of Ownership

A PERSON MAY BE DECLARED OWNER BUT HE MAY NOT BE ENTITLED TO POSSESSION

33. Heirs of Soriano v. Court of Appeals


G.R. No. 128177, August 15, 2001
Ynares-Santiago, J.

FACTS:
A certain Adriano Soriano originally owned a piece of land located in Lingayen, Pangasinan. His heirs
subsequently leased the property to Spouses de Vera and Villasista for a period of 15 years, with Roman
Soriano, son of Adriano Soriano acting as caretaker of the property during the duration of the lease. Upon
the death of Adriano the lot was divided into two and given to his heirs. Spouses Abalos, private respondents
in this case, bought one whole lot and ¾ of the other lot. The lots in question were subsequently registered
in the name of Spouses Abalos. The latter was also declared by the court to be the undisputed owners
thereof. Aggrieved, petitioner filed case against Spouses Abalos, questioning their ownership of the land
and to annul the decision of ejectment in the subject property. While the case was pending, Roman Soriano
is still in possession of the land claiming the rights of “Security of Tenure” as a tenant of the land.

ISSUE:
May a winning party in a land registration case effectively eject the possessor thereof?

HELD:
No, a party who won in a land registration case cannot effectively eject the possessor.

Possession and ownership are distinct legal concepts. Possession is the holding of a thing or the enjoyment
of a right. Literally, to possess means to actually and physically occupy a thing with or without right. A
judgment of ownership does not necessarily include possession as a necessary incident. Such declaration
pertains only to OWNERSHIP and does not automatically include possession. This is especially true in the
case at bar wherein petitioner is occupying the land allegedly in the concept of an agricultural tenant. The
court says “allegedly” due to the fact that there is still a pending case in the DARAB (Department of Agrarian
Reform and Adjudication Board) on the issue. The issue of ownership of the subject land has been laid to
rest by final judgment; however the right of possession is yet to be resolved. The Tenancy Act, which
protects the rights of agricultural tenants, may limit the exercise of rights by the lawful owners. The exercise
of the rights of ownership yields to the exercise of the rights of an agricultural tenant. Since the rights of
Soriano to possess the land are still pending litigation in the DARAB he is protected from dispossession of
the land until final judgment of said court unless Soriano’s occupancy is found by the court to be unlawful.

45
OWNERSHIP
Concept of Ownership

MERE TOLERANCE OF THE OWNER DOES NOT ALLOW THE POSSESSION TO BE IN THE
CONCEPT OF AN OWNER

34. Garcia v. Court of Appeals


G.R. No. 133140, August 10, 1999
Puno, J.

FACTS:
Spouses Magpayos mortgaged a parcel of land located at Makati City to PBCom to secure a loan. A Deed
of Real Estate Mortgage was registered and annotated on the Magpayos TCT. When the said spouses
failed to fulfill their obligation upon its maturity, the mortgage was extrajudicially foreclosed and at the public
auction sale, PBCom was the highest bidder bought the land. PBCom filed a petition for issuance of a writ
of possession, which was granted by the court.

During the time of the controversy, petitioner, who was Mrs. Magpayo’s brother was in possession of the
land. Upon service of the writ of possession to him, he refused to honor it and filed a motion for intervention
in PBCom’s petition, which motion was denied.

Petitioner filed a Motion for Summary Judgment, which the lower court granted. It held that the mortgage
executed by the Spouses Magpayos was void. It ruled that that the time of the execution of the Mortgage,
the spouses were not yet the owners of the property. This is evident from the fact that new Torrens title was
issued to the Spouses only after the execution of the mortgage. The vendor of the land, Atty. Pedro V.
Garcia, father of both parties and whose name was stated in the old Transfer Certificate Title, was not in
possession of the land at the time of the execution of the mortgage and hence could not deliver the property
merely by the execution of the document. At the time, the property was in the possession of the plaintiff.

Upon appeal of PBCom, the CA revered the lower court’s ruling. The CA opined that it appearing the Vendor
Atty. Garcia had control of the property which was registered in his name and that the deed of sale was
likewise registered, the sale was consummated and the Magpayos were free to exercise the attributes of
ownership including the right to mortgage the land. Even if the Magpayos’ title to the land was issued four
days following the execution of the deed of real estate mortgage is of no moment, for registration under the
Torrens system does no vest ownership but is intended only to confirm and register the title which one may
already have on the land.

Petitioner filed a motion of reconsideration, but it was denied. Hence this petition.

ISSUE:
Whether or not the TCT issued after the execution of the mortgage contract is void.

HELD:
No, the Mortgage to PBCom by the Spouses Magpayo is valid notwithstanding that the transfer certificate
of title over the property was issued to them after the mortgage contract was entered into.

Registration of the property does not confer ownership. It is merely evidence of such ownership over a
particular property. The deed of sale operates as a formal or symbolic delivery of the property sold and
authorizes the buyer to use the document as proof of ownership.

46
OWNERSHIP
Concept of Ownership

DISPOSITION OF HIS PROPERTY BY THE OWNER IS AN ATTRIBUTE OF OWNERSHIP

35. Rodil Enterprises v. Court of Appeals


G.R. Nos. 129609 & 135537, November 29, 2001
Bellosillo, J.

FACTS:
The Republic of the Philippines (Republic) leased the O’Racca Building to petitioner Rodil. Republic
acquired ownership of the O’Racca Building, a former alien property, by virtue of RA 477. Petitioner then
sublease the Building to private respondents known as the Association. The lease contract between Rodil
and the Republic was renewed for another 15 years.

Subsequently, a new law was enacted known as BP 233, which authorized the sale of “former alien
properties” classified as commercial and industrial, and the O’RACCA building was classified as commercial
property. Both Rodil and the Association wanted to purchase the property and so they both offered their
price, the former to the Republic, and the latter, to DGSREPM. Despite the offer to purchase the property,
Republic, through BSRPMO, granted Rodil’s request for another renewal of the lease contract. However,
DGSREPM recommended the suspension of the approval, and as a result a memorandum was issued
disapproving the renewal contract in favor of Rodil.

Rodil filed an action for specific performance, damages and injunction with prayer for TRO against
respondents, particularly for the Association to enjoin in collecting rentals from the occupants or sub-lessees
of the O’RACCA building. However, both Rodil and the Republic, through OSG, filed a joint motion to dismiss
the specific performance, which was granted by the lower court. The order of dismissal was appealed by
the Association to the CA.

The lower court granted the actions of Rodil. However, the Court of Appeals set aside the decisions of the
lower court, specifically the order of dismissal, hence this petition.

Rodil Argues that the Republic, the only defendant who is a real party in interest, signified its assent to
having the action dismissed. Assuming arguendo that the Association was a real party in interest, its
counterclaim was nonetheless unmeritorious.

On the other hand, the Association argues that it’s counterclaim in filed against Rodil cannot be dismissed
because the trial court has not passed upon it.

ISSUE:
Can the Republic, the real owner of the property, assent to the dismissal of the action for specific
performance.

HELD:
Yes, because the Republic, as the owner of the O’Racca Building, merely exercised his right of ownership.

The owner has the right to enjoy and dispose of a thing, without other limitations than those established by
law. Every owner has the freedom of disposition over his property. It is an attribute of ownership, and this
rule has no exception. The Republic being the owner of the disputed property enjoys the prerogative to
enter into a lease contract with Rodil in the exercise of its jus disponendi. Hence, as lessor the Republic has
the right to eject usurpers of the leased property where the factual elements required for relief in an action
for unlawful detainer are present.

47
OWNERSHIP
Concept of Ownership

A MORTGAGEE IS NOT REQUIRED TO BE IN ACTUAL POSSESSION OF THE MORTGAGED


PROPERTY

36. Isaguirre v. De Lara


G.R. No. 138053, May 31, 2000
Gonzaga-Reyes, J.

FACTS:
Respondent approached petitioner, who was married to her, to help out in her financial difficulties. They
agreed to execute a document denominated as a “Deed of Salle and Special Cession of Rights and
Interests” (Deed of Sale), whereby respondent sold a 250 square meter portion of Lot No. 502, together
with the two-story commercial and residential structure standing thereon, in favor of petitioner, for and in
consideration of the sum of Php 5,000.

Petitioner filed a sales application over the subject property on the basis of the Deed of Sale. His application
was granted, resulting in the issuance of an OCT in favor of his name. Meanwhile, respondent’s sales
application over the entire 1,000 sq m. of the subject property (including the 250 sq. m.) was also granted,
resulting in the issuance of an OCT in respondent’s favor.

To avoid the overlapping of titles, petitioner filed an action for quieting of title and damages. The lower court
ruled in favor of petitioner. But when appealed to the CA, it reversed the lower court’s decision. It ruled that
the contract was not a sale, but an equitable mortgage. Respondent filed a motion for execution, praying
for the immediate delivery of possession of the subject property, which motion was granted by the lower
court. Then a writ of possession was filed by respondent, which was again granted by court.

Petitioner filed a motion for reconsideration over the decision, asserting that he had the right of retention
over the property until the payment of the loan and the value of the improvements he had introduced on the
property.

ISSUE:
Is a mortgagee is entitled to retain possession of the Property so long as the mortgage loan remains unpaid.

HELD:
No, a mortgagee has no better right to possess the property over the owner.

As a general rule, the mortgagor retains possession of the mortgaged property since a mortgage is merely
a lien and title to the property does not pass to the mortgagee. However, even though a mortgagee does
not have possession of the property, there is no impairment of his security since the mortgage directly and
immediately subjects the property upon which it is imposed, whoever the possessor may be, to the fulfillment
of the obligation for whose security it was constituted.

The trial court correctly issued the writ of possession. Such writ was but a necessary consequence of the
decision affirming the validity of the OCT in the name of respondent, while at the same time nullifying the
OCT in the name of petitioner is an essential attribute of ownership.

48
OWNERSHIP
Concept of Ownership

RES JUDICATA WILL NOT APPLY BECAUSE AN EJECTMENT CASE INVOLVES A DIFFERENT
CAUSE OF ACTION FROM AN ACCION PUBLICIANA OR ACCION REINVINDICATORIA

37. Custodio v. Corrado


G.R. No. 146082, July 30, 2004
Quisumbing, J.

FACTS:
This is a review on certiorari of the Decision of the Court of Appeals (CA) and its Resolution denying the
motion for reconsideration. The CA affirmed the decision of the Regional Trial Court (RTC) reversing the
decision of the Municipal Trial Court (MTC) dismissing respondent Rosendo F. Corrado’s Complaint for
Recovery of Possession and Ownership with Injunction and Damages, in Civil Case No. 120.

Respondent Corrado filed an ejectment case against petitioner Custodio with the MTC docketed as Civil
Case No. 116. This case was dismissed and affirmed by the RTC upon appeal. Two years later, respondent
filed with the same MTC another complaint for recovery of possession and damages against petitioner,
docketed as Civil Case No. 120, and which is the core case subject of the present petition.

The MTC rendered judgment dismissing the complaint. Respondent appealed the MTC decision to the RTC,
which set aside and reversed the MTC decision. On appeal, the CA ruled that the principle of res judicata is
inapplicable because there is no identity of causes of action between Civil Case Nos. 116 and 120. It
stressed that the former is an ejectment suit which was dismissed for failure of respondent to state the date
of deprivation of possession while the latter is for recovery of possession, and not ejectment.

ISSUE:
Is the principle of res judicata is applicable in this case?

HELD:
No. The principle of res judicata is inapplicable because Civil Case No. 116 for ejectment was not decided
on the merits and its cause of action is different from Civil Case No. 120 for recovery of possession and
ownership. In Civil Case No. 116, the case as found by the MTC is an ejectment suit and for failure of
plaintiff-private respondent to state the date when he was deprived of his possession, the court held that it
did not entitle him to file an ejectment suit against herein defendant-petitioner. In Civil Case No. 120, the
cause of action is for recovery of possession and not ejectment. These are two separate causes of action
and therefore the principle of res judicata does not apply to the present case.

Indeed, an ejectment case such as Civil Case No. 116, involves a different cause of action from an accion
publiciana or accion reinvindicatoria, such as Civil Case No. 120, and the judgment of the former shall not
bar the filing of another case for recovery of possession as an element of ownership. A judgment in a forcible
entry or detainer case disposes of no other issue than possession and establishes only who has the right
of possession, but by no means constitutes a bar to an action for determination of who has the right or title
of ownership. Incidentally, we agree with the findings of the RTC that Civil Case No. 120 is not an accion
publiciana but more of an accion reinvindicatoria as shown by the respondents allegation in the complaint
that he is the registered owner of the subject lot and that the petitioner had constructed a bungalow thereon
and had been continuously occupying the same since then.

49
OWNERSHIP
Concept of Ownership

AN ACTION FOR RECONVEYANCE OF A PROPERTY IS THE SOLE REMEDY OF A LANDOWNER


WHOSE PROPERTY HAS BEEN WRONGFULLY OR ERRONEOUSLY REGISTERED IN ANOTHER'S
NAME AFTER ONE YEAR FROM THE DATE OF THE DECREE SO LONG AS THE PROPERTY HAS
NOT PASSED TO AN INNOCENT PURCHASER FOR VALUE

38. Abejaron v. Nabasa


G.R. No. 84831, June 20, 2001
Puno, J.

FACTS:
Petitioner Abejaron filed this petition for review on certiorari to annul the respondent court's Decision and
Resolution reversing the trial court's decision and declaring respondent Nabasa the owner of the subject
lot.

Petitioner Abejaron avers that he is the actual and lawful possessor and claimant of a 118-square meter
portion of a 175-square meter residential lot in Silway, General Santos City. In 1945, petitioner Abejaron
and his family started occupying the 118-square meter land. At that time, the land had not yet been
surveyed. They fenced the area and built thereon a family home. Knowing that the disputed land was public
in character, petitioner declared only his house, and not the disputed land, for taxation purposes.

Later on, the Bureau of Lands surveyed the area in controversy. Abejaron merely watched them do the
survey and did not thereafter apply for title of the land on the belief that he could not secure title over it as it
was government property. Without his knowledge and consent, Nabasa applied for and caused the titling in
his name the entire Lot 1, including petitioner Abejaron's 118-square meter portion. Nabasa was issued
Original Certificate of Title (OCT). As the title included petitioner Abejaron’s 118-square meter portion of the
lot, he filed a protest with the Bureau of Lands against Nabasa's title and application. The case was
eventually dismissed.

Petitioner Abejaron filed against respondent Nabasa an action for reconveyance with damages seeking
reconveyance of his 118-square meter portion of Lot 1, Block 5, Psu-154953. After trial on the merits, the
RTC ruled in favor of petitioner in its reconveyance case declaring the possession and occupancy of
Abejaron over 118 square meters of lot in good faith and thereby declaring the inclusion of said portion in
the OCT issued in the name of Nabasa erroneous.

Upon appeal to the CA, the Court reversed the decision of the RTC stating that the only basis for
reconveyance is actual fraud. It ruled that Abejaron failed to substantiate the existence of actual fraud. There
was no proof of irregularity neither in the issuance of title nor in the proceedings incident thereto nor was
there a claim that fraud intervened in the issuance of the title, thus, the title has become indefeasible.

ISSUE:
Has the petitioner acquired title over the land in dispute?

HELD:
No. An action for reconveyance of a property is the sole remedy of a landowner whose property has been
wrongfully or erroneously registered in another's name after one year from the date of the decree so long
as the property has not passed to an innocent purchaser for value. The action does not seek to reopen the
registration proceeding and set aside the decree of registration but only purports to show that the person
who secured the registration of the property in controversy is not the real owner thereof. Fraud is a ground
for reconveyance. For an action for reconveyance based on fraud to prosper, it is essential for the party
seeking reconveyance to prove by clear and convincing evidence his title to the property and the fact of
fraud.
As admitted by the petitioner, he has never declared the disputed land for taxation purposes. While tax
receipts and tax declarations are not incontrovertible evidence of ownership, they become strong evidence
of ownership acquired by prescription when accompanied by proof of actual possession of the property or
supported by other effective proof. Even the tax declarations and receipts covering his house do not bolster
his case as the earliest of these was dated 1950.

50
Petitioner's evidence does not constitute the "well-nigh incontrovertible" evidence necessary to acquire title
through possession and occupation of the disputed land at least since January 24, 1947 as required by Sec.
48(b) of the Public Land Act, as amended by R.A. 1942. The basic presumption is that lands of whatever
classification belong to the State and evidence of a land grant must be "well-nigh incontrovertible." As
petitioner Abejaron has not adduced any evidence of title to the land in controversy, whether by judicial
confirmation of title, or homestead, sale, or free patent, he cannot maintain an action for reconveyance.

51
OWNERSHIP
Concept of Ownership

AS LONG AS THE ALLEGATIONS DEMONSTRATE A CAUSE OF ACTION FOR FORCIBLE ENTRY,


THE FIRST LEVEL COURT ACQUIRES JURISDICTION OVER THE SUBJECT MATTER

39. Javier v. Lumontad


G.R. No. 203760, December 3, 2014
Perlas-Bernabe, J.

FACTS:
Assailed in this petition for review on certiorari are the Decision and the Resolution of the Court of Appeals
which set aside the Regional Trial Court’s decision finding that the action instituted by petitioner was not
one for forcible entry, but for recovery of ownership and possession, hence, within the original jurisdiction
of the latter. The CA remanded the case to the RTC for trial on the merits.

Petitioner Javier filed an action for forcible entry against respondent Lumontad. Petitioner alleged in his
complaint that his father Vicente was the owner of a 360- square meter parcel of land and petitioner’s family
lived in a residential house erected on this land. Upon his father’s death, petitioner, together with his mother,
continued their possession over the same. The respondent gained entry into the subject land and started to
build a two (2)-storey building (subject building) on a 150 sq. m. portion thereof, despite petitioner’s vigorous
objections and protests.

The MTC dismissed the complaint. Upon appeal to the RTC, the Court found that petitioner, being the owner
and possessor of the property in question, has the right to be respected in his possession and that
respondent forcibly and unlawfully deprived him of the same. Dissatisfied, an appeal to the CA was made
which ruled that the issue of possession of the subject land is intimately intertwined with the issue of
ownership, such that the former issue cannot be determined without ruling on who really owns such land.
Thus, it remanded the case to the RTC for trial on the merits in the exercise of the latter’s original jurisdiction
in an action for recovery of ownership and possession.

ISSUE:
Does the CA correctly set aside the RTC Ruling and ordered the remand of the case to the latter court for
trial on the merits in an action for recovery of ownership and possession?

HELD:
No. In forcible entry, the complaint must necessarily allege that one in physical possession of a land or
building has been deprived of that possession by another through force, intimidation, threat, strategy or
stealth. The plaintiff must allege that he, prior to the defendant’s act of dispossession by force, intimidation,
threat, strategy or stealth, had been in prior physical possession of the property. This requirement
is jurisdictional, and as long as the allegations demonstrate a cause of action for forcible entry, the court
acquires jurisdiction over the subject matter.

A plain reading of petitioner’s complaint shows that the required jurisdictional averments, so as to
demonstrate a cause of action for forcible entry, have all been complied with. Said pleading alleges that
petitioner, as the original owner’s, i.e., Vicente’s, successor-in-interest, was in prior physical possession of
the subject land but was eventually dispossessed of a 150 sq. m. portion thereof on March 26, 2007 by
respondent who, through force and intimidation, gained entry into the same and, thereafter, erected a
building thereon. Clearly, with these details, the means by which petitioner’s dispossession was effected
cannot be said to have been insufficiently alleged as mistakenly ruled by the MTC and later affirmed by the
CA. The “how” (through unlawful entry and the construction of the subject building), “when” (March 26,
2007), and “where” (a 150 sq. m. portion of the subject land) of the dispossession all appear on the face of
the complaint.

Verily, ejectment cases fall within the original and exclusive jurisdiction of the first level courts by express
provision of Section 33 (2) of Batas Pambansa Blg. 129,in relation to Section 1, Rule 70, of the Rules of
Court. Even in cases where the issue of possession is closely intertwined with the issue of ownership, the
first level courts maintain exclusive and original jurisdiction over ejectment cases, as they are given the
authority to make an initial determination of ownership for the purpose of settling the issue of possession. It
must be clarified, however, that such adjudication is merely provisional and would not bar or prejudice an

52
action between the same parties involving title to the property. It is, therefore, not conclusive as to the issue
of ownership.

53
OWNERSHIP
Concept of Ownership

THERE IS NO FORUM-SHOPPING FOR FAILURE TO DISCLOSE IN THE CERTIFICATION ON NON-


FORUM SHOPPING OF THE UNLAWFUL DETAINER CASE, A COMPLETE STATEMENT OF THE
STATUS OF THE PENDING ACTION FOR RECOVERY OF OWNERSHIP OF PROPERTY

40. Bradford United Church of Christ, Inc. v. Ando


G.R. No. 195669, May 30, 2016
Del Castillo, J.

FACTS:
Assailed in this Petition for Review on Certiorari are the Decision of the Court of Appeals (CA) which
dismissed the Petition in CA-GR. SP No. 01935 and its Resolution which denied petitioner's Motion for
Reconsideration.

Petitioner Bradford United Church of Christ, Inc. (BUCCI) filed a Complaint for unlawful detainer and
damages against herein respondents Dante Ando et al. in their capacities as Members of the Mandaue
Bradford Church Council, the Mandaue Bradford Church (MBC), and the United Church of Christ in the
Philippines, Inc. (UCCPI).

The MTCC ruled that BUCCI failed to mention in its certification against non-forum-shopping a complete
statement of the present status of another case concerning the recovery of ownership of certain parcels of
land earlier filed before the Regional Trial Court (RTC) by the UCCPI and the MBC against BUCCI.

The recovery of ownership case also involved Lot 3-F, the same parcel of land subject of the unlawful
detainer case, and yet another parcel of land, denominated simply as Lot 3-C. The RTC of Mandaue City-
rendered its judgment in the recovery of ownership case against therein plaintiffs UCCPI and MBC and in
favor of therein defendant BUCCI.

The case before the MTCC was eventually dismissed on failure to comply with the rule on certification
against forum shopping. The RTC and CA affirmed the MTCC’s decision.

ISSUE:
Is BUCCI guilty of forum-shopping?

HELD:
No, BUCCI is not guilty of forum-shopping when it failed to disclose in the certification on non-forum
shopping of the unlawful detainer case a complete statement of the status of the action for recovery of
ownership of property then pending before RTC Mandaue City. Here, there is only identity of parties
between the summary action of unlawful detainer and the land ownership recovery case. However, the
issues raised are not identical or similar in the two cases. The issue in the unlawful detainer case is which
party is entitled to, or should be awarded, the material or physical possession of the disputed parcel of land,
(or possession thereof as a fact); whereas the issue in the action for recovery of ownership is which party
has the right to be recognized as lawful owner of the disputed parcels of land.

The CA thus erred in holding that, "[a]n adjudication in respondents' recovery of ownership case would
constitute an adjudication of petitioner BUCCI's unlawful detainer case, such that the court handling the
latter case would be bound thereby and could not render a contrary ruling in the issue of physical or material
possession."

What really distinguishes an action for unlawful detainer from a possessory action (accion publiciana) and
from a reinvindicatory action (accion reinvindicatoria) is that the first is limited to the question of possession
de facto. An unlawful detainer suit (accion interdictal) together with forcible entry are the two forms of an
ejectment suit that may be filed to recover possession of real property. Aside from the summary action of
ejectment, accion publiciana or the plenary action to recover the right of possession and accion
reinvindicatoria or the action to recover ownership which includes recovery of possession, make up the
three kinds of actions to judicially recover possession.

54
OWNERSHIP
Concept of Ownership

DOCTRINE OF SELF-HELP CAN ONLY BE EXERCISED AT THE TIME OF ACTUAL OR THREATENED


DISPOSSESSION. THUS, WHEN POSSESSION HAS ALREADY BEEN LOST, THE OWNER MUST
RESORT TO JUDICIAL PROCESS FOR THE RECOVERY OF PROPERTY

41. German Management and Services Inc. v. Court of Appeals


GR Nos. 76217 & L-76216, September 14, 1989
Fernan, C.J.

FACTS:
This is a petition for review under Rule 65 seeking the reversal of the decision of Court of Appeals stating
that private respondents in this case have a right to commence an action for forcible entry regardless of the
legality or illegality of possession of the subject real property.

Spouses Jose are the owners of a parcel of land situated in Sitio Inarawan, Antipolo, Rizal, with an area of
232,942 square meters and covered by TCT No. 50023. On February 26, 1982, the spouses Jose executed
a special power of attorney authorizing petitioner German Management Services to develop their property
covered by TCT No. 50023 into a residential subdivision.

Finding that part of the property was occupied by private respondents and twenty other persons, petitioner
advised the occupants to vacate the premises but the latter refused. Nevertheless, petitioner proceeded
with the development of the subject property which included the portions occupied and cultivated by private
respondents. It deprived private respondents of their property without due process of law by: (1) forcibly
removing and destroying the barbed wire fence enclosing their farmholdings without notice; (2) bulldozing
the rice, corn fruit bearing trees and other crops of private respondents; and (3) trespassing, coercing and
threatening to harass private respondents from their respective farmholdings.

Private respondents filed an action for forcible entry against the petitioner. The MTC and RTC dismissed
private respondents' complaint for forcible entry. On appeal, the Appellate Court gave due course to their
petition and held that since private respondents were in actual possession of the property at the time they
were forcibly ejected by petitioner, private respondents have a right to commence an action for forcible entry
regardless of the legality or illegality of possession.

ISSUE:
Are the private respondents entitled to file a forcible entry case against petitioner?

HELD:
Yes. Notwithstanding petitioner's claim that it was duly authorized by the owners to develop the subject
property, private respondents, as actual possessors, can commence a forcible entry case against petitioner
because ownership is not in issue. Forcible entry is merely a quieting process and never determines the
actual title to an estate. Title is not involved.

In the case at bar, it is undisputed that at the time petitioner entered the property, private respondents were
already in possession thereof. There is no evidence that the spouses Jose were ever in possession of the
subject property. On the contrary, private respondents' peaceable possession was manifested.

Both the Municipal Trial Court and the Regional Trial Court have rationalized petitioner's drastic action of
bulldozing and destroying the crops of private respondents on the basis of the doctrine of self-help
enunciated in Article 429 of the New Civil Code. Such justification is unavailing because the doctrine of self-
help can only be exercised at the time of actual or threatened dispossession which is absent in the case at
bar. When possession has already been lost, the owner must resort to judicial process for the recovery of
property. This is clear from Article 536 of the Civil Code which states, "(I)n no case may possession be
acquired through force or intimidation as long as there is a possessor who objects thereto. He who believes
that he has an action or right to deprive another of the holding of a thing, must invoke the aid of the
competent court, if the holder should refuse to deliver the thing."

55
OWNERSHIP
Concept of Ownership

THE DOCTRINE OF SELF-HELP CANNOT BE INVOKED AGAINST A LAWFUL CO-POSSESSOR OF


THE LAND

42. Caisip v. People


GR No. L-28716, November 18, 1970
Concepcion, C.J.

FACTS:
This is a petition for review on certiorari of a decision of the Court of Appeals which affirmed that of the
Court of First Instance of Batangas, convicting Petitioners Felix Caisip et. al of the crime of Grave Coercion.

Spouses Gloria Cabalag and Marcelino Guevarra cultivated a parcel of land known as Lot 105-A of
Hacienda Palico situated in Sitio Bote-bote, Barrio Tampisao, Nasugbu, Batangas. The overseer of the
hacienda is petitioner Felix Caisip and the owner of the same is Roxas y Cia. The latter acquired a court
ruling against the spouses Gloria and Marcelino for forcible entry which orders them to vacate the premises
within 20 days. The order was carried out June 6, 1959, giving them until June 26 to vacate it.

On June 17, Gloria was seen by Felix Caisip weeding the portion of Lot 105-A, which is a ricefield. The
latter bade her to stop what she was doing and to leave the premises. When Gloria refused, Caisip called
for Sgt. Rjales and Cpl. Villadelrey to help him shoo her away. Gloria stuck to her attitude and still refused
to stop and leave so the two police officers, by means of force, stopped her and dragged her away. As a
result, the clothes of Gloria got torn. One of Gloria’s neighbours caught sight of the event and asked the
officers to release her. Gloria was later turned over to the police on duty for interrogation.

A case filed against the petitioners, Caisip and the officers, for Grave Coercion. One of their defenses was
Article 429 of the Civil Code based on the Doctrine of Self-Help.

ISSUE:
Can petitioners Caisip et. al validly invoke Article 429?

HELD:
No. Art. 429 of the Civil Code which provides that “The owner or lawful possessor of a thing has the right to
exclude any person from the enjoyment and disposal thereof. For this purpose, he may use such force as
may be reasonably necessary to repel or prevent an actual or threatened unlawful physical invasion or
usurpation of his property." cannot be used as a defense of the petitioner to justify their action.

Having been given 20 days from June 6, 1959, within which to vacate Lot 105-A, Gloria did not, on June
17, 1959 — or within said period — invade or usurp said lot. She had merely remained in possession thereof,
even though the hacienda owner may have become its co-possessor. Appellants did not "repel or prevent
in actual or threatened ... physical invasion or usurpation." They expelled Gloria from a property of which
she and her husband were in possession even before the action for forcible entry was filed against them on
May 17, 1958, despite the fact that the Sheriff had explicitly authorized them to stay in said property up to
June 26, 1959, and had expressed the view that he could not oust them therefrom On June 17, 1959,
without a judicial order therefor.

It is, accordingly, clear that appellants herein had, by means of violence, and without legal authority therefor,
prevented the complainant from "doing something not prohibited by law," (weeding and being in Lot 105-A),
and compelled her "to do something against" her will (stopping the weeding and leaving said lot), "whether
it be right or wrong ," thereby taking the law into their hands, in violation of Art. 286 of the Revised Penal
Code.

56
OWNERSHIP
Concept of Ownership

THE PRINCIPLE OF SELF-HELP AUTHORIZES THE LAWFUL POSSESSOR TO USE FORCE, NOT
ONLY TO PREVENT A THREATHENED UNLAWFUL INVASION OR USURPATION THEREOF; IT IS
SORT OF SELF-DEFENSE

43. People v. Pletcha, Jr.


No. 19029-CR, June 27, 1977
Presbitero, J.

FACTS:
Tito Pletcha Jr., a farmer, owns a land which has been cultivating for 19 years. In the morning of June 10,
1973, eight men were seen by Pletcha constructing fence and wires on a piece of land in Hacienda Gaspit.
It has been alleged that a private corporation sought to take over the aforementioned land. Claiming actual
possession and ownership, Pletcha asked the group to desist from fencing pending the resurvey of the land.
However, Pletcha was ignored by the eight men.

Pletcha thereafter came back with a bolo, without any chance, however, to actually use it or threaten them,
because as soon as the group saw him they ran away. This prompted the eight men to file a complaint for
grave coercion.

Tito Pletcha invoked the doctrine of self-help under Article 429 of the Civil Code, giving him the right to use
reasonable force to exclude any person threatening his peaceful ownership.

ISSUE:
Can Pletcha Jr. validly invokes self-help, hence, shall not be convicted of Grace Coercion.

HELD:
Yes. The principle of self-help authorizes the lawful possessor to use force not only to prevent a threatened
unlawful invasion or usurpation thereof; it is sort of self-defense. The use of such necessary force to protect
proprietary or possessory rights constitutes a justifying circumstance under our penal laws. The appellant
need not rush to court to seek redress before reasonably resisting the invasion of property. The situation
required immediate action and Article 429 gave him the self-executory mechanics of self-defense and self-
reliance.

Article 429 confirms the right of the appellant, an owner and lawful possessor, to use reasonable force to
repel an invasion, usurpation, actual, threatened or physical, of his property.

57
OWNERSHIP
Concept of Ownership

BEING LEGITIMATE POSSESSORS OF THE LAND AND HAVING EXERCISED LAWFUL MEANS TO
PROTECT THEIR POSSESSION, RESPONDENTS WERE NOT GUILTY OF UNLAWFUL OCCUPATION

44. Diamond Farms, Inc. v. Diamond Farm Workers Multi-Purpose Cooperative


G.R. No. 19299, July 18, 2012
Villarama, Jr., J.

FACTS:
Petitioner is a corporation engaged in commercial farming of bananas. It owned1,023.8574 hectares of land
in Carmen, Davao. A big portion of this land measuring 958 hectares was initially deferred for acquisition
and distribution under the Comprehensive Agrarian Reform Program (CARP).

Thereafter, 698.8897 hectares of the 958-hectare land were awarded to members of the Diamond Agrarian
Reform Beneficiaries Multi-Purpose Cooperative (DARBMUPCO). Petitioner, however, maintained
management and control of 277.44 hectares of land, including a portion measuring 109.625 hectares (109-
hectare land). The 109 hectares was subsequently awarded to 278 CARP beneficiaries and the TCT of
petitioner Diamond Farms were cancelled.

In 2002, the petitioner filed unlawful occupation alleging that on November 1995, it was holder of TCT
covering the 109 hectares of land and grew export quality banana. Petitioner alleges that DAR’s order
distributing land was not yet final on account of appeals. Petitioner insists that prior to its receipt of the
corresponding payment for the land from the government or deposit in its favor of the compensation for the
land in cash or in LBP bonds, respondents cannot be deemed lawful possessors of the subject land and the
valuable improvements thereon, citing Section 16 (e) of the CARL. According to petitioner, "it has yet to
receive any compensation for the lands acquired by the government."

Respondent farmers refused to do their work and forcibly occupied 74-hectares which prohibited petitioner
from harvesting its produce. As a result, petitioner installed workers on a CARP-covered land when the DAR
has already identified the CARP beneficiaries of the land and has already ordered the distribution of the
land to them serves no other purpose than to create an impermissible roadblock to installing the legitimate
beneficiaries on the land.

ISSUE:
Are the actions taken by the respondents lawful and within the contemplation of Article 429?

HELD:
Yes. Actions taken by respondents to guard the land are reasonable and necessary to protect their
legitimate possession and prevent precisely what petitioner attempted to do. Such course was justified
under Article 429 of the Civil Code which reads “the owner or lawful possessor of a thing has the right to
exclude any person from the enjoyment and disposal thereof. For this purpose, he may use such force as
may be reasonably necessary to repel or prevent an actual or threatened unlawful physical invasion or
usurpation of his property.”

Being legitimate possessors of the land and having exercised lawful means to protect their possession,
respondents were not guilty of unlawful occupation.

58
OWNERSHIP
Concept of Ownership

THE LAWFUL OWNER OF A PROPERTY HAVE THE RIGHT, UNDER ARTICLE 429 OF THE CIVIL
CODE, TO EXCLUDE ANY PERSON FROM ITS ENJOYMENT AND DISPOSAL

45. Spouses Fuentes v. Roca


G.R. No. 178902, April 21, 2010
Abad, J.

FACTS:
Spouses Rosario (wife) and Tarciano (husband) had been living separately for 30 years since 1958. In 1982
and during the marriage, Tarciano bought a piece of land from his parents. Tarciano decided to sell the land
to the Spouses Fuentes on 1989. Atty. Plagata prepared the documents of sale. The agreement required
the Spouses Fuentes to pay a downpayment and the balance after six months on the condition that Tarciano
clear the lot of the structures and occupants and secure the consent of his estranged wife, Rosario, to the
sale. The parties relied on Atty. Plagata’s word that he went to see Rosario and had her sign the affidavit of
consent. Upon Tarciano's compliance with the other conditions, a new title was transferred to the Fuentes
spouses. Tarciano passed away followed by his wife, who died nine months afterwards.

Eight years later on in 1997, the children of Tarciano and Rosario, respondents Rocas filed an action for
annulment of sale and reconveyance of the land against the Fuentes spouses before the Regional Trial
Court. The Rocas claimed that the sale to the spouses was void since Tarciano's wife, Rosario, did not give
her consent to it. Her signature on the affidavit of consent had been forged. They thus prayed that the
property be reconveyed to them upon reimbursement of the price that the Fuentes spouses paid Tarciano.
The Fuentes spouses denied the Rocas' allegations. They pointed out that the claim of forgery was personal
to Rosario and she alone could invoke it. The Fuentes also counter that the four-year prescriptive period for
nullifying the sale on ground of fraud had already lapsed.

ISSUE:
May the heirs of Rosario (respondent Rocas) bring an action to annul the sale of conjugal property without
her consent after her death?

HELD:

Yes. The sale was void from the beginning. Article 124 of the Family Code provides that without the other
spouse's written consent or a court order allowing the sale on a conjugal property, the same would be void.
Here, the Rocas filed an action against the Fuentes spouses in 1997 for annulment of sale and
reconveyance of the real property that Tarciano sold without their mother's (his wife's) written consent. The
passage of time did not erode the right to bring such an action.

Consequently, the land remained the property of Tarciano and Rosario despite that sale. When the two
died, they passed on the ownership of the property to their heirs, namely, the respondent Rocas. As lawful
owners, the Rocas had the right, under Article 429 of the Civil Code, to exclude any person from its
enjoyment and disposal.

However, the Spouses Fuentes should be entitled, among other things, to recover from Tarciano's heirs,
the Rocas, the P200,000.00 that they paid him, with legal interest until fully paid, chargeable against his
estate. The Fuentes spouses appear to have acted in good faith in entering the land and building
improvements on it. Atty. Plagata, whom the parties mutually entrusted with closing and documenting the
transaction, represented that he got Rosario's signature on the affidavit of consent. The Fuentes spouses
had no reason to believe that the lawyer had violated his commission and his oath. Further, the notarized
document appears to have comforted the Fuentes spouses that everything was already in order when
Tarciano executed a deed of absolute sale in their favor. In fact, they paid the balance due him. The Fuentes
spouses are deemed as possessors in good faith. According to Article 526 of the Civil Code, he is deemed
a possessor in good faith if he is not aware that there exists in his title or mode of acquisition any flaw which
invalidates it.

As possessor in good faith, the Fuentes spouses were under no obligation to pay for their stay on the
property prior to its legal interruption by a final judgment against them. What is more, they are entitled under
Article 448 to indemnity for the improvements they introduced into the property with a right of retention until

59
the reimbursement is made. The Rocas shall of course have the option, pursuant to Article 546 of the Civil
Code, of indemnifying the Fuentes spouses for the costs of the improvements or paying the increase in
value which the property may have acquired by reason of such improvements.

60
OWNERSHIP
Concept of Ownership

THE IMPOSITION BY AN OWNER OF SECURITY MEASURES ON HIS PROPERTY EMANATES FROM


THE ATTRIBUTES OF OWNERSHIP UNDER ARTICLE 429 OF THE CIVIL CODE

46. Saluday v. People


G.R. No. 215305. April 3, 2018
Carpio, Acting C.J.

FACTS: On 5 May 2009, the bus that petitioner boarded on was flagged down by Task Force Davao of the
Philippine Army at a checkpoint. SCAA Buco, a member of the Task Force, requested all male passengers
to disembark from the vehicle while allowing the female passengers to remain inside. He then boarded the
bus to check the presence and intercept the entry of any contraband, illegal firearms or explosives, and
suspicious individuals. SCAA Buco checked all the baggage and personal effects of the passengers, but a
small, gray-black pack bag on the seat at the rear of the bus caught his attention. He lifted the bag and
found it too heavy for its small size. Afterwards, SCAA Buco asked who the owner of the bag was, to which
the bus conductor answered that Saluday and his brother were the ones seated at the back. SCAA Buco
then requested Saluday to board the bus and open the bag. When Saluday opened it, the bag revealed the
contents of firearms. Saluday was prosecuted and convicted of illegal possession of high-powered firearm,
ammunition, and explosive under Presidential Decree No. 1866, 4 as amended (PD 1866). He assails the
legality of the search and seizure conducted.

ISSUE: What is the basis of owners of private property in imposing security measures on their property
without violating the constituional right against unreasonable searches and seizures?

HELD: Article 429 of the Civil Code as an attribute of ownership.

In Costabella Corp. v. Court of Appeals, the Court explained, “[c]onsidering that the petitioner operates a
hotel and beach resort in its property, it must undeniably maintain a strict standard of security within its
premises. Otherwise, the convenience, privacy, and safety of its clients and patrons would be
compromised." Similarly, shopping malls install metal detectors and body scanners, and require bag
inspection as a requisite for entry. Needless to say, any security lapse on the part of the mall owner can
compromise public safety.

Concededly, a bus, a hotel and beach resort, and a shopping mall are all private property whose owners
have every right to exclude anyone from entering. At the same time, however, because these private
premises are accessible to the public, the State, much like the owner, can impose non-intrusive security
measures and filter those going in. The only difference in the imposition of security measures by an owner
and the State is, the former emanates from the attributes of ownership under Article 429 of the Civil Code,
while the latter stems from the exercise of police power for the promotion of public safety. Necessarily, a
person's expectation of privacy is diminished whenever he or she enters private premises that are
accessible to the public.

In view of the foregoing, the bus inspection conducted by Task Force Davao at a military checkpoint
constitutes a reasonable search. Bus No. 66 of Davao Metro Shuttle was a vehicle of public transportation
where passengers have a reduced expectation of privacy.

61
OWNERSHIP
Concept of Ownership

IT IS WITHIN THE RIGHT OF OWNERS TO FENCE AND ENCLOSE THEIR PROPERTY AND NO
DAMAGES CAN BE RECOVERED BY PERSONS INCIDENTALLY INJURED

47. Spouses Custodio v. Court of Appeals


G.R. No. 116100, February 9, 1996
Regalado, J.

FACTS:
A civil case for the grant of an easement of right of way was filed by Pacifico Mabasa (defendant herein)
against Cristino Custodio, et. al. (petitioners). During the pendency of this case, Mabasa died and was
substituted by his surviving spouse. In his complaint, he alleged that he owns a parcel of land surrounded
by other immovables pertaining to defendants herein. When the said property was purchased by Mabasa,
there were tenants occupying the premises. Sometime in 1982, Mabasa discovered that there had been
built an adobe fence in the passageway to the property making it narrower in width. Said adobe fence was
first constructed by Spouses Santos along their property which is also along the passageway. Eventually,
the adobe fence was extended in such a way that the entire passageway was enclosed. And it was then
that the remaining tenants of said apartment located in defendant’s property vacated the area.

The trial court granted the easement of right of way and ordered petitioner spouses to give defendant
permanent access — ingress and egress, to the public street; and indemnity to petitioners. The prayer for
damages was denied.

Mabasa’s heirs appealed the dispositive portion of the decision regarding the denial of damages in their
favor. The Court of Appeals modified the decision and awarded damages in favor of defendant.

ISSUE:
Whether or not the defendant may recover damages for losses incurred in the form of unrealized rentals by
reason of the closure of the passageway.

HELD:
No. In the case at bar, although there was damage, there was no legal injury.

The act of petitioners in constructing a fence within their lot is a valid exercise of their right as owners, hence
not contrary to morals, good customs or public policy. The law recognizes in the owner the right to enjoy
and dispose of a thing, without other limitations than those established by law. It is within the right of
petitioners, as owners, to enclose and fence their property. Article 430 of the Civil Code provides that
"(e)very owner may enclose or fence his land or tenements by means of walls, ditches, live or dead hedges,
or by any other means without detriment to servitudes constituted thereon."

At the time of the construction of the fence, the lot was not subject to any servitudes. There was no easement
of way existing in favor of private respondents, either by law or by contract. The fact that private respondents
had no existing right over the said passageway is confirmed by the very decision of the trial court granting
a compulsory right of way in their favor after payment of just compensation. It was only that decision which
gave private respondents the right to use the said passageway after payment of the compensation and
imposed a corresponding duty on petitioners not to interfere in the exercise of said right.

Hence, prior to said decision, petitioners had an absolute right over their property and their act of fencing
and enclosing the same was an act which they may lawfully perform in the employment and exercise of said
right. To repeat, whatever injury or damage may have been sustained by private respondents by reason of
the rightful use of the said land by petitioners is damnum absque injuria.

A person has a right to the natural use and enjoyment of his own property, according to his pleasure, for all
the purposes to which such property is usually applied. As a general rule, therefore, there is no cause of
action for acts done by one person upon his own property in a lawful and proper manner, although such
acts incidentally cause damage or an unavoidable loss to another, as such damage or loss is damnum
absque injuria. When the owner of property makes use thereof in the general and ordinary manner in which
the property is used, such as fencing or enclosing the same as in this case, nobody can complain of having

62
been injured, because the inconvenience arising from said use can be considered as a mere consequence
of community life.

63
OWNERSHIP
Concept of Ownership

THE OWNER OF A THING CANNOT MAKE USE THEREOF IN A MANNER AS TO INJURE THE RIGHTS
OF A THIRD PERSON

48. Spouses Andamo v. Intermediate Appellate Court


G.R. No.74761, November 6, 1990
Fernan, C.J.

FACTS:
This is a petition whether a corporation, which has built through its agents, waterpaths, water conductors
and contrivances within its land, thereby causing inundation and damage to an adjacent land, can be held
civilly liable for damages.

Spouses Andamo are the owners of a parcel of land in Silang, Cavite which is adjacent to that of private
respondent, Missionaries of Our Lady of La Salette, Inc., a religious corporation. Within the land of
respondent corporation, waterpaths and contrivances, including an artificial lake, were constructed, which
allegedly inundated and eroded petitioners' land, caused a young man to drown, damaged petitioners' crops
and plants, washed away costly fences, endangered the lives of petitioners and their laborers during rainy
and stormy seasons, and exposed plants and other improvements to destruction.

Spouses Andamo instituted a criminal action; thereafter a civil action for damages was filed. The trial court
suspended the hearings over the civil case, the suspension was later affirmed by the IAC.

ISSUE:
Should respondent corporation be held liable for damages caused to Spouses Andamo?

HELD:
Yes. It must be stressed that the use of one's property is not without limitations. Article 431 of the Civil Code
provides that "the owner of a thing cannot make use thereof in such a manner as to injure the rights of a
third person." SIC UTERE TUO UT ALIENUM NON LAEDAS.

Moreover, adjoining landowners have mutual and reciprocal duties which require that each must use his
own land in a reasonable manner so as not to infringe upon the rights and interests of others. Although we
recognize the right of an owner to build structures on his land, such structures must be so constructed and
maintained using all reasonable care so that they cannot be dangerous to adjoining landowners and can
withstand the usual and expected forces of nature. If the structures cause injury or damage to an adjoining
landowner or a third person, the latter can claim indemnification for the injury or damage suffered.

64
OWNERSHIP
Concept of Ownership

SOLE ISSUE IN UNLAWFUL DETAINER CASES IS MATERIAL POSSESSION

49. Heirs of Mariano v. City of Naga


G.R. No.197743, March 12, 2018
Tijam, J.

FACTS:
This is a petition for review under Rule 45 assailing the decision of the CA reversing the MTC and the RTC
in upholding the unlawful detainer case filed by petitioners.

The Lopez’ owner of City Heights Subd. offered to construct the Naga City Hall within the premises of the
subdivision. The area where the hall will be built will be donated on the condition that it will be the subdivision
that will construct the city hall. The area donated was registered with Macario and Gimenez.

Petitioners contended that the plan to donate the area did not materialize as the contract to build the City
Hall was not awarded to the subdivision but to a different contractor. Petitioners claimed that the Macario
and officers of the subdivision met with Naga’s mayor to demand the return of the property; the mayor
assured them that the city would buy the property. Macario died without receiving payment. Petitioners as
heirs demanded that the city vacate and return the property. Failing to do so, they filed a complaint for
unlawful detainer.

The City countered that the donation actually took place, as evidenced by a Deed of Donation. It also
countered that the property has long been declared in the city’s name for tax purposes. It further contended
that it could not be ejected from the premises as it possessed the rights of a builder in good faith.

ISSUE:
Does the mere invocation of ownership by the defendant render the action for unlawful detainer dismissible?

HELD:
No.

The sole issue for resolution in an unlawful detainer case is physical or material possession of the property
involved, independent of any claim of ownership by any of the parties - possession de facto and not
possession de jure.

When the defendant, however, raises the defense of ownership in his pleadings and the question of
possession cannot be resolved without deciding the issue of ownership, the issue of ownership shall be
resolved only to determine the issue of possession, or more particularly, to determine who between the
parties has the better right to possess the property. Nonetheless, the adjudication is merely provisional and
would not bar or prejudice an action between the same parties involving title to the property.

65
OWNERSHIP
Concept of Ownership

REQUISITE FOR A VALID CAUSE OF ACTION FOR UNLAWFUL DETAINER THAT POSSESSION WAS
INITIALLY LAWFUL AND TURNED UNLAWFUL UPON EXPIRATION OF THE RIGHT TO POSSESS

50. Eversley Childs Sanitarium v. Spouses Barbarona


G.R. No. 195814, April 4, 2018
Leonen, J.

FACTS:
This is a Petition for Review on Certiorari assailing the CA’s decision upholding the MTC and RTC ordering
petitioner to vacate the disputed property.

Eversley is a public health facility operated by the DOH for the treatment to patients with leprosy. Since
1930, it has occupied a portion of land in Mandaue, Cebu. Spouses Barbarona alleged that they are the
owners of the said lot by virtue of a TCT. They claim that they have acquired the property from Spouses
Alba whose ownership was covered by an OCT. Spouses Barbarona filed a complaint for ejectment before
MTC Mandaue on May 6, 2005. The Spouses Barbarona alleged that they had sent demand letters and
that the occupants were given until April 15, 2005 to vacate the premises. They further claimed that despite
the lapse of the period, the occupants refused to vacate.

In their Answer, the occupants alleged that since they had been in possession of the property for more than
70 years, the case was effectively one for recovery of possession, which was beyond the jurisdiction of the
Municipal Trial Court.

The MTC ordered the occupants to vacate the property finding the action was one of unlawful detainer which
was affirmed by the RTC. The CA upheld the lower court’s decision upon appeal.

ISSUE:
Whether the Spouses’ complaint was for unlawful detainer?

HELD:
The spouses’ complaint was not for unlawful detainer. The proper remedy should have been to file an accion
publiciana or reinvindicatoria to assert their right of possession or ownership.

Respondents' Complaint before the Municipal Trial Court states: “That [the occupants] are presently
occupying the above-mentioned property of the [Spouses Barbarona] without color [of] right or title. Such
occupancy is purely by mere tolerance. Indeed, [the occupants'] occupying the lot owned by [the Spouses
Barbarona] is illegal and not anchored upon any contractual relations with the [Spouses Barbarona.]”

Indeed, no mention has been made as to how petitioner came to possess the property and as to what acts
constituted tolerance on the part of respondents or their predecessors-in-interest to allow petitioner's
occupation. A requisite for a valid cause of action in an unlawful detainer case is that possession must be
originally lawful, and such possession must have turned unlawful only upon the expiration of the right to
possess. It must be shown that the possession was initially lawful; hence, the basis of such lawful
possession must be established. If, as in this case, the claim is that such possession is by mere tolerance
of the plaintiff, the acts of tolerance must be proved. Respondents failed to state when petitioner's
possession was initially lawful, and how and when their dispossession started. All that appears from the
Complaint is that petitioner's occupation "is illegal and not anchored upon any contractual relations with
[respondents.]"

66
OWNERSHIP
Concept of Ownership

THE FACT OF TOLERANCE IS OF UTMOST IMPORTANCE IN AN ACTION FOR UNLAWFUL


DETAINER

51. Javelosa v. Tapus


G.R. No. 204361, July 4, 2018
Reyes, Jr., J.

FACTS:
This is a Petition for Review on Certiorari assailing the CA’s decision dismissing the case for unlawful
detainer filed by Javelosa before the lower court.

The subject property in Aklan is registered with the petitioner by a TCT. The property was originally covered
by an OCT which Javelosa acquired by donation from Tirol. It was occupied by Tapus and other
respondents. The respondents’ predecessor was assigned as a caretaker of the property, and therefore
possessed and occupied a portion thereof upon the tolerance and permission of Tirol. In 2003, petitioner’s
daughter learned that a relative of Tapus offered to sell the said property. After the parties failed to
compromise, the petitioner sent a demand letter to the respondents. The demand was unheeded which
prompted petitioner to file for an unlawful detainer.

In their Answer, she and her predecessors-in-interest have been occupying the subject property since time
immemorial. She emphasized that they are actual, adverse and exclusive possessors under a claim of
ownership. She further averred that they are indigenous occupants and tribal settlers of the land in dispute,
and hence their rights are protected by law.

The MTC ordered the occupants to vacate the which was affirmed by the RTC. The CA reversed the lower
court’s decision upon appeal ruling that the the petitioner failed to show that the respondents occupied the
subject property pursuant to her tolerance, and that such permission was present from the very start of their
occupation.

ISSUE:
Whether the all the requisites in filing an action for unlawful detainer was complied with?

HELD:
No, the complaint stated that (i) the respondents occupied the subject property upon the tolerance of the
petitioner; (ii) the petitioner sent the respondents a demand to vacate sometime in October 2003; (iii) the
same demand was unheeded; and (iv) the action for unlawful detainer was filed within one year from the
date of the demand.

The petitioner failed to adduce evidence to establish that the respondents' occupation of the subject property
was actually effected through her tolerance or permission. Unfortunately, the petitioner failed to prove how
and when the respondents entered the subject lot, as well as how and when the permission to occupy was
purportedly given. In fact, she was conspicuously silent about the details on how the permission to enter
was given, save for her bare assertion that the respondents' occupied the premises as caretakers thereof.

It cannot be gainsaid that the fact of tolerance is of utmost importance in an action for unlawful detainer.
Without proof that the possession was legal at the outset, the logical conclusion would be that the
defendant's possession of the subject property will be deemed illegal from the very beginning, for which,
the action for unlawful detainer shall be dismissed. Tolerance cannot be presumed from the owner's failure
to eject the occupants from the land. Rather, "tolerance always carries with it 'permission' and not merely
silence or inaction for silence or inaction is negligence, not tolerance."

67
OWNERSHIP
Concept of Ownership

THE OWNER OF A THING CANNOT MAKE USE THEREOF IN SUCH A MANNER AS TO INJURE THE
RIGHTS OF OTHER PERSONS; SIC UTERE TUO UT ALIENUM NON LAEDAS

52. Spouses Andamo v. Intermediate Appellate Court


G.R. No. 74761, November 6, 1990
Fernan, J.

FACTS:
This is a petition for certiorari, prohibition and mandamus filed by petitioner Spouses Andamo challenging
the order of the respondent IAC and the RTC suspending further hearings in a civil case until after judgment
in the related criminal case.

Petitioner spouses Andamo are the owners of a parcel of land situated in Silang, Cavite which is adjacent
to that of private respondent, Missionaries of Our Lady of La Salette, Inc., a religious corporation. Within the
land of respondent corporation, waterpaths and contrivances, including an artificial lake, were constructed,
which allegedly inundated and eroded petitioners’ land, caused a young man to drown, damaged petitioners’
crops and plants, washed away costly fences, endangered the lives of petitioners and their laborers during
rainy and stormy seasons, and exposed plants and other improvements to destruction.

Thus, petitioners first instituted a criminal action against the officers and directors respondent corporation
for destruction by means of injunction under Article 324 of the Revised Penal Code. After one year,
petitioners filed another action against respondent corporation, this time a civil case, for damages with
prayer for the issuance of a writ of preliminary injunction before the same court.

ISSUE:
Can a corporation, which has built through its agents, waterpaths, water conductors and contrivances within
its land, thereby causing inundation and damage to an adjacent land, be held civilly liable for damages?

HELD:
Yes, a corporation which causes damage to another by the use of its property may be held liable for
damages. It must be stressed that the use of one’s property is not

Article 431 of the Civil Code provides that "the owner of a thing cannot make use thereof in such a manner
as to injure the rights of a third person." SIC UTERE TUO UT ALIENUM NON LAEDAS. Moreover, adjoining
landowners have mutual and reciprocal duties which require that each must use his own land in a
reasonable manner so as not to infringe upon the rights and interests of others. Although we recognize the
right of an owner to build structures on his land, such structures must be so constructed and maintained
using all reasonable care so that they cannot be dangerous to adjoining landowners and can withstand the
usual and expected forces of nature. If the structures cause injury or damage to an adjoining landowner or
a third person, the latter can claim indemnification for the injury or damage suffered.

In this case, the waterpaths and contrivances built by respondent corporation have inundated the land of
petitioners. There is therefore, an assertion of a causal connection between the act of building these
waterpaths and the damage sustained by petitioners. Such action if proven constitutes fault or negligence
which may be the basis for the recovery of damages.

Thus, a corporation, which has built through its agents, waterpaths, water conductors and contrivances
within its land, thereby causing inundation and damage to an adjacent land can be held civilly liable for
damages under Articles 2176 and 2177 of the Civil Code.

68
OWNERSHIP
Concept of Ownership

IN THE CONTEXT OF EXPROPRIATION PROCEEDINGS, THE SOIL HAS NO VALUE SEPARATE


FROM THAT OF THE EXPROPRIATED LAND

THE OWNERSHIP OF LAND EXTENDS TO THE SURFACE AS WELL AS TO THE SUBSOIL UNDER IT

53. Republic v. Rural Bank of Kabacan


GR No. 185124, January 25, 2012
Sereno, J.

FACTS:
This is a Petition for Review on Certiorari under Rule 45 filed by petitioner National Irrigation Administration
(NIA) seeking the reversal of the decision of CA affirming the judgement of the RTC.

Petitioner is a GOCC under R.A. 3601. It is primarily responsible for irrigation development and
management in the country. To carry out its purpose, petitioner was specifically authorized under P.D. 552
to exercise the power of eminent domain. As petitioner needed some parcels of land for the purpose of
constructing the Malitubog-Marigadao Irrigation Project, it filed with the RTC of Kabacan, Cotabato a
Complaint for the expropriation of a portion of 3 parcels of lands: Lot No. 3080, Lot No. 455, and Lot No.
3039.

The RTC created a committee to determine the fair market value of the expropriated properties to establish
the just compensation to be paid to the owners. The committee added to its computation the value of the
earthfill excavated from portions of Lot Nos. 3039 and 3080, to which the petitioners objected. The RTC
ruled that the inclusion of the value of the soil is correct. On the other hand, the CA deleted the inclusion of
the value of the soil excavated from the properties in the just compensation but affirmed RTC’s valuation of
the improvements in the said lands.

ISSUE:
Is excavated soil distinct and separate from the expropriated lands, therefore, it must be included in the
computation of just compensation?

HELD:
No, excavated soil is not distinct and separate from the expropriated lands, thus, it must not be inclyded in
the computation of just compensation. In the context of expropriation proceedings, the soil has no value
separate from that of the expropriated land.

In National Power Corporation v. Ibrahim, et al, the SC held that rights over lands are indivisible. As stated
in ART. 437, “The owner of a parcel of land is the owner of its surface and of everything under it, and he
can construct thereon any works or make any plantations and excavations which he may deem proper,
without detriment to servitudes and subject to special laws and ordinances. He cannot complain of the
reasonable requirements of aerial navigation.” Thus, The ownership of land extends to the surface as well
as to the subsoil under it.

In the construction of irrigation projects, petitioner must necessarily make excavations in order to build the
canals. Indeed, it is preposterous that petitioner will be made to pay not only for the value of the land but
also for the soil excavated from such land when such excavation is a necessary phase in the building of
irrigation projects. That petitioner will make use of the excavated soil is of no moment and is of no concern
to the landowner who has been paid the fair market value of his land. As pointed out by the OSG, the law
does not limit the use of the expropriated land to the surface area only. Further, petitioner, now being the
owner of the expropriated property, has the right to enjoy and make use of the property in accordance with
its mandate and objectives as provided by law. To sanction the payment of the excavated soil is to allow
the landowners to recover more than the value of the land at the time when it was taken, which is the true
measure of the damages, or just compensation, and would discourage the construction of important public
improvements.

Therefore, since in the context of expropriation proceedings, the soil has no value separate from that of the
expropriated land, the excavated soil should not be included in the computation of just compensation.

69
OWNERSHIP
Concept of Ownership

A LAND CANNOT BE CLASSIFIED SIMULTANEOUSLY AS AN AGRICULTURAL LAND IN REGARDS


TO THE SURFACE LAND AND MINERAL LAND IN REGARDS TO ITS SUB-SURFACE

54. Republic v. Court of Appeals and De la Rosa


G.R. Nos. L-43938, L-44081, & L-44092, April 15, 1988
Cruz, J.

FACTS:
This is an appeal filed by both petitioner Benguet and Atok from the judgment of the CA affirming that the
surface rights of respondents de la Rosas over the land while at the same time reserving the sub-surface
rights of petitioners Benguet and Atok by virtue of their mining claims. On the other hand, the Republic has
filed its own petition for review and reiterates its argument that neither the private respondents nor the two
mining companies have any valid claim to the land because it is not alienable and registerable.

Respondent Jose de la Rosa on his own behalf and on behalf of his three children, Victoria, Benjamin and
Eduardo filed for an application for registration of a parcel of land. The land, situated in Tuding, Itogon,
Benguet Province, was divided into 9 lots. According to the application, Lots 1-5 were sold to Jose de la
Rosa and Lots 6-9 to his children by Mamaya Balbalio and Jaime Alberto, respectively, in 1964. In support
of the application, both Balbalio and Alberto testified that they had acquired the subject land by virtue of
prescription.

The application was separately opposed by petitioners Benguet Consolidated, Atok Big Wedge and by the
Republic of the Philippines, through the Bureau of Forestry Development, as to lots 1-9.

Both petitioners Benguet and Atok support their opposition from a mining claim they respectively own. On
the other hand, the Bureau of Forestry Development also interposed its objection, arguing that the land
sought to be registered was covered by the Central Cordillera Forest Reserve Moreover, by reason of its
nature, it was not subject to alienation under the Constitutions of 1935 and 1973.

The RTC denied the application, holding that the applicants had failed to prove their claim of possession
and ownership of the land sought to be registered. CA reversed the decision of the RTC and recognized
the claims of the applicant, but subject to the rights of Benguet and Atok respecting their mining claims. In
other words, the Court of Appeals affirmed the surface rights of the de la Rosas over the land while at the
same time reserving the sub-surface rights of Benguet and Atok by virtue of their mining claims. CA arrived
in its decisions by applying the Regalian doctrine.

ISSUE:
Can a land be classified as half agricultural and half mineral lands?

HELD:
No, a land cannot be used for both mining and non-mining purposes simultaneously. The Regalian doctrine
was misapplied in this case by the CA.

The Court feels that the rights over the land are indivisible and that the land itself cannot be half agricultural
and half mineral. The classification must be categorical; the land must be either completely mineral or
completely agricultural. In the instant case, as already observed, the land which was originally classified as
forest land ceased to be so and became mineral — and completely mineral — once the mining claims were
perfected. As long as mining operations were being undertaken thereon, or underneath, it did not cease to
be so and become agricultural, even if only partly so, because it was enclosed with a fence and was
cultivated by those who were unlawfully occupying the surface.

In regard to the application of CA of the Regalian doctrine, the flaw in the reasoning of the respondent court
is in supposing that the rights over the land could be used for both mining and non-mining purposes
simultaneously. The correct interpretation is that once minerals are discovered in the land, whatever the
use to which it is being devoted at the time, such use may be discontinued by the State to enable it to extract
the minerals therein in the exercise of its sovereign prerogative. The land is thus converted to mineral land
and may not be used by any private party, including the registered owner thereof, for any other purpose that

70
will impede the mining operations to be undertaken therein, For the loss sustained by such owner, he is of
course entitled to just compensation under the Mining Laws or in appropriate expropriation proceedings.

Therefore, a land cannot be classified as both agricultural in the surface and mineral land in the sub-soil.

71
OWNERSHIP
Right of Accession

A BONUS THAT BEARS NO IMMEDIATE, BUT ONLY A REMOTE ACCIDENTAL RELATION TO A


LAND CANNOT BE CONSIDERED AS A CIVIL FRUIT OF THE SAID LAND

55. Bachrach Motor Co., Inc. v. Talisay-Silay Milling Co., Inc.


G.R. No. 35223, September 17, 1931
Romualdez, J.

FACTS:
This is an appeal filed by intervenor-appellant Philippine National Bank (PNB) seeking the reversal of the
judgment of the trial court ruling that a bonus is not a civil fruit.

Defendant Talisay-Silay Milling Co., Inc., was indebted to the PNB. To secure the payment of its debt, it
succeeded in inducing its planters, among whom was Mariano Lacson Ledesma, to mortgage their land to
the creditor bank. And in order to compensate those planters for the risk they were running with their
property under the mortgage, the aforesaid central, by a resolution, undertook to credit the owners every
year with the payment of a bonus equal to two per centum of the debt secured according to yearly balance.

Bachrach Motor Co., Inc. filed a complaint against defendant Talisay-Silay Milling for the delivery of the
amount of P13,850 or promissory notes or other instruments of credit for that sum payable on 30 June 1930,
as bonus in favor of Mariano Lacson Ledesma.

PNB filed a third party claim alleging a preferential right to receive any amount which Mariano Lacson
Ledesma might be entitled to from the defendant Talisay-Silay Milling Co. as bonus, because that would be
civil fruits of the land mortgaged to said bank by said debtor for the benefit of the central referred to, and by
virtue of a deed of assignment. Defencant Talisay-Silay answered the complaint that Mariano Lacson
Ledesma’s credit (P7,500) belonged to Cesar Ledesma, a purchaser in good faith. At the trial all the parties
agreed to recognize and respect the sale made in favor of Cesar Ledesma of the P7,500 part of the credit
in question. Upon conclusion of the hearing, the court held that the Bachrach Motor Co., Inc., had a preferred
right over the bonus.

ISSUE:
Is bonus considered as a civil fruit of the mortgaged property?

HELD:
No, the bonus is not obtained from that land but from something else, it is not civil fruits of that land, and
the bank's contention is untenable.

Article 355 of the Civil Code considers three things as civil fruits: First, the rents of buildings; second, the
proceeds from leases of lands; and, third, the income from perpetual or life annuities, or other similar
sources of revenue. It may be noted that according to the context of the law, the phrase "u otras analogas"
refers only to rent or income, for the adjectives "otras" and "analogas" agree with the noun "rentas," as do
also the other adjectives "perpetuas" and "vitalicias." That is why we say that by "civil fruits" the Civil Code
understands one of three and only three things, to wit: the rent of a building, the rent of land, and certain
kinds of income.

The said bonus bears no immediate, but only a remote accidental relation to the land mentioned, having
been granted as compensation for the risk of having subjected one's land to a lien in favor of the bank, for
the benefit of the entity granting said bonus. If this bonus be income or civil fruits of anything, it is income
arising from said risk, or, if one chooses, from Mariano Lacson Ledesma's generosity in facing the danger
for the protection of the central, but certainly it is not civil fruits or income from the mortgaged property,
which, as far as this case is concerned, has nothing to do with it.

Therefore, the said bonus cannot be considered as civil fruits of the said mortgaged property.

72
OWNERSHIP
Right of Accession

RENT IS A CIVIL FRUIT THAT BELONGS TO THE OWNER OF THE PROPERTY PRODUCING IT BY
RIGHT OF ACCESSION. OWNERSHIP OF THE THING SOLD IS ACQUIRED BY THE BUYER ONLY
UPON DELIVERY THEREOF, THAT IS, WHEN IT IS PLACED IN THE CONTROL AND POSSESSION
OF THE BUYER

56. Equatorial Realty Development, Inc. v. Mayfair Theater, Inc


G.R. No. 133879, November 21, 2001
Panganiban, J.

FACTS:
This is a Petition for Review under Rule 45 filed by Petitioner Equatorial Realty Development, Inc. seeking
the reversal of the judgment of the RTC dismissing the claim of petitioner for unpaid back rentals.

Carmelo & Bauermann, Inc. ("Camelo") used to own two 2-storey buildings located at Claro M. Recto
Avenue, Manila. Carmelo entered into a two Contract of Lease with Mayfair Theater Inc. ("Mayfair") for a
period of 20 years. Both leases contained a provision granting Mayfair a right of first refusal to purchase the
subject properties. However, in July 1978, within the 20-year-lease term, the subject properties were sold
by Carmelo to Equatorial Realty Development, Inc. ("Equatorial") for the total sum of P11,300,000, without
their first being offered to Mayfair. Mayfair filed a Complaint with the RTC for the annulment of the Deed of
Absolute Sale between Carmelo and Equatorial. The RTC ruled in favor of Carmelo and Equatorial. On
appeal, the CA completely reversed and set aside the judgment of the lower court. The SC denied the
Petition for Review and ruled that the Deed of Absolute Sale between petitioners Equatorial Realty
Development, Inc. and Carmelo & Bauermann, Inc. be rescinded. The Decision stated that "Equatorial has
received rents from Mayfair during all the years that this controversy has been litigated”.

Meanwhile, barely five months after Mayfair had submitted its Motion for Execution before the RTC,
Equatorial filed with the RTC, an action for the collection of a sum of money against Mayfair, claiming
payment of rentals or reasonable compensation for the defendant's use of the subject premises after its
lease contracts had expired.

ISSUE:
Is Equatorial entitled to back rentals?

HELD:
No, Equatorial is not entitled to back rentals because it was never put in actual and effective control or
possession of the property.

Ownership of the thing sold is a real right, which the buyer acquires only upon delivery of the thing to him
"in any of the ways specified in articles 1497 to 1501, or in any other manner signifying an agreement that
the possession is transferred from the vendor to the vendee." This right is transferred, not merely by
contract, but also by tradition or delivery. Non nudis pactis sed traditione dominia rerum transferantur. There
is said to be delivery if and when the thing sold "is placed in the control and possession of the vendee."
Thus, it has been held that while the execution of a public instrument of sale is recognized by law as
equivalent to the delivery of the thing sold, such constructive or symbolic delivery, being merely presumptive,
is deemed negated by the failure of the vendee to take actual possession of the land sold.

Rent is a civil fruit that belongs to the owner of the property producing it by right of accession. Consequently
and ordinarily, the rentals that fell due from the time of the perfection of the sale to petitioner until its
rescission by final judgment should belong to the owner of the property during that period. Under the factual
environment of this controversy, Equatorial was never put in actual and effective control or possession of
the property because of Mayfair's timely objection. The rental payments made by Mayfair should not be
construed as a recognition of Equatorial as the new owner. They were made merely to avoid imminent
eviction.

Therefore, since there was no actual and effective control or possession of the property by Equitorial
because of the timely objection of Mayfair, it is not entitled to back rentals.

73
OWNERSHIP
Right of Accession

THERE IS NO ACCRETION WITH ARTIFICIAL AND MAN-MADE DEPOSITS

57. Rex Daclison v. Eduardo Baytion


G.R. No. 219811, April 6, 2016
Mendoza, J.

FACTS:
Respondent Baytion alleges that he was a co-owner of a parcel of land consisting of 1,500 square meters,
a portion of which was leased to one Leonida. When the lease expired, petitioner and other persons acting
under her took possession of the portion leased and occupied by Leonida without the prior knowledge and
consent of respondent. Since then, petitioner had been occupying the contested portion. Upon learning of
petitioner’s unauthorized entry into the subject portion of the property, respondent demanded that he vacate
it. Despite oral and written demands to vacate, petitioner refused to do so. Hence, respondent filed a
Complaint for Forcible Entry and Damages against petitioner.

In his answer, petitioner averred that in 1978, respondent leased the subject portion to Antonio dela
Cruz (Antonio); that ten or fifteen years later, a stone walling, called a riprap, was erected at the creek lying
beside the subject property, leaving a deep down-sloping area; that Antonio acquired possession of the said
down-slope when he had the same filled up until it was leveled with the leased portion; that Antonio paid for
the right to possess the same, and respondent promised that he would no longer bother him if they would
just transfer to the filled-up and plane-leveled property; that on account of the said promise, Antonio and
Ernanie vacated the leased area and transferred their business to the filled-up portion; that despite the fact
that they already vacated the leased portion of the property, respondent still filed a complaint with the
barangay claiming that the filled-up portion was part of his property.

Petitioner insists that the filled-up portion between the riprap constructed by the government and the
property of respondent, which they possess, is outside of the land co-owned by the latter. Respondent
basically posits that although the disputed portion is outside the description of his property, it forms an
integral part of the latter because it is an accretion, construction, or improvement on the property and, under
the law, any accretion or anything built thereon belongs to him and his co-owners.

ISSUE:
Can the portion between the land co-owned by Baytion and the constructed riprap be owned through the
right of accretion?

HELD:
No, the respondent’s contention that he owns that portion by reason of accretion is misplaced.

Article 457 of the New Civil Code provides: To the owners of lands adjoining the banks of rivers belongs the
accretion which they gradually receive from the effects of the current of the waters. In other words, the
following requisites must concur in order for an accretion to be considered, namely: (1) that the deposit be
gradual and imperceptible; (2) that it be made through the effects of the current of the water; and,(3) that
the land where accretion takes place is adjacent to the banks of rivers.

In the case at bench, this contested portion cannot be considered an accretion. To begin with, the land
came about not by reason of a gradual and imperceptible deposit. The deposits were artificial and man-
made and not the exclusive result of the current from the creek adjacent to his property. Respondent failed
to prove the attendance of the indispensable requirement that the deposit was due to the effect of the current
of the river or creek. Alluvion must be the exclusive work of nature and not a result of human intervention.

Hence, the deposits being man-made, there is no accretion.

74
OWNERSHIP
Right of Accession

NO RECOVERY OF BUILDINGS FOR GOVERNMENT PROJECTS UNDER ARTICLE 445 OF CIVIL


CODE

58. Land Bank of the Philippines v. Perez


G.R. No. 166884, June 13, 2012
Brion, J.

FACTS:
Petitioner Land Bank of the Philippines (LBP) is a government financial institution and the official depository
of the Philippines. Respondents are the officers and representatives of Asian Construction and
Development Corporation (ACDC), a corporation engaged in the construction business.

Petitioner extended a credit accommodation to ACDC through the execution of an Omnibus Credit Line
Agreement (Agreement) between LBP and ACDC. In various instances, ACDC used the Letters of
Credit/Trust Receipts Facility of the Agreement to buy construction materials. The respondents, as officers
and representatives of ACDC, executed trust receipts in connection with the construction materials, with a
total principal amount of ₱52,344,096.32. The trust receipts matured, but ACDC failed to return to petitioner
the proceeds of the construction projects or the construction materials subject of the trust receipts. Petitioner
sent ACDC a demand letter, dated May 4, 1999, for the payment of its debts, including those under the
Trust Receipts Facility in the amount of ₱66,425,924.39. ACDC failed to comply with the demand letter
hence, petitioner filed a complaint for estafa against the respondents. Respondents alleged that ACDC
acted as a subcontractor for government projects such as the Metro Rail Transit, the Clark Centennial
Exposition and the Quezon Power Plant in Mauban, Quezon. Its clients for the construction projects, which
were the general contractors of these projects, have not yet paid them; thus, ACDC had yet to receive the
proceeds of the materials that were the subject of the trust receipts and were allegedly used for these
constructions. As there were no proceeds received from these clients, no misappropriation thereof could
have taken place.

ISSUE:
Can there be recovery of buildings or construction (to which the subject materials were used) despite
knowing that such are for government projects?

HELD:
No, the buildings or construction cannot be recovered.

Article 445 provides that: whatever is built, planted or sown on the land of another and the improvements
or repairs made thereon, belong to the owner of the land, subject to the provisions of the following articles.

At the onset of these transactions, petitioner knew that ACDC was in the construction business and that the
materials that it sought to buy under the letters of credit were to be used for the following projects: the Metro
Rail Transit Project and the Clark Centennial Exposition Project. Petitioner had in fact authorized the delivery
of the materials on the construction sites for these projects, as seen in the letters of credit it attached to its
complaint. Clearly, they were aware of the fact that there was no way they could recover the buildings or
constructions for which the materials subject of the alleged trust receipts had been used.

The materials were used for the construction of an immovable property on a property of public domain. As
an immovable property, the ownership of whatever was constructed with those materials would presumably
belong to the owner of the land, under Article 445 of the Civil Code, which in this case, is the government.

75
OWNERSHIP
Right of Accession

REMOVAL OF STRUCTURES BY THE OWNER OF THE LAND IS AN OPTION ONLY WHEN AFTER
THE HE CHOOSES TO SELL HIS LAND, THE OTHER PARTY FAILS TO PAY FOR THE SAME

59. Ignacio v. Hilario


G.R. No. L-175, April 30, 1946
Moran, C.J.

FACTS:
This is a petition for certiorari arising from a case in the Court of First Instance of Pangasinan between the
respondents Elias Hilario and his wife Dionisia Dres as plaintiffs, and the petitioners Damian, Francisco and
Luis, all surnamed Ignacio, as defendants, concerning the ownership of a parcel of land, partly rice-land
and partly residential. After the trial of the case, the lower court rendered judgment holding plaintiffs as the
legal owners of the whole property but conceding to defendants the ownership of the houses and granaries
built by them on the residential portion with the rights of a possessor in good faith, in accordance with Article
361 of the Civil Code.

The plaintiffs prayed for an order of execution alleging that since they chose neither to pay defendants for
the buildings nor to sell to them the residential lot, said defendants should be ordered to remove the structure
at their own expense and to restore plaintiffs in the possession of said lot. Defendants objected to this
motion which, after hearing, was granted by Judge Natividad.

ISSUE:
Is the removal of buildings from the land an option granted to builders, planters, or sowers in good faith if
the plaintiffs-respondents neither choose to pay for such buildings nor to sell the land?

HELD:
No, the removal of buildings from the land is not an option for the builders, planters, or sowers in good faith
should the landowners neither choose to pay for the same nor to sell the land.

Article 361 provides that: The owner of land on which anything has been built, sown or planted in good faith,
shall have the right to appropriate as his own the work, sowing or planting, after the payment of the indemnity
stated in articles 453 and 454, or to oblige the one who built or planted to pay the price of the land, and the
one who sowed, the proper rent.

Article 453 provides that: Necessary expenses shall be refunded to every possessor; but only the possessor
in good faith may retain the thing until such expenses are made good to him. Useful expenses shall be
refunded to the possessor in good faith with the same right of retention, the person who has defeated him
in the possession having the option of refunding the amount of the expenses or paying the increase in value
which the thing may have acquired in consequence thereof.

The owner of the building erected in good faith on a land owned by another, is entitled to retain the
possession of the land until he is paid the value of his building, under article 453. The owner of the land,
upon the other hand, has the option, under article 361, either to pay for the building or to sell his land to the
owner of the building. But he cannot, as respondents here did, refuse both to pay for the building and to sell
the land and compel the owner of the building to remove it from the land where it is erected. He is entitled
to such remotion only when, after having chosen to sell his land, the other party fails to pay for the same.
But this is not the case before us.

Hence, not having chosen to pay for the building nor to sell the land, the landowners cannot order the builder
in good faith to remove the building from the land.

76
OWNERSHIP
Right of Accession

ARTICLE 448 OF THE CIVIL CODE APPLIES TO LAND HELD IN CO-OWNERSHIP

60. Ignao v. Intermediate Appellate Court


G.R. No. 72876, January 18, 1991
Fernan, C.J.

FACTS:
Petitioner and his uncles private respondents Juan Ignao and Isidro Ignao were co-owners of a parcel of
land with an area of 534 square meters situated in Cavite. Pursuant to an action for partition filed by
petitioner, the then Court of First Instance of Cavite directed the partition of the land, alloting 133.5 square
meters or 2/8 thereof to private respondents, and giving the remaining portion with a total area of 266.5
square meters to petitioner. However, no actual partition was ever effected.

On July 17, 1978, petitioner instituted a complaint for recovery of possession of real property against private
respondents before the Court of First Instance of Cavite. In his complaint, petitioner alleged that the area
occupied by the two (2) houses built by private respondents exceeded the 133.5 square meters previously
alloted to them by the trial court. Consequently, the lower court conducted an ocular inspection. It was found
that the houses of private respondents actually encroached upon a portion of the land belonging to
petitioner. Upon agreement of the parties, the trial court ordered a licensed geodetic engineer to conduct a
survey to determine the exact area occupied by the houses of private respondents. The survey subsequently
disclosed that the house of Juan occupied 42 square meters while that of Isidro occupied 59 square meters
of Florencio's land or a total of 101 square meters.

In its decision, the trial court ruled that although private respondents occupied a portion of petitioner's
property, they should be considered builders in good faith. Furthermore, the trial court stated that pursuant
to Article 448 of the Civil Code, the owner of the land (Florencio) should have the choice to either appropriate
that part of the house standing on his land after payment of indemnity or oblige the builders in good faith
(Juan and Isidro) to pay the price of the land. However, the trial court observed that based on the facts of
the case, it would be useless and unsuitable for Florencio to exercise the first option since this would render
the entire houses of Juan and Isidro worthless. The trial court then applied the ruling in the similar case
of Grana vs. Court of Appeals, where the Supreme Court had advanced a more "workable solution". Thus,
it ordered Florencio to sell to Juan and Isidro those portions of his land respectively occupied by the latter.
Petitioner Florencio Ignao appealed to the Intermediate Appellate Court. The Appellate Court affirmed the
decision of the trial court.

ISSUE:
Whether or not Article 448 is applicable to land owned in common by the parties?

HELD:
Yes, Article 448 is applicable to land owned in common by the parties.

In Spouses del Campo vs. Abesia, the Court ruled that: “The court a quo correctly held that Article 448 of
the Civil Code cannot apply where a co-owner builds, plants or sows on the land owned in common for then
he did not build, plant or sow upon land that exclusively belongs to another but of which he is a co-owner.
The co-owner is not a third person under the circumstances, and the situation is governed by the rules of
co-ownership. However, when, as in this case, the ownership is terminated by the partition and it appears
that the home of defendants overlaps or occupies a portion of 5 square meters of the land pertaining to
plaintiffs which the defendants obviously built in good faith, then the provisions of Article 448 of the new
Civil Code should apply. Manresa and Navarro Amandi agree that the said provision of the Civil Code may
apply even when there is a co-ownership if good faith has been established. In other words, when the co-
ownership is terminated by a partition and it appears that the house of an erstwhile co-owner has
encroached upon a portion pertaining to another co-owner which was however made in good faith, then the
provisions of Article 448 should apply to determine the respective rights of the parties.

77
OWNERSHIP
Right of Accession

NO AUTOMATIC APPROPRIATION UPON THE FAILURE OF THE BUILDER TO PAY THE VALUE OF
THE LAND, WHEN SUCH IS DEMANDED BY THE LAND-OWNER, UNDER ARTICLE 445

61. Filipinas Colleges, Inc. v. Timbang


G.R. No. L-12812, September 29, 1959
Barrera, J.

FACTS:
In a case before the CA, it was declared that petitioner has acquired the rights of the respondent spouses
in and to lot No. 2- and in consideration thereof, the former was ordered to pay the spouses Timbang the
amount of P15,807.90. Meanwhile, Maria Gervacio Blas was declared to be a builder in good faith of the
school building constructed on the lot in question and entitled to be paid the amount of P19,000.00 for the
same.

Petitioner, purchaser of the said building, was ordered to deliver to Blas stock certificate for 108 shares of
their corporation with a par value of P10,800.00 and to pay Blas the sum of P8,200.00 of the house. In case
Filipinas Colleges, Inc. failed to deposit the value of the land, petitioner would lose all its rights to the land
and the respondent spouses would then become the owners thereof. In that eventuality, the Timbangs
would make known to the court their option under Art. 448 of the Civil Code whether they would appropriate
the building in question, in which even they would have to pay Filipinas Colleges, Inc. the sum of
P19,000.00, or would compel the latter to acquire the land and pay the price thereof.

Filipinas Colleges, Inc. having failed to pay within the time prescribed, the respondent spouses, in
compliance with the judgment of the Court of Appeals, made known to the court their decision that they had
chosen not of appropriate the building but to compel Filipinas Colleges, Inc., for the payment of the sum of
P32,859,34. The motion having been granted, a writ of execution was issued. Appellee Blas in turn filed a
motion for execution of her judgment of P8,200.00 representing the unpaid portion of the price of the house
sold to Filipinas Colleges, Inc. over the object of the Timbangs, the court granted the motion and the
corresponding writ of execution was issued. Levy having been made on the house in virtue of the writs of
execution, the Sheriff sold the building in public auction in favor of the spouses Timbang, as the highest
bidders, in the amount of P5,750.00. Personal properties of petitioner were also auctioned for P245.00 in
favor of the respondent spouses.

Respondent contended that because the builder in good faith has failed to pay the price of the land after
the owners thereof exercised their option under Article 448 of the Civil Code, the builder lost his right of
retention provided in Article 546 and by operation of Article 445, the appellants as owners of the land
automatically became the owners ipso facto, the execution sale of the house in their favor was superfluous.

ISSUE:
Upon failure of the builder to pay the value of the land under Arts. 448 and 546 of the Civil Code, when such
is demanded by the land-owner, does the latter automatically become the owner of the improvement?

HELD:
No, the landowner does not automatically become the owner of the improvement upon failure of the buidler
to pay the value of the land.

Under the terms of these article, it is true that the owner of the land has the right to choose between
appropriating the building by reimbursing the builder of the value thereof or compelling the builder in good
faith to pay for his land. Even this second right cannot be exercised if the value of the land is considerably
more than that of the building. In addition to the right of the builder to be paid the value of his improvement,
Article 546 gives him the corollary right of retention of the property until he is indemnified by the owner of
the land.

There is nothing in the language of these two articles 448 and 546, which would justify the conclusion of
appellants that, upon the failure of the builder to pay the value of the land, when such is demanded by the
land-owner, the latter becomes automatically the owner of the improvement under Article 445. The reliance
by the respondent on the case of Bernardo v. Bataclan is misplaced. Although it is true it was declared
therein that in the event of the failure of the builder to pay the land after the owner thereof has chosen this

78
alternative, the builder's right of retention provided in Article 546 is lost, nevertheless there was nothing said
that as a consequence thereof, the builder loses entirely all rights over his own building.

79
OWNERSHIP
Right of Accession

THE OPTION GIVEN BY LAW EITHER TO RETAIN THE PREMISES AND PAY FOR THE
IMPROVEMENTS THEREON OR TO SELL THE SAID PREMISES TO THE BUILDER IN GOOD FAITH
BELONGS TO THE OWNER OF THE PROPERTY

62. Manotok Realty, Inc. v. Tecson


G.R. No. L-47475, August 19, 1988
Gutierrez, Jr., J.

FACTS:
This is a Petition for Mandamus, where petitioner questions the denial of his motion to exercise option and
for execution of judgment on the grounds that under Articles 448 and 546 of the Civil Code, the exercise of
option belongs to the owner of the property, who is the petitioner herein, and that upon finality of judgment,
the prevailing party is entitled, as a matter of right, to its execution which is only a ministerial act on the part
of the respondent judge.

Petitioner filed a complaint for recovery of possession and damages against Nilo Madlangawa for the
recovery of possession of a parcel of land, but the trial court declared the latter a builder and possessor in
good faith.

Upon finality of the decision, the petitioner filed with the trial court, presided over by respondent judge, a
motion for the approval of petitioner’s exercise of option and for satisfaction of judgment, praying that the
court issue an order: a) approving the exercise of petitioner’s option to appropriate the improvements
introduced by the private respondent on the property; b) thereafter, private respondent be ordered to deliver
possession of the property in question to the petitioner.

The CFI denied the motion and held that under peculiar circumstances which intervened, such as the
construction of improvements, it thereby made the action not legally proper. After a denial of its motion for
reconsideration, the petitioner filed the present petition for mandamus.

ISSUE:
Should the motion for the approval of the exercise of option be granted?

HELD:
Yes, the motion is proper and should be approved.

Under Articles 448 and 546 of the Civil Code, the exercise of option belongs to the owner of the property.

The respondent judge cannot deny the issuance of a writ of execution because the private respondent was
adjudged a builder in good faith or on the ground of "peculiar circumstances which supervened after the
institution of this case, like, for instance, the introduction of certain major repairs of and other substantial
improvements..." because the option given by law either to retain the premises and pay for the
improvements thereon or to sell the said premises to the builder in good faith belongs to the owner of the
property.

Moreover, the repairs and improvements introduced by the said respondents after the complaint was filed
cannot be considered to have been built in good faith, much less justify the denial of the petitioner’s exercise
of option. Since the improvements have been gutted by fire, and therefore, the basis for private respondent’s
right to retain the premises has already been extinguished without the fault of the petitioner, there is no
other recourse for the private respondent but to vacate the premises and deliver the same to herein
petitioner.

80
OWNERSHIP
Right of Accession

THE OWNER OF THE LAND SHOULD CHOOSE BETWEEN INDEMNIFYING THE OWNER OF THE
IMPROVEMENTS OR REQUIRING THE LATTER TO PAY FOR THE LAND

63. Bernardo v. Bataclan


G.R. No. L-44606, November 28, 1938
Laurel, Jr., J.

FACTS:
Petitioner learned when he entered into the premises of the property purchased from Pastor Samonte, the
latter authorised respondent to make improvements thereon. In a civil case to secure possession, the court
ruled that respondent was a builder and possessor in good faith and was entitled to reimbursement for the
works and improvements,

The court gave the petitioner 30 days within which to choose between the sale of the land or to buy the
works. Petitioner decided to sell the land to the respondent but the latter informed the court that he is unable
to pay the sum required. The court then awarded the respondent 30 days to purchase the land or else the
property will be sold in a public auction.

In the auction sale, Toribio Teodoro was the highest bidder. The purchaser sought judicial remedy for the
possession of the property.

ISSUE:
Did defendant lose his right to retain the property pending payment for indemnity?

HELD:
Yes, the court ruled that the right to retain the property has already been lost.
The principle of accession provides that the owner of property acquires not only that which it produces but
that which is united to it either naturally or artificially. Whatever is built, planted or sown on the land of
another, and the improvements or repairs made thereon, belong to the owner of the land. Where, however,
the planter, builder, or sower has acted in good faith, a conflict of rights arises between the owners and it
becomes necessary to protect the owner of the improvements without causing injustice to the owner of the
land. The law provides a just and equitable solution by giving the owner of the land the option to acquire the
improvements after payment of the proper indemnity or to oblige the builder or planter to pay for the land
and the sower to pay the proper rent. It is the owner of the land who is allowed to exercise the option
because his right is older and because, by the principle of accession, he is entitled to the ownership of the
accessory thing. In the case before us, the plaintiff, as owner of the land, chose to require the defendant,
as owner of the improvements, to pay for the land.

The petitioner expressed his desire to require the respondent to pay for the value of the land. The said
respondent could have become owner of both land and improvements and continued in possession thereof.
But he said he could not pay and the land was sold at public auction to Toribio Teodoro. The law, as we
have already said, requires no more than that the owner of the land should choose between indemnifying
the owner of the improvements or requiring the latter to pay for the land. When he failed to pay for the land,
the respondent lost his right of retention.

81
OWNERSHIP
Right of Accession

ORDINARY ACQUISITIVE PRESCRIPTION, IN THE CASE OF IMMOVABLE PROPERTY, REQUIRES


POSSESSION OF THE THING IN GOOD FAITH AND WITH JUST TITLE, FOR A PERIOD OF TEN
YEARS

64. Heirs of Durano, Sr. v. Spouses Uy


G.R. No. 136456, October 24, 2000
Gonzaga-Reyes, J.

FACTS:
Petitioners seek the reversal of the decision and resolution of the CA finding that the respondents are
builders in good faith.

In a case for damages against respondents, petitioners alleged that respondents filed complaints against
the former for invasion of the latter’s properties. As a counterclaim, the respondents demanded the return
of their properties claiming that in August 1970, they received mimeographed notices signed by Durano Sr.
informing them that the land they occupied was purchased by Durano & Co. from Cebu Portland Cement
Company (Cepoc); that at that time, they were already tilling and residing in the land in question. However,
before many of them could even receive the notice in 1970 that Durano & Co. bought the land from Cepoc,
employees of Durano & Co. proceeded to bulldoze the land, destroying plantings and improvements made
therein.

On September 15, 1970, Durano & Co. sold the subject land to Durano III. On the other hand, petitioners
alleged that the property originally belonged to Cepoc and was sold to Durano & Co., and later on to Durano
III. But Durano III claimed that he only learned of the bulldozing when the respondents already filed
complaints.

ISSUE:
Whether or not the respondents are builders in good faith and thereby acquired the said property through
ordinary acquisitive prescription?

HELD:
Yes, the court ruled that the respondents are builders in good faith and their possession has already ripened
into ownership by acquisitive prescription.

In the case at bar, the respondents acquired the properties by purchase or inheritance and ever since were
in actual, continuous, open, and adverse possession. They exercised rights of ownership over the lands,
including the regular payment of taxes and introduction of plantings and improvements. They were unaware
of anyone claiming to be the owner of these lands other than themselves until the notices of demolition in
1970— and at the time each of them had already completed the ten-year prescriptive period either by their
own possession or by obtaining from the possession of their predecessors-in-interest.

The petitioners on the other hand cannot claim good faith. A purchaser of a parcel of land cannot close his
eyes to facts which should put a reasonable man upon his guard, such as when the property subject of the
purchase is in the possession of persons other than the seller. A buyer who could not have failed to know
or discover that the land sold to him was in the adverse possession of another is a buyer in bad faith. Since
petitioners knew fully well the defect in their titles, they were correctly held by the Court of Appeals to be
builders in bad faith.

Based on the provisions under the Civil Code, the owner of the land has three alternative rights: (1) to
appropriate what has been built without any obligation to pay indemnity therefor, or (2) to demand that the
builder remove what he had built, or (3) to compel the builder to pay the value of the land. In any case, the
landowner is entitled to damages. The Court sustains the return of the properties to respondents and the
payment of indemnity as being in accord with the reliefs under the Civil Code.

82
OWNERSHIP
Right of Accession

THE RIGHT TO CHOOSE BETWEEN APPROPRIATING THE IMPROVEMENT OR SELLING THE LAND
ON WHICH THE IMPROVEMENT STANDS TO THE BUILDER, PLANTER OR SOWER, IS GIVEN TO
THE OWNER OF THE LAND

65. Ballatan v. Court of Appeals


G.R. No. 125683, March 2, 1999
Puno, J.

FACTS:
This is a petition for review on certiorari of the decision of the Court of Appeals, seeking the reversal of the
judgment rendered by the CA in finding the respondents as builders in good faith.

In 1985, petitioner Ballatan constructed her house on Lot No. 24. During the construction, she noticed that
the concrete fence and side pathway of the adjoining house of respondent Winston Go encroached on the
entire length of the eastern side of her property. Go, on the other hand, claims that the fence and pathway
were built within the parameters of his father’s lot and that this lot was surveyed by engineer Jose Quedding,
the authorized surveyor of Araneta Institute of Agriculture (AIA).

Ballatan made written demands to the respondent to dismantle and move their improvements and since the
latter did not answer, the petitioner filed an accion publiciana in court. The court found that Go encroached
42 square meters from the property of Ballatan and Yao encroached 37 square meters on Go’s property,
all of which were in good faith.

ISSUE:
May petitioner demand the dismantling of the improvements made by the respondents who were found to
be builders in good faith?

HELD:
No, the petitioner may not demand the dismantling of the improvements made by the respondents who were
found by the court to be builders in good faith.

The owner of the land on which anything has been built, sown or planted in good faith shall have the right
to appropriate as his own the building, planting or sowing, after payment to the builder, planter or sower of
the necessary and useful expenses, and in the proper case, expenses for pure luxury or mere pleasure.
The owner of the land may also oblige the builder, planter or sower to purchase and pay the price of the
land. If the owner chooses to sell his land, the builder, planter or sower must purchase the land, otherwise
the owner may remove the improvements thereon

Applying Artice 448 of the Civil Code, petitioners, as owners of Lot No. 24, may choose to purchase the
improvement made by respondents Go on their land, or sell to respondents Go the subject portion. If buying
the improvement is impractical as it may render the Go's house useless, then petitioners may sell to
respondents Go that portion of Lot No. 24 on which their improvement stands. If the Go's are unwilling or
unable to buy the lot, then they must vacate the land and, until they vacate, they must pay rent to petitioners.
Petitioners, however, cannot compel respondents Go to buy the land if its value is considerably more than
the portion of their house constructed thereon. If the value of the land is much more than the Go's
improvement, then respondents Go must pay reasonable rent. If they do not agree on the terms of the lease,
then they may go to court to fix the same.

83
OWNERSHIP
Right of Accession

POSSESSORS IN GOOD FAITH ARE ENTITLED TO INDEMNITY FOR THE IMPROVEMENTS THEY
INTRODUCED INTO THE PROPERTY WITH A RIGHT OF RETENTION UNTIL THE REIMBURSEMENT
IS MADE; HOWEVER, SUBJECT TO THE OPTION OF THE LANDOWNER UNDER ARTICLE 448

66. Spouses Fuentes v. Roca


G.R. No. 178902, April 21, 2010
Abad, J.

FACTS:
This is a petition for review on certiorari of the decision of the CA, seeking the reversal of its finding the sale
of the subject land void from the beginning and did not transfer ownership to the petitioners.

In 1988, Tarciano T. Roca offered to sell a lot to petitioners. The sale was to take effect in six months, and
within six months, Tarciano was to clear the lot of structures and occupants and secure the consent of his
estranged wife, Rosario Gabriel Roca (Rosario), to the sale. If Tarciano was unable to comply with these
conditions, the Fuentes spouses would become owners of the lot without any further formality and payment.

The parties left their signed agreement with Atty. Plagata who then worked on the other requirements of the
sale. According to him, he went to see Rosario in one of his trips to Manila and had her sign an affidavit of
consent. After 6 months, a new title was issued in the name of the spouses who immediately constructed a
building on the lot. Then, Tarciano passed away, followed by his wife Rosario who died nine months after.

Eight years later in 1997, the children of Tarciano and Rosario, the respondents, filed an action for
annulment of sale and re-conveyance of the land against the Fuentes spouses before the RTC, claiming
that the sale to the spouses was void since Rosario, did not give her consent to it. Her signature on the
affidavit of consent had been forged.

The RTC ruled in favor of the petitioner sustaining the validity of the sale. On appeal, the CA reversed. The
CA found sufficient evidence of forgery, but since the Fuentes spouses were builders in good faith, they
were entitled under Article 448 to payment of the value of the improvements they introduced on the lot.

ISSUE:
Are the Fuentes entitled to the value of the improvements made on the land despite the sale being void?

HELD:
Yes, the Fuentes spouses entitled to the value of the improvements they made on the land despite the sale
being void. The sale was void from the beginning. Consequently, the land remained the property of Tarciano
and Rosario despite that sale. However, the Spouses Fuentes appear to have acted in good faith in entering
the land and building improvements on it. The Fuentes spouses had no reason to believe that Atty. Plagata
had violated his commission and his oath.

The owner of the land on which anything has been built, sown or planted in good faith, shall have the right
to appropriate as his own the works, sowing or planting, after payment of the indemnity provided for in
Articles 546 and 548, or to oblige the one who built or planted to pay the price of the land, and the one who
sowed, the proper rent. However, the builder or planter cannot be obliged to buy the land if its value is
considerably more than that of the building or trees. In such case, he shall pay reasonable rent, if the owner
of the land does not choose to appropriate the building or trees after proper indemnity. The parties shall
agree upon the terms of the lease and in case of disagreement, the court shall fix the terms thereof.

As possessor in good faith, the Fuentes spouses were under no obligation to pay for their stay on the
property prior to its legal interruption by a final judgment against them. What is more, they are entitled under
Article 448 to indemnity for the improvements they introduced into the property with a right of retention until
the reimbursement is made.

The Rocas shall of course have the option, pursuant to Article 546 of the Civil Code, of indemnifying the
Fuentes spouses for the costs of the improvements or paying the increase in value which the property may
have acquired by reason of such improvements.

84
OWNERSHIP
Right of Accession

THE RULE THAT ARTICLE 448 OF THE NEW CIVIL CODE IS INAPPLICABLE WHERE THERE IS A
CONTRACTUAL RELATION BETWEEN THE PARTIES ADMITS OF EXCEPTIONS

67. Communities Cagayan, Inc. v. Spouses Nanol


G.R. No. 176791, November 14, 2012
Del Castillo, J.

FACTS:
In this Petition for Review on Certiorari under Rule 45, Petitioner Communities Cagayan, Inc. (CCI) assails
the order of the RTC in ordering recovery of possession subject to its payment of the total monthly
installments and the value of the new house minus the cost of the original house to respondents Spouses
Arsenio and Angeles Nanol and anybody claiming rights under them (respondents).

Respondent spouses entered into a Contract to Sell with CCI whereby the latter agreed to sell the former a
house and two lots for the price of P368,000. The spouses obtained a loan, whereby the property was used
as the collateral, from Capitol Development Bank. Accordingly, the titles were transferred in the names of
the latter and submitted to Capitol Development Bank for loan processing. Unfortunately, the bank collapsed
and closed before it could release the loan.

The respondent spouses entered into another Contract to Sell with CCI over the same property and for the
same price. This time they availed of CCI’s in-house financing with the undertaking that the loan will be paid
for four years. In the interim, Arsenio Nanol demolished the original house and constructed a three-story
house allegedly valued at P3.5M. Arsenio died thereby leaving his wife to pay for monthly amortizations.
She eventually failed to pay the same, thus prompting CCI to send a Notice of Delinquency and Cancellation
of Contract to Sell. CCI filed with the RTC a Complaint for Cancellation of Title, Recovery of Possession,
Reconveyance and Damages against respondents. The RTC declared the Deed of Absolute Sale Void and
ordered the TCTs issued in favor of respondent spouses cancelled. Possession of the house and lots were
also ordered to be turned over to CCI subject to the latter’s payment of their total monthly installments and
the value of the new house minus the cost of the original house.

CCI seeks to delete the order of reimbursement on the ground that the respondent spouses were in bad
faith when they renovated and improved the house, which was not yet their own. On the other hand, Angeles
Nanol, instead of squarely answering the legal issues posited by CCI, maintained that the Deed of Absolute
Sale is valid.

ISSUE:
Should Article 448 of the New Civil Code be applied in the case at bar despite the existence of a Contract
to Sell between the parties?

HELD:
Yes, the instant case is an exception.

As a rule, Article 448 on builders on good faith does not apply where there is a contractual relation between
the parties. Jurisprudence provides that the Supreme Court had already construed good faith beyond its
limited definition. It is important to point out that the parties failed to attach a copy of the Contract to Sell
thus constraining the court to apply the said provision. Moreover, the determination on whether respondents
are indeed builders in good faith or in bad faith is a factual question and is thus beyond the scope of a
petition under Rule 45.

For its failure to elevate the case to the CA and in directly appealing the case to the Supreme Court, CCI is
deemed to have waived all factual issues. As such, the court is again constrained to rely on the presumption
of good faith on the part of respondents. In addition, good faith is presumed on the part of the respondents
and CCI failed to rebut it. There was also no evidence that CCI opposed or objected to the improvements
introduced by respondents. In light of the foregoing, Article 448 is applicable in this case.

85
OWNERSHIP
Right of Accession

THERE IS NO INCONSISTENCY THAT EXISTS BETWEEN THE FINDING OF GOOD FAITH ON THE
PART OF THE BUILDER AND THE RELIEFS GRANTED TO THE OWNER UNDER ARTICLE 448 OF
THE NEW CIVIL CODE

68. Angeles v. Pascual


G.R. No. 157150, September 21, 2011
Bersamin, J.

FACTS:
This is a petition for review on certiorari questioning the decision of the Court of Appeals.

Neighbors Regidor Pascual and Pedro Angeles were registered owners of adjacent parcels of land whereby
the former owned Lot 4 while the latter owned Lot 5. Each of them built a house on his respective lot,
believing all the while that his respective lot was properly delineated. It was not until Metropolitan Bank and
Trust Company (Metrobank), as the highest bidder in the foreclosure sale of the adjacent Lot 3, caused the
relocation survey of Lot 3 that the geodetic engineer discovered that Pascual’s house had encroached on
Lot 3. As a consequence, Metrobank successfully ejected Pascual. In turn, Pascual caused the relocation
survey of his own lot and discovered that Angeles had encroached on his lot. Pascual demanded rentals
for the use of the encroached area from Angeles, or the removal of Angeles’s house. Angeles refused the
demand. Before the RTC, Pascual sued Angeles for the recovery of possession and damages. Both the
RTC and the CA ruled in favor of Pascual. The CA, however, modified the ruling of the RTC by applying
Article 448 of the New Civil Code and ruling that Angeles is a builder in good faith. The CA laid down the
following options for Pascual – (a) to either buy the portion of Angeles’ house or (b) to sell to Angeles the
portion of his land that was occupied. Angeles argues that the said options were contrary to its finding of
good faith.

ISSUE:
Was the application of Article 448 of the New Civil Code by the Court of Appeals correct and proper?

HELD:
Yes, the appellate court correctly and properly applied the provision.

Article 448 contemplates a person building, or sowing, or planting in good faith on land owned by another.
The law presupposes that the land and the building or plants are owned by different persons. Good faith
consists in the belief of the builder that the land he is building on is his and in his ignorance of a defect or
flaw in his title.

With the unassailable finding that Angeles’ house straddled the lot of Pascual, and that Angeles had built
his house in good faith, Article 448 of the Civil Code, which spells out the rights and obligations of the owner
of the land as well as of the builder, is unquestionably applicable. Consequently, the land being the principal
and the building the accessory, preference is given to Pascual as the owner of the land to make the choice
as between appropriating the building or obliging Angeles as the builder to pay the value of the land.
Contrary to the insistence of Angeles, there is no inconsistency that exists between the finding of good faith
in his favor and the grant of reliefs set forth in Article 448.

86
OWNERSHIP
Right of Accession

INTRODUCTION OF VALUABLE IMPROVEMENTS ON THE LEASED PREMISES DOES NOT GIVE A


LESSEE THE RIGHT OF RETENTION AND REIMBURSEMENT WHICH RIGHTFULLY BELONG TO A
BUILDER IN GOOD FAITH

69. Sulo Sa Nayon, Inc. v. Nayong Pilipino Foundation


G.R. No. 170923, January 20, 2009
Puno, J.

FACTS:
Respondent Nayong Pilipino Foundation is the owner Nayong Pilipino Complex. Petitioner Philippine Village
Hotel, Inc., formerly called Sulo sa Nayon, Inc., is a domestic corporation. Respondent leased a portion of
the Nayong Pilipino Complex to petitioner for the construction and operation of a hotel building, to be known
as the Philippine Village Hotel. The lease contract was for an initial period of 21 years, or until May 1996
and may be renewed for another 25 years. On March 7, 1995, petitioner sent respondent a letter notifying
the latter of their intention to renew the contract for another 25 years, or until 2021. A voluntary addendum
to the Lease Agreement was executed, providing, in part, that the monthly rental shall be subject to an
increase of 20% at the end of every 3-year period. At the time of the renewal of the lease contract, the rent
already amounted to P725,780. Beginning 2001, petitioner defaulted in its payments thus pushing
respondent to demand the former to pay the arrears and vacate the premises. There is no question as to
the liability of petitioner to respondent as to the rentals. As computed by respondent, the arrears of petitioner
as of July 31, 2001 already amounted to P26,183,225.14. The problem lies with the improvements
introduced by the lessee i.e. a first-class hotel with complete facilities and other improvements.

Respondent filed a complaint for unlawful detainer before the MeTC which ruled in its favor. According to
the MeTC, the improvements made by a lessee on leased premises are not valid reasons for their retention
thereof. The court added that although the Contract of Lease stipulates that the building and all
improvements in the leased premises belong to herein petitioner, such will not defeat the right of herein
respondent to its property. Failure to pay rentals was in violation of their contract. It added that herein
petitioner can invoke Article 1678. Upon appeal, the RTC modified the ruling of the MeTC. The RTC
considered the elements of permanency of the construction and substantial value of the improvements as
well as the undisputed ownership over the land improvements of herein petitioner. The court applied Article
448, in relation to Article 546 of the New Civil Code. The CA ruled that the RTC erroneously applied the
rules on accession, as found in the said provisions. It stated that herein petitioner are mere lessees of the
subject premises and as such, cannot validly claim that they are builders in good faith in order to solicit the
application of the provisions in their favor.

ISSUE:
Should the rules on accession, as found in Articles 448 and 546 of the New Civil Code, apply to the instant
case?

HELD:
No, the provisions should not be applied.

A lessee is neither a builder in good faith or bad faith that would call for the application of Articles 448 and
546 of the New Civil Code. His rights are governed by Article 1678. The introduction of valuable
improvements on the leased premises does not give the lessee the right of retention and reimbursement
which rightfully belongs to a builder in good faith. Otherwise, such a situation would allow the lessee to
easily “improve” out the lessor out of its property.

Article 448 is manifestly intended to apply to a case where one build, plants, or sows on land in which he
believes himself to have a claim of title, and not to lands where the only interest of the builder, planter, or
sower is that of a holder, such as a tenant. In fact, as lessees, they recognize that the respondent is the
owner of the land. Petitioner cannot be a considered a builder in good faith just because they introduced
improvements to the land which are of substantial value. Such a situation would allow a lessee to easily
“improve” the lessor out of its property.

87
OWNERSHIP
Right of Accession

ARTICLE 448 OF THE NEW CIVIL CODE IS INAPPLICABLE IN CASES INVOLVING CONTRACTS OF
SALE WITH RIGHT OF REPURCHASE

70. Spouses Narvaez v. Spouses Alciso


G.R. No. 165907, July 27, 2009
Carpio, J.

FACTS:
This is a petition for review on certiorari under Rule 45 which seeks to challenge the decision of the CA
affirming with modification the decision of the RTC.

The land in controversy is a 1,329-square meter parcel of land situated in Pico, La Trinidad, Benguet. Larry
Ogas, the owner, sold the said property to his daughter, herein respondent, Rose Ogas Alciso. Respondent
entered into a Deed of Sale with Right to Repurchase, selling the property to Jaime Sansano. After
exercising her right to repurchase, respondent entered into another Deed of Absolute Sale, this time to
Celso Bate. Thereafter, Bate entered into a Deed of Sale of Realty, selling the property to Spouses
Dominador and Lilia Narvaez, herein petitioners. The Spouses Narvaez built a commercial building on the
property amounting to P300,000. Alciso demanded that a stipulation be included in the Deed of Sale of
Realty allowing her to repurchase the property from the Spouses Narvaez, to which Bate and the latter
complied. Alciso alleged that she informed Spouses Narvaez that she wanted to repurchase the property.
The spouses demanded Alciso to pay them the amount of P300,000 but the latter was only willing to pay
P150,000. The parties failed to reach an agreement on the repurchase price.

The Court of Appeals held that Article 448 is applicable, therefore, Alciso could either (a) appropriate the
commercial building after the payment of indemnity or (b) oblige the Spouses Narvaez to pay the price of
the land, unless the price was considerably more than that of the building. It further ruled that the Spouses
having built the improvement on the land they own and registered in their names, they are likened to builders
in good faith.

ISSUE:
Is Article 448 applicable in cases involving Contracts of Sale with a Right of Repurchase?

HELD:
No, it is not applicable

Article 448 does not apply to a case where the owner of the land is the builder, sower or planter who then
later loses ownership of the land by sale or donation. The provision applies only in cases where a person
constructs a building on the land of another in good or in bad faith, as the case may be. It does not apply to
a case where a person constructs a building on his own land, for then there can be no question as to the
good or bad faith on the part of the builder. Elsewise stated, where the true owner himself is the builder of
the works on his own land, the issue of good faith or bad faith is entirely irrelevant.

The Spouses Narvaez built the commercial building on the land that they own. Besides, to compel them to
buy the land, which they own, would be absurd. Alciso, however, may still exercise her right of redemption
by paying petitioners the price of the sale, the expenses of the contract, legitimate payments made by
reason of the sale, and the necessary and useful expenses made on the subject property.

88
OWNERSHIP
Right of Accession

IMMEDIATE REMOVAL OF THE STRUCTURES CANNOT BE GRANTED HENCE THE OWNER MUST
EXERCISE EITHER OF THE TWO OPTIONS PROVIDED IN ARTICLE 448 OF THE NEW CIVIL CODE

71. Department of Education v. Tuliao


G.R. No. 205664, June 9, 2014
Mendoza, J.

FACTS:
In this petition for review on certiorari under Rule 45, petitioner Department of Education (DepEd) assails
the decision of the CA dismissing its petition for review.

Respondent Mariano Tuliao filed an action for recovery of possession and removal of structure with
damages against DepEd with the MTCC. He alleged that he was the registered owner of the subject parcel
of land and that a portion thereof was allowed by his predecessors-in-interest to be used by the Atulayan
Elementary School (AES) as an access road for the school children in going to and from the school. Upon
discovering that a structure was being constructed on the land, he demanded that the DepEd cease and
desist and vacate the property but the refused. Tuliao likewise demanded payment for reasonable rent but
his demand was also ignored. The MTCC ruled in favor of Tuliao. With respect to the structures constructed,
the court ruled that Tuliao may exercise either of the two options provided in Article 448 of the New Civil
Code. The decision was affirmed in the RTC and the CA.

ISSUE:
Is the ruling of the MTCC, in ordering respondent Tuliao to exercise either of his two options provided in
Article 448, instead of granting his prayer for removal of structures, fair and proper?

HELD:
Yes, the MTCC was fair when it stated that it could not order the immediate removal of the structures and
directed Tuliao to exercise his option under Article 448.

Tuliao was the registered owner of the subject property and thus, had a right of action against the holder
and possessor of the said property. The Supreme Court remarked that the RTC, despite affirming the MTCC
decision, made an interesting suggestion. The RTC opined that that the case was impressed with public
interest and it was the paramount interest of the pupils who would be prejudiced by the finality and execution
of the appealed decision. It strongly suggested that the DepEd, or if unable, the City Government of
Tuguegarao City, be requested to pay Tuliao the just compensation of the land in question the amount of
which to be determined by a panel of three commissioners appointed by the court and whose determination
was to be approved by the said court. The Supreme Court added that if it would not be feasible or practical
for DepEd, its remedy is to file an action for expropriation.

89
OWNERSHIP
Right of Accession

TO BE DEEMED A BUILDER IN GOOD FAITH, IT IS ESSENTIAL THAT A PERSON ASSERTS TITLE


TO THE LAND ON WHICH HE BUILDS, I.E., THAT HE BE A POSSESSOR IN THE CONCEPT OF
OWNER, AND THAT HE BE UNAWARE THAT THERE EXISTS IN HIS TITLE OR MODE OF
ACQUISITION ANY FLAW WHICH INVALIDATES IT

72. Department of Education v. Casibang


G.R. No. 192268, January 27, 2016
Peralta, J.

FACTS:
The 7,532 square meter portion of Lot 115 registered under the name of Juan Cepeda, the respondents'
late father, is the property in controversy. Cepeda allowed the construction and operation of Solana North
Central School under the supervision of DepEd upon the request of Mayor Caronan. Cepeda died in 1983
but respondents and other descendants of Cepeda continued to tolerate the use and possession of the
property by the school.

Sometime between October 31, 2000 and November 2, 2000, the respondents entered and occupied a
portion of the property to which the school officials demanded for them to vacate the property but they
refused and asserted Cepeda's ownership of the lot.

DepEd filed a Complaint for Forcible Entry and Damages against respondents before the MCTC which ruled
in favor of the petitioner. On appeal, the RTC affirmed the decision of the MCTC. Responded demanded
the petitioner to either pay rent, purchase the area occupied, or vacate the premises but DepEd did not
heed the demand. Hence, on March 16, 2004, the respondents filed an action for Recovery of Possession
and/or Sum of Money against the DepEd. The latter alleged that it owned the subject property because it
was purchased by civic-minded residents of Solana, Cagayan from Cepeda and that contrary to
respondents' claim that the occupation is by mere tolerance, the property has always been occupied and
used adversely, peacefully, continuously and in the concept of owner for almost forty (40) years. It insisted
that respondents lost their right by laches.

RTC declared respondents as the owner thereof to which the CA affirmed. Hence, the present petition.

ISSUE:
Is DepEd is a builder in good faith despite being a possessor by mere tolerance?

HELD:
Yes, DepEd is a builder in good faith.

To be deemed a builder in good faith, it is essential that a person asserts title to the land on which he
builds, i.e., that he be a possessor in the concept of owner, and that he be unaware that there exists in his
title or mode of acquisition any flaw which invalidates it.

Despite being a possessor by mere tolerance, the DepEd is considered a builder in good faith, since Cepeda
permitted the construction of building and improvements to conduct classes on his property. Hence, Article
448 may be applied in the case at bar. Thus, the two options available to the respondents as landowners
are: (a) they may appropriate the improvements, after payment of indemnity representing the value of the
improvements introduced and the necessary and useful expenses defrayed on the subject lots; or (b) they
may oblige the DepEd to pay the price of the land. However, it is also provided under Article 448 that the
builder cannot be obliged to buy the land if its value is considerably more than that of the improvements and
buildings. If that is the case, the DepEd is not duty-bound to pay the price of the land should the value of
the same be considerably higher than the value of the improvement introduced by the DepEd on the subject
property. In which case, the law provides that the parties shall agree on the terms of the lease and, in case
of disagreement, the court shall fix the terms thereof.

90
OWNERSHIP
Right of Accession

THE BUILDER IN GOOD FAITH CAN COMPEL THE LANDOWNER TO MAKE A CHOICE BETWEEN
APPROPRIATING THE BUILDING BY PAYING THE PROPER INDEMNITY OR OBLIGING THE
BUILDER TO PAY THE PRICE OF THE LAND

73. Briones v. Macabagdal


G.R. No. 150666, August 3, 2010
Villarama, Jr., J.

FACTS:
This is a petition for review on certiorari under Rule 45 which seeks to reverse the decision of the CA
affirming the decision of the RTC ordering petitioners to remove the improvements they have made on the
disputed property or to pay respondent-spouses Jose and Fe Macabagdal the prevailing price of the land
as compensation.

Respondent-spouses purchased from Vergon Realty Investments Corporation (Vergon) Lot No. 2-R, a 325-
square-meter land located in Vergonville Subdivision No. 10 at Las Piñas City, Metro Manila. On the other
hand, petitioners are the owners of Lot No. 2-S, which is adjacent to Lot No. 2-R. Petitioners constructed a
house on Lot No. 2-R which they thought was Lot No. 2-S. After being informed of the mix up by Vergon’s
manager, respondent-spouses immediately demanded petitioners to demolish the house and vacate the
property. Petitioners, however, refused to heed their demand.

Respondent-spouses filed an action to recover ownership and possession of the said parcel of land with
the RTC of Makati City. Petitioners insisted that the lot on which they constructed their house was the lot
which was consistently pointed to them as theirs by Vergon’s agents over the seven (7)-year period they
were paying for the lot. They interposed the defense of being buyers in good faith and impleaded Vergon
as third-party defendant claiming that because of the warranty against eviction, they were entitled to
indemnity from Vergon in case the suit is decided against them. The RTC ruled in favor of respondent-
spouses and found that petitioners’ house was undoubtedly built on Lot No. 2-R. On appeal, the CA affirmed
the RTC’s finding.

ISSUE:
Are the petitioners considered as builders in good faith in constructing a house on Lot No. 2-R which they
thought was Lot No. 2-S?

HELD:
Yes, they are builders in good faith.

When a person builds in good faith on the land of another, Article 448 of the Civil Code governs, which
provides that “the owner of the land on which anything has been built, sown or planted in good faith, shall
have the right to appropriate as his own the works, sowing or planting, after payment of the indemnity
provided for in Articles 546 and 548, or to oblige the one who built or planted to pay the price of the land,
and the one who sowed, the proper rent. However, the builder or planter cannot be obliged to buy the land
if its value is considerably more than that of the building or trees. In such case, he shall pay reasonable
rent, if the owner of the land does not choose to appropriate the building or trees after proper indemnity.
The parties shall agree upon the terms of the lease and in case of disagreement, the court shall fix the terms
thereof.”

The builder in good faith can compel the landowner to make a choice between appropriating the building by
paying the proper indemnity or obliging the builder to pay the price of the land. The choice belongs to the
owner of the land, a rule that accords with the principle of accession, i.e., that the accessory follows the
principal and not the other way around. However, even as the option lies with the landowner, the grant to
him, nevertheless, is preclusive. He must choose one. He cannot, for instance, compel the owner of the
building to remove the building from the land without first exercising either option. It is only if the owner
chooses to sell his land, and the builder or planter fails to purchase it where its value is not more than the
value of the improvements, that the owner may remove the improvements from the land. The owner is
entitled to such remotion only when, after having chosen to sell his land, the other party fails to pay for the
same.

91
Therefore, the respondent-spouses have the option to appropriate the house on the subject land after
payment to petitioners of the appropriate indemnity or to oblige petitioners to pay the price of the land,
unless its value is considerably more than the value of the structures, in which case petitioners shall pay
reasonable rent.

92
OWNERSHIP
Right of Accession

ARTICLE 448 COVERS ONLY CASES IN WHICH THE BUILDERS, SOWERS OR PLANTERS BELIEVE
THEMSELVES TO BE THE OWNERS OF THE LAND OR, AT LEAST, TO HAVE A CLAIM OF TITLE
THERETO

74. Spouses Macasaet v. Spouses Macasaet


G.R. Nos. 154391-92, September 30, 2004
Panganiban, J.

FACTS:
This is a petition for review under Rule 45 assailing the decision of the CA in sustaining the finding of the
MTCC and RTC that petitioners Ismael and Teresita had been occupying the subject lots only by the
tolerance of respondents Vicente and Rosario. Thus, possession of the subject lots by petitioners became
illegal upon their receipt of respondents’ letter to vacate it.

Petitioners and Respondents are first-degree relatives. Ismael is the son of respondents, and Teresita is his
wife. On December 10, 1997, the respondents filed with the MTCC of Lipa City an ejectment suit against
their children. Respondents alleged that they were the owners of two parcels of land covered by TCT Nos.
T-78521 and T-103141, situated at Banay-banay, Lipa City; that by way of a verbal lease agreement,
petitioners occupied these lots in March 1992 and used them as their residence and the situs of their
construction business; and that despite repeated demands, petitioners failed to pay the agreed rental of
P500 per week.

Petitioners denied the existence of any verbal lease agreement. They claimed that respondents had invited
them to construct their residence and business on the subject lots in order that they could all live near one
other, employ Marivic (the sister of Ismael), and help in resolving the problems of the family. They added
that it was the policy of respondents to allot the land they owned as an advance grant of inheritance in favor
of their children. Thus, they contended that the lot covered by TCT No. T-103141 had been allotted to Ismael
as advance inheritance. On the other hand, the lot covered by TCT No. T-78521 was allegedly given to
petitioners as payment for construction materials used in the renovation of respondents house.

ISSUE:
Is Article 448 providing for the rights of a builder in good faith applicable if the possession of the lot is not
by mere tolerance but by invitation?

HELD:
Yes, Art. 448 is applicable.

When a person builds in good faith on the land of another, the applicable provision is Article 448, which
provides that “the owner of the land on which anything has been built, sown or planted in good faith, shall
have the right to appropriate as his own the works, sowing or planting, after payment of the indemnity
provided for in Articles 546 and 548, or to oblige the one who built or planted to pay the price of the land,
and the one who sowed, the proper rent. However, the builder or planter cannot be obliged to buy the land
if its value is considerably more than that of the building or trees. In such case, he shall pay reasonable
rent, if the owner of the land does not choose to appropriate the building or trees after proper indemnity.
The parties shall agree upon the terms of the lease and in case of disagreement, the court shall fix the terms
thereof.”

This Court has ruled that this provision covers only cases in which the builders, sowers or planters believe
themselves to be owners of the land or, at least, to have a claim of title thereto. It does not apply when the
interest is merely that of a holder, such as a mere tenant, agent, or usufructuary. From these
pronouncements, good faith is identified by the belief that the land is owned or that, by some title, one has
the right to build, plant, or sow thereon.

The established facts of this case show that respondents fully consented to the improvements introduced
by petitioners. In fact, because the children occupied the lots upon their invitation, the parents certainly
knew and approved of the construction of the improvements introduced thereon. Therefore, petitioners may
be deemed to have been in good faith when they built the structures on those lots.

93
OWNERSHIP
Right of Accession

ONE IS CONSIDERED IN GOOD FAITH IF HE IS NOT AWARE THAT THERE EXISTS IN HIS TITLE OR
MODE OF ACQUISITION ANY FLAW WHICH INVALIDATES IT

75. Spouses Aquino v. Spouses Aguilar


G.R. No. 182754, June 29, 2015
Sereno, C.J.

FACTS:
This is a petition for review on certiorari filed under Rule 45 of the Rules of Court wherein petitioner-spouses
Crispin and Teresa Aquino assails the decision of the CA modifying the decisions of both the MeTC and the
RTC ordering the respondents to vacate the property. The CA ruled that although respondent-spouses
Eusebio and Josefina Aguilar cannot be considered builders in good faith, they should still be reimbursed
for the improvements they have introduced on petitioners’ property.

Petitioner-spouses Aquino, are the owners of a house and lot. Since 1981, this property has been occupied
by Teresa's sister, respondent Josefina Vela Aguilar with her husband Eusebio and their family, who stayed
on the property with the consent and approval of petitioners. While respondents were in possession of the
property, the house previously constructed therein was demolished, and a three-storey building built in its
place. Respondents occupied half of the third floor of this new building for the 20 years without payment of
rental. On 2003, petitioners sent a letter to respondents demanding the surrender of the property.
Respondents failed to heed this demand, prompting petitioners to file a Complaint for ejectment against
them before the office of the barangay captain of Guadalupe Viejo. Failing to amicably settle the case,
petitioners filed a complaint with the MeTC praying for the respondents to vacate the portion of the building
they were occupying and to pay a reasonable amount for the use and enjoyment of the premises.

Respondents claimed that they had contributed to the improvement of the property and the construction of
the building, both in terms of money and management/supervision services. Petitioners purportedly agreed
to let them contribute to the costs of construction in exchange for the exclusive use of a portion of the
building. According to the appellate court, respondents were aware that their right to possess the property
had a limitation, because they were not the owners thereof. They knew that their occupation of the building
was by mere tolerance or permission of petitioners, who were the registered owners of the property.

ISSUE:
Can petitioners be considered as builders in good faith if they are aware that there is a limitation to their
right to possess the property?

HELD:
No, they are not builders in good faith.

The term "builder in good faith" as used in reference to Article 448 of the Civil Code, refers to one who, not
being the owner of the land, builds on that land believing himself to be its owner and unaware of the land,
builds on that land, believing himself to be its owner and unaware of the defect in his title or mode of
acquisition. The essence of good faith lies in an honest belief in the validity of one's right, ignorance of a
superior claim, and absence of intention to overreach another.

In this case, the Spouses Aguilar cannot be considered as builders in good faith on account of their
admission that the subject lot belonged to the Spouses Aquino when they constructed the building. At the
onset, petitioners were aware of a flaw in their title and a limit to their right to possess the property. By law,
one is considered in good faith if he is not aware that there exists in his title or mode of acquisition any flaw
which invalidates it.

Hence, they are not considered as builders in good faith.

94
OWNERSHIP
Right of Accession

PERSONS WHOSE OCCUPATION OF A REALTY IS BY SHEER TOLERANCE OF ITS OWNERS ARE


NOT POSSESSORS IN GOOD FAITH

76. Spouses Esmaquel v. Coprada


G.R. No. 152423, December 15, 2010
Peralta, J.

FACTS:
This is is a petition for review on certiorari under Rule 45 seeking to set aside the decision of the CA affirming
the ruling of the MCTC dismissing the case by reason of laches, which was reversed by the RTC ruling that
respondent's occupation of the subject property was by virtue of petitioners' tolerance and permission.

Petitioner-spouses Esmaquel are the registered owners of a parcel of land situated in Laguna. In 1945,
respondent Coprada was able to persuade the petitioners to allow her and her family to use and occupy the
land for their residence, under the condition that they will vacate the premises should petitioners need to
use the same. Respondent and her family were allowed to construct their residential house. Petitioners then
verbally demanded that respondent vacate the subject land, but the latter refused. Thus, petitioners sent a
letter to respondents demanding to vacate the subject premises. However, respondent still ignored said
demand prompting petitioners to bring a complaint before the barangay authorities. As no settlement was
reached, petitioners filed an ejectment case before the MCTC.

Respondent admitted that petitioners are the registered owners of the subject land. However, she averred
that in 1945, it was Emiliana Coprada, petitioner Victoria Sordevilla's mother and original owner of the
subject land, who gave permission to her late husband Brigido Coprada and his family o occupy the lot as
their permanent abode. Thereafter, Brigido and his family cleared the area and built therein a nipa hut to
dwell in. When Emiliana died, the ownership of the property was inherited by her only child, petitioner
Victoria Sordevilla. Respondent alleged that sometime in the early 1960s, petitioner Victoria offered the said
lot for sale to respondent, who readily agreed. The purchase price was fully paid in 1962 however the
agreement was never reduced to writing. After the sale, respondent built on the subject land a semi-concrete
structure. Respondent stated that petitioners' claim is barred by laches. Even granting, without admitting,
that respondent's claim of ownership over the property is improper because petitioners are the registered
owners thereof, respondent argued that she is a builder in good faith, because she was able to build the
structure on the subject lot with the prior permission of the owner.

ISSUE:
Is the respondent a possesor in good faith, hence, has the right to retain the property?

HELD:
No, respondent cannot be considered as a possesor in good faith, hence, no right to retain the property.

It is well settled that both Article 448 and Article 546 of the New Civil Code, which allow full reimbursement
of useful improvements and retention of the premises until reimbursement is made, apply only to a
possessor in good faith, i.e., one who builds on land with the belief that he is the owner thereof. Verily,
persons whose occupation of a realty is by sheer tolerance of its owners are not possessors in good faith.

In the present case, respondent failed to present evidence to substantiate her allegation that a portion of
the land was sold to her in 1962. In fact, when petitioners sent a letter to the respondent, demanding her to
vacate the subject property, the respondent, in reply to the said letter, never mentioned that she purchased
the subject land in 1962. Since respondent's occupation of the subject property was by mere tolerance, she
has no right to retain its possession under Article 448 of the Civil Code. She is aware that her tolerated
possession may be terminated any time and she cannot be considered as builder in good faith. At the time
respondent built the improvements on the premises in 1945, she knew that her possession was by mere
permission and tolerance of the petitioners.

Hence, respondent is not entitled to the retention of the land since she is deemed to be a builder in bad
faith.

95
OWNERSHIP
Right of Accession

ONE IS CONSIDERED IN GOOD FAITH IF HE IS NOT AWARE THAT THERE EXISTS IN HIS TITLE OR
MODE OF ACQUISITION ANY FLAW WHICH INVALIDATES IT

77. Heirs of Mariano v. City of Naga


G.R. No. 197743, March 12, 2018
Tijam, J.

FACTS:
This is a petition for review on certiorari assailing the amended decision rendered by the CA which
reconsidered its Decision, annulling the Decision of the RTC of Naga City and reinstating the decision of
the MTC of Naga City dismissing the ejectment case instituted by petitioners on the ground of lack of
jurisdiction.

In 1954, the officers of the City Heights Subdivision, wrote to Monico Imperial, the mayor of the City of Naga,
offering to construct the Naga City Hall within the premises of the Subdivision. The City's Municipal Board
subsequently passed Resolution No. 75 asking the Subdivision for a bigger area, hence, the Subdivision
amended its offer and agreed to donate five hectares of land registered in the names of Macario Mariano
and Jose A. Gimenez. Thereafter, the Municipal Board adopted Resolution No. 89 accepting the
Subdivision's offer of donation and its proposed contract.

According to the City, the City Mayor of Naga and the registered landowners, executed a Deed of Donation
of the parcel of land to be used as the City Hall site, public plaza and public market. By virtue of said Deed,
the City entered the property and began construction of the government center. It also declared the five-
hectare property in its name for tax purposes. Thereafter, LTO, NBI, DOLE, PPC, the Fire Department and
other government agencies and instrumentalities entered the same property and built their offices thereon.
In contrast, petitioners averred that the landowners' plan to donate five hectares to the City did not
materialize as the contract to build the City Hall was not awarded to the Subdivision. Petitioners claimed
that Macario and officers of the Subdivision met with Mayor Imperial to demand the return of the land as the
condition for the donation that the subdivision shall undertake the construction of the City Hall therein, was
not complied with, when the City eventually awarded the construction contract to a local contractor,
Francisco O. Sabaria. Mayor Imperial assured them that the City would buy the property from them,
however, it did not materialize. Macario then wrote to the Subdivision's General Manager Lopez Jr.,
demanding for the City's payment for the subject lot. When the City did not comply, petitioners, as the heirs,
filed a Complaint for unlawful detainer.

ISSUE:
Is the City of Naga considered a builder in good faith in constructing the government center when the Deed
of Donation was not executed?

HELD:
No, City of Naga is not considered a builder in good faith.

By law, one is considered in good faith if he is not aware that there exists in his title or mode of acquisition
any flaw which invalidates it. The essence of good faith lies in an honest belief in the validity of one's right,
ignorance of a superior claim, and absence of intention to overreach another.

In this case, it shows that the City knew of a substantial flaw in its claim over the subject property. The
proposed donation was conditioned on the award of the construction contract to the Subdivision. By its
Resolution No. 89, the City accepted the proposal with all its conditions. Thus, the City could not have been
unaware that by awarding the same construction contract to Sabaria, it no longer had any cause to continue
occupying the subject property as the condition for the proposed donation had not been satisfied.
Accordingly, it should have vacated the subject property. However, it stayed on and allowed Sabaria to
undertake the construction.

Therefore, it cannot be said that the City was of an honest belief that it had a valid right to the subject
property or that its actions had not overreached the landowners. Accordingly, it cannot be considered to
have acted in good faith.

96
OWNERSHIP
Right of Accession

ARTICLE 448 OF THE CIVIL CODE MAY APPLY EVEN WHEN THERE WAS CO-OWNERSHIP IF GOOD
FAITH HAS BEEN ESTABLISHED

78. Spouses del v. Abesia


G.R. No. L-49219, April 15, 1988
Gancayco, J.

FACTS:
In this appeal from the decision of the CFI of Cebu, certified to this Court by CA on account of the question
of law involved, the sole issue is the applicability of the provisions of Article 448 of the Civil Code relating to
a builder in good faith when the property involved is owned in common.

Plaintiff-spouses Del Campo and defendant Bernarda Abesia are co-owners pro indiviso of a parcel of land
in Cebu. An action for partition was filed by the plaintiffs. The houses of plaintiffs and defendants were
surveyed and shown on the sketch plan. The house of defendants occupied the portion with an area of 5
square meters of the land pertaining to plaintiffs.

Defendants argue that the rights of a builder in good faith under art. 448 of the New Civil Code should apply
to them with respect to that part of their house occupying a portion of the lot assigned to plaintiffs-appellees.

ISSUE:
Will Art. 448 apply when the part of the house of the defendants occupied a portion of the lot of the plaintiffs
after the termination of co-ownership?

HELD:
Yes. Art. 448 of the Civil Code may apply even when there was co-ownership, if good faith has been
established.

Article 448 of the Civil Code cannot apply where a co-owner builds, plants or sows on the land owned in
common for then he did not build, plant or sow upon land that exclusively belongs to another but of which
he is a co-owner. However, when, as in this case, the co-ownership is terminated by the partition and it
appears that the house of defendants overlaps or occupies a portion of 5 square meters of the land
pertaining to plaintiffs which the defendants obviously built in good faith, then the provisions of Article 448
of the new Civil Code should apply. Manresa and Navarro Amandi agree that the said provision of the Civil
Code may apply even when there was co-ownership if good faith has been established.

In the case at bar, the plaintiffs have the right to appropriate said portion of the house of defendants upon
payment of indemnity to defendants as provided for in Article 546 of the Civil Code. Otherwise, the plaintiffs
may oblige the defendants to pay the price of the land occupied by their house. However, if the price asked
for is considerably much more than the value of the portion of the house of defendants built thereon, then
the latter cannot be obliged to buy the land. The defendants shall then pay the reasonable rent to the
plaintiffs upon such terms and conditions that they may agree. In case of disagreement, the trial court shall
fix the terms thereof. Of course, defendants may demolish or remove the said portion of their house, at their
own expense, if they so decide.

97
OWNERSHIP
Right of Accession

THE APPLICATION BY ANALOGY OF THE RULES OF ACCESSION UNDER ARTICLE 447

79. Pacific Farms, Inc. v. Esguerra


G.R. No. L-21783, November 29,1969
Castro, J.

FACTS:
This is an appeal by the defendant-appellant Carried Lumber Company (Company) from the decision of the
CFI of Pangasinan, annulling the levy and certificate of sale covering six buildings owned by the plaintiff-
appellee Pacific Farms, Inc., to satisfy a money judgment against the Insular Farms, Inc., the plaintiff's
predecessor-in-interest over the buildings.

Defendant-appellee Company furnished lumber and construction materials to the Insular Farms, Inc. which
the latter used in the construction of the six buildings in its compound. Out of the total procurement price of
P15,000, the amount of P4,710.18 remains outstanding and unpaid. Thus, the Company instituted civil case
D-775 with the CFI of Pangasinan to recover the balance. The trial court sustained the Company’s claim
and thereafter issued a writ of execution.

Asserting absolute and exclusive ownership of the buildings in question, herein appellee Pacific Farms, Inc.
filed a complaint against the appellant Company and the sheriff with the court a quo, praying that judgment
be rendered, declaring null and void the levy and judicial sale of the six buildings. The trial court annulled
the levy and the Certificate of Sale.

ISSUE:
Will Article 447, which contemplates a principal and an accessory, apply by analogy in the case at bar with
respect to the construction of buildings through the use of materials owned by another person

HELD:
Yes. Article 447 applies by analogy in the case at bar.

Art. 447 of the New Civil Code contemplates a principal and an accessory, the land being considered the
principal, and the plantings, constructions or works, the accessory. The owner of the land who in good faith
- whether personally or through another - makes constructions or works thereon, using materials belonging
to somebody else, becomes the owner of the said materials with the obligation however of paying for their
value. The owner of the materials, on the other hand, is entitled to remove them, provided no substantial
injury is caused to the landowner. Otherwise, he has the right only to reimbursement for the value of his
materials.

The appellee claims that it owns the six buildings constructed out of the lumber and construction materials
furnished by the appellant is indubitable. Therefore, applying article 447 by analogy, we perforce consider
the buildings as the principal and the lumber and construction materials that went into their construction as
the accessory. Thus the appellee Pacific Farms, Inc., if it does own the six buildings, must bear the obligation
to pay for the value of the said materials. On the other hand, the appellant Company, which apparently has
no desire to remove the materials, and, even if it were minded to do so, cannot remove them without
necessarily damaging the buildings, has the corresponding right to recover the value of the unpaid lumber
and construction materials.

Therefore, the appellee who benefited from the lumber and materials that went into the construction of the
six buildings should shoulder the compensation due to the appellant as unpaid furnisher of materials.

98
OWNERSHIP
Right of Accession

THE PROVISION ON ARTICLES 448 AND 546 ON INDEMNITY MAY BE APPLIED BY ANALOGY

80. Pecson v. Court of Appeals


G.R. No. 115814, May 26, 1995
Davide, Jr., J.

FACTS:
This is a petition for review on certiorari which seeks to set aside the decision of the CA affirming in part the
order of the RTC citing Article 448 of the Civil Code, stating that private respondent opted to appropriate
the improvement introduced by petitioner on the subject lot, giving rise to the right of petitioner to be
reimbursed of the cost of constructing under Article 546, and of the right to retain the improvements until he
is reimbursed of the cost of the improvements

Petitioner Pedro P. Pecson was the owner of a commercial lot located in Kamias Street, Quezon City, on
which he built a four-door two-storey apartment building. For his failure to pay realty taxes amounting to
P12,000.00, the lot was sold at public auction by the City Treasurer of Quezon City to Mamerto Nepomuceno
who in turn sold it to the private respondents, the spouses Juan Nuguid and Erlinda Tan-Nuguid. Petitioner
challenged the validity of the auction sale. Both the CA and the trial court agreed that the apartment building
was not included in the auction sale of the commercial lot.

Thereafter, the private respondents moved for delivery of possession of the lot and the apartment building,
citing Article 546 of the Civil Code. The parties agree that the petitioner was a builder in good faith of the
apartment building on the theory that he constructed it at the time when he was still the owner of the lot.

ISSUE:
Will Articles 448 and 526 apply by analogy where the owner of the land is the builder, sower, or planter who
then later loses ownership of the land by sale or donation?

HELD:
Yes, Articles 448 and 526 apply by analogy in the case at bar.

By its clear language, Article 448 refers to a land whose ownership is claimed by two or more parties, one
of whom has built some works, or sown or planted something. The building, sowing or planting may have
been made in good faith or in bad faith. The rule on good faith laid down in Article 526 of the Civil Code
shall be applied in determining whether a builder, sower or planter had acted in good faith. Article 448 does
not apply to a case where the owner of the land is the builder, sower, or planter who then later loses
ownership of the land by sale or donation. Elsewise stated, where the true owner himself is the builder of
works on his own land, the issue of good faith or bad faith is entirely irrelevant. Thus, in strict point of law,
Article 448 is not apposite to the case at bar. Nevertheless, we believe that the provision therein on
indemnity may be applied by analogy considering that the primary intent of Article 448 is to avoid a state of
forced co-ownership.

Since the private respondents have opted to appropriate the apartment building, the petitioner is thus
entitled to the possession and enjoyment of the apartment building, until he is paid the proper indemnity, as
well as of the portion of the lot where the building has been constructed. This is so because the right to
retain the improvements while the corresponding indemnity is not paid implies the tenancy or possession in
fact of the land on which it is built, planted or sown. The petitioner not having been so paid, he was entitled
to retain ownership of the building and, necessarily, the income therefrom.

99
OWNERSHIP
Right of Accession

POSSESSION IN GOOD FAITH CONTINUES TO BE ENJOYED BY THE SUCCESSOR, UNTIL THE


CONTRARY IS PROVED

81. Tecnogas Philippines Manufacturing Corp. v. Court of Appeals


G.R. No. 108894, February 10, 1997
Panganiban, J.

FACTS:
This is an appeal from the decision of the CA that petitioner cannot be considered in good faith because as
a land owner, it is presumed to know the metes and bounds of his own property, especially if the same are
reflected in a properly issued certificate of title.

The parties in this case are owners of adjoining lots in Parañaque. When petitioner Tecnogas Philippines
Manufacturing Corporation purchased the land from Pariz Industries, the buildings and other structures
were already in existence. Private respondent Eduardo Uy was unaware that a portion of the building of
petitioner encroached into a portion of his property until after 1971 when he hired a surveyor, following his
purchase of another adjoining lot, to survey all his newly acquired lots. Upon being apprised of the
encroachment, petitioner immediately offered to buy the area occupied by its building but respondent
refused the offer.

Private respondent insists on the removal of the encroaching structures. Whereas, petitioner prays that
private respondent be ordered to sell the land.

ISSUE:
1) Can petitioner invoke good faith in order to enjoy the benefit granted under Article 448 even if it is
not the builder of the offending structures but possesses them as buyer.
2) What, then, is the benefit granted under Article 448?

HELD:
1. Yes, petitioner can invoke good faith, hence, may enjoy the beneift granted under Articles 448.

It is presumed that possession continues to be enjoyed in the same character in which it was acquired, until
the contrary is proved. Good faith consists in the belief of the builder that the land he is building on is his,
and his ignorance of any defect or flaw in his title. Article 527 of the Civil Code presumes good faith.

The record is not clear as to who actually built those structures, but it may well be assumed that petitioner’s
predecessor-in-interest, Pariz Industries, did so. Since no proof exists to show that the encroachment over
a narrow, needle-shaped portion of private respondent’s land was done in bad faith by the builder of the
encroaching structures, the latter should be presumed to have built them in good faith. Hence, such good
faith, by law, passed on to Pariz’s successor, petitioner in this case.

2. The obvious benefit to the builder under Article 448 is that, instead of being outrightly ejected from the
land, he can compel the landowner to make a choice between the two options: (1) to appropriate the building
by paying the indemnity required by law, or (2) sell the land to the builder. The landowner cannot refuse to
exercise either option and compel instead the owner of the building to remove it from the land.

In the present case, the private respondent’s insistence on the removal of the encroaching structures cannot
be granted. This is not one of the remedies bestowed upon him by law. It would be available only if and
when he chooses to compel the petitioner to buy the land at a reasonable price but the latter fails to pay
such price. This has not taken place. Hence, his options are limited to: (1) appropriating the encroaching
portion of petitioner’s building after payment of proper indemnity, or (2) obliging the latter to buy the lot
occupied by the structure. He cannot exercise a remedy of his own liking.

100
OWNERSHIP
Right of Accession

GOOD FAITH OF THE BUILDER IS DETERMINED BY HIS STATE OF MIND AT THE TIME HE BUILT
THE IMPROVEMENTS

82. Pleasantville Development Corp. v. Court of Appeals


G.R. No. 79688, February 1, 1996
Panganiban, J.

FACTS:
This is a petition for review on certiorari assailing the decision of the CA reversing the ruling of the RTC that
respondent Kee was a builder in bad faith.

Respondent Wilson Kee bought on installment Lot 8, Phase II of Pleasantville Subdivision from C.T. Torres
Enterprises, Inc. (CTTEI), the exclusive real estate agent of petitioner. CTTEI through its employee, Zenaida
Octaviano, accompanied Kee’s wife, Donabelle, to inspect Lot 8. Unfortunately, the parcel of land pointed
by Octaviano was Lot 9. Thereafter, respondent Kee proceeded to construct his residence, a store, an auto
repair shop and other improvements on the Lot 9, believing that it was his lot. After discovering that Lot 9
was occupied by Kee, Jardinico (owner of Lot 9) confronted him and demanded that the latter remove all
improvements and vacate Lot 9. Kee refused to vacate. Thus, a complaint for ejectment was filed against
him.

Petitioner contends that respondent Kee was a builder in bad faith.

ISSUE:
Is a lot buyer who constructs improvements on the wrong property erroneously delivered by the owner’s
agent, a builder in good faith?

HELD:
Yes. Respondent Kee is a builder in good faith.

Good faith consists in the belief of the builder that the land he is building on is his and his ignorance of any
defect or flaw in his title. And as good faith is presumed, petitioner has the burden of proving bad faith on
the part of Kee.

At the time he built improvements on Lot 8, Kee believed that said lot was what he bought from petitioner.
He was not aware that the lot delivered to him was not Lot 8. Upon Kee’s receipt of the map, his wife went
to the subdivision site accompanied by CTTEI’s employee, Octaviano, who authoritatively declared that the
land she was pointing to was indeed Lot 8. Having full faith and confidence in the reputation of CTTEI, and
because of the company’s positive identification of the property, Kee saw no reason to suspect that there
had been a misdelivery. Hence, respondent Kee is a builder in good faith as the petitioner failed to prove
otherwise.

101
OWNERSHIP
Right of Accession

LESSEES CANNOT BE CONSIDERED AS POSSESSORS OR BUILDERS IN GOOD FAITH. BEING


MERE LESSEES, THEY KNEW THAT THEIR POSSESSION WOULD CONTINUE ONLY FOR THE LIFE
OF THE LEASE

83. Geminiano v. Court of Appeals


G.R. No. 120303, July 24, 1996
Davide Jr., J.

FACTS:
This is a petition for review on certiorari assailing the decision of the CA affirming the RTC ruling ordering
the petitioners Geminiano to reimburse the private respondents of the value of the house in question and
other improvements and allowed the latter to retain the premises until reimbursement was made.

The lot in question was originally owned by the petitioners’ mother, Paulina Amado vda. de Geminiano. On
a 12-square-meter portion of that lot stood the petitioners’ unfinished bungalow, which the petitioners sold
to the private respondents Dominador and Mary Nicolas. Subsequently, the petitioners’ mother executed a
contract of lease over a 126 square-meter portion of the lot, including that portion on which the house stood,
in favor of the private respondents. The latter then introduced additional improvements and registered the
house in their names. The lot in question was the subject of a suit, which resulted in its acquisition previously
by one Lee, who sold the lot to Salcedo, who in turn sold it the spouses Dionisio. Subsequently, the lot was
registered in the petitioners’ name by virtue of a Deed of Quitclaim executed by spouses Dionisio. The
petitioners sent a letter to private respondents demanding that she vacate the premises and pay the rentals
in arrears. Upon failure to heed the demand, the petitioners filed with the MTCC a complaint for unlawful
detainer and damages. The MTCC ruled that Articles 448 and 546, which allow possessors in good faith to
recover the value of improvements and retain the premises until reimbursed, did not apply to lessees like
the private respondents, because the latter knew that their occupation of the premises would continue only
during the life of the lease.

The private respondents claim they are builders in good faith, hence, Article 448 should apply. They rely on
the lack of title of the petitioners' mother at the time of the execution of the contract of lease, as well as the
alleged assurance made by the petitioners that the lot on which the house stood would be sold to them.

ISSUE:
Can the private respondent, as mere lessees, be considered as builders in good faith?

HELD:
No, the private respondent, as mere lessees, cannot be considered as builders in good faith.

The private respondents, as lessees, who had undisturbed possession for the entire term under the lease,
are estopped to deny their landlord's title, or to assert a better title not only in themselves, but also in some
third person while they remain in possession of the leased premises and until they surrender possession to
the landlord. This estoppel applies even though the lessor had no title at the time the relation of lessor and
lessee was created, and may be asserted not only by the original lessor, but also by those who succeed to
his title.

It is undisputed that the private respondents came into possession of a 126 square-meter portion of the said
lot by virtue of a contract of lease executed by the petitioners' mother in their favor. The juridical relation
between the petitioners' mother as lessor, and the private respondents as lessees, is therefore well
established, and carries with it a recognition of the lessor's title. Therefore, being mere lessees, the private
respondents knew that their occupation of the premises would continue only for the life of the lease. Plainly,
they cannot be considered as possessors nor builders in good faith.

Also, there is no need to apply by analogy the provisions of Article 448 on indemnity as was done in Pecson
vs. Court of Appeals, because the situation sought to be avoided and which would justify the application of
that provision, is not present in this case. Suffice it to say, "a state of forced co-ownership" would not be
created between the petitioners and the private respondents. For, as correctly pointed out by the petitioners,
the rights of the private respondents as lessees are governed by Article 1678 of the Civil Code which allows
reimbursement to the extent of one-half of the value of the useful improvements.

102
OWNERSHIP
Right of Accession

ONE IS CONSIDERED IN GOOD FAITH IF HE IS NOT AWARE THAT THERE EXISTS IN HIS TITLE OR
MODE OF ACQUISITION ANY FLAW WHICH INVALIDATES IT

84. Heirs of Mariano v. City of Naga


G.R. No. 197743, March 12, 2018
Tijam, J.

FACTS:
This is a petition for review on certiorari assailing the amended decision rendered by the CA which
reconsidered its Decision, annulling the Decision of the RTC of Naga City and reinstating the decision of
the MTC of Naga City dismissing the ejectment case instituted by petitioners on the ground of lack of
jurisdiction.

In 1954, the officers of the City Heights Subdivision, wrote to Monico Imperial, the mayor of the City of Naga,
offering to construct the Naga City Hall within the premises of the Subdivision. The City's Municipal Board
subsequently passed Resolution No. 75 asking the Subdivision for a bigger area, hence, the Subdivision
amended its offer and agreed to donate five hectares of land registered in the names of Macario Mariano
and Jose A. Gimenez. Thereafter, the Municipal Board adopted Resolution No. 89 accepting the
Subdivision's offer of donation and its proposed contract.

According to the City, the City Mayor of Naga and the registered landowners, executed a Deed of Donation
of the parcel of land to be used as the City Hall site, public plaza and public market. By virtue of said Deed,
the City entered the property and began construction of the government center. It also declared the five-
hectare property in its name for tax purposes. Thereafter, LTO, NBI, DOLE, PPC, the Fire Department and
other government agencies and instrumentalities entered the same property and built their offices thereon.
In contrast, petitioners averred that the landowners' plan to donate five hectares to the City did not
materialize as the contract to build the City Hall was not awarded to the Subdivision. Petitioners claimed
that Macario and officers of the Subdivision met with Mayor Imperial to demand the return of the land as the
condition for the donation that the subdivision shall undertake the construction of the City Hall therein, was
not complied with, when the City eventually awarded the construction contract to a local contractor,
Francisco O. Sabaria. Mayor Imperial assured them that the City would buy the property from them,
however, it did not materialize. Macario then wrote to the Subdivision's General Manager Lopez Jr.,
demanding for the City's payment for the subject lot. When the City did not comply, petitioners, as the heirs,
filed a Complaint for unlawful detainer.

ISSUE:
Is the City of Naga considered a builder in good faith in constructing the government center when the Deed
of Donation was not executed?

HELD:
No, City of Naga is not considered a builder in good faith.

By law, one is considered in good faith if he is not aware that there exists in his title or mode of acquisition
any flaw which invalidates it. The essence of good faith lies in an honest belief in the validity of one's right,
ignorance of a superior claim, and absence of intention to overreach another.

In this case, it shows that the City knew of a substantial flaw in its claim over the subject property. The
proposed donation was conditioned on the award of the construction contract to the Subdivision. By its
Resolution No. 89, the City accepted the proposal with all its conditions. Thus, the City could not have been
unaware that by awarding the same construction contract to Sabaria, it no longer had any cause to continue
occupying the subject property as the condition for the proposed donation had not been satisfied.
Accordingly, it should have vacated the subject property. However, it stayed on and allowed Sabaria to
undertake the construction.

Therefore, it cannot be said that the City was of an honest belief that it had a valid right to the subject
property or that its actions had not overreached the landowners. Accordingly, it cannot be considered to
have acted in good faith.

103
OWNERSHIP
Right of Accession

AS A GENERAL RULE, ARTICLE 448 ON BUILDERS IN GOOD FAITH DOES NOT APPLY WHERE
THERE IS A CONTRACTUAL RELATION BETWEEN THE PARTIES

85. Leviste Management System, Inc. v. Legaspi Towers 200, Inc.


G.R. No. 199353, April 4, 2018
Leonardo-De Castro, J.

FACTS:
This is a petition under Rule 45, assailing the decision of the CA affirming the ruling of the RTC that Article
448 is applicable, thus, ordering respondent Legaspi Towers 200, Inc. (Legaspi Towers) to exercise its
option to appropriate the additional structure constructed by the petitioner Leviste Management System,
Inc. (LEMANS) or if the option to appropriate is not exercised, for the parties to agree upon the terms of the
lease.

Legaspi Towers is a condominium building located at Paseo de Roxas, Makati City consisting of seven (7)
floors, with a unit on the roof deck and two levels above said unit called Concession 2 and Concession 3.
The use and occupancy of the condominium building is governed by the Master Deed. Petitioner LEMANS
bought Concession 3 where he subsequently decided to build another unit called Concession 4 of which,
he was able to secure a building permit for its construction. Despite respondent Legaspi Corporation’s notice
that the construction of Concession 4 was illegal, petitioner LEMANS refused to stop its construction. Due
to this, respondent Legaspi Corporation forbade the entry of LEMANS’ construction materials to be used in
Concession 4 in the condominium. The petitioner LEMANS filed a complaint with the RTC, praying among
others that a writ of mandatory injunction be issued to allow the completion of the construction of Concession
4. The RTC issued the writ prayed for by Lemans.

ISSUE:
Is Article 448 on builders in good faith applicable where there is a contractual relation between the parties?

HELD:
No, as a general rule, Article 448 on builders in good faith does not apply where there is a contractual
relation between the parties.

Articles 448 and 546 of the Civil Code on builders in good faith are therefore inapplicable in cases covered
by the Condominium Act where the owner of the land and the builder are already bound by specific
legislation on the subject property (the Condominium Act), and by contract (the Master Deed and the By-
Laws of the condominium corporation). This Court has ruled that upon acquisition of a condominium unit,
the purchaser not only affixes his conformity to the sale; he also binds himself to a contract with other unit
owners.

In the case at bar, however, the land belongs to a condominium corporation, wherein the builder, as a unit
owner, is considered a stockholder or member in accordance with Section 10 of the Condominium Act. The
builder is therefore already in a co-ownership with other unit owners as members or stockholders of the
condominium corporation, whose legal relationship is governed by a special law, the Condominium Act.
Therefore, the application of Article 448 to the present situation is highly iniquitous, in that an owner, also
found to be in good faith, will be forced to either appropriate the illegal structure (and impliedly be burdened
with the cost of its demolition) or to allow the continuance of such an illegal structure that violates the law
and the Master Deed, and threatens the structural integrity of the condominium building upon the payment
of rent.

104
OWNERSHIP
Right of Accession

ACCRETIONS BELONG TO THE RIPARIAN OWNERS UPON WHOSE LANDS THE ALLUVIAL
DEPOSITS WERE MADE

86. Agustin v. Intermediate Appellate Court


G.R. Nos. 66075-76, July 5, 1990
Grino-Aquino, J.

FACTS:
This is a petition for review assailing the decision of the IAC affirming the decision of the RTC in awarding
the lot and its accretions to private respondents.

The Cagayan River separates the towns of Solana on the west and Tuguegarao on the east in the province
of Cagayan. In 1919 the lands east of the river were covered by the Tuguegarao Cadastre. In 1925, An
original certificate of title was issued for land east of the Cagayan River owned by petitioner Eulogio Agustin.
As the years went by, the Cagayan River moved gradually eastward, depositing silt on the western bank
continuing until1968. In 1950, all lands west of the river were included in the Solana Cadastre. Among these
occupying lands covered by the Solana Cadastre were private respondents. Through the years, the
Cagayan River eroded lands of the Tuguerarao Cadastre on its eastern bank among which was petitioner’s
lot depositing the alluvium as accretion on the land possessed by private respondent on the western bank.
However, in 1968, after a big flood, the Cagayan River changed its course, returned to its 1919 bed, and,
in the process, cut across the lands of private respondents whose lands were transferred on the eastern, or
Tuguegarao, side of the river. To cultivate those lots they had to cross the river. In April, 1969, while the
private respondents and their tenants were planting corn on their lots located on the eastern side of the
Cagayan River, the petitioners, accompanied by the mayor and some policemen of Tuguegarao, claimed
the same lands as their own and drove away the private respondents from the premises. Private
respondents filed a complaint to recover their lot and their accretions.

ISSUE:
1. Does the private respondent own the accretion?
2. If yes, does such ownership of private respondents over the accretion not affected by the sudden and
abrupt change in the course of the Cagayan River when it reverted to its old bed?

HELD:
Yes, the private respondents own the accretion and such ownership was not affected by the sudden and
abrupt change in the course of the Cagayan River when it reverted to its old bed.

1. Article 457 of the New Civil Code provides that “To the owners of lands adjoining the banks of rivers
belong the accretion which they gradually receive from the effects of the current of the waters.” Accretion
benefits a riparian owner when the following requisites are present: (1) that the deposit be gradual and
imperceptible; (2) that it resulted from the effects of the current of the water; and (3) that the land where
accretion takes place is adjacent to the bank of a river.

As found by the trial court, Cagayan River did move year by year from 1919 to 1968 or for a period of 49
years. Within this period, the alluviun deposited on the other side has become greater in area than the
original lands of the plaintiffs in both cases. Still the addition in every year is imperceptible in nature, one
could not discern it but can be measured after the lapse of a certain time. Also, the appellate court confirmed
that the accretion on the western bank of the Cagayan River had been going on from 1919 up to 1968 or
for a period of 49 years. It was gradual and imperceptible.

2. Articles 459 and 463 of the New Civil Code apply in the case at bar.

Art. 459. Whenever the current of a river, creek or torrent segregates from an estate on its bank a known
portion of land and transfers it to another estate, the owner of the land to which the segregated portion
belonged retains the ownership of it, provided that he removes the same within two years.

Art. 463. Whenever the current of a river divides itself into branches, leaving a piece of land or part thereof
isolated, the owner of the land retains his ownership. He also retains it if a portion of land is separated from
the estate by the current.

105
In the case at bar, the sudden change of course of the Cagayan River as a result of a strong typhoon in
1968 caused a portion of the lands of the private respondents to be separated from the estate by the current.
The private respondents have retained the ownership of the portion that was transferred by avulsion to the
other side of the river.

106
OWNERSHIP
Right of Accession

ACCRETION DOES NOT AUTOMATICALLY BECOME REGISTERED LAND JUST BECAUSE THE LOT
WHICH RECEIVES SUCH ACCRETION IS COVERED BY TORRENS TITLE

87. Cureg v. Intermediate Appellate Court


G.R. No. 73465, September 7, 1989
Medialdea, J.

FACTS:
In a petition for review under Rule 45 of the Rules of Court, petitioner assails the decision of the IAC
declaring private respondent the absolute owner of a parcel of land.

Private respondents Domingo Apostol and others filed a complaint for quieting of title and damages with
preliminary injunction against herein petitioners. The complaint alleged that private respondents are the
successors-in-interest of the late Francisco Gerardo, and that since time immemorial, together with his
predecessors-in-interest have been in actual, open, peaceful and continuous possession, under a bona fide
claim of ownership and adverse to all other claimants, of their motherland, situated in Cabagan, Isabela.
Said land was declared for taxation purposes in the name of Francisco Gerardo. Upon the latter’s death,
the ownership and possession of the motherland was succeeded by his only issue, Domingo Gerardo who,
together with three (3) legal or forced heirs, one of private respondents herein have also been in actual,
open, peaceful and continuous possession of the same. Thereafter, respondents verbally sold the
motherland to one of the co-respondents which verbal sale and conveyance was reduced into writing by the
vendors who executed an Extra-Judicial Partition with Voluntary Reconveyance. About the time of the
execution of the Extra-Judicial Partition, their motherland already showed/manifested signs of accretion of
about three (3) hectares on the north caused by the northward movement of the Cagayan River. On
September 15, 1982, private respondent subsequently declared the motherland and its accretion for tax
purposes. Private respondents also alleged that sometime about the last week of September and/or the first
week of October 1982, when they were about to cultivate their motherland together with its accretion, they
were prevented and threatened petitioners from continuing to do so. Petitioners' answer alleged that the
subject land is an accretion to their registered land and that petitioners have been in possession and
cultivation of the accretion for many years now.

ISSUE:
Does the subject land or “accretion” (which is bounded on the north by the Cagayan River) belong to the
private respondents and not to the petitioners when the petitioners’ Original Certificate of Title states clearly
that the petitioners’ land is bounded on its north by the Cagayan River?

HELD:
No, the subject land or “accretion” belong to the petioners.

The allegation of private respondents that they were in possession of the "motherland" through their
predecessors-in-interest had not been proved by substantial evidence. The assailed decision of the
respondent court, stated that since the "motherland" exists, it is also presumed that private respondents
were in possession of the "subject land" through their predecessors-in-interest since prior to July 26, 1894.
The trial court relied on the testimony one of the private respondents in this case, an interested and biased
witness, regarding their possession of the "motherland." The foregoing considerations indubitably show that
the alleged “motherland” claimed by private respondents is nonexistent. The “subject land” is an alluvial
deposit left by the northward movement of the Cagayan River and pursuant to Article 457 of the New Civil
Code: “To the owners of land adjoining the banks of river belong the accretion which they gradually receive
from the effects of the current of the waters.”

However, it should be noted that the area covered by OCT No. P-19093 is only four thousand five hundred
eighty four (4,584) square meters. The accretion attached to said land is approximately five and a half (5.5)
hectares. The increase in the area of petitioners’ land, being an accretion left by the change of course or
the northward movement of the Cagayan River does not automatically become registered land just because
the lot which receives such accretion is covered by a Torrens title. As such, it must also be placed under
the operation of the Torrens System.

107
OWNERSHIP
Right of Accession

REGISTRATION DOES NOT PROTECT THE RIPARIAN OWNER FROM ACCRETION

88. Viajar v. Court of Appelas


G.R. No. 77294, December 12, 1988
Medialdea, J.

FACTS:
This is a petition for review on certiorari of the decision of the CA affirming the decision of CFI declaring
defendant-appellees Leonor Ladrido, et. al. as owner of the parcel of land.

The spouses Ricardo Ladrido and Leonor Ladrido were the owners of Lot No. 7511 situated in Pototan,
Iloilo which contained an area of 154,267 square meters and was registered in the names of the spouses
under TCT No. T-21940. Spouses Rosendo Te and Ana Te were also the registered owners of a parcel of
land described in their title as Lot No. 7340 of the Cadastral Survey of Pototan. Spouses Te sold this lot to
plaintiffs-appelants Angelica Viajar and Celso Viajar. A Torrens title was later issued in their names. Later,
Angelica had Lot No. 7340 relocated and found out that the property was in the possession of Ricardo Y.
Ladrido. Consequently, she demanded its return but Ladrido refused. Plaintiffs-appellants instituted a civil
action for recovery of possession and damages against Ricardo Ladrido. The facts admitted by the parties
during the pre-trial show that the piece of real property which used to be Lot No. 7340 consisted of 20,089
square meters and that at the time of the cadastral survey in 1926, Lot No. 7511 and Lot No. 7340 were
separated by the Suague River.

Petitioners contend, Article 457 of the New Civil Code must be construed to limit the accretion mentioned
therein as accretion of unregistered land to the riparian owner, and should not extend to registered land.
Thus, the lot in question having remained the registered land of the petitioners, then the private respondents
cannot acquire title there in derogation to that of the petitioners, by accretion, for that will defeat the
indefeasibility of a Torrens Title.

ISSUE:
Does the registration under the Torrens system protects the riparian owner against the dimunition of area
of his land due to accretion?

HELD:
No, registration under the Torrens system protects the riparian owner against the dimunition of area of his
land due to accretion.

In Payatas Estate Improvement Co. vs. Tuason, the court ruled that Art. 366 provides that "any accretions
which the banks of rivers may gradually receive from the effects of the current belong to the owners of the
estates bordering thereon." Accretions of that character are natural incidents to land bordering on running
streams and are not affected by the registration laws. It follows that registration does not protect the riparian
owner against diminution of the area of his land through gradual changes in the course of the adjoining
stream. Also, in the case of C.N. Hodges vs. Garcia, the court ruled that “It clearly appearing that the land
in question has become part of defendant's estate as a result of accretion, it follows that said land now
belongs to him. The fact that the accretion to his land used to pertain to plaintiffs estate, which is covered
by a Torrens Certificate of Title, cannot preclude him (defendant) from being the owner thereof. Registration
does not protect the riparian owner against the diminution of the area of his land through gradual changes
in the course of the adjoining stream. Accretions which the banks of rivers may gradually receive from the
effect of the current become the property of the owners of the banks (Art. 366 of the Old Civil Code; Art.
457 of the New). Such accretions are natural incidents to land bordering on running streams and the
provisions of the Civil Code in that respect are not affected by the Registration Act.

The trial court found that the change in the course of the Suague River was gradual and this finding was
affirmed by the respondent Court of Appeals. The established facts indicate that the eastern boundary of
Lot No. 7511 was the Suague River based on the cadastral plan. For a period of more than 40 years (before
1940 to 1980) the Suague River overflowed its banks yearly and the property of the defendant gradually
received deposits of soil from the effects of the current of the river. The consequent increase in the area of
Lot No. 7511 due to alluvion or accretion was possessed by the defendants whose tenants plowed and

108
planted the same with corn and tobacco. The Court does not find any valid reason to disturb this finding of
fact.

109
OWNERSHIP
Right of Accession

ALLUVION MUST BE THE EXCLUSIVE WORK OF NATURE

89. Vda. de Nazareno v. Court of Appels


G.R. No. 98045, June 26, 1996
Romero, J.

FACTS:
This is a petition for review challenging the decision of the CA affirming ruling of the RTC in dismissing
complaint filed by the petitioners Desamparado Vda. de Nazareno and Leticia Nazareno Tapia.

A parcel of land situated in Cagayan de Oro City is said to have been formed as a result of sawdust dumped
into the dried-up Balacanas Creek and along the banks of the Cagayan River. Private respondents Jose
Salasalan and Leo Rabaya leased the subject lots on which their houses stood from one Antonio Nazareno,
petitioners’ predecessor-in-interest. Private respondents allegedly stopped paying rentals.

Before he died, Antonio Nazareno caused the approval by the Bureau of Lands of the survey plan
designated with a view to perfecting his title over the accretion area being claimed by him. Before the
approved survey plan could be released to the applicant, the private respondents protested it before the
Bureau of Lands. The report of the Land Investigator, made in compliance with the order of the District Land
Officer, recommended the Survey Plan, in the name of Antonio Nazareno, be cancelled and that private
respondents be directed to file appropriate public application. Based on the report, the Regional Director of
the Bureau of Lands rendered a decision ordering an amendment to the survey plan of Nazareno by
segregating therefrom the areas occupied by the private respondents. Antonio Nazareno filed a motion for
reconsideration with the Undersecretary of Natural Resources and OIC of the Bureau of Lands, which was
denied. Petitioners claim that the subject land is private land being an accretion to his titled property,
applying Article 457 of the Civil Code.

ISSUE:
Can the subject land formed as a result of sawdust dumped into the dried-up Balacanas Creek and along
the banks of the Cagayan river be considered as accretion belonging to the petitioner?

HELD:
No, the subject land cannot be considered as an accretion belonging to the petitioner.

Article 457 of the Civil Code provides that “to the owners of land adjoining the banks of rivers belong the
accretion which they gradually receive from the effects of the current of the waters.” In the case of Meneses
v. CA, it was held that accretion, as a mode of acquiring property under Article 457 of the Civil Code, requires
the concurrence of these requisites: (1) that the deposition of soil or sediment be gradual and imperceptible;
(2) that it be the result of the action of the waters of the river (or sea); and (3) that the land where accretion
takes place is adjacent to the banks or rivers (or the sea coast). These are called the rules on alluvion which
if present in a case, give to the owners of lands adjoining the banks of rivers or streams any accretion
gradually received from the effects of the current of waters.

In Hilario v. City of Manila, it was held that the word “current” indicate the participation of the body of water
in the ebb and flow of waters due to high and low tide. And, in Republic v. CA, it was ruled that the
requirement that the deposit should due to the effect of the current of the river is indispensable. This
excludes from Article 457 of the Civil Code all deposits caused by human intervention. Putting it differently,
alluvion must be the exclusive work of nature. Thus, in Tiongco v. Director of Lands, et al., where the land
was not formed solely by the natural effect of the water current of the river bordering said land but is also
the consequence of the direct and deliberate intervention of man, it was deemed a man-made accretion
and, as such, part of the public domain.

The application of the rules on alluvion cannot be made in the present case as the first and second
requirements of the rules were not met. The subject land was the direct result of the dumping of sawdust
by the Sun Valley Lumber Co. consequent to its sawmill operations. By the petitioners’ own admission, the
accretion was formed by the dumping of boulders, soil and other filling materials on portions of the
Balacanas Creek and the Cagayan River bounding their land. It cannot be claimed, therefore, that the
accumulation was gradual and imperceptible, resulting from the action of the waters or the current of the

110
Balacanas Creek and the Cagayan River. As the accretion site was the result of the late Antonio Nazareno’s
labor, the same would still be part of the public domain.

111
OWNERSHIP
Right of Accession

ACCRETION IS NOT APPLICABLE ON SEA BANK

90. Heirs of Navarro v. Intermediate Appellate Court


G.R. No. 68166, February 12, 1997
Hermosisima, Jr., J.

FACTS:
This is a petition assaling the decision of IAC in reversing the ruling of the lower court that the subject
property is a foreshore land and, being a part of the public domain, it cannot be the subject of land
registration proceedings.

Sinforoso Pascual filed an application for foreshore lease covering a tract of foreshore land in Bataan. His
registered property is bounded on the east by the Talisay River, on the west by the Bulacan River, and on
the north by the Manila Bay. The Talisay River and the Bulacan River flow down towards the Manila Bay
and act as boundaries of the applicant's registered land on the east and on the west. This application was
denied. Subsequently, petitioners' predecessor-in-interest, also now deceased, Emiliano Navarro, filed a
fishpond application with the Bureau of Fisheries covering twenty five (25) hectares of foreshore land also
in Bataan. Initially the application was denied, eventually however the grant was given. Pascual claimed
that this land is an accretion to his property, The Talisay River as well as the Bulacan River flow downstream
and meet at the Manila Bay thereby depositing sand and silt on his property resulting in an accretion thereon.
He claimed the accretion as the riparian owner.

The Director of Lands, represented by the Assistant Solicitor General, filed an opposition thereto stating
that neither Pascual nor his predecessors-in-interest possessed sufficient title to the subject property, the
same being a portion of the public domain and, therefore, it belongs to the Republic of the Philippines. On
the other hand, anchoring their claim of ownership on Article 457 of the Civil Code, petitioners vigorously
argue that the disputed 14-hectare land is an accretion caused by the joint action of the Talisay and Bulacan
Rivers which run their course on the eastern and western boundaries, respectively, of petitioners' own tract
of land.

ISSUE:
Can the petitioners rightfully claim the accretion on a sea bank under the principle of accretion?

HELD:
No, the petitioner’s cannot claim the accretion on a sea bank under the principle of accretion.

The principle of accretion is only applicable to owners whose estates are adjacent to rivers as stated in
Article 457 of the Civil Code. The disputed land is an accretion not on a river bank but on a sea bank, or on
what used to be the foreshore of Manila Bay which adjoined petitioners' own tract of land on the northern
side. As such, the applicable law is not Article 457 of to Civil Code but Article 4 of the Spanish Law of Waters
of 1866.

The disputed property is an accretion on a sea bank, Manila Bay being an inlet or an arm of the sea; as
such, the disputed property is, under Article 4 of the Spanish Law of Waters of 1866, part of the public
domain. As part of the public domain, the herein disputed land is intended for public uses, and "so long as
the land in litigation belongs to the national domain and is reserved for public uses, it is not capable of being
appropriated by any private person, except through express authorization granted in due form by a
competent authority. Only the executive and possibly the legislative departments have the right and the
power to make the declaration that the lands so gained by action of the sea is no longer necessary for
purposes of public utility or for the cause of establishment of special industries or for coast guard services.

Petitioners utterly fail to show that either the executive or legislative department has already declared the
disputed land as qualified, under Article 4 of the Spanish Law of Waters of 1866, to be the property of
petitioners as owners of the estates adjacent thereto.

112
OWNERSHIP
Right of Accession

DRYING UP OF RIVER IS NOT ACCRETION HENCE BELONGS TO THE STATE

91. Republic v. Santos III


G.R. No. 160453, November 12, 2012
Bersamin, J.

FACTS:
This is an appeal assailing the decision of the CA affirming the ruling of the RTC in declaring the respondents
Arcadio Ivan Santos III and Arcadio Santos, Jr. as true and absolute owner of the subject land which was
previously a part of the Parañaque River which became an orchard after it dried up.

Alleging continuous and adverse possession of more than ten years, respondent Arcadio Ivan Santos III
applied for the registration of Lot 4998-B RTC in Parañaque City. The property was located in Barangay
San Dionisio, Parañaque City, and was bounded in the Northeast by Lot 4079 belonging to respondent
Arcadio Santos, Jr. (Arcadio, Jr.), in the Southeast by the Parañaque River, in the Southwest by an
abandoned road, and in the Northwest by Lot 4998-A also owned by Arcadio Ivan. Respondent Arcadio Ivan
amended his application for land registration to include Arcadio, Jr. as his co-applicant because of the
latter’s co-ownership of the property. He alleged that the property had been formed through accretion and
had been in their joint open, notorious, public, continuous and adverse possession for more than 30 years.

The City of Parañaque (the City) opposed the application for land registration, stating that it needed the
property for its flood control program; that the property was within the legal easement of 20 meters from the
river bank; and that assuming that the property was not covered by the legal easement, title to the property
could not be registered in favor of the applicants for the reason that the property was an orchard that had
dried up and had not resulted from accretion.

ISSUE:
Can the subject land, which was previously a part of the Parañaque River that became an orchard after it
dried up, be acquired through the process of accretion?

HELD:
No, the subject land, which was a dried-up riverbed cannot be acquired through the process of accretion

By law, accretion, the gradual and imperceptible deposit made through the effects of the current of the
water, belongs to the owner of the land adjacent to the banks of rivers where it forms. The drying up of the
river is not accretion. Hence, the dried-up riverbed belongs to the State as property of public dominion, not
to the riparian owner, unless a law vests the ownership in some other person.

The RTC and the CA grossly erred in treating the dried-up riverbed as an accretion that became
respondents' property pursuant to Article 457 of the Civil Code. That land was definitely not an accretion.
The process of drying up of a river to form dry land involved the recession of the water level from the river
banks, and the dried-up land did not equate to accretion, which was the gradual and imperceptible
deposition of soil on the river banks through the effects of the current. In accretion, the water level did not
recede and was more or less maintained. Hence, respondents as the riparian owners had no legal right to
claim ownership of Lot 4998-B. Considering that the clear and categorical language of Article 457 of the
Civil Code has confined the provision only to accretion, we should apply the provision as its clear and
categorical language tells us to. The State exclusively owned Lot 4998-B and may not be divested of its
right of ownership. Article 502 of the Civil Code expressly declares that rivers and their natural beds are
public dominion of the State. It follows that the river beds that dry up, like Lot 4998-B, continue to belong to
the State as its property of public dominion, unless there is an express law that provides that the dried-up
river beds should belong to some other person.

113
OWNERSHIP
Right of Accession

ALLUVION MUST BE THE EXCLUSIVE WORK OF NATURE

92. Daclison v. Baytion


G.R. No. 219811, April 6, 2016
Mendoza, J.

FACTS:
Respondent Baytion filed a complaint for Forcible Entry and Damages with Prayer for Issuance of
Preliminary Mandatory Injunction against petitioner Daclison. Respondent alleged that he was a co-owner
of a parcel of land consisting of 1,500 sq.m where he inherited and his siblings from their parents, and as
agreed upon, it was to be administered by him. As administrator, he leased portions of the property to third
persons. Erected on the property was a one-story building divided into seven units or stalls. One stall was
occupied by Leonida Dela Cruz who used it for selling rocks, pebbles and similar construction materials.

According to Baytion, Leonida’s lease expired. Daclison took possession of the portion leased and occupied
by Leonida without the prior knowledge and consent of Baytion. Oral and written demands to vacate were
given. Daclison refused prompting Baytion to file the abovementioned complaint.

According to Daclison, Baytion leased the subject portion to Antonio Dela Cruz where the latter started a
business. 10 or 15 years later, a stone walling called a riprap, was erected at the creek lying beside Baytion’s
property, leaving a deep down-sloping area; and that Antonio negotiated with an engineer so he could be
in possession of the said down-slope; and that Antonio had it filled up until it was leveled with the lease
portion and he paid for the right to possess the same.

Both the RTC and the C.A. ruled in favor of respondent. Baytion basically posits that although the disputed
portion is outside the description of the property covered by TCT No. 221507, it forms an integral part of the
latter because it is an accretion, construction, or improvement on the property and, under the law, any
accretion or anything built thereon belongs to him and his co-owners.

ISSUE:
Is the constructed down-slope an accretion?

HELD:
No, Baytion’s contention that he owns that portion by reason of accretion is misplaced. Article 457 of the
New Civil Code provides that to the owners of lands adjoining the banks of rivers belongs the accretion
which they gradually receive from the effects of the current of the waters.

The following requisites must concur in order for an accretion to be considered (1) that the deposit be
gradual and imperceptible; (2) that it be made through the effects of the current of the water; and,
(3) that the land where accretion takes place is adjacent to the banks of rivers.

The contested portion cannot be considered an accretion. To begin with, the land came about not by reason
of a gradual and imperceptible deposit. The deposits were artificial and man-made and not the exclusive
result of the current from the creek adjacent to his property. Baytion failed to prove the attendance of the
indispensable requirement that the deposit was due to the effect of the current of the river or creek. Alluvion
must be the exclusive work of nature and not a result of human intervention.

114
OWNERSHIP
Right of Accession

ALLUVIAL DEPOSITS ALONG THE BANKS OF A CREEK OR A RIVER AUTOMATICALLY BELONGS


TO THE OWNER OF THE ESTATE TO WHICH IT MAY HAVE BEEN ADDED

93. Heirs of Narvasa, Sr. v. Imbornal


G.R. No. 182908, August 6, 2014
Perlas-Bernabe, J.

FACTS:
This is petition for review on certiorari of the Decision of the CA which reversed and set aside the Decision
of the RTC declaring (a) the descendants of Ciriaco Abrio as the exclusive owners of the Motherland (b) the
descendants of respondent Victoriano Imbornal (respondent Victoriano) as the exclusive owners of the first
accretion (First Accretion) and (c) the descendants of Pablo Imbornal (Pablo) as the exclusive owners of
the second accretion (Second Accretion).

Basilia Imbornal (Basilia) had four (4) children, namely, Alejandra, Balbina, Catalina, and Pablo. Petitioners
are the heirs and successors-in-interest of Francisco, son of Alejandra. On the other hand, respondents
Victoriano et al., all surnamed Imbornal, are the descendants of Pablo. During her lifetime, Basilia owned a
parcel of land which she conveyed to her three (3) daughters. Meanwhile, Catalina’s husband, Ciriaco Abrio
(Ciriaco), applied for and was granted a homestead patent over a 31,367-sq.m. riparian land (Motherland)
and was later issued a certificate of title over the said land. Ciriaco and his heirs had since occupied the
northern portion of the Motherland, while respondents occupied the southern portion. Sometime in 1949,
the First Accretion adjoined the southern portion of the Motherland. A certificate of title was issued in the
name of respondent Victoriano covering the First Accretion. In 1971, the Second Accretion, more or less,
abutted the First Accretion on its southern portion. A certificate of title was issued in the names of all the
respondents covering the Second Accretion.

Claiming rights over the entire Motherland, petitioners filed a complaint for reconveyance. They anchored
their claim on the allegation that Ciriaco, with the help of his wife Catalina, urged Balbina and Alejandra to
sell the Sabangan property, and that Ciriaco used the proceeds therefrom to fund his then pending
homestead patent application over the Motherland. In return, Ciriaco agreed that once his homestead patent
is approved, he will be deemed to be holding the Motherland — which now included both accretions — in
trust for the Imbornal sisters. In other words, petitioners are alleging that Ciriaco agreed to hold the same
in trust for their predecessors-in-interest Alejandra and Balbina upon issuance of the title in his name.

ISSUE:
Are the petitioners entitled to the Motherland and by extension the First and Second Accretion?

HELD:
No. The burden of proving the existence of a trust is on the party asserting its existence, and such proof
must be clear and satisfactorily show the existence of the trust and its elements. In this case, it cannot be
said, merely on the basis of the oral evidence offered by petitioners that the Motherland had been either
mistakenly or fraudulently registered in favor of Ciriaco. As the CA had aptly pointed out, a homestead
patent award requires proof that the applicant meets the stringent conditions set forth under Commonwealth
Act No. 141, as amended, which includes actual possession, cultivation, and improvement of the
homestead. It must be presumed, therefore, that Ciriaco underwent the rigid process and duly satisfied the
strict conditions necessary for the grant of his homestead patent application. As such, it is highly implausible
that the Motherland had been acquired and registered by mistake or through fraud as would create an
implied trust between the Imbornal sisters and Ciriaco. Hence, when an OCT covering the Motherland was
issued in his name pursuant to a Homestead Patent, Ciriaco’s title to the Motherland had become
indefeasible.

Article 457 of the Civil Code states the rule on accretion as follows: To the owners of lands adjoining the
banks of rivers belong the accretion which they gradually receive from the effects of the current of the
waters. Accordingly, therefore, alluvial deposits along the banks of a creek or a river do not form part of the
public domain as the alluvial property automatically belongs to the owner of the estate to which it may have
been added. The only restriction provided for by law is that the owner of the adjoining property must register
the same under the Torrens system; otherwise, the alluvial property may be subject to acquisition through
prescription by third persons.

115
In this case, petitioners are not the riparian owners of the Motherland to which the First Accretion had
attached, hence, they cannot assert ownership over the First Accretion. Consequently, as the Second
Accretion had merely attached to the First Accretion, they also have no right over the Second Accretion.
Neither were they able to show that they acquired these properties through prescription as it was not
established that they were in possession of any of them. Therefore, whether through accretion or,
independently, through prescription, the discernible conclusion is that petitioners’ claim of title over the First
and Second Accretions had not been substantiated, and, as a result, said properties cannot be reconveyed
in their favor.

116
OWNERSHIP
Quieting of Title

A FREE PATENT ISSUED OVER A PRIVATE LAND IS NULL AND VOID AND PRODUCES NO LEGAL
EFFECTS WHATSOEVER

94. Heirs of Tappa v. Heirs of Bacud


G.R. No. 187633, August 4, 2016
Jardeleza, J.

FACTS:
This is a Petition for Review on Certiorari under Rule 45 of the Revised Rules of Court assailing the Decision
of the CA which reversed and set aside the Decision of the RTC declaring the petitioners to be the absolute
owners of the parcel of land.

Petitioners Delfin Tappa (Delfin) and Maria Tappa (Spouses Tappa) filed a complaint for Quieting of Title,
Recovery of Possession and Damages (Complaint) against respondents Jose Bacud (Bacud), Henry
Calabazaron (Calabazaron), and Vicente Malupeng (Malupeng). In their complaint, Petitoners alleged that
they are the registered owners of the parcel of land, that they inherited the parcel of land from their father,
Lorenzo Tappa (Lorenzo). In their Answer, respondents claimed that the original owner of the parcel of land
was Genaro Tappa (Genaro) who had two children, Lorenzo and Irene. Upon Genaro’s death, the property
passed on to Lorenzo and Irene by operation of law; and they became ipso facto co-owners of the property.

Respondents presented before the RTC a joint affidavit (1963 Affidavit) signed by petitioner. The 1963
affidavit stated that Genaro originally owned Lot No. 3341. It further stated that one-half (1/2) of the property
was owned by Lorenzo; but that the whole property was declared as his, only for taxation purposes.

The RTC ruled in favor of the petitioners declaring that there was no document in the hands of respondents
can overcome the title of petitioner. The CA, however, ruled that the two indispensable requisites for an
action to quiet title under Articles 476 and 477 of the Civil Code were not met.

ISSUE:
Did the petitioners meet the requisites needed for an action for quieting of title to proser?

HELD:
No, for an action to quiet title to prosper, two indispensable requisites must concur, namely: (1) the plaintiff
or complainant has a legal or an equitable title to or interest in the real property subject of the action; and
(2) the deed, claim, encumbrance or proceeding claimed to be casting cloud on his title must be shown to
be in fact invalid or inoperative despite its prima facie appearance of validity or legal efficacy. Petitioners
failed to meet these two requisites.

First, Spouses Tappa’s claim of legal title over the parcel of land by virtue of the free patent and the
certificate of title issued in their name cannot stand. At the time of the application for free patent, the parcel
of land had already become private land by virtue of the open, continuous, exclusive, and notorious
possession by respondents. Hence, such parcel of land had been removed from the coverage of the Public
Land Act which governs public patent applications.

The settled rule is that a free patent issued over a private land is null and void, and produces no legal effects,
whatsoever. Private ownership of land — as when there is a prima facie proof of ownership like a duly
registered possessory information or a clear showing of open, continuous, exclusive, and notorious
possession, by present or previous occupants — is not affected by the issuance of a free patent over the
same land, because the Public Land Law applies only to lands of the public domain. The Director of Lands
has no authority to grant free patent to lands that have ceased to be public in character and have passed
to private ownership.

The second requisite for an action to quiet title is likewise wanting. A cloud on a title exists when (1) there
is an instrument (deed, or contract) or record or claim or encumbrance or proceeding; (2) which is apparently
valid or effective; (3) but is, in truth and in fact, invalid, ineffective, voidable, or unenforceable or extinguished
(or terminated) or barred by extinctive prescription; and (4) and may be prejudicial to the title.

117
The 1963 Affidavit is no doubt an instrument, which appears to be valid. It is dated and appears to be
executed and signed by petitioner. It is also notarized by a public notary. It states that Genaro originally
owns the land described, and that one-half (1/2) of which is actually owned by Irene as a coheir. This is
contrary to the claim of petitioner that the property was solely Lorenzo’s. Respondents’ argue that this
affidavit evidences the title of their predecessor-in-interest over Lot No. 3341 and effectively, theirs.

The 1963 Affidavit however, was not proven to be, in fact, invalid, ineffective, voidable or unenforceable, or
extinguished (or terminated) or barred by extinctive prescription. The CA pointed out that, aside from the
testimony of petitioner, no other evidence was presented to prove the claim of force and intimidation, hence,
it is at most, self-serving. Also, the 1963 Affidavit was duly notarized and, as such, is considered a public
document, and enjoys the presumption of validity as to its authenticity and due execution.

118
OWNERSHIP
Quieting of Title

AN ORDINARY CIVIL ACTION FOR DECLARATION OF NULLITY OF FREE PATENTS AND


CERTIFICATES OF TITLE IS NOT THE SAME AS AN ACTION FOR REVERSION

95. Spouses Galang v. Spouses Reyes


G.R. No. 184746, August 15, 2012
Mendoza, J.

FACTS:
This petition for review on certiorari under Rule 45 seeks to reverse and set aside the April 9, 2008 Decision
of the (CA) reversing the decision of the RTC dismissing the case of the complainant.

The respondents Reyeses alleged that they owned two properties: (1) a subdivision project known as
Ponderosa Heights Subdivision (Ponderosa), and (2) an adjoining property.; that the properties were
separated by the Marigman Creek, which dried up sometime in 1980 when it changed its course and passed
through Ponderosa; that the Petitioners Galangs, by employing manipulation and fraud, were able to obtain
a certificate of title over the dried up creek bed from the Department of Environment and Natural Resources
(DENR), through its Provincial Office (PENRO). The petitioners in their Answer denied that the land subject
of the complaint was part of a creek and countered that OCT No. P-928 was issued to them after they had
complied with the free patent requirements of the DENR, through the PENRO; that they and their
predecessor-in-interest had been in possession, occupation, cultivation, and ownership of the land for quite
some time.

The RTC dismissed the complaint for lack of cause of action and for being an erroneous remedy. The RTC
ruled that the land, having been acquired through a homestead patent, was presumably public land.
Therefore, only the State can institute an action for the annulment of the title covering it. The CA reversed
and set aside the RTC decision and ordered the cancellation of OCT No. P-928 and the reconveyance of
the land to the Respondents.

ISSUE:
Should the complaint be dismissed because of lack of cause of action and for being an erroneous remedy?

HELD:
No, the petitioners state that the property was formerly a public land, titled in their names by virtue of a Free
Patent issued by the DENR. Thus, they posit that the Respondents do not have the personality and authority
to institute any action for annulment of title because such authority is vested in the Republic of the
Philippines, through the Office of the Solicitor General.

In this regard, the Petitioners are mistaken. The action filed by the Respondents seeks the transfer to their
names of the title registered in the names of the Petitioners. In their Complaint, they alleged that: first, they
are the owners of the land, being the owners of the properties through which the Marigman creek passed
when it changed its course; and second, the Petitioners illegally dispossessed them by having the same
property registered in their names. It was not an action for reversion which requires that the State be the
one to initiate the action in order for it to prosper.

It is obvious that private respondents allege in their complaint all the facts necessary to seek the nullification
of the free patents as well as the certificates of title covering Lot 1015 and Lot 1017. Clearly, they are the
real parties in interest in light of their allegations that they have always been the owners and possessors of
the two (2) parcels of land even prior to the issuance of the documents of title in petitioners’ favor, hence
the latter could only have committed fraud in securing them.

119
OWNERSHIP
Quieting of Title

A CLOUD ON TITLE CONSIST OF ANY INSTRUMENT, RECORD, CLAIM, ENCUMBRANCE OR


PROCEEDING; WHICH IS APPARENTLY VALID OR EFFECTIVE BUT IS IN TRUTH AND IN FACT
INVALID, INEFFECTIVE, VOIDABLE OR UNENFORCEABLE AND MAY BE PREJUDICIAL TO THE
TITLE SOUGHT TO BE QUIETED

96. Green Acres Holdings, Inc. v. Cabral


G.R. Nos. 175542 & 183205, June 5, 2013
Villarama, J.

FACTS:
Victoria Cabral was the original owner of a parcel of land. The land was placed under the coverage of P.D.
No. 27, and on March 23, 1993, three Emancipation Patents were issued to the spouses Enrique Moraga
and Victoria Soriano (Spouses Moraga) as follows: EP No. 496039 with an area of 861 square meters; EP
No. 496040 with an area of 2,159 square meters; and EP No. 496041 with an area of 8,941 square meters.
The Spouses Moraga thereafter caused the cancellation of EP No. 496041 and its conversion to TCT No.
256260 (M).

On August 29, 1994, Cabral filed a complaint before the PARAD seeking the cancellation of the
Emancipation Patents issued to the Spouses Moraga on the grounds that these were obtained through
fraud and that the land is not suitable for rice and corn production and has long been classified as residential,
commercial, industrial and nonagricultural land by the Zoning Administrator of the Housing and Land Use
Regulatory Board. The PARAD rendered a decision denying the petition for cancellation of the Emancipation
Patents and dismissing the complaint for lack of merit. Cabral appealed the decision to the DARAB.

While the appeal was pending, the Spouses Moraga subdivided the lot covered by TCT No. 256260 (M)
into three smaller lots and sold the lots to Filcon Ready Mixed Inc. On April 29, 1999, Green Acres purchased
five lots from Filcon including the three subject properties. Except for an already cancelled annotation of a
real estate mortgage in favor of Philippine Commercial International Bank (PCI Bank), the titles were free
from any annotations, liens, notices, claims or encumbrances. On April 30, 1999, new titles were issued in
the name of Green Acres. Green Acres then constructed a warehouse building complex on the said lots.

On January 17, 2001, the DARAB resolved Cabral’s appeal and rendered judgment ordering the
cancellation of the titles issued in the names of the Spouses Moraga and those of Filcon for having been
illegally acquired.

On April 19, 2001, Green Acres filed a Complaint for Quieting of Title, Damages with Application for
Preliminary Injunction and Writ of Preliminary Attachment against Cabral, the Spouses Moraga, Filcon, the
DARAB and the Registry of Deeds of Meycauayan, Bulacan. Green Acres sought to quiet its title and alleged
that it is a purchaser in good faith and for value, claiming that it had no notice or knowledge of any adverse
claim, lien, or encumbrance on the properties.

ISSUE:
Does the said DARAB decision in favor of Cabral constitutes a cloud on Green Acres’ title over the subject
properties?

HELD:
Yes, There is no dispute as to the first requisite since Green Acres has legal title over the subject properties.
The issue lies in the second requisite.

A cloud on title consists of (1) any instrument, record, claim, encumbrance or proceeding; (2) which is
apparently valid or effective; (3) but is in truth and in fact invalid, ineffective, voidable, or unenforceable; and
(4) may be prejudicial to the title sought to be quieted.

This Court holds that the DARAB decision in favor of Cabral satisfies all four elements of a cloud on title.
As Green Acres correctly points out, the DARAB decision, a final one at that, is both an “instrument” and a
“record.”

120
Also, the DARAB decision is apparently valid and effective. It is a final decision that has not been reversed,
vacated or nullified. It is likewise apparently effective and may be prejudicial to Green Acres’ titles since it
orders the cancellation of the titles of the Spouses Moraga and Filcon all from which Green Acres derived
its titles. However, as discussed above, it is ineffective and unenforceable against Green Acres because
Green Acres was not properly impleaded in the DARAB proceedings nor was there any notice of lis pendens
annotated on the title of Filcon so as to serve notice to Green Acres that the subject properties were under
litigation. As such, Green Acres is an innocent purchaser for value. Green Acres’ arguments are meritorious.

121
OWNERSHIP
Quieting of Title

LANDS WITHIN THE BAGUIO TOWNSITE RESERVATION BELONG TO THE PUBLIC DOMAIN AND
ARE NO LONGER REGISTRABLE UNDER THE LAND REGISTRATION ACT

97. Heirs of Pocdo v. Avila


G.R. No. 199146, March 19, 2014
Carpio, J.

FACTS:
In June 2000, Pacifico Pocdo, who was later substituted by his heirs upon his death, filed a complaint to
quiet title over a 1,728-square meter property (disputed property) located in Camp 7, Baguio City. Pacifico
claimed that the disputed property originally belonged to Pacifico’s father, Pocdo Pool. The disputed
property is allegedly different from the one- hectare portion alloted to Polon Pocdo, the predecessor-in-
interest of the defendants Arsenia Avila and Emelinda Chua, in a partition made by the heirs of Pocdo Pool.
Pacifico alleged that the defendants unlawfully claimed the disputed property, which belonged to Pacifico.

The instant case, before the Regional Trial Court of Baguio City, was filed by Pacifico Pocdo against Arsenia
Avila and Emelinda Chua in June 2000, just after the RED set aside Suaking’s revocation on April 28, 2000
and ordered the restoration of Avila’s Certificate of Exclusion. Since then, the judicial proceedings have run
parallel to the administrative case.

The Regional Trial Court dismissed the case for lack of jurisdiction. The trial court held that the DENR had
already declared the disputed property as public land, which the State, through the DENR, has the sole
power to dispose. Thus, the claim of petitioners to quiet title is not proper since they do not have title over
the disputed property.

ISSUE:
Does the petitioners have title to the property that would support an action for quieting of title?

HELD:
No ,the DENR Decision was affirmed by the Office of the President which held that lands within the Baguio
Townsite Reservation belong to the public domain and are no longer registrable under the Land Registration
Act.[7] The Office of the President ordered the disposition of the disputed property in accordance with the
applicable rules of procedure for the disposition of alienable public lands within the Baguio Townsite
Reservation, particularly Chapter X of Commonwealth No. 141 on Townsite Reservations and other
applicable rules.

Having established that the disputed property is public land, the trial court was therefore correct in
dismissing the complaint to quiet title for lack of jurisdiction. The trial court had no jurisdiction to determine
who among the parties have better right over the disputed property which is admittedly still part of the public
domain.

Under Articles 476 and 477 of the Civil Code, the two indispensable requisites in an action to quiet title are:
(1) that the plaintiff has a legal or equitable title to or interest in the real property subject of the action; and
(2) that there is a cloud on his title by reason of any instrument, record, deed, claim, encumbrance or
proceeding, which must be shown to be in fact invalid or inoperative despite its prima facie appearance of
validity.

In this case, petitioners, claiming to be owners of the disputed property, allege that respondents are
unlawfully claiming the disputed property by using void documents, namely the “Catulagan” and the Deed
of Waiver of Rights. However, the records reveal that petitioners do not have legal or equitable title over the
disputed property, which forms part of Lot 43, a public land within the Baguio Townsite Reservation. It is
clear from the facts of the case that petitioners’ predecessors-in-interest, the heirs of Pocdo Pool, were not
even granted a Certificate of Ancestral Land Claim over Lot 43, which remains public land. Thus, the trial
court had no other recourse but to dismiss the case.

122
CO-OWNERSHIP
Concept

AN ACTION FOR PARTITION DOES NOT PRESCRIBE. UNTIL NO PHYSICAL DIVISION IS EFFECTED,
SHARES OF CO-OWNERS REMAIN IDEAL

98. Del Blanco v. Iintermediate Appellate Court


G.R. No. L-72694, December 1, 1987
Paras, J.

FACTS:
In a document executed in the Municipality of San Rafael, Bulacan, three brothers, Benedicto, Jose and
Manuel Pansacola, entered into an agreement which provided, among others: (1) That they will purchase
from the Spanish Government the lands comprising the Island of Cagbalite which is located within the
boundaries of the Municipality of Mauban, Province of Tayabas (now Quezon) and has an approximate area
of 1,600 hectares; (2) That the lands shall be considered after the purchase as their common property; (3)
That the co-ownership includes Domingo Arce and Baldomera Angulo, minors at that time represented by
their father, Manuel Pansacola (Fr. Manuel Peña) who will contribute for them in the proposed purchase of
the Cagbalite Island; (4) That whatever benefits may be derived from the Island shall be shared equally by
the co-owners in the following proportion: Benedicto Pansacola — ¼ share; Jose Pansacola — ¼ share;
and, Domingo Arce and Baldomera Angulo — 2/4 shares which shall be placed under the care of their
father, Manuel Pansacola (Fr. Manuel Peña).

The co-owners entered into the actual possession and enjoyment of the Island purchased by them from the
Spanish Government. On 1868 they agreed to modify the terms and conditions of the agreement wherein
the new agreement provided for a new sharing and distribution of the lands, comprising the Island of
Cagbalite and whatever benefits may be derived therefrom, as follows: "(a) The first one-fourth (1/4) portion
shall belong to Don Benedicto Pansacola; (b) The second one fourth (1/4) portion shall belong to Don Jose
Pansacola; (c) The third one-fourth (1/4) portion shall henceforth belong to the children of their deceased
brother, Don Eustaquio Pansacola, namely: Don Mariano Pansacola, Maria Pansacola and Don Hipolito
Pansacola; (d) The fourth and last one-fourth (1/4) portion shall belong to their nephews and nieces (1)
Domingo Arce, (2) Baldomera Angulo, (3) Marcelina Flores, (4) Francisca Flores, (5) Candelaria dela Cruz,
and (6) Gervasio Pansacola who, being all minors, are still under the care of their brother, Manuel Pansacola
(Fr. Manuel Peña). The latter is the real father of said minors."

About one hundred years later, on November 18, 1968, private respondents brought a special action for
partition in the Court of First Instance of Quezon, under the provisions of Rule 69 of the Rules of Court,
including as parties the heirs and successors-in-interest of the co-owners of the Cagbalite Island in the
second contract of co-ownership dated April 11, 1968.

ISSUE/S:
(1) Is the Cagbalite Island is still undivided property owned in common by the heirs and successors-in-
interest of the brothers, Benedicto, Jose and Manuel Pansacola?
(2) Does an action for partition prescribe?

HELD:
(1) Yes, the property is still undivided. There is nothing in all agreements that suggests that actual or physical
partition of the Island had really been made by either the original owners or their heirs or successors-in-
interest. The agreement entered into in 1859 simply provides for the sharing of whatever benefits can be
derived from the island.
(2) An action for partition does not prescribe. Under Article 403 of the Old Civil Code, now Article 497,
provides that the assignees of the co-owners may take part in the partition of the common property, and
Article 400 of the Old Code, now Article 494 provides that each co-owner may demand at any time the
partition of the common property, a provision which implies that the action to demand partition is
imprescriptible or cannot be barred by laches (Budlong vs. Pondoc, 79 SCRA 24 [1977]). An action for
partition does not lie except when the co-ownership is properly repudiated by the co-owner (Jardin vs.
Hollasco, 117 SCRA 532 [1982]).

123
CO-OWNERSHIP
Concept

A CO-OWNER HAS THE RIGHT TO ALIENATE HIS PRO-INDIVISO SHARE IN THE CO-OWNED
PROPERTY EVEN WITHOUT THE CONSENT OF HIS CO-OWNERS

99. Heirs of Dela Rosa v. Batongbacal


G.R. No. 179205, July 30, 2014
Perez, J.

FACTS:
The subject property consists of a 3,750 square meter-portion of the 15,001 square meters parcel of land
situated in Barrio Saog, Marilao, Bulacan denominated as Lot No. 1, and registered under the names of
Reynaldo Dela Rosa, Eduardo Dela Rosa, Araceli Dela Rosa and Zenaida Dela Rosa.

Sometime in 1984, Reynaldo offered to sell the subject property to Guillermo Batongbacal and Mario
Batongbacal for P50.00 per square meter or for a total of P187,500.00. Pursuant to the agreement,
Reynaldo received an advance payment of ₱31,500.00 leaving a balance of ₱156,000.00. As shown in the
document denominated as Resibo and signed by Reynaldo on 18 February 1987, the parties agreed that
the amount of ₱20,000.00 as part of the advance payment shall be paid upon the delivery of the Special
Power-of-Attorney which would authorize Reynaldo to alienate the subject property on behalf of his co-
owners and siblings namely, Eduardo, Araceli and Zenaida. The balance thereon shall be paid in
₱10,000.00 monthly installments until the purchase price is fully settled

Consequently, Guillermo and Mario initiated an action for Specific Performance or Rescission and Damages
before the Regional Trial Court of Malolos, Bulacan, seeking to enforce their Contract to Sell dated 18
February 1987. Mario and Guillermo asserted that they have a better right over the subject property and
alleged that the subsequent sale thereof effected by Reynaldo to third persons is void as it was done in bad
faith. It was prayed in the Complaint that Reynaldo be directed to deliver the SPA and, in case of its
impossibility, to return the amount of ₱31,500.00 with legal interest and with damages in either case.

Reynaldo in his Answer countered that the purported Contract to Sell is void, because he never gave his
consent thereto. Reynaldo insisted that he was made to understand that the contract between him and the
Batongbacals was merely an equitable mortgage whereby it was agreed that the latter will loan to him the
amount of ₱3 l ,500.00 payable once he receives his share in the proceeds of the sale of the land registered
under TCT No. T-107449.

Following the pre-trial conference without the parties reaching an amicable settlement, trial on the merits
ensued. The RTC dismissed the case and ordered Reynaldo to return to the former the sum of ₱28,000.00
with 12% annual interest. Reynaldo failed to convince the court a quo that the contract he entered into with
Mario was an equitable mortgage.

ISSUE:
Is the sale of Reynaldo of his undivided share in the property valid and enforceable?

HELD:
Yes, the subject of the Contract to Sell was limited only to '14 pro-indiviso share of Reynaldo consisting an
area of 3,750 square meter and not the entire 15,001-square meter parcel of land. As a co-owner of the
subject property, Reynaldo's right to sell, assign or mortgage his ideal share in the property held in common
is sanctioned by law.

Pursuant to Aricle 493, a co-owner has the right to alienate his pro-indiviso share in the co-owned property
even without the consent of his co-owners. This right is absolute and in accordance with the well-settled
doctrine that a co-owner has a full ownership of his pro-indiviso share and has the right to alienate, assign
or mortgage it, and substitute another person for its enjoyment. In other words, the law does not prohibit a
co-owner from selling, alienating, mortgaging his ideal share in the property held in common.

124
CO-OWNERSHIP
Concept

A CO-OWNER EVEN THROUGH THE COURT CANNOT COMPEL OTHER CO-OWNERS TO SELL
THEIR RESPECTIVE SHARES
|
100. Arambulo v. Nolasco
G.R. No. 189420, March 16, 2014
Perez, J.

FACTS:
Petitioners Raul V. Arambulo and Teresita A. Dela Cruz, along with their mother Rosita Vda. de Arambulo,
and siblings Primo V. Arambulo, Ma. Lorenza A. Lopez, Ana Maria V. Arambulo, Maximiano V. Arambulo,
Julio V. Arambulo and Iraida Arambulo Nolasco (Iraida) are co-owners of two (2) parcels of land located in
Tondo, Manila, with an aggregate size of 233 square meters.

When Iraida passed away, she was succeeded by her husband, respondent Genaro Nolasco and their
children, Iris Abegail Nolasco, Ingrid Aileen Arambulo and respondent Jeremy Spencer Nolasco.

On 8 January 1999, petitioners filed a petition for relief under Article 491 of the Civil Code with the RTC of
Manila, alleging that all of the co-owners, except for respondents, have authorized petitioners to sell their
respective shares to the subject properties; that only respondents are withholding their consent to the sale
of their shares.

ISSUE:
Can the respondents as co-owners be compelled by the court to give their consent to the sale of their shares
in the co-owned properties?

HELD:
No, Under Art. 493. “Each co-owner shall have the full ownership of his part and of the fruits and benefits
pertaining thereto, and he may therefore alienate, assign or mortgage it, and even substitute another person
in its enjoyment, except when personal rights are involved. But the effect of the alienation or the mortgage,
with respect to the co-owners, shall be limited to the portion which may be allotted to him in the division
upon the termination of the co-ownership”

The Court henceforth, cannot compel the respondents to give their consent to the sale of their shares.
Pertinent to this case, Article 493 dictates that each one of the parties herein as co-owners with full
ownership of their parts can sell their fully owned part. The sale by the petitioners of their parts shall not
affect the full ownership by the respondents of the part that belongs to them. Their part which petitioners
will sell shall be that which may be apportioned to them in the division upon the termination of the co-
ownership. With the full ownership of the respondents remaining unaffected by petitioners' sale of their
parts, the nature of the property, as co-owned, likewise stays. In lieu of the petitioners, their vendees shall
be co-owners with the respondents. |||

125
CO-OWNERSHIP
Concept

BETWEEN DISMISSAL WITH PREJUDICE UNDER RULE 17, SECTION 3 AND THE RIGHT GRANTED
TO CO-OWNERS UNDER ARTICLE 494 OF THE CIVIL CODE, THE LATTER MUST PREVAIL

101. Quintos v. Nicolas


G.R. No. 210252, June 16, 2014
Velasco Jr., J.

FACTS:
Petitioners ar siblings, heir parents, Bienvenido and Escolastica Ibarra, were the owners of the subject
property, a 281 sq.m.parcel of land situated along Quezon Ave in Tarlac. By 1999, both Bienvenido and
Escolastica had already passed away, leaving to their ten children ownership over the subject property.
Subsequently, sometime in 2002, respondent siblings brought an action for partition against petitioners. The
RTC dismissed the action due to failure of the parties to appear despite due notice.

Having failed to secure a favorable decision for partition, respondent siblings instead resorted to executing
a Deed of Adjudication to transfer the property in favor of the ten siblings. As a result, TCT No. 318717 was
canceled and in lieu thereof, TCT No. 390484 was issued in its place by the Registry of Deeds of Tarlac in
the names of the ten heirs of the Ibarra spouses. ScC
DET
Subsequently, respondent siblings sold their 7/10 undivided share over the property in favor of their co-
respondents, the spouses Recto and Rosemarie Candelario. By virtue of a Deed of Absolute Sale executed
in favor of the spouses Candelario and an Agreement of Subdivision purportedly executed by them and
petitioners, TCT No. 390484 was partially canceled and TCT No. 434304 was issued in the name of the
Candelarios, covering the 7/10 portion.

Petitioners filed a complaint for Quieting of Title and Damages against respondents wherein they alleged
that during their parents' lifetime, the couple distributed their real and personal properties in favor of their
ten children. Upon distribution, petitioners alleged that they received the subject property and the house
constructed thereon as their share. They likewise averred that they have been in adverse, open, continuous,
and uninterrupted possession of the property for over four decades and are, thus, entitled to equitable title
thereto. They also deny any participation in the execution of the aforementioned Deed of Adjudication dated
September 21, 2004 and the Agreement of Subdivision.
|||
ISSUE:
Is the the respondents' counterclaim for partition is already barred by laches or res judicata?

HELD:
No, the law generally does not favor the retention of co-ownership as a property relation, and is interested
instead in ascertaining the co-owners' specific shares so as to prevent the allocation of portions to remain
perpetually in limbo. Thus, the law provides that each co-owner may demand at any time the partition of
the thing owned in common.

Between dismissal with prejudice under Rule 17, Sec. 3 and the right granted to co-owners under Art. 494
of the Civil Code, the latter must prevail. To construe otherwise would diminish the substantive right of a co-
owner through the promulgation of procedural rules. Such a construction is not sanctioned by the principle,
which is too well settled to require citation, that a substantive law cannot be amended by a procedural rule.
The counterclaim for partition is not barred by laches. As an equitable defense, laches does not concern
itself with the character of the petitioners' title, but only with whether or not by reason of the respondents'
long inaction or inexcusable neglect, they should be barred from asserting this claim at all, because to allow
them to do so would be inequitable and unjust to petitioners. As correctly appreciated by the lower courts,
respondents cannot be said to have neglected to assert their right over the subject property. They cannot
be considered to have abandoned their right given that they filed an action for partition sometime in 2002,
even though it was later dismissed.

126
CO-OWNERSHIP
Concept

DEPOSITORS IN A JOINT ACCOUNT ARE CO-OWNERS AS FAR AS THE BANK IN CONCERNED, AS


BETWEEN THE ACCOUNT HOLDERS, THEIR RIGHT AGAINST EACH OTHER MAY DEPEND ON
WHAT THEY HAVE AGREED UPON

102. Apique v. Fahnestich


G.R. No. 205705, August 5, 2015
Perlas-Bernabe, J.

FACTS:
This is a petition for certiorari which ordered petitioner Dominador M. Apique (Dominador) to return to
respondent Evangeline Apique Fahnenstich (Evangeline) the amount of P980,000.00, plus 6% interest per
annum reckoned from the filing of the complaint up to the finality of the decision and, thereafter, 12% interest
on the total amount demanded until its full satisfaction.

Dominador and Evangeline are siblings who used to live with their parents until Evangeline left for Germany
to work sometime in 1979. On 1995, Evangeline executed a General and Special Powers of Attorney
constituting Dominador as her attorney-in-fact to purchase real property for her, and to manage or supervise
her business affairs in the Philippines. As Evangeline was always in Germany, she opened a joint savings
account on January 18, 1999 with Dominador at the Claveria Branch of the Philippine Commercial
International Bank in Davao City, which later became Equitable PCI Bank,and now Banco de Oro, under
Savings Account No. 1189-02819-5 (subject account).

On February 11, 2002, Dominador withdrew the amount of P980,000.00 from the subject account and,
thereafter, deposited the money to his own savings account with the same bank, under Savings Account
No. 1189-00781-3. Evangeline demanded the return of the amount withdrawn from the joint account, but to
no avail.

ISSUE:
Is Evangeline entitled to the return of the amount of P980,000.00 Dominador withdrew from their joint
savings account with EPCIB, plus legal interest thereon?

HELD:

Yes, there is no dispute that the account opened by Evangeline and Dominador under Savings Account No.
1189-02819-5 with EPCIB was a joint "OR" account. It is also admitted that: (a) the account was opened for
a specific purpose,i.e., to facilitate the transfer of needed funds for Evangeline's business projects; and (b)
Dominador may withdraw funds therefrom "if" there is a need to meet Evangeline's financial obligations
arising from said projects. Hence, while Dominador is a co-owner of the subject account as far as the bank
is concerned — and may, thus, validly deposit and/or withdraw funds without the consent of his co-depositor,
Evangeline — as between him and Evangeline,his authority to withdraw, as well as the amount to be
withdrawn, is circumscribed by the purpose for which the subject account was opened.

Under Article 485 of the Civil Code, q joint account is one that is held jointly by two or more natural persons,
or by two or more juridical persons or entities. Under such setup, the depositors are joint owners or co-
owners of the said account, and their share in the deposits shall be presumed equal, unless the contrary is
proved,

The common banking practice is that regardless of who puts the money into the account, each of the named
account holder has an undivided right to the entire balance, and any of them may deposit and/or withdraw,
partially or wholly, the funds without the need or consent of the other, during their lifetime. Nevertheless, as
between the account holders, their right against each other may depend on what they have agreed upon,
and the purpose for which the account was opened and how it will be operated.

Under the foregoing circumstances, Dominador's right to obtain funds from the subject account was, thus,
conditioned on the necessity of funds for Evangeline's projects. Admittedly, at the time he withdrew the
amount of P980,000.00 from the subject account, there was no project being undertaken for Evangeline.
Moreover, his claim that the said amount belonged to him, as part of the compensation promised by Holgar
for his services as administrator of the business affairs of Evangeline, was correctly rejected by the CA,

127
considering the dearth of competent evidence showing that Holgar: (a) undertook to pay Dominador the
amount of P1,000,000.00 for his services as administrator of Evangeline's various projects; and (b) remitted
such amount to the subject account for the benefit of Dominador. Having failed to justify his right over the
amount withdrawn, Dominador is liable for its return, as correctly adjudged by the CA.

128
CO-OWNERSHIP
Concept

ANY ONE OF THE CO-OWNERS MAY BRING AN ACTION IN EJECTMENT

103. Catedrilla v. Lauron


G.R. No. 179011, April 15, 2013
Peralta, J.

FACTS:
On February 12, 2003, petitioner Rey Castigador Catedrilla filed with the Municipal Trial Court (MTC) of
Lambunao, Iloilo a Complaint for ejectment against the spouses Mario and Margie Lauron. Respondents
Mario and Margie Lauron, through the tolerance of the heirs of Lilia, constructed a residential building of
strong materials on the northwest portion of Lot No. 5 covering an area of one hundred square meters; that
the heirs of Lilia made various demands for respondents to vacate the premises and even exerted earnest
efforts to compromise with them but the same was unavailing; and that petitioner reiterated the demand on
respondents to vacate the subject lot on January 15, 2003, but respondents continued to unlawfully withhold
such possession.

In their Answer, respondents claimed that petitioner had no cause of action against them, since they are
not the owners of the residential building standing on petitioner's lot, but Mildred Kascher (Mildred), sister
of respondent Margie, as shown by the tax declaration in Mildred's name.

The MTC rendered its Decision, in favor of the plaintiff ordering the defendants to vacate the lot in question
and restore possession. Respondents filed their appeal with the RTC which merely affirmed the lower court’s
decision. However, the CA reversed and set aside the said decision. The CA found that petitioner's co-heirs
to the subject lot should have been impleaded as co-plaintiffs in the ejectment case against respondents,
since without their presence, the trial court could not validly render judgment and grant relief in favor of
petitioner.

ISSUE:
Did the non-inclusion of the co-heirs as indipensable parties made the complaint fatally defective?

HELD:
No, petitioner can file the action for ejectment without impleading his co-owners. In Wee v. De Castro,
petitioner therein argued that the respondent cannot maintain an action for ejectment against him, without
joining all his co-owners. Article 487 of the New Civil Code is explicit on this point: “Any one of the co-owners
may bring an action in ejectment.”

This article covers all kinds of action for the recovery of possession, i.e., forcible entry and unlawful detainer
(accion interdictal), recovery of possession (accion publiciana), and recovery of ownership (accion de
reivindicacion). As explained by the renowned civilest, Professor Arturo M. Tolentino: A co-owner may bring
such an action, without the necessity of joining all the other co-owners as co-plaintiffs, because the suit is
deemed to be instituted for the benefit of all. If the action is for the benefit of the plaintiff alone, such that he
claims possession for himself and not for the co-ownership, the action will not prosper.

129
CO-OWNERSHIP
Rights and Obligations of Each Co-Owner

EACH COOWNER OR TENANT IN COMMON OF UNDIVIDED REALTY HAS THE SAME RIGHTS
THEREIN AS THE OTHERS; HE MAY USE AND ENJOY THE SAME WITHOUT OTHER LIMITATION
EXCEPT THAT HE MUST NOT PREJUDICE THE RIGHTS OF HIS CO-OWNERS

104. Pardell v. Bartolome


G.R. No. L-4656, November 18, 1912
Torres, J.

FACTS:
Petitioner Vicenta Ortiz y Felin de Pardell and respondent Matilde Ortiz y Felin Bartolome were the existing
heirs of the late Miguel Ortiz and Calixta Felin. On 1888, Matilde and co-defendant Gaspar de Bartolome y
Escribano took it upon themselves without an judicial authorization or even extra judicial agreement the
administration of the properties of the late Calixta and Miguel. These properties included a house in Escolta
Street,Vigan, Ilocos Sur; a house in Washington Street, Vigan,Ilocos Sur; a lot in Magallanes Street, Vigan,
IlocosSur; parcels of rice land in San Julian and Sta. Lucia;and parcels of land in Candon, Ilocos Sur, they
took upon themselves the administration and enjoyment of the properties left by Calixta and collected the
rents, fruits, and products thereof, to the serious detriment of Vicenta’s interest. Despite repeated demands
to divide the properties and the fruits accruing therefrom, Sps Gaspar and Matilde had been delaying the
partition and delivery of the said properties by means of unkempt promises and other excuses.

Vicenta filed a petition for partition with damages in the RTC. The RTC absolved Matilde from payment of
damages. It held that the revenues and the expenses were compensated by the residence enjoyed by the
defendant party, that no losses or damages were either caused or suffered, nor likewise any other expense
besides those aforementioned. Counsel for Matilde took an exception to the judgment and moved for a new
trial on the grounds that the evidence presented did not warrant the judgment rendered and that the latter
was contrary to law. That motion was denied by the lower court. Thus, this petition.

ISSUE:
Is a co-owner is required to pay for rent in exclusively using the co-owned property?

HELD:
No, Article 394 of the Civil Code prescribes: “Each co-owner may use the things owned in common, provided
he uses them in accordance with their object and in such manner as not to injure the interests of the
community nor prevent the co-owners from utilizing them according to their rights.”

Matilde Ortiz and her husband occupied the upper story, designed for use as a dwelling, in the house of
joint ownership; but the record shows no proof that, by so doing, the said Matilde occasioned any detriment
to the interests of the community property, nor that she prevented her sister Vicenta from utilizing the said
upper story according to her rights. It is to be noted that the stores of the lower floor were rented and an
accounting of the rents was duly made to the plaintiffs.

Each co-owner of realty held pro indiviso exercises his rights over the whole property and may use and
enjoy the same with no other limitation than that he shall not injure the interests of his coowners, for the
reason that, until a division be made, the respective part of each holder can not be determined and every
one of the coowners exercises together with his other coparticipants, joint ownership over the pro indiviso
property, in addition to his use and enjoyment of the same.

As the hereditary properties of the joint ownership of the two sisters, Vicenta Ortiz, plaintiff, and Matilde
Ortiz, defendant, were situated in the Province of Ilocos Sur, and were in the care of the last named, assisted
by her husband, while the plaintiff Vicenta with her husband was residing outside of the said province the
greater part of the time between 1885 and 1905, when she left these Islands for Spain, it is not at all strange
that delays and difficulties should have attended the efforts made to collect the rents and proceeds from the
property held in common and to obtain a partition of the latter, especially during several years when, owing
to the insurrection, the country was in a turmoil; and for this reason, aside from that founded on the right of
co-ownership of the defendants, who took upon themselves the administration and care of the property of
joint tenancy for purposes of their preservation and improvement, these latter are not obliged to pay to the
plaintiff Vicenta one-half of the rents which might have been derived from the upper story of the said house
on Calle Escolta, and, much less, because one of the living rooms and the storeroom thereof were used for

130
the storage of some belongings and effects of common ownership between the litigants. The defendant
Matilde, therefore, in occupying with her husband the upper floor of the said house, did not injure the
interests of her coowner, her sister Vicenta, nor did she prevent the latter from living therein, but merely
exercised a legitimate right pertaining to her as a coowner of the property.

131
CO-OWNERSHIP
Extinguishment of Co-Ownership

RIGHT OF REDEMPTION MAY BE EXERCISED BY A CO-OWNER OR HIS SUCCESSOR-IN-INTEREST


ONLY BEFORE PARTITION, OR BEFORE THE COMMUNITY IS TERMINATED

105. Caro v. Court of Appeals


G.R. No. L-46001, March 25, 1982
Guerrero, J.

FACTS:
Alfredo Benito, Benjamin Benito and Mario Benito were the original co-owners of two parcels of land.

Sometime in January 1957, Mario died. His surviving wife, Basilia Lahorra and his father, Saturnino Benito
had thereafter been appointed as joint administrators of Mario's estate.

Sometime in 1959, Benjamin Benito, one of the co-owners, executed a deed of absolute sale of his one-
third undivided portion over said parcels of land in favor of herein petitioner, Luz Caro.

In 1960, Alfredo, Luz, Saturnino and Basilia orally agreed to subdivide the property. A petition for subdivision
was then filed for the purpose, accompanied by the affidavits of Alfredo and Saturnino stating they agree to
the segregation of the parcel of land.

Pursuant thereto, a subdivision plan was then made, wherein they allocated Lot I-C to Luz. A TCT over the
same lot was then finally issued to her.

Nevertheless, Basilia alleged that it was only sometime in May 1966, did she learn about the transfer of Lot
I-C by Benjamin to Luz, when she received a special proceeding-related pleading containing an allegation
to that effect. Basilia then sent a written offer to redeem to Luz in August 1966, allegedly pursuant to a right
of legal redemption under the Civil Code.

ISSUES:
1) Is the subdivision made by the parties in 1960 valid?
2) Assuming that aforementioned subdivision is valid, can the right of redemption still be exercised?

HELD:
1) YES. Citing Hernandez vs. Andal, the Court held that an agreement of partition, though oral, is
valid and consequently binding upon the parties; hence, the subdivision made in 1960 is valid and
must be respected.

2) NO. The Court then remarked that such partition or subdivision being valid, the community is
likewise terminated and according to Caram v. CA, there is no longer reason to sustain any right of
legal redemption.

Moreover, citing Saturnino v. Paulino, the right of redemption under the Civil Code may be exercised by a
co-owner only before partition. In this case, the right was asserted not only after partition, but after the
property inherited has already been subdivided into several parcels, which were assigned by lot.

Thus, it may be concluded that right of redemption may no longer be exercised after a co-owned land has
already been subdivided.

132
CO-OWNERSHIP
Extinguishment of Co-Ownership

THE PROPER REMEDY AGAINST A BUYER WHO SUBSTITUTED A CO-OWNER BY VIRTUE OF A


SALE OF THE COMMUNITY PROPERTY, IS NOT AN ACTION FOR NULLIFICATION OF SUCH SALE
BUT AN ACTION FOR DIVISION OF THE COMMUNITY PROPERTY

106. Bailon-Casilao v. Court of Appeals


G.R. No. 78178, April 15, 1988
Cortes, J.

FACTS:
In this case, there was a parcel of land co-owned by 6 Bailon siblings—each owning a 1/6 share. Two of
the siblings, Rosalia and Gaudencio, sold a portion of the lot to Donato Delgado. Rosalia then sold the
remainder of the land to Ponciana de Lanuza.

Ponciana eventually also acquired the portion acquired by Donato, as a separately titled lot, in a subsequent
and separate transaction with the latter. The husband of Ponciana, by virtue of an SPA then managed to
sell the entire property as two separate lots, to Celestino Afable Sr., herein respondent.

The other co-owners, Delia and heirs of Nenita, herein petitioners then filed a case for recovery of the
property with damages against Celestino.

ISSUES:
1) Is the sale made by the two co-owners, Rosalia and Gaudencio, as well as the subsequent
transactions thereto, valid?
2) Is the action for recovery of property the proper remedy in the present case?

HELD:
1) YES. The Court held that as a co-owner is entitled to sell his undivided share under Art. 493 of the
Civil Code, a sale of the entire property by one co-owner without the consent of the other co-owners
is valid. However, only the rights of the co-owner-seller may be transferred; thus, the effect of the
sale, although valid, is that the buyer merely substitutes the seller co-owner.

2) NO. The proper action in case of sale or of the thing owned in common by one or some of the co-
owners to a third person who substituted the co-owner or co-owners, is not an action for nullification
of the sale but an action for division of the common property. The Court held that as a buyer
substitutes the seller or sellers as a co-owner of the latter’s proportionate share, he is a legitimate
proprietor and possessor in joint ownership of the common property claimed. Hence, an action for
division of the common property, thereby terminating co-ownership thereof, is the proper remedy
in the present case.

133
CO-OWNERSHIP
Extinguishment of Co-Ownership

IF DEFENDANT IN AN ACTION FOR PARTITION ASSERTS ADVERSE AND EXCLUSIVE TITLE, THE
COURT SHOULD NOT DISMISS THE ACTION BUT SHOULD INSTEAD DETERMINE IF THE PLAINTIFF
IS A CO-OWNER OR NOT

107. Roque v. Intermediate Appellate Court


G.R. No. 75886, August 30, 1988
Feliciano, J.

FACTS:
On September 21, 1959, the intestate heirs of Januario Avendafio executed a document for extrajudicial
settlement, allocating 1/4 shares each to the heirs or their representatives.

On September 28, 1959, co-owners Illuminada, Gregorio, Miguel, Bernardino, Bienvenido, Numeriano and
Rufina, all surnamed Avendaño, in consideration of the aggregate amount of P500.00, transferred their
collective and undivided 3/4 share in the parcel of land, to respondents Ernesto Roque and Victor Roque.

Emesto and Victor then purportedly sold a three-fourths (3/4) undivided portion of Lot No. 1549 to their half-
sister, petitioner Concepcion Roque, for the same amount. When Concepcion sought a subdivision plan to
claim her 3/4 undivided share however, the heirs of Ernesto and Victor, herein respondents instead asserted
their title over the same and rejected her plan to divide the land.

Attempts at amicable settlement having fallen through, petitioner Concepcion, on December 6, 1977, filed
a Complaint for "Partition with Specific Performance", claiming legal ownership of an undivided threefourths
(3/4) portion of lot in controversy.

The RTC ruled in favour of herein petitioner, however the IAC reversed the decision and ruled for the
dismissal of the complaint. The IAC reasoned that from the moment an alleged co-owner asserts an adverse
title, the proper remedy of an aggrieved co-owner may only be accion reivindicatoria or action for recovery
of title and possession, not an action for partition.

ISSUE:
Is the IAC correct in ruling for the dismissal for the action for partition?

HELD:
NO. The Court held that in case defendants in an action for partition assert exclusive title in themselves
adversely to the plaintiff, the court should not dismiss the action but resolve the question of whether the
plaintiff is co-owner or not.

Should the trial court find that the plaintiff was unable to sustain his claimed status as co-owner, or that the
defendants are or have become the sole and exclusive owners of the property involved, the court will
necessarily have to dismiss the action for partition. This result would be reached, not because the wrong
action was commenced by the plaintiff, but rather because the plaintiff having been unable to show co-
ownership rights in himself, no basis exists for requiring the defendants to submit to partition the property
at stake.

If, upon the other hand, the court after trial should find the existence of co-ownership among the parties
litigant, the court may and should order the partition of the property in the same action.

134
CO-OWNERSHIP
Extinguishment of Co-Ownership

RIGHT TO PARTITION MAY BE BARRED WHEN A CO-OWNER SUCCESSFULLY ACQUIRES


EXCLUSIVE TITLE OVER THE PROPERTY BY PRESCRIPTION

108. Delima v. Court of Appeals


G.R. No. L-46296, September 24, 1991
Medialdea, J.

FACTS:
Lino Delima acquired Lot No. 7758 of the Talisay-Minglanilla Friar Lands Estate in Cebu by sale on
installments from the government. He later died in 1921, leaving as his only heirs three brothers and a sister
namely: Eulalio Delima, Juanita Delima, Galileo Delima and Vicente Delima.

After his death, TCT No. 2744 of the property in question was issued on August 3, 1953 in the name of the
Legal Heirs of Lino Delima, deceased, represented by Galileo Delima. On September 22, 1953, Galileo
Delima, now substituted by respondents, executed an affidavit of "Extra-judicial Declaration of Heirs."

Based on this affidavit, TCT No. 2744 was cancelled and TCT No. 3009 was issued on February 4,1954 in
the name of Galileo Delima alone to the exclusion of the other heirs.Galileo Delima declared the lot in his
name for taxation purposes and paid the taxes thereon from 1954 to 1965.

On February 29, 1968, petitioners, who are the surviving heirs of Eulalio and Juanita Delima, filed with the
Court of First Instance of Cebu an action for reconveyance and/or partition of property and for the annulment
of TCT No. 3009 with damages against their uncles Galileo Delima and Vicente Delima,. Vicente Delima
was joined as party defendant by the petitioners for his refusal to join the latter in their action.

ISSUE:
Did the respondent acquire title over the lot by prescription, to the exclusion of the other sibling co-owners?

HELD:
YES. The Court held that the moment one of the co-owners claims that he is the absolute and exclusive
owner of the properties and denies the others any share therein, the question involved is no longer one of
partition but of ownership. In such case, the imprescriptibility of the action for partition can no longer be
invoked or applied when one of the co-owners has adversely possessed the property as exclusive owner
for a period sufficient to vest ownership by prescription.

Since an action for reconveyance of land based on implied or constructive trust prescribes after 10 years, it
is from the date of the issuance of such title that the effective assertion of adverse title for purposes of the
statute of limitations is counted. Evidence shows that TCT No. 2744 in the name of the legal heirs of Lino
Delima, represented by Galileo Delima, was cancelled by virtue of an affidavit executed by Galileo Delima
and that on February 4, 1954, Galileo Delima obtained the issuance of a new title in Ms name numbered
TCT No. 3009 to the exclusion of his co-heirs. The issuance of this new title constituted an open and clear
repudiation of the trust or co-ownership, and the lapse of 10 years of adverse possession by Galileo Delima
from February 4, 1954 was sufficient to vest title in him by prescription.

As the certificate of title was notice to the whole world of his exclusive title to the land, such rejection was
binding on the other heirs and started as against them the period of prescription. Hence, when petitioners
filed their action for reconveyance and/or to compel partition on February 29, 1968, such action was already
barred by prescription. Whatever claims the other co-heirs could have validly asserted before can no longer
be invoked by them at this time.

135
CO-OWNERSHIP
Extinguishment of Co-Ownership

WHEN AN ACTION TO COMPEL SALE OF PROPERTY UNDER ARTICLE 498 IS FILED AND
THEREAFTER GRANTED, THE CO-OWNERSHIP IS DEEMED TERMINATED

109. Aguilar v. Court of Appeals


G.R. No. 76351, October 29, 1993
Bellosillo, J.

FACTS:
Petitioner Virgilio and respondent Senen are brothers. Virgilio is the youngest of seven children of the late
Maximiano Aguilar, while Senen is the fifth. On October 28, 1969, the two brothers purchased a house and
lot in Parañaque where their father could spend and enjoy his remaining years in a peaceful neighborhood.
Initially, the brothers agreed that Virgilio's share in the co-ownership was two-thirds while that of Senen was
one-third. By virtue of a written memorandum dated February 23, 1970, Virgilio and Senen agreed that
henceforth their interests in the house and lot should be equal. The brothers agreed that the deed of sale
would be executed and the title registered in the meantime in the name of Senen. It was further agreed that
Senen would take care of their father and his needs since Virgilio and his family were staying in Cebu. After
Maximiano Aguilar died in 1974, petitioner demanded from private respondent that the latter vacate the
house and that the property be sold and proceeds thereof divided among them. Because of the Senen’s
refusal, Virgilio filed an action to compel the sale.

ISSUE:
Was the co-ownership terminated upon the filing and the granting of the action to compel sale of the property
and ejectment of a co-owner?

HELD:
Yes. Art. 494 of the Civil Code provides that no co-owner shall be obliged to remain in the co-ownership,
and that each co-owner may demand at any time partition of the thing owned in common insofar as his
share is concerned. Corollary to this rule, Art. 498 of the Code states that whenever the thing is essentially,
indivisible and the co-owners cannot agree that it be, allotted to one of them who shall indemnify the others,
it shall be sold and its proceeds accordingly distributed.

This is resorted to: (1) when the right to partition the property is invoked by any of the co-owners but because
of the nature of the property it cannot be subdivided or its subdivision would prejudice the interests of the
co-owners, and (2) the co-owners are not in agreement as to who among them shall be allotted or assigned
the entire property upon proper reimbursement of the co-owners. In one case, the Court upheld the order
of the trial court directing the holding of a public sale of the properties owned in common pursuant to Art.
498 of the Civil Code.

Hence, hen petitioner filed an action to compel the sale of the property and the trial court granted the petition
and ordered the ejectment of respondent, the co-ownership was deemed terminated and the right to enjoy
the possession jointly also ceased. Thereafter, the continued stay of respondent and his family in the house
prejudiced the interest of petitioner as the property should have been sold and the proceeds divided equally
between them. To this extent and from then on, respondent should be held liable for monthly rentals until
he and his family vacate.

136
CO-OWNERSHIP
Extinguishment of Co-Ownership

FOLLOWING THE PRINCIPLE UNDER ARTICLE 494, AN ACTION FOR PARTITION DOES NOT
PRESCRIBE

110. Tomas Claudio Memorial College, Inc. v. Court of Appeals


G.R. No. 124262 October 12, 1999
Quisumbing, J.

FACTS:
On December 13, 1993, private respondents filed an action for Partition before the Regional Trial Court of
Morong, Rizal. They alleged that their predecessor-in-interest, Juan De Castro, died intestate in 1993 and
they are his only surviving and legitimate heirs.

They also alleged that their father owned a parcel of land designated as Lot No. 3010 located at Barrio San
Juan, Morong, Rizal. They further claim that in 1979, without their knowledge and consent, said lot was sold
by their brother Mariano to petitioner. The sale was made possible when Mariano represented himself as
the sole heir to the property.

Private respondents filed an action for partition, contending that the sale made by Mariano affected only his
undivided share to the lot in question but not the shares of the other co-owners equivalent to 4/5 of the
property. Petitioner refuted that their right to partition of the property has already prescribed

ISSUE:
Did the action for partition prescribe?

HELD:
NO. Firstly, the Court remarked that even if a co-owner sells the whole property as his, the sale will affect
only his own share but not those of the other co-owners who did not consent to the sale; hence, the
transferee gets only what corresponds to his grantor's share in the partition of the property owned in
common, thereby making him a co-owner of the property.

In the light of the foregoing, the Court then held that petitioner's defense of prescription against an action
for partition is a vain proposition. Pursuant to Article 494 of the Civil Code, no co-owner shall be obliged to
remain in the co-ownership. Such co-owner may demand at any time the partition of the thing owned in
common, insofar as his share is concerned.

Moreoever, citing Budlong vs. Bondoc, the Court interpreted Art. 494 to mean that the action for partition is
imprescriptible. It cannot be barred by prescription, for the law provides, no prescription shall lie in favor of
a co-owner or co-heirs as long as he expressly or impliedly recognizes the co-ownership.

137
CO-OWNERSHIP
Extinguishment of Co-Ownership

ENTERING INTO A MORTGAGE CONTRACT OR PAYING LAND TAXES IN ONE’S OWN NAME
WITHOUT OUSTING OTHER CO-OWNERS, NOT REPUDIATION OF CO-OWNERSHIP

111. Robles v. Court of Appeals


G.R. No. 123509, March 14, 2000
Panganiban, J.

FACTS:
Petitioners were the children of Silvino, and Hilario Robles is their half-brother. The task of cultivating their
inherited land was assigned to Lucio, while the payment of the land taxes was entrusted to Hilario. For
unknown reasons, the tax declaration of the parcel of land in the name of Silvino was cancelled and
transferred to Exequiel Ballena.

Exequiel then secured a loan from Antipolo Rural Bank using the tax declaration as security. Somehow, the
tax declaration was then transferred to the name of Antipolo Rural Bank and later, to the name of respondent
spouses, Hilario and Andrea Robles. Andrea then secured a loan from Cardona Rural Bank using the tax
declaration for the lot in question as security. For failure to pay the mortgage debt, the property was
foreclosed with Cardona Rural Bank emerging as the highest bidder. The bank sold the property to spouses
Vergel and Ruth Santos.

In September 1987, petitioners discovered the mortgage and attempted to redeem the property but were
unsuccessful. In 1988, the spouses Santos took possession of the property and were able to secure a Free
Patent therefor. Petitioners then filed an action for quieting of title. Respondents questioned their standing
to sue for quieting of title, contending that petitioners no longer have any interest to the property in question
due to the mortgage executed thereon by Hilario, and the consequent foreclosure thereof by the Bank.

ISSUE:
Did respondent Hilario acquire exclusive title to the property by prescription, to the exclusion of his sibling
co-owners?

HELD:
NO. The Court held that in order that the title may prescribe in favor of a co-owner, the following requisites
must concur: (1) the co-owner has performed unequivocal acts of repudiation amounting to an ouster of the
other co-owners; (2) such positive acts of repudiation have been made known to the other co-owners; and
(3) the evidence thereof is clear and convincing.

In the present case, Hilario did not have possession of the subject property; neither did he exclude the
petitioners from the use and the enjoyment thereof, as they had indisputably shared in its fruits. Likewise,
his act of entering into a mortgage contract with the bank cannot be construed to be a repudiation of the co-
ownership. Neither should his payment of land taxes in his name, as agreed upon by the co-owners, be
construed as a repudiation of the co-ownership.

138
CO-OWNERSHIP
Extinguishment of Co-Ownership

AN AFFIDAVIT OF SELF-ADJUDICATION IS NOT AN ACT OF REPUDIATION BY VIRTUE OF WHICH


A CO-OWNER MAY ACQUIRE EXCLUSIVE TITLE OVER COMMUNITY PROPERTY BY
PRESCRIPTION

112. Galvez v. Court of Appeals


G.R. No. 157954, March 24, 2006
Chico-Nazario, J.

FACTS:
Timotea F. Galvez died intestate on April 28, 1965. She left behind her children, Ulpiano and Paz Galvez.
Ulpiano predeceased Timotea and was survived by his son, Porfirio Galvez. Timotea left a parcel of land
situated at Pagdaraoan, San Fernando, La Union.

Considering that all the other compulsory heirs of Timotea already received their respective shares, the
property passed by succession, both to Timotea's daughter, Paz, and to the former's grandson, Porfirio, the
latter succeeding by right of representation as the son of Ulpiano.

Porfirio was surprised to discover sometime in 1994 that on May 4, 1970, Paz executed an affidavit of
adjudication stating that she is the true and lawful owner of the said property.

It was by virtue of the said affidavit of adjudication did Paz sell the land to Carlos Tam, also sometime in
1994. Porfirio then sought nullification of the affidavit of adjudication and legal redemption of the entire lot.

ISSUE:
Did petitioner Paz successfully acquire exclusive title to the lot by prescription after executing the affidavit
of adjudication?

HELD:
NO—The Court held that Paz effected no clear and evident repudiation of the co-ownership. The execution
of the affidavit of self-adjudication does not constitute such sufficient act of repudiation as contemplated
under the law as to effectively exclude Porfirio from the property.

Citing jurisprudence, the Court remarked that where a co-heir was excluded from his legal share by the
other co-heir who represented himself as the only heir, such act of exclusion does not constitute repudiation.
For title to prescribe in favor of a co-owner there must be a clear showing that he has repudiated the claims
of the other co-owners and the latter has been categorically advised of the exclusive claim he is making to
the property in question. The rule requires a clear repudiation of the co-ownership duly communicated to
the other co-owners. It is only when such unequivocal notice has been given that the period of prescription
will begin to run against the other co-owners and ultimately divest them of their own title if they do not
seasonably defend it.

139
CO-OWNERSHIP
Extinguishment of Co-Ownership

REDEMPTION BY ONE CO-OWNER OF THE CO-OWNED PROPERTY DOES NOT TERMINATE CO-
OWNERSHIP; FRAUDULENT REGISTRATION BY ONE CO-OWNER SOLE UNDER HIS NAME
CONSTITUTES AN IMPLIED TRUST IN FAVOR OF THE OTHERS; REPUDIATION MUST BE MADE
KNOWN TO THE OTHER CO-OWNERS BEFORE PRESCRIPTION MAY SET IN

113. Adille v. Court of Appeals


G.R. No. L-44546, January 29, 1988
Sarmiento, J.

FACTS:
This is an appeal by way of certiorari. Herein private respondents, the Asejos, half-siblings of petitioner
Adille, filed before the trial court a case for partition with accounting. The trial court dismissed the case in
favor of petitioner Adille who claimed that he became the absolute owner by virtue of his redemption of the
entire co-owned property, and subsequent registration of the entire property under his sole name and
prescription. The CA reversed the ruling of the trial court.

The partition case stemmed from a parcel of land owned by the mother of herein parties--Adille from the
first marriage, and the Asejos from the second marriage. The mother sold the subject land in pacto de retro
to certain third persons, but she died before the lapse of the redemption period of three years.

During the redemption period, however, petitioner Adille was able to repurchase the entire property and
thereafter execute a deed of extra-judicial partition representing his self to be the only heir and child of his
mother, finally registering the same to his name. The Asejos maintained that the redemption made by
petitioner Adille did not vest sole ownership to the latter.

ISSUE:

Did Adille’s redemption and subsequent registration of the property in his name constitute acts of repudiation
by which he could have acquired exclusive title over the lot by prescription?

HELD:
NO—The Court held that while it is true that a vendee a retro may not be compelled to consent to a partial
redemption under the law, the redemption by one co-heir or co-owner of the property in its totality does not
vest in him ownership over it. The law does not give to the redeeming co-owner the right to the entire
property nor does it state that such redemption is a mode of terminating a co-ownership. Moreover,
registration is not a mode of acquiring ownership. Petitioner’s act of registration instead shows that he is
guilty of fraud, and thus holds the entire property in trust, with the private respondents as the beneficiaries
thereof.

Finally, it may not be said that the foregoing acts constitute acts of repudiation by which petitioner Adille
could have acquired exclusive title over the lot by prescription. The present case has failed to satisfy all the
following elements of repudiation: (1) a co-owner repudiates the co-ownership; (2) such an act of repudiation
is clearly made known to the other co-owners; (3) the evidence thereon is clear and conclusive, and (4) he
has been in possession through open, continuous, exclusive, and notorious possession of the property for
the period required by law. Especially, by keeping the other co-owners in the dark about the repudiation
made by a co-owner, it cannot be said that the repudiation was made known to the others; hence, it may
not give rise to prescription.

140
CO-OWNERSHIP
Extinguishment of Co-Ownership

EJECTMENT SUIT BY A CO-OWNER, DEEMED INSTITUTED FOR THE BENEFIT OF ALL;


EJECTMENT SUIT BY A CO-OWNER UNDER CLAIM OF EXCLUSIVE OWNERSHIP WILL NOT
PROSPER

114. Adlawan v. Adlawan


G.R. No. 161916, January 20, 2006
Ynares-Santiago, J.

FACTS:

This is a petition for review. The instant ejectment suit stemmed from the parties’ dispute over a lot and the
house built thereon, registered in the name of the late Dominador Adlawan. In his complaint, petitioner
Arnelito claimed to be the sole heir (acknowledged illegitimate child) of Dominador and thus he executed
an affidavit adjudicating to himself the lot and the house built thereon. That out of respect and generosity to
respondents who are the siblings of his father, he granted their plea to occupy the subject property provided
they would vacate the same should his need for the property arise. On the other hand, respondents Narcisa
and Emeterio denied that they begged petitioner to allow them to stay on the questioned property and
stressed that they have been occupying the lot and the house standing thereon since birth. The subject lot
was originally registered in the name of their deceased father and the ancestral home built thereon was
owned by their deceased parents. That ownership of the lot was transferred to the name of Dominador
through a simulated deed of sale solely for the purpose of obtaining a loan to finance the renovation of their
ancestral home. Notwithstanding the execution of the simulated deed, Dominador, then single, never
disputed his parents’ ownership of the lot. He and his wife, Graciana, did not disturb respondents’
possession of the property until they died in 1987 and 1997 respectively. Respondents argued that even if
petitioner is indeed Dominador’s acknowledged illegitimate son, his right to succeed is doubtful because
Dominador was survived by his wife, Graciana.

The MTC dismissed the unlawful detainer complaint holding that the establishment of petitioner’s filiation
and the settlement of the estate of Dominador are conditions precedent to the accrual of petitioner’s action
for ejectment. The RTC reversed the decision of the MTC holding that the title of Dominador over the lot
cannot be collaterally attacked. The CA set aside the decision of the RTC and reinstated the judgment of
the MTC. It ratiocinated that petitioner and the heirs of Graciana are co-owners of subject lot. As such,
petitioner cannot eject respondents from the property via an unlawful detainer suit filed in his own name and
as the sole owner of the property.

ISSUE:

Will the instant case for ejectment filed by the petitioner claiming exclusive ownership prosper?

HELD:

NO—First of all, the death of Graciana did not make petitioner the absolute owner because the share of
Graciana passed to her relatives by consanguinity and not to petitioner with whom she had no blood
relations. The CA thus correctly held that petitioner has no authority to institute the instant action as the sole
owner.

Moreover, under Art. 487 of the Civil Code, any co-owner instituting ejectment proceedings is deemed to
have instituted the same for all for all. Hence, if the co-owner expressly states that he is bringing the case
only for himself, the action should not be allowed to prosper. In the instant case, it is not disputed that
petitioner brought the suit for unlawful detainer in his name alone and for his own benefit to the exclusion of
the heirs of Graciana as he even executed an affidavit of self- adjudication over the disputed property.

141
CO-OWNERSHIP
Condominium Act

OWNERSHIP OF UNIT INCLUDES ONLY THE FOUR WALLS, CEILINGS, WINDOWS AND DOORS
THEREOF, NOT ROOF OR “AIR SPACE” ABOVE IT; RULES ON BUILDER IN GOOD FAITH DO NOT
APPLY TO PROPERTIES RECORDED UNDER AND GOVERNED BY THE CONDOMINIUM ACT

115. Leviste Management System, Inc. v. Legaspi Towers 200, Inc.


G.R. No. 199353, April 4, 2018
Leonardo-De Castro, J.

FACTS:
These are consolidated petitions under Rule 45 filed by Leviste Management System, Inc. (LEMANS) and
Legaspi Towers 200, Inc. (Legaspi Towers), both assailing the of the CA of affirming the Decision of the
RTC.

Legaspi Towers is a condominium building with 7 floors, a unit on the roof deck and two levels above said
unit called Concession 2 and Concession 3. The use and occupancy of the condominium building is
governed by the Master Deed with Declaration of Restrictions of Legaspi Towers (hereafter "Master Deed")
annotated on the transfer certificate of title of the developer, Legaspi Towers Development Corporation. In
1989, LEMANS decided to build another unit (hereafter "Concession 4") on the roof deck of Concession 3.
LEMANS was able to secure the building permit and commenced the construction thereof. Despite Legaspi
Corporation's notice that the construction of Concession 4 was illegal, LEMANS refused to stop its
construction. Legaspi Towers also argues that Concession 4 is an illegal construction, for being in violation
of the Condominium Act and the By Laws of Legaspi Towers. Legaspi Towers stresses that LEMANS failed
to comply with the Condominium Act, which requires the consent of the registered owners of the
condominium project for the amendment of the Master Deed. LEMANS contends that its construction of
another floor was in the exercise of its rights. The trial court held that the air space which LEMANS claims
is not on top of its unit but also on top of the condominium itself, owned and operated by defendant Legaspi
Towers.

As regards the appeal of Legaspi Towers, the CA held that while Concession 4 is indeed a nuisance,
LEMANS has been declared a builder in good faith and Concession 4 was built in good faith, thus it cannot
be demolished.

ISSUES:
1) Does LEMANS own the air space above Concession 3 and thus have the right to build Concession
4?
2) Is LEMANS a builder in good faith and thus Legaspi Towers does not have the right to demolish
Concession 4?

HELD:
1) NO—As correctly pointed out by the private respondent Legaspi, the air space wherein Concession
4 was built is not only above Concession 3, but above the entire condominium building. The
petitioner's [LEMANS'] ownership of Concession 3 does not necessarily extend to the area above
the same, which is actually the "air space" of the entire condominium building. The ownership of
the air space above Concession 3 is not a necessary incident of the ownership of Concession 3.
Pursuant to Secs. 2, 3(d) and 6 of R.A. No. 4726, otherwise known as The Condominium Act, what
a unit includes is only the four walls, ceilings, windows and doors thereof. It certainly does not
include the roof or the areas above it.

2) NO—Arts. 448 and 546 of the Civil Code on builders in good faith are inapplicable in cases covered
by the Condominium Act where the owner of the land and the builder are already bound by specific
legislation on the subject property (the Condominium Act), and by contract (the Master Deed and
the By-Laws of the condominium corporation).

It is recognized in jurisprudence that, as a general rule, Art. 448 on builders in good faith does not apply
where there is a contractual relation between the parties. The construction by LEMANS of Concession 4
contravenes the Master Deed by adding a third level above the roof deck.

142
As pointed out by Legaspi Towers and shown in the records, the Master Deed was never amended to reflect
the building of Concession 4. Furthermore, LEMANS failed to procure the consent of the registered owners
of the condominium project as required in the last paragraph of Section 4 of the Condominium Act. The By-
Laws of Legaspi Towers specifically provides that extraordinary improvements or additions must be
approved by the members in a regular or special meeting called for the purpose prior to the construction.
Instead of procuring the required consent by the registered owners of the condominium project pursuant to
the Condominium Act, or having Concession 4 approved by the members in a regular or special meeting
called for the purpose pursuant to the By-Laws, LEMANS merely had an internal arrangement with the then
president of Legaspi Towers. The same, however, cannot bind corporations, which includes condominium
corporations such as Legaspi Towers, as they can act only through their Board of Directors.

143
CO-OWNERSHIP
Condominium Act

OWNER-DEVELOPER MAY BE A MEMBER OF A CONDOMINIUM CORPORATION; OWNERSHIP OF


UNIT NEED NOT BE THROUGH SALE OR PURCHASE, IN ORDER TO BE ENTITLED TO SUCH
MEMBERSHIP IN CONDOMINIUM CORPORATION

116. Lim v. Moldex Land, Inc.


G.R. No. 206038, January 25, 2017
Mendoza, J.

FACTS:
Lim is a registered unit owner of Golden Empire Tower, a condominium project of Moldex, a real estate
company engaged in the construction and development of high-end condominium projects and in the
marketing and sale of the units thereof to the general public. Condocor, a non-stock, non-profit corporation,
is the registered condominium corporation for the Golden Empire Tower. Lim, as a unit owner of Golden
Empire Tower, is a member of Condocor. Moldex became a member of Condocor on the basis of its
ownership of the 220 unsold units in the Golden Empire Tower. Lim claimed that the individual respondents
(who are representatives of Moldex) are non-unit buyers, but all are members of the Board of Directors of
Condocor, having been elected during its organizational meeting in 2008 and re-elected in 2012. Lim
asserted that only unit buyers are entitled to become members of Condocor. She insisted that under P.D.
No. 957, a condominium corporation is an association of homeowners for the purpose of managing the
condominium project, among others. Thus, it must be composed of actual unit buyers or residents of the
condominium project. Lim further averred that the ownership contemplated by law must result from a sale
transaction between the owner-developer and the purchaser. She advanced the view that the ownership of
Moldex was only in the nature of an owner-developer and only for the sole purpose of selling the units.

ISSUE:
May an owner-developer such as Moldex be a member of a condominium corporation such as Condocor?

HELD:
YES— Lim's reliance of P.D. No. 957 is misplaced. There is no provision in P.D. No. 957 which states that
an owner-developer of a condominium project cannot be a member of a condominium corporation.

It is erroneous to argue that the ownership must result from a sale transaction between the owner-developer
and the purchaser. Such interpretation would mean that persons who inherited a unit, or have been donated
one, and properly transferred title in their names cannot become members of a condominium corporation.

Moreover, although the Condominium Act provides for the minimum requirement for membership in a
condominium corporation, a corporation's articles of incorporation or by-laws may provide for other terms of
membership, so long as they are not inconsistent with the provisions of the law, the enabling or master
deed, or the declaration of restrictions of the condominium project. Here, pursuant to Secs. 2 and 10 of the
Condominium Act, the Master Deed and Declaration of Restrictions, as well as the By-Laws of Condocor,
Moldex, as the owner of 220 unsold units and the parking slots and storage areas attached thereto, has
automatically become a member of Condocor upon the latter's creation.

It is noted that a condominium corporation is not just a management body of the condominium project. It
also holds title to the common areas, including the land, or the appurtenant interests in such areas. Clearly,
a homeowners association is different from a condominium corporation.

144
CO-OWNERSHIP
Condominium Act

COMMON SPACES DO NOT FORM PART OF A CONDOMINIUM UNIT

117. Poole-Blunden v. Union Bank of the Philippines


G.R. No. 205838, November 29, 2017
Leonen, J.

FACTS:
In March 2001, Poole-Blunden came across a public auction advertisement placed by Union Bank in the
Manila Bulletin of several properties including subject condominium unit, identified as Unit 2-C of T-Tower
Condominium (the "Unit"). The Unit was advertised to have an area of 95 square meters. Thinking that it
was sufficient and spacious enough for his residential needs, Poole-Blunden decided to register for the sale
and bid on the unit. About a week prior to the auction, Poole-Blunden, accompanied by a representative of
Union Bank visited the unit for inspection. Poole-Blunden did not doubt the unit's area as advertised. After
placing the winning bid, Poole-Blunden entered into a Contract to Sell with Union Bank. Eventually, Poole-
Blunden occupied the unit after fully paying the bid price. He took rough measurements of the Unit, which
indicated that its floor area was just about 70 square meters, not 95 square meters, as advertised by Union
Bank. Poole-Blunden got in touch with an officer of Union Bank to raise the matter, but no action was taken.
He then asked for a rescission of the Contract to Sell, along with a refund of the amounts he had paid, in
the event that it was conclusively established that the area of the unit was less than 95 square meters. In a
letter, Union Bank informed Poole-Blunden that after inquiring with the Housing and Land Use Regulatory
Board (HLURB), the Homeowners' Association of T-Tower Condominium, and its appraisers, the Unit was
confirmed to be 95 square meters, inclusive of the terrace and the common areas surrounding it.

ISSUE:
Are outside spaces such as a terrace, and common areas part of a condominium unit’s total area?

HELD:
No. Section 6(a) of the Condominium Act specifies the reckoning of a condominium unit's bounds. It also
specifies that areas of common use "are not part of the unit": Section 6. Unless otherwise expressly provided
in the enabling or master deed or the declaration of restrictions, the incidents of a condominium grant are
as follows: (a) The boundary of the unit granted are the interior surfaces of the perimeter walls, floors,
ceilings, windows and doors thereof. The following are not part of the unit bearing walls, columns, floors,
roofs, foundations and other common structural elements of the building; lobbies, stairways, hallways, and
other areas of common use, elevator equipment and shafts, central heating, central refrigeration and central
air-conditioning equipment, reservoirs, tanks, pumps and other central services and facilities, pipes, ducts,
flues, chutes, conduits, wires and other utility installations, wherever located, except the outlets thereof
when located within the unit. Thus, the unit sold to petitioner was deficient in relation to its advertised area.

145
SOME SPECIAL PROPERTIES
Waters

DAMAGE OCCASIONED BY ARTIFICIALLY COLLECTED WATER IN MAN-MADE LAGOONS BY A


HIGHER ESTATE ENTITLES THE SERVIENT ESTATE TO COMPENSATION

118. Spouses Ermino v. Golden Village Homeowners Association, Inc.


G.R. No. 180808, August 15, 2018
Caguioa, J.

FACTS:
Spouses Ermino are residents of Alco Homes Subdivision located beside Golden Village Subdivision. The
Hilltop City Subdivision is found at the upper portion of Alco Homes, making it a higher estate, while Golden
Village is located beside Alco Homes, which makes both Alco Homes and Golden Village lower estates vis-
a-vis Hilltop City Subdivision.

After continuous heavy rains which caused a large volume of water to fall from the hilltop subdivision to the
subdivision below, which also directly hit Spouses Ermino’s house, their fence, furniture, appliances and
car were all damaged.

Spouses Ermino then filed a complaint for damages against E.B. Villarosa, the developer of Hilltop City
Subdivision, and Golden Village Homeowners Association (GVHAI). Spouses Ermino blamed E.B. Villarosa
for negligently failing to observe DENR Rules and Regulations, and failing to provide retaining walls and
other flood control devices which could have prevented the softening of the soil and consequent inundation.

ISSUE:
Is Respondent Villarosa liable for damages caused by the burdensome flooding of water caused by
constructions in a higher estate?

HELD:
YES. Article 637 of the Civil Code and Article 50 of the Water Code provides that the owner of the higher
estate cannot make works which will increase the natural flow of water which the lower estate is obliged to
receive.

Thus, when the waters which flow from a higher state are those which are artificially collected in man-made
lagoons, any damage occasioned thereby entitles the owner of the lower or servient estate to compensation.

Here, it is evident that the bulldozing and construction works done by E.B. Villarosa gave Alco Homes and
Golden Village's obligation, as lower estates, more burdens than what the law contemplated. The bulldozing
and flattening of the hills led to the softening of the soil that could then be easily carried by the current of
water whenever it rained.

146
SOME SPECIAL PROPERTIES
Waters

LGU CANNOT ISSUE AN ORDINANCE ARROGATING UNTO ITSELF THE POWER TO CONTROL AND
REGULATE USE OF WATER WHICH PERTAINS TO THE STATE, THROUGH THE NWRB

119. City of Batangas v. Philippine Shell Petroleum Corp.


G.R. No. G.R. No. 195003, June 7, 2017
Caguioa, J.

FACTS:
Pilipinas Shell Petroleum Corporation (PSPC) owns and operates a refinery in Tabangao, Batangas City.
The Sangguniang Panlungsod of the City of Batangas enacted an Ordinace which requires heavy industries
operating along the portions of Batangas Bay within the territorial jurisdiction of Batangas City to construct
desalination plants to facilitate the use of seawater as coolant for their industrial facilities.

PSPC then filed against Batangas City and the Sangguniang Panlungsod a Petition for Declaration of Nullity
before the RTC praying that the Assailed Ordinance be declared null and void for contravening the Water
Code of the Philippines and encroaching the power of the National Water Resources Board to regulate and
control the Philippines’ Water resources.

ISSUE:
Can the control and regulation of the use of water be made subject of a city ordinance?

HELD:
NO. The Water Code governs the ownership, appropriation, utilization, exploitation, development,
conservation and protection of water resources. Under Article 3 thereof, water resources are placed under
the control and regulation of the government through the NWRB (then, the NWRC).

In turn, the privilege to appropriate and use water is one which is exclusively granted and regulated by the
State through water permits issued by the NWRB. Once granted, these water permits continue to be valid
save only for reasons spelled out under the Water Code itself. Conversely, the power to modify, suspend,
cancel or revoke water permits already issued also rests with NWRB.

There is no doubt, therefore, that the Assailed Ordinance effectively contravenes the provisions of the Water
Code as it arrogates unto Batangas City the power to control and regulate the use of ground water which,
by virtue of the provisions of the Water Code, pertains solely to the NWRB. By enacting the Assailed
Ordinance, Batangas City acted in excess of the powers granted to it as an LGU, rendering the Assailed
Ordinance ultra vires.

147
SOME SPECIAL PROPERTIES
Waters

RIGHT TO APPROPRIATE WATER OR TO TAKE OR DIVERT WATERS FROM A NATURAL SOURCE


MAY ONLY BE AFTER A WATER PERMIT THEREFOR IS ISSUED BY THE NWRB

120. First Mega Holdings Corp. v. Guiguinto Water District


G.R. No. 208383; June 8, 2016
Perlas-Bernabe, J.

FACTS:
Petitioner filed with the NWRB a Water Permit Application (WPA) for the installation of a deep well that
would supply the water resources requirements of its gasoline station and commercial complex in Guiguinto,
Bulacan. Respondent filed its Protest against the petitioner’s WPA averring that: a) the water level in
Guiguinto is at critical level and the water exploration to be conducted by petitioner would hamper the water
requirements of the municipality and be detrimental to its water service and b) petitioner disregarded and
violated existing laws, rules and regulations because it had already started drilling operations before it
sought the NWRB’s approval.

ISSUE:
Should the petitioner’s WPA be denied for failing to secure a permit before starting to drill for extraction of
ground water?

HELD:
YES. It should be emphasized that it is only through a duly issued water permit that any person acquires
the right to appropriate water, or to take or divert waters from a natural source in the manner and for any
purpose allowed by law.

In the present case, the NWRB correctly denied petitioner's WPA for its flagrant disregard of the Water
Code and its IRR. Records show that petitioner drilled a deep well and installed a water pump without having
first secured the necessary permit to drill. Moreover, despite the NWRB's Cease and Desist Order refraining
it from operating the water pump, petitioner extracted water from the deep well.

It should be remembered that where extraction of ground water is sought, a permit to drill must first be
secured from the NWRB. Before such permit to drill is issued, the NWRB shall conduct a field investigation
to determine any adverse effect that may be caused to public or private interests.

Only after it has determined that the application meets the requirements and is not prejudicial to any public
or private interests shall it issue the permit to drill which shall be regarded as a temporary permit, until the
rate of water withdrawal/yield of the well has been determined and assessed, and the application is finally
either approved and a water permit is issued subject to such conditions as the NWRB may impose, or
disapproved and returned to the applicant, stating the reasons therefor.

148
SOME SPECIAL PROPERTIES
Waters

NATIONAL POWER CORP. MAY TRANSFER OPERATION AND POSSESSION OF MULTI-PURPOSE


HYDROPOWER FACILITIES BUT SHOULD RETAIN THE WATER PERMIT THEREFOR

121. Initiatives for Dialogue and Empowerment Through Alternative Legal Services, Inc. v. Power
Sector Assets and Liabilities Management Corp.
G.R. No. 192088, October 9, 2012
Villarama, J.

FACTS:
PSALM is a GOCC tasked to manage the orderly sale, disposition, and privatization of the NPC’s assets.
In line with the privatization of the 246-Megawatt Angat Hydro-Electric Power Plant (AHEPP), PSALM’s
Board of Directors approved and confirmed the issuance of the Notice of Award to the highest bidder, Korea
Water Resources Corporation (K-Water).

ISSUE:
Does the sale of the power plant automatically transfer the water rights of NPC to the buyers of its multi-
purpose hydropower facilities?

HELD:
NO. The law, specifically EPIRA, does not authorize NPC to assign or transfer its water rights in case of
transfer of operation and possession of multi-purpose hydropower facilities.

Since only the power plant is to be sold and privatized, the operation of the non-power components such
as the dam and reservoir, including the maintenance of the surrounding watershed, should remain under
the jurisdiction and control of NPC which continue to be a government corporation. There is therefore no
necessity for NPC to transfer its permit over the water rights to K-Water. Pursuant to its purchase and
operation/management contracts with K-Water, NPC may authorize the latter to have beneficial use of the
water in the dam to generate electricity but NPC should retain its water permit.

That the EPIRA itself did not ordain any transfer of water rights leads us to infer that Congress intended
NPC to continue exercising full supervision over the dam, reservoir and, more importantly, to remain in
complete control of the extraction or diversion of water from the Angat River. In this way, the State’s full
supervision and control over the country’s water resources is also assured notwithstanding the privatized
power generation business.

149
SOME SPECIAL PROPERTIES
Waters

EASEMENTS ESTABLISHED FOR PUBLIC OR COMMUNAL USE ARE CONSIDERED PUBLIC LAND
UNDER SPECIAL LAWS AND SHALL NOT BE RETAINED BY THE OWNER IN THE SAME CONCEPT
AS IN THE CIVIL CODE

122. Pilar Development Corp. v. Dumadag


G.R. No. 194336, March 11, 2013
Peralta, J.

FACTS:
Petitioner filed a complaint for accion publiciana with damages against respondents for allegedly building
their shanties, without its knowledge and consent, in its property in Las Piñas City.

It claims that said property is duly registered and was designated as an open space of Pilar Village
Subdivision for recreational activities. The lower court dismissed the complaint, finding that the land being
occupied by respondent is situated on the sloping area going down and leading towards the Mahabang Ilog
Creek, and within the designated three-meter legal easement; thus, the lower court held that it should be
considered as public property and part of public dominion under Art. 502 of the Civil Code and likewise
cannot be owned by the petitioner.

ISSUE:
Can petitioner retain ownership of the subject property occupied by the respondents within the three-meter
strip reserved for public easement?

HELD:
NO. The Court remarked that neither the respondent may occupy the same. While Art. 630 of the Civil Code
provides for the general rule that "the owner of the servient estate retains the ownership of the portion on
which the easement is established, Art. 635 thereof is specific in saying that "all matters concerning
easements established for public or communal use shall be governed by the special laws and regulations
relating thereto.

Thus, Art. 51 of the Water Code of the Philippines provides, “The banks of rivers and streams and the
shores of the seas and lakes throughout their entire length and within a zone of three (3) meters in urban
areas . . . are subject to the easement of public use in the interest of recreation, navigation, floatage, fishing
and salvage”. Moreover, according to a series of DENR issuances, when titled lands are subdivided or
consolidated-subdivided into lots for residential, commercial or industrial purposes the segregation of the
three (3) meter wide strip along the banks of rivers or streams shall be observed and be made part of the
required easement pursuant to the above cited provision.

Following the foregoing special provisions of law, the strip shall be preserved and unlike the concept of
easement in the Civil Code, shall not be retained in private ownership and instead, will be public land.

150
POSSESSION
Concept of Possession

POSSESSTION IN THE CONCEPT OF OWNER MAY BE PROVED BY MATERIAL OCCUPATION;


NEITHER TAX RECEIPTS NOR DECLARATION OF OWNERSHIP FOR TAXATION PURPOSES ALONE
CONSTITUTE EVIDENCE OF OWNERSHIP NOR THE RIGHT TO POSSESS REALTY FLOWING
THEREFROM

123. Rizal Cement Co. v. Villareal


G.R. No. L-30272, February 28, 1985
Cuevas, J.

FACTS:
Private respondents filed an application for registration alleging that the lots were assessed for taxation
purposes and that applicants have acquired the said lands by purchase from spouses Cervo and Guillermo,
as evidenced by a Deed of Sale. However, petitioner filed an opposition thereto, alleging that it is the owner
of the unregistered land and that it has been in continuous and adverse possession of said land since 1911
and has been religiously paying the real estate tax from the time it had acquired said property from the
previous owner.

Petitioner submitted the survey plan which was approved on 1912, a sketch plan of the geographical position
of the real properties, tax declaration and real estate tax receipts. Respondent, on the other hand, presented
three witnesses to testify as to their continuous and actual possession of the subject lot.

ISSUE:
Who between the parties possessed the subject lot in the concept of an owner?

HELD:
THE RESPONDENTS—As aptly found by the appellate court, it was respondents, rather than petitioner,
who possess the property in the concept of an owner. Possession, as a legal concept, is acquired by the
material occupation of a thing or the exercise of a right or by the fact it is subject to the action of our will, or
by the proper acts and legal formalities established for acquiring such right.

Being an attribute of ownership, repondent’s possession of the land goes far to tip the scale in their favor.
The right to possess flows from ownership. No person will suffer adverse possession by another of what
belongs to him.

Petitioner's evidence, in contract, merely consists of tax receipts, tax declaration and a survey plan. They
are not conclusive and indisputable basis of one's ownership of the property in question. Assessment alone
is of little value as proof of title. Mere tax declaration does not vest ownership of the property upon the
declarant. Settled is the rule that neither tax receipts nor declaration of ownership for taxation purposes
alone constitutes sufficient evidence of ownership or of the right to possess realty. They must be supported
by other effective proofs. Neither can the survey plan or technical descriptions prepared at the instance of
the party concerned be considered in his favor, the same being self-serving.

151
POSSESSION
Concept of Possession

POSSESSION AS A FACT CANNOT BE RECOGNIZED IN TWO DIFFERENT PERSONALITIES; ONE


LONGER IN POSSESSION, PREFERRED; POSSESSION IN GOOD FAITH CEASES WHEN DEFECT IN
TITLE IS KNOWN TO POSSESSOR

124. Wong v. Carpio


G.R. No. L-50264, October 21, 1991
Bidin, J.

FACTS:
Respondent-plaintiff Manuel Mercado acquired his rights to possess the land in litigation, from William Giger
by virtue of a deed of sale. Plaintiff began harvesting only the coconut fruits and he paid the taxes on the
land for Mr. Giger. He went periodically to the land to make copra but he never placed any person on the
land in litigation to watch it. Neither did he reside on the land as he is a businessman and storekeeper by
occupation and resides at Lower Sta. Maria, Davao del Sur while the land in litigation is at Colongan, Sta.
Maria. Neither did he put any sign or hut to show that he is in actual possession. He knew defendants’
laborers were in the land in suit as early as August 1976 and that they have a hut there but he did not do
anything to stop them. Instead plaintiff was happy that there were people and a hut on the land in suit.

Petitioner-defendant Ignacio Wong went to the land in litigation to find out if there were other people residing
there or claiming it besides the owner and he found none. So, defendant Ignacio Wong bought the parcel
of land in litigation from William Giger. Ignacio Wong asked for the delivery of the title to him and so he has
in his possession TCT in the name of William Giger. Mr. Wong declared the land in suit for taxation purposes
in his name. The defendant Wong placed laborers on the land in suit, built a small farm house after making
some clearings and fenced the boundaries. He also placed signboards.

Plaintiff Manuel Mercado again went to the land in suit to make copras. That was the time the matter was
brought to the attention of the police. Then defendant Wong ordered the hooking of the coconuts from the
land in litigation and nobody disturbed him; however, defendant received a copy of plaintiff’s complaint for
forcible entry sometime in November 1976, yet continued in his possession of the land.

ISSUES:
1) Did private respondent Mercado have actual and lawful possession of the land?
2) Did petitioner Wong possess the land in good faith?

HELD:
1) YES. Possession as a fact cannot be recognized at the same time in two different personalities
except in the cases of co-possession. Should a question arise regarding the fact of possession, if
there are two possessions, the one longer in possession is preferred. In this case, private
respondent clearly possessed the land longer than petitioner did and pursuant to a sale transaction
ahead of the latter. Hence, private respondent may be said to have been the one in actual and
lawful possession of the land.

2) NO. Before November 1976, he did possess the same in good faith; however, upon receiving a
copy of the complaint for forcible entry, he may no longer be said to have possessed the land in
good faith.

Possession in good faith ceases from the moment defects in the title are made known to the possessors,
by extraneous evidence or by suit for recovery of the property by the true owner. Whatever may be the
cause or the fact from which it can be deduced that the possessor has knowledge of the defects of his title
or mode of acquisition, it must be considered sufficient to show bad faith.

152
POSSESSION
Concept of Possession

THERE IS POSSESSION IF A PERSON WAS ABLE TO SUBJECT THE PROPERTY TO THE ACTION
OF HIS WILL

125. Somodio v. Court of Appeals


G.R. No.82680, August 15, 1994
Quiason, J.

FACTS:
This is a petition for review on certiorari under Rule 45 to reverse and set aside the Decision and the
Resolution of the CA.

Wilfredo Mabugat and Nicanor Somodio bought a lot in General Santos City. They partitioned the property
into two portions, with Somodio taking the western part. Immediately after the partition, Somodio took
possession of his portion and planted thereon ipil-ipil trees, coconut trees and other fruit-bearing. He also
began constructing a house on the said lot but because of his employment, he left the unfinished structure
to his uncle. Then Respondent Ebenecer Purisima entered the subject lot and constructed a house thereon.
This prompted Somodio to file complaint for forcible entry.

Purisima averred that the lot was a portion of the land subject of his application for miscellaneous sales
patent with the Bureau of Lands. He contended that his father had surveyed a parcel of land including the
subject lot and that his father’s survey plan was approved by the Director of Lands in 1960.

ISSUE:
Who between the claimants had priority in possession of the subject property?

HELD:
Petitioner Somodio had priority possession. Following Art. 531 of the Civil Code, it is sufficient that one was
able to subject the property to the action of his will. Possession in the eyes of the law does not mean that a
man has to have his feet on every square meter of ground before it can be said that he is in possession.

Petitioner took possession of the property sometime in 1974 when he planted the property to coconut trees,
ipil- ipil trees and fruit trees. In 1976, he started the construction of a building on the property. It is immaterial
that the building was unfinished and that he left for Kidapawan for employment reasons and visited the
property only intermittently. Possession in the eyes of the law does not mean that a man has to have his
feet on every square meter of ground before it can be said that he is in possession. It is sufficient that
petitioner was able to subject the property to the action of his will.

Even if the Court of Appeals is correct in its finding that petitioner started introducing improvements on the
land only in 1981, he still enjoyed priority of possession because respondent Purisima entered the premises
only in 1983. Moreover, neither is the fact that respondent Purisima's father surveyed the property of help
to his cause. Respondent Purisima's father surveyed the land for the Small Farmers Fishpond Association,
Inc., not for himself. Although respondent Purisima claims that Lot No. 6328-X was in payment of his fee for
the services of his father and that he caused the construction of a perimeter wall in the area, these facts do
not mean that respondent Purisima himself had prior possession. Neither did he present proof that when
his father allegedly took possession of the land, and 1983, when said respondent himself entered the land,
his father ever exercised whatever right of possession he should have over the property.

153
POSSESSION
Concept of Possession

PAYMENT OF RENTALS IS AN ACKNOWLEDGEMENT THAT ANOTHER HAS A SUPERIOR RIGHT


OF POSSESSION

126. Maglucot-Aw v. Maglucot


G.R. No.132518, March 28, 2000
Kapunan, J.

FACTS:
This petition for review on certiorari assails the Decision, of the CA which reversed and set aside the
Decision, of the RTC, in an action for recovery of possession and damages.

Sometime in 1963, Guillermo Maglucot rented a portion of Lot No. 1639-D(subject lot). Subsequently,
Leopoldo and Severo, both surnamed Maglucot, rented portions of subject lot in 1964 and 1969,
respectively, and each paying rentals therefor. Said respondents built houses on their corresponding leased
lots. They paid the rental amount of P100.00 per annum to Mrs. Ruperta Salma, who represented the heirs
of Roberto Maglucot, petitioners predecessor-in-interest. In December 1992, however, said respondents
stopped paying rentals claiming ownership over the subject lot. Petitioners thus filed the
complaint. Petitioners assert that respondents are estopped from claiming to be co-owners of the subject
lot in view of the mutual agreement in 1946, judicial confirmation in 1952, and respondents’ acquiescence’
because they themselves exclusively exercised ownership over the land beginning 1952 up to the present.

ISSUE:
Was respondent Maglucot holder of the land in the concept of an owner

HELD:
NO. The payment of rentals by respondents reveal that they are mere lessees. As such, the possession of
respondents over Lot No. 1639-D is that of a holder and not in the concept of an owner. One who possesses
as a mere holder acknowledges in another a superior right which he believes to be ownership, whether his
belief be right or wrong. Since the possession of respondents were found to be that of lessors of petitioners,
it goes without saying that the latter were in possession of Lot No. 1639-D in the concept of an owner from
1952 up to the time the present action was commenced.

The records show that respondents were paying rent for the use of a portion of Lot No. 1639-D. Had they
been of the belief that they were co-owners of the entire Lot No. 1639 they would not have paid rent.

154
POSSESSION
Concept of Possession

POSSESSION CANNOT BE ACQUIRED THROUGH FORCE OR VIOLENCE

127. Cequena v. Bolante


G.R. No.137944, April 6, 2000
Panganiban, J.

FACTS:
Before us is a Petition for Review on Certiorari of the Decision of the CA. reversing the decision of the RTC.

Margarito Mendoza and Sinforoso Mendoza are brothers. Petitioners were daughters of Margarito. Prior to
1954, a land was originally declared for taxation purposes in the name of Sinforoso Mendoza, father of
respondent Honorata Mendoza Bolante. When Sinforoso died, Margarito took possession of the land and
cultivated it with his son Miguel. At the same time, respondent and her mother continued residing on the lot.
On the basis of an affidavit, the tax declaration in the name of Sinforoso was cancelled and subsequently
declared in the name of Margarito Mendoza. Honorata and Miguel Mendoza, son of Margarito, during the
cadastral survey had a dispute on the ownership of the land. A case ensued wherein the trial court declared
petitioners (heirs of Margarito) as owners of the land.

The CA ruled that the respondent was the preferred possessor under Article 538 of the Civil Code because
she was in notorious, actual, exclusive and continuous possession of the land since 1985. Petitioners
dispute this ruling. They contend that she came into possession through force and violence, contrary to
Article 536 of the Civil Code.

ISSUE:
Was respondent the preferred possessor of the land

HELD:
Yes. We concede that despite their dispossession in 1985, the petitioners did not lose legal possession
because possession cannot be acquired through force or violence. To all intents and purposes, a possessor,
even if physically ousted, is still deemed the legal possessor. However, possession by the petitioners does
not prevail over that of the respondent. Possession by the former before 1985 was not exclusive, as the
latter also acquired it before 1985. The records show that the petitioners' father and brother, as well as the
respondent and her mother were simultaneously in adverse possession of the land. Before 1985, the subject
land was occupied and cultivated by the respondent's father (Sinforoso), who was the brother of petitioners'
father (Margarito). When Sinforoso died, Margarito took possession of the land and cultivated it with his son
Miguel. At the same time, respondent and her mother continued residing on the lot. When respondent came
of age in 1948, she paid realty taxes for the years 1932-1948. Margarito declared the lot for taxation in his
name in 1953 and paid its realty taxes beginning 1952. When he died, Miguel continued cultivating the land.
As found by the CA, the respondent and her mother were living on the land, which was being tilled by Miguel
until 1985 when he was physically ousted by the respondent.

Based on Article 538 of the Civil Code, the respondent is the preferred possessor because, benefiting from
her father's tax declaration of the subject lot since 1926, she has been in possession thereof for a longer
period. On the other hand, petitioners' father acquired it’s possession only in 1952.

155
POSSESSION
Concept of Possession

UNLAWFUL DETAINER INVOLVES THE PERSON’S WITHHOLDING FROM ANOTHER OF THE


POSSESSION OF THE REAL PROPERTY TO WHICH THE LATTER IS ENTITLED, AFTER THE
EXPIRATION OR TERMINATION OF THE FORMER’S RIGHT TO HOLD POSSESSION UNDER THE
CONTRACT, EITHER EXPRESSED OR IMPLIED

128. Carbonilla v. Abiera


G.R. No. 177637, July 26, 2010
Nachura, J.

FACTS:
This is a petition for review assailing the decision of the CA dismissing petitioner Dr. Dioscoro Carbonilla’s
(Carbonilla) complaint for ejectment against Respondents Marcelo and Maricris Abiera.

Carbonilla alleged that he is the owner of a land covered by a certificate of title, and declared for assessment
and taxation purposes in his name. He also contended that he is the owner of the residential building
standing on the land, which building he acquired through a Deed of Extrajudicial Settlement of Estate.
Carbonilla maintained that the respondents occupied the building by mere tolerance of the previous owners.
Carbonilla demanded the respondents to leave the premises but they failed and refused to do so.
Respondents on the other hand averred that they occupied the building as owners, having inherited it from
their ascendants, but was not declared for tax purposes. However, as to the land, which they also inherited
from their ascendant, was declared for tax purposes.

ISSUE:
Is the petitioner, as the owner of the land, entitled to the possession of the building situated therein?

HELD:
No. While Carbonilla may have proven his ownership of the land, he failed to present any evidence to
substantiate his claim of ownership or right to the possession of the building.

A requisite for a valid cause of action in an unlawful detainer case is that possession must be originally
lawful, and such possession must have turned unlawful only upon the expiration of the right to possess. It
must be shown that the possession was initially lawful; hence, the basis of such lawful possession must be
established. If, as in this case, the claim is that such possession is by mere tolerance of the plaintiff, the
acts of tolerance must be proved.

Carbonilla failed to prove that respondents’ possession was based on his alleged tolerance. A bare
allegation of tolerance will not suffice. He must, at least, show overt acts indicative of his or his predecessor’s
permission to occupy the subject property. Tolerance must be present right from the start of possession
sought to be recovered to be within the purview of unlawful detainer. Mere tolerance always carries with it
"permission" and not merely silence or inaction for silence or inaction is negligence, not tolerance.

Hence, for Carbonilla’s failure to prove his ownership and right to possess the building, the petition was
dismissed without prejudice to the proper remedy available to Carbonilla.

156
POSSESSION
Loss of Possession

THE POSSESSOR MAY LOSE HIS POSSESSION: (1) BY THE ABANDONMENT OF THE THING; (2)
BY TRANSFER TO ANOTHER FOR A GOOD OR VALUABLE CONSIDERATION; (3) BY THE
DESTRUCTION OR TOTAL LOSS OF THE THING OR BY THE THING BECOMING UNMARKETABLE;
(4) BY THE POSSESSION OF ANOTHER, EVEN AGAINST THE WILL OF THE FORMER POSSESSOR,
IF THE NEW POSSESSION HAS LASTED MORE THAN ONE YEAR

129. Aragon v. Insular Government


G.R. No. L-6019, March 25, 1911
Carson, J.

FACTS:
This is an appeal from a decree of the Court of Land Registration adjudicating title to a parcel of land in
favor of the appellee, Juan Aragon (Aragon) and ordering its registry in accordance with the provisions of
"The Land Registration Act."

In the year 1892 a possessory title to the land in question was duly registered in favor of one of the
predecessors in interest of Aragon. From time immemorial, Aragon and his predecessors in interest have
been in possession of the land in question, under an undisputed claim of ownership. The land in question
was originally well above the ebb and flow of the tide and it was only in later years have the waters risen to
such a height along the shores of the Bay of Manila at the point of covering the land in question completely
at high tide, though the receding waters left it completely bare at low tide. However, it does not definitely
appear whether this is due to changes in the current and flow of the waters in the bay, or to the gradual
sinking of the land along the coast.

The Government of the Philippine Islands (Government) objected to the application for registry on the
ground that, as it alleges, the land in question is a part of the public domain, as defined in Art. 339 (1), of
the Civil Code (now Art. 420 (1)) and also as defined in Article 1 of "The Law of Waters"

ISSUE:
Should the ownership and possession of the land along the shores of the Manila Bay be granted to Aragon?

HELD:
Yes, the ownership and possession of the land may still be adjudicated in favor of Aragon. Article 460 of
that code is as "ART. 460. The possessor may lose his possession: 1.) By the abandonment of the thing.
2.) By transfer to another for a good or valuable consideration. 3.) By the destruction or total loss of the
thing or by the thing becoming unmarketable. 4.) By the possession of another, even against the will of the
former possessor, if the new possession has lasted more than one year.

If the Government is justified in disturbing the possession of the applicants, it can only be on the ground
that they have abandoned their property, or that it has been totally destroyed and has now become a part
of the public domain by the erosive action of the sea. In this case, Aragon and his predecessors in interest
have never abandoned their possession under a claim of ownership of the land. There has been no such
destructive or total loss of the property as would justify a holding that the owners have lost possession.
Doubtless the property has been injured by the erosive action of the sea. Doubtless the owners in order to
profitably enjoy the possession of this property will be compelled to make some relatively small expenditures
by way of a "fill" or a retaining wall. The Government's claim of ownership, on the ground that this is a part
of the shore of Manila Bay, necessarily falls to the ground. The owners of the land in question have never
in fact nor in intent abandoned it, and that keeping in mind its location and actual condition it cannot be said
to have been totally destroyed for the purposes for which it was held by them, so as to have become a part
of the shore of the Bay of Manila.

Hence, having no total destruction of the property or abandonment on the part of Aragon and his
predecessors in interest, the ownership and possession of the lot in question must be awarded to Aragon.

157
POSSESSION
Loss of Possession

THE REAL RIGHT OF POSSESSION IS NOT LOST TILL AFTER THE LAPSE OF TEN YEARS

130. Catholic Vicar Apostolic of the Mountain Province v. Court of Appeals


G.R. Nos. 80294-95, March 23, 1990
Gancayco, J.

FACTS:
This is a case filed by Catholic Vicar (petitioner), questioning the previous decision of the SC which upheld
the CA, affirming the RTC’s decision ordering the petitioner to return and surrender the lots in question to
the heirs of Egmidio Octaviano and Juan Valdez (private respondents).

The case involves a land registration case where petitioner and the heirs of Egmidio Octaviano and Juan
Valdez (private respondents) were asking for confirmation of their alleged imperfect titles to the lots in
question under Section 49 (b) of the Public Land Act. The petitioner was in possession of the said property
as borrower in commodatum from private respondents since 1906, but in 1951 petitioner repudiated the
trust when it declared the property for tax purposes under its name. When it filed its application for
registration of the said property in 1962, petitioner had been in adverse possession of the same for at least
11 years. In its decision, the CA ruled that neither the petitioner nor the private respondents are entitled to
the confirmation of imperfect title over said two lots.

ISSUE:
May the Court order the petitioner to return and surrender the lots to the private respondents when it was
found that neither the petitioner nor the private respondent are entitled to the confirmation of imperfect title
over said lots?

HELD:
No. When the Court found that neither of the claiming parties are entitled to the confirmation of imperfect
title, the lots in question remained part of the public lands.

Art. 555 of the Civil Code provides that:


“A possessor may lose his possession:
xxx
(4) By the possession of another, subject to the provisions of Article 537, if the new possession has
lasted longer than one year. But the real right of possession is not lost till after the lapse of ten
years.”

From the foregoing provision of the law, the real right of possession of private respondents over the property
was lost or no longer exists after the lapse of 10 years that petitioner had been in adverse possession
thereof. Thus, the action for recover of possession of said property filed by private respondents against
petitioner must fail.

Hence, the trial court and the Court of Appeals erred in declaring the private respondents to be entitled to
the possession thereof. Said lots are part of the public domain.

158
POSSESSION
Loss of Possession

LANDS COVERED BY A TITLE CANNOT BE ACQUIRED BY PRESCRIPTION OR ADVERSE


POSSESSION

131. Supapo v. Spouses de Jesus


G.R. No. 198356, April 20, 2015
Villarama, Jr., J.

FACTS:
This is a petition for review on certiorari filed by petitioners Esperanza and Romeo Supapo (petitioners) to
assail the decision of the CA which dismissed the complaint for accion publiciana.

Petitioners filed a complaint for accion publiciana with the MeTC against respondents, seeking to compel
the latter to vacate the subject lot covered by TCT registered and titled under the name of the petitioners.
Petitioners did not reside on the subject lot. They also did not employ an overseer but they made sure to
visit at least twice a year. During one of their visits in 1992, they saw two houses built on the subject lot.
The houses were built without their knowledge and permission.

The respondents point out that the petitioners filed the complaint for accion publiciana on March 7, 2008 or
more than ten years after the certificate to file action was issued on November 25, 1992. The respondents
contend that the petitioners may no longer recover possession of the subject property. They posit that the
petitioners have lost their right to recover possession because of laches.

ISSUE:
Whether or not the action for accion publiciana has already prescribed after 10 years if the subject land is
covered by a Torrens title?

HELD:
No. The action is imprescriptible if the property in question is covered by a Torrens title. Section 47 of P.D.
No. 1529 states that “[r]egistered land not subject to prescriptions. No title to registered land in derogation
of the title of the registered owner shall be acquired by prescription or adverse possession.

Lands covered by a title cannot be acquired by prescription or adverse possession. A claim of acquisitive
prescription is baseless when the land involved is a registered land because of Article 1126 of the Civil
Code in relation to Act 496 (now, Section 47 of P.D. No. 1529. In addition to the imprescriptibility, the person
who holds a Torrens Title over a land is also entitled to the possession thereof. The right to possess and
occupy the land is an attribute and a logical consequence of ownership. Corollary to this rule is the right of
the holder of the Torrens Title to eject any person illegally occupying their property. Again, this right is
imprescriptible.

Hence, the petitioner's right to recover possession of the subject lot is not barred by prescription.

159
POSSESSION
Theory of Irrevindicability

ONE WHO HAS BEEN UNLAWFULLY DEPRIVED OF PERSONAL PROPERTY MAY RECOVER IT
FROM ANY PERSON POSSESSING IT

132. EDCA Publishing & Distributing Corp. v. Santos


G.R. No. 80298, April 26, 1990
Cruz, J.

FACTS:
The case calls for the interpretation of Article 559 of the Civil Code and raises the particular question of
when a person may be deemed to have been "unlawfully deprived" of movable property in the hands of
another.

A person identifying himself as Professor Jose Cruz (Cruz) placed an order by telephone with the petitioner
company (EDCA) for 406 books, payable on delivery. EDCA prepared the corresponding invoice and
delivered the books as ordered, for which Cruz issued a personal check. Cruz sold 120 books to private
respondent Leonor Santos (Santos). Upon suspicion over a second order placed by Cruz, EDCA went to
the police, which set a trap and arrested Cruz who turned out to be an impostor. EDCA sought the assistance
of the police which seized the 120 books without warrant. Private respondents sued for recovery of the
books after demand for their return was rejected by EDCA. Ownership of the books was recognized in the
private respondents by the MTC, which was sustained by the RTC, which was in turn sustained by the CA.
The petitioner contended that the private respondents have not established their ownership of the disputed
books because they have not even produced a receipt to prove they had bought the stock. It averred that
the owner who has been unlawfully deprived of personal property is entitled to its recovery except only
where the property was purchased at a public sale, in which event its return is subject to reimbursement of
the purchase price.

ISSUE:
May the petitioner seize the books from private respondent because it has been unlawfully deprived of the
books due to the dishonored check issued by the impostor?

HELD:
No. Actual delivery of the books having been made, Cruz acquired ownership over the books which he
could then validly transfer to the private respondents. The fact that he had not yet paid for them to EDCA
was a matter between him and EDCA and did not impair the title acquired by the private respondents to the
books.

A person relying on the seller’s title who buys a movable property from him would have to surrender it to
another person claiming to be the original owner who had not yet been paid the purchase price therefor.
The buyer in the second sale would be left holding the bag, so to speak, and would be compelled to return
the thing bought by him in good faith without even the right to reimbursement of the amount he had paid for
it

Unlike in the cases invoked by EDCA, it has yet to be established in the case at bar that EDCA has been
unlawfully deprived of the books. Although the title of Cruz was presumed under Article 559 by his mere
possession of the books, these being movable property, Santos nevertheless demanded more proof before
deciding to buy them. It would certainly be unfair now to make the private respondents bear the prejudice
sustained by EDCA as a result of its own negligence. The remedy of EDCA is not against the private
respondents but against the impostor, who has apparently caused all the trouble.

Hence, EDCA is not said to have been unlawfully deprived of the said books by the respondent so as to
recover the books from the latter.

160
POSSESSION
Theory of Irrevindicability

UNDER ARTICLE 599 OF THE NEW CIVIL CODE, TITLE OF THE POSSESSOR OF A MOVABLE
PROPERTY IS NOT THAT OF OWNERSHIP, BUT IS MERELY A PRESUMPTIVE TITLE SUFFICIENT
TO SERVE AS BASIS FOR ACQUISITIVE PRESCRIPTION

133. De Garcia v. Court of Appeals


G.R. No. L-20264, January 30, 1971
Fernando, J.

FACTS:
This is a petition for certiorari to review the decision of the CA.

Private respondent Angelina Guevarra assisted by her spouse Juan Guevarra sought to recover a diamond
ring which Angelina bought from R. Rebullida Inc. She recognized the said ring in the finger of Mrs. Garcia
herein petitioner, and inquired where she bought it to which the latter answered from her comadre. Angelina
explained that the ring was stolen from her house. She asked to fit the same, and it fit her finger. She also
had the same ring examined by R. Rebuilda Inc. who affirmed that the said ring was the same one bought
by Angelina from their store. Angelina returned the ring, and demanded its return from petitioners who
refused to do so even after the service of the writ of replevin by the sheriff claiming the ring was lost.

For her part, Mrs. De Garcia stated that she bought the ring from Mrs. Miranda who got it from Ms. Angelita
Hinahon who in turn got it from Aling Petring, and the ring could be similar but not the same ring purchased
by the plaintiff. She invoked Article 541 of the Civil Code, which provides: 'A possessor in the concept of
owner has in his favor the legal presumption that he possesses with a just title and he cannot be obliged to
show or prove it."

ISSUE:
Whether the possession of Mrs. De Garcia of the ring was in good faith and defeats the claims of Guevarra?

HELD:
No, Mrs. De Garcia’s possession was not in good faith. The controlling provision is Article 559 of the Civil
Code, as authoritative interpreted in Cruz v. Pahati, the right of the owner cannot be defeated even by proof
that there was good faith by the acquisition by the possessor. The only exception the law allows is when
there is acquisition in good faith of the possessor at a public sale, in which case the owner cannot obtain its
return without reimbursing the price.

Under Art. 559 of the NCC, the title of the possessor is not that of ownership, but is merely a presumptive
title sufficient to serve as a basis of acquisitive prescription. It is for the very reason that the title established
by the first clause of Art. 559 is only a presumptive title sufficient to serve as a basis for acquisitive
prescription, that the clause immediately following provides that `one who has lost any movable or has been
unlawfully deprived thereof, may recover it from the person in possession of the same.'

Respondent Angelina D. Guevara, having been unlawfully deprived of the diamond ring in question, was
entitled to recover it from petitioner Consuelo S. de Garcia who was found in possession of the same. The
owner can recover the same once she can show illegal deprivation. Respondent Court of Appeals was so
convinced from the evidence submitted that the owner of the ring in litigation is such respondent. Hence,
the decision of the CA was affirmed.

161
POSSESSION
Theory of Irrevindicability

THE ONLY EXCEPTION THE LAW ALLOWS IS WHEN THERE IS ACQUISITION IN GOOD FAITH OF
THE POSSESSOR AT A PUBLIC SALE, IN WHICH CASE THE OWNER CANNOT OBTAIN ITS RETURN
WITHOUT REIMBURSING THE PRICE

134. Dizon v. Suntay


G.R. No. L-30817, September 29, 1972
Fernando, J.

FACTS:
Lourdes Suntay and Clarita Sison entered into a transaction where Suntay’s ring was delivered to Sison for
sale on commission. After the lapse of considerable amount of time, Suntay demanded the return of the
ring. Sison did not comply because without the knowledge of Suntay, Sison’s niece Melia pledged the ring
to Dominador Dizon’s pawnshop. After sometime, Sison delivered the pawnshop ticket to Suntay. Suntay
sued Sison for estafa. She also wrote a letter to Dizon asking for the return of the ring to which Dizon refused
prompting Suntay to file the present action for the recovery of the ring.

ISSUE:
Whether Suntay is entitled to the return of her ring.

HELD:
Yes, Suntay is entitled to the return of the ring. In De Gracia v. Court of Appeals, "The controlling provision
is Article 559 of the Civil Code. It reads thus: 'The possession of movable property acquired in good faith is
equivalent to a title. Nevertheless, one who has lost any movable or has been unlawfully deprived thereof
may recover it from the person in possession of the same. If the possessor of a movable lost of which the
owner has been unlawfully deprived, has acquired it in good faith at a public sale, the owner cannot obtain
its return without reimbursing the price paid therefor.'

There is a reiteration of this principle in Aznar v. Yapdiangco. Thus: 'Suffice it to say in this regard that the
right of the owner to recover personal property acquired in good faith by another, is based on his being
dispossessed without his consent. The common law principle that were one of two innocent persons must
suffer by a fraud perpetrated by another, the law imposes the loss upon the party who, by his misplaced
confidence, has enabled the fraud to be committed, cannot be applied in a case which is covered by an
express provision of the new Civil Code, specifically Article 559. Between a common law principle and a
statutory provision, the latter must prevail in this jurisdiction."

The phrase ‘unlawfully deprived’ means ‘. . . it extends to all cases where there has been no valid
transmission of ownership including depositary or lessee who has sold the same. It is believed that the
owner in such a case is undoubtedly unlawfully deprived of his property and may recover the same from a
possessor in good faith.’

The only exception the law allows is when there is acquisition in good faith of the possessor at a public sale,
in which case the owner cannot obtain its return without reimbursing the price. As authoritatively interpreted
in Cruz v. Pahati, the right of the owner cannot be defeated even by proof that there was good faith in the
acquisition by the possessor.

162
POSSESSION
Theory of Irrevindicability

AS A GENERAL RULE, OWNERSHIP SHALL PASS FROM THE VENDOR TO THE VENDEE UPON THE
ACTUAL OR CONSTRUCTIVE DELIVERY OF THE THING SOLD EVEN IF THE PURCHASE PRICE HAS
NOT YET BEEN PAID

135. Ledesma v. Court of Appeals


G.R. No. 86051, September 1, 1992
Davide Jr, J.

FACTS:
Armando Suarez representing himself to be Jojo Consunji (“Jojo”), purchased purportedly for his father, a
certain Rustico T. Consunji, two (2) brand new motor vehicles from Citiwide Motors, Inc. (Citiwide)--- an
Isuzu Gemini and Holden Premier Model. The motor vehicles were delivered to “Jojo” who in turn issued
Citiwide a Manager’s Check of Philippine Commercial and Industrial Bank (PCIB) as full payment of the
motor vehicles. The check was dishonored by the bank on the ground of being tampered with. Citiwide
reported the criminal act, and learned the real identity of Armando Suarez who had a long line of criminal
cases against him for estafa using the same modus operandi. Citiwide was able to recover the Holden
Premier vehicle, while the Isuzu was transferred to one Jaime Ledesma at the time of the institution of this
action for replevin. RTC ruled in favor of Ledesma. The same was reversed by the CA hence this petition.

ISSUE:
Whether Citiwide was unlawfully deprived of the cars when it sold the same to Rustico Consunji, through
Jojo

HELD:
No, respondent was not unlawfully deprived of the car by false pretenses. There are three (3) requisites to
make possession of movable property equivalent to title, namely: (a) the possession should be in good faith;
(b) the owner voluntarily parted with the possession of the thing; and (c) the possession is in the concept of
owner. Undoubtedly, one who has lost a movable or who has been unlawfully deprived of it cannot be said
to have voluntarily parted with the possession thereof. This is the justification for the exceptions found under
the second sentence of Article 559 of the Civil Code.

In the case of EDCA Publishing and Distributing Corp. v. Santos with almost the same factual milieu, the
SC ruled that: "The contract of sale is consensual and is perfected once agreement is reached between the
parties on the subject matter and the consideration.”

It is clear that ownership in the thing sold shall not pass to the buyer until full payment of the purchase price
only if there is a stipulation to that effect. Otherwise, the rule is that such ownership shall pass from the
vendor to the vendee upon the actual or constructive delivery of the thing sold even if the purchase price
has not yet been paid. Non-payment only creates a right to demand payment or to rescind the contract, or
to criminal prosecution in the case of bouncing checks. But absent the stipulation above noted, delivery of
the thing sold will effectively transfer ownership to the buyer who can in turn transfer it to another."

There was a perfected unconditional contract of sale between private respondent and the original vendee.
The former voluntarily caused the transfer of the certificate of registration of the vehicle in the name of the
first vendee — even if the said vendee was represented by someone who used a fictitious name — and
likewise voluntarily delivered the cars and the certificate of registration to the vendee’s alleged
representative Title thereto was forthwith transferred to the vendee. The subsequent dishonor of the check
because of the alteration merely amounted to a failure of consideration which does not render the contract
of sale void, but merely allows the prejudiced party to sue for specific performance or rescission of the
contract, and to prosecute the impostor for estafa under Article 315 of the Revised Penal Code.

163
POSSESSION
Theory of Irrevindicability

THE BURDEN OF PROOF RESTS ON PETITIONER TO SHOW THAT THE PROPERTY WERE
UNLAWFULLY TAKEN

136. Subic Bay Legend Resorts and Casinos Inc. v. Fernandez


G.R. No. 193426, September 29, 2014
Del Castillo, J.

FACTS:
This is a petition for review on certiorari assailing the decision of CA affirming in toto the decision of the
RTC.

Respondent’s brother Ludwin Fernandez visited Legenda Hotel and Casino (Legenda) operated by Subic
Bay Legend Resorts and Casinos Inc. (Subic Bay). CCTV footage revealed that Ludwin changed USD 5000
chips into smaller denominations. Legenda admitted in its brief that its surveillance staff paid close attention
to Ludwin simply because it was "unusual" for a Filipino to play using dollar-denominated chips. Ludwin with
his brother Deoven were accosted by security and interrogated separately. The ultimatum was simple, they
confess the chips were given by a certain employee, Michael Cabrera, or they will not be released for
questioning. Due to hunger pangs and fatigue, they relented. Later on, they recanted their statement.

Bernard filed a civil case for recovery of sum of money with damages against petitioner, on the premise
that, he went to Legenda with his brothers Ludwin and Deoven; that he handed over Legenda casino chips,
which was illegally confiscated by Legenda and was refused to be returned despite the demand.

RTC ruled in favor of Fernandez stating that the chips were in possession of Fernandez as payment for car
services he rendered to a Chinese national. While defendant contends that the chips were stolen from the
casino and the lawful owner of the same. The burden of proof rests on petitioner to prove that the chips
were stolen. The proof adduced is however, wanting. CA affirmed the same stating that petitioner did not
even file a criminal case against Cabrera.

ISSUE:
Whether the chips were stolen from petitioner, and be subject to Art. 559 of the NCC

HELD:
No, hence Art. 559 shall stand. The Supreme Court gave more credence to the facts upheld by the CA
which were the same facts affirmed by the RTC. A question of fact cannot be raised in a certiorari. Though
casino chips do not constitute legal tender, there is no law which prohibits their use or trade outside of the
casino which issues them. In any case, it is not unusual – nor is it unlikely – that respondent could be paid
by his Chinese client at the former' s car shop with the casino chips in question; said transaction, if not
common, is nonetheless not unlawful.

These chips are paid for anyway; petitioner would not have parted with the same if their corresponding
representative equivalent - in legal tender, goodwill, or otherwise – was not received by it in return or
exchange. Given this premise - that casino chips are considered to have been exchanged with their
corresponding representative value - it is with more reason that this Court should require petitioner to prove
convincingly and persuasively that the chips it confiscated from Ludwin and Deoven were indeed stolen
from it; if so, any Tom, Dick or Harry in possession of genuine casino chips is presumed to have paid for
their representative value in exchange therefor. If petitioner cannot prove its loss, then Article 559 cannot
apply; the presumption that the chips were exchanged for value remains.

164
POSSESSION
Fruits

A PERSON WHO HAS BEEN ORDERED TO LEAVE CERTAIN PREMISES IS ORDINARILY NOT
PROHIBITED FROM TAKING WITH HIM HIS OWN EFFECTS AND POSSESSION, UNLESS THERE IS
AN EXPRESS PROHIBITION TO THIS EFFECT

137. Azarcon v. Eusebio


G.R. No. L-11977, April 29, 1959
Labrador, J.

FACTS:
This is an appeal from the order of CA finding petitioners Leonardo Azarcon, Manuel Azarcon and Esteban
Abobo guilty for contempt of court, and ordering each of them to pay a fine of P100 and remove the
improvements they have constructed on the land.

Respondent Victor Eusebio and petitioners had a dispute over the possession of a parcel of public land.
Victor filed a lease application for the land containing an area of 349 hectares. A portion of which was of
which was occupied by petitioners under a homestead application. Before the same could be settled,
Eusebio filed a complaint with the RTC ordering the petitioners to vacate the area occupied by them which
overlaps with his and pay damages. Leonardo answered that they were granted a homestead application
in the same area. Pending appeal to the CA, the lower court issued a writ for execution of judgment ordering
petitioners to be removed from the premises and respondent be restituted to the same. The same was set
aside upon filing of the supersedeas bond by the petitioners. However, they failed to notify the CA of the
approval of the supersedeas bond.

Evidence shows that in spite of the receipt by the defendants of the notice of the writ of execution of
petitioners nevertheless entered the land to gather palay which was then pending harvest. We gather further
from the record that the rice found on the disputed land at the time of the service of the order of execution
had been planted by petitioners, who appear to have been in possession of the land from 1951.

ISSUE:
Whether it was lawful for herein petitioners to gather the palay pending harvest even after the execution of
the writ

HELD:
Yes, petitioners had the right to gather the palay under Art 545 of the NCC.
Under the law a person who is in possession and who is being ordered to leave a parcel of land while
products thereon are pending harvest, has the right to a part of the net harvest, as expressly provided by
Article 545 of the Civil Code.
While the court order of October 3, 1955 ordered the defendant-appellant to move out from the premises, it
did not prohibit them from gathering the crop then existing thereon. As the order of execution did not
expressly prohibit the defendants-appellants from gathering the pending fruits, which fruits were the result
of their possession and cultivation of the land, it cannot be said that the defendants-appellants committed
an act which is clear violation of the courts' order.
Their act in harvesting the pending fruits was not only justified by law but was not expressly prohibited by
the court's order, and was even ratified when the court ordered the suspension of the execution. There was,
therefore, no open, clear and contumacious refusal to obey a definite order of the court such as would
constitute contempt. Furthermore, a person who has been ordered to leave certain premises is ordinarily
not prohibited from taking with him his own effects and possession, unless there is an express prohibition
to this effect.

165
POSSESSION
Fruits

POSSESSORS SHALL BE DEEMED IN BAD FAITH FROM THE SERVICE OF THE SUMMONS

138. Cordero v. Cabral


G.R.No. L-36789, July 25, 1983
Abad Santos, J.

FACTS:
Felipa Cordero and her children Mauro, Casimiro and Elisea all surnamed Ocampo sued Victoria Cabral,
Alejandro Berboso and Dalmacio Montaos.

Mr. Gregorio Z. Ocampo of Meycauayan, Bulacan, husband of the plaintiff Felipa Cordero and father of the
other plaintiffs surnamed Ocampo who died, and that said deceased left several properties, which were
inherited by the plaintiffs, one of which is a parcel of land described. the plaintiffs herein took possession of
the properties left by him, among others is the afore-described parcel of land which is a riceland, but they
found out that the southern portion of the same was possessed by the defendants herein, Victoria P. Cabral,
Alejandro Berboso and Dalmacio Montaos and that the defendant Victoria P. Cabral claimed to be the owner
of said portion while her co-defendants co-possessed the same as her tenants; That the plaintiffs demanded
of the defendants to surrender to the former possession of the afore-mentioned portion of land and/or vacate
it but they refused and failed to do so, and the defendant Victoria P. Cabral continued claiming to be the
owner of the same while her co-defendants continued recognizing her as the owner thereof instead of the
plaintiffs;

Defendants and the predecessors in interest before her are the real owners, and have been in actual,
adverse, peaceful and continuous possession, of that portion of land claimed by the plaintiffs in their
complaint, deceased Gregorio Z. Ocampo and/or his heirs, and their predecessors in interest have openly
admitted, acknowledged and recognized the defendant Victoria P. Cabral and her predecessors in interest
as the real owners of said portion of land

It can be seen that the thrust of the Complaint is that a piece of land covered by T.C.T. No. 14513 in the
name of Gregorio Z. Ocampo was illegally possessed by the defendants. Upon the other hand, the thrust
of the Answer is that "the defendant Victoria P. Cabral is the real owner of Lot No. 5-B, plan Psd-11496,
with an area of 4,303 square meters, more or less, erroneously or fraudulently included in the property
described in Transfer Certificate of Title No. 14513 of the Register of Deeds of the Province of Bulacan,
registered in the name of the deceased Gregorio Z. Ocampo and now claimed by the herein plaintiffs."

ISSUE:
Are the defendants considered as possessors in bad faith and thus entitled to the fruits

HELD:
YES. They are possessors in bad faith.

The defendants, by their own admission, are in possession of the disputed land. There is no evidence that
they were possessors in bad faith. However, their good faith ceased when they were served with summons
to answer the complaint. (Art. 528, Civil Code; Tacas vs. Tobon, 53 Phil. 356 [1929].) As possessors in bad
faith from the service of the summons they "shall reimburse the fruits received and those which the legitimate
possessor could have received, ... (Art. 549, Civil Code.)

Therefore, defendants shall vacate and surrender the land in question to the plaintiffs, and the defendants
shall also account for the fruits thereof pursuant to Article 549 of the Civil Code from the service of the
summons.

166
POSSESSION
Expenses

NECESSARY EXPENSES ARE THOSE MADE FOR THE PRESERVATION OF THE THING

139. Mendoza v. De Guzman


G.R. No. L-28721; October 5, 1928
Malcolm, J.

FACTS:
In the cadastral proceedings, the CFI adjudicated the above mentioned land in favour of petitioners pro
indivoso and de Guzman with the right to retention until the latter is indemnified for the improvements
already existing on the land. By virtue of the judgment, de Guzman moved for a writ of possession which
the court granted in 1924. From 1916 until 1924, Mendoza was in possession of the land, thereafter de
Guzman was in dominion of the land.

Being unable to agree as to the amount for the improvements of the land the petitioners requested the CFI
to: a. fix the value of the necessary and useful expenses incurred by de Guzman in introducing the
improvements; b. require the de Guzman to render an accounting of the fruits received by him and order
that the value of the fruits be applied to the payment of the necessary and useful expenses; and c. decree
the restitution of the possession to the plaintiffs. In their answer, de Guzman asked for ₱6,000.00.

The trial court resolved the questions presented by holding (1) that in accordance with the provisions of
articles 435 and 454 in relation with article 361 of the Civil Code, the value of the "indemnization" to be paid
to the defendant should be fixed according to the necessary and useful expenses incurred by him in
introducing "las plantaciones en cuestion"; (2) that the plaintiffs as the owner of the property have the right
to make their own "las plantaciones hechas por el demandado" upon payment in the form indicated in No.
1, the defendant having the right to retain the land until the expenditures have been refunded; (3) that the
defendant is obliged to render a detail and just account of the fruits and other profits received by him from
the property for their due application; and (4) that the value of the fruits received by the defendant should
first be applied to the payment of the "indemnizacion," and in that it exceeds the value of the
"indemnizacion," the excess shall be returned to the plaintiffs.

ISSUE:
Should the excess of the indemnification be returned to the petitioners

HELD:
Yes. Necessary expenses have been variously described by the Spanish commentators as those made for
the preservation of the thing as those without which the thing would deteriorate or be lost as those that
augment the income of the things upon which they are expanded. Among the necessary expenditures are
those incurred for cultivation, production, upkeep, etc. Here the plaintiffs have chosen to take the
improvements introduced on the land and are disposed to pay the amount of the necessary and useful
expenses incurred by the defendant. Inasmuch as the retentionist, who is not exactly a posessor in good
faith with in the meaning of the law, seeks to be reimbursed for the necessary and useful expenditures, it is
only just that he should account to the owners of the estate for any rents, fruits, or crops he has gathered
from it.

167
POSSESSION
Expenses

THE BRUDEN OF PROVING BAD FAITH ON THE PART OF POSSESSOR RESTS UPON THE PERSON
ALLEGING IT

140. Robles v. Hermanos


G.R. No. L-16736, December 22, 1921
Romualdez, J.

FACTS:
Anastasia de la Rama died, leaving six children and some properties, among which is house No. 4 on Iznart
Street in the city of Iloilo, concerning which a controversy arose which developed into the three cases now
under consideration.

The children and heirs of Anastasia de la Rama entered into partnership with Lizarraga Hermanos in
liquidation and settlement of their accounts, by virtue of which the competent court awarded to said
partnership the properties left by the deceased, including the aforesaid house No. 4 on Iznart Street.

Evarista Robles, one of the aforesaid heirs, since before the death of her mother Anastasia de la Rama,
has been with her husband occupying the aforesaid house No. 4 on Iznart Street, at the beginning, by
permission of her mother, later on by the consent of her coheirs, and lastly by agreement with the
partnership, Lizarraga Hermanos, to whom it had been awarded, having made some improvements on the
house, the value of which is fixed at four thousand five hundred pesos (P4,500), and paying to said
partnership forty pesos (P40) monthly as rent of the upper story.

Evarista Robles refused to pay such a new rate of rent and to vacate the house, and Lizarraga Hermanos
brought suit against her for ejectment. Evarista Robles sued Lizarraga Hermanos afterwards to recover the
value of the improvements, and demanded, in another action, that said value be noted on the certificate of
title as an encumbrance.

ISSUE:
Is Evarista Robles the owner of the aforesaid improvements and has the right to demand payment of their
value?

HELD:
YES. The expenditures incurred in these improvements were not necessary inasmuch as without them the
house would have continued to stand just as before, but were useful, inasmuch as with them the house
better serves the purpose for which it was intended, being used as a residence, and the improvements
consisting of the addition of a dining room, kitchen, closet, and bathroom in the lower and upper stories of
the house, and a stable, suitable as a coach house and dwelling, it is beyond doubt that such improvements
are useful to the building. One of the chiefs of the firm of Lizarraga Hermanos, on the occasion of a luncheon
in the house, on noting the improvements, could not refrain from expressing that such improvements added
much to the value of the building.

Now then, was Evarista Robles a possessor in good faith when she made those improvements? Article 434
provides that "good faith is always presumed and the burden of proving bad faith on the part of the possessor
rests upon the person alleging it." Lizarraga Hermanos did not allege, nor prove in the first instance the bad
faith characterizing Evarista Robles' possession, who, as shown in the records and heretofore stated, began
to occupy the house by permission of the former owner, her mother Anastasia de la Rama, and continued
later in the occupation by the consent of her coheirs, and afterwards by considering herself the future owner
of the building by virtue of the contract with the present owner, Lizarraga Hermanos.

We find that in the court below the presumption of good faith in favor of Evarista Robles' possession at the
time she made the improvements on the property was neither disputed nor discussed, but on the contrary,
there is positive evidence sufficient to support the conclusion that when she made the improvements on the
aforesaid building she was possessing it in good faith.

If the improvements are useful and Evarista Robles' possession was in good faith, the conclusion set out in
article If 453 of the Civil Code, supra, is inevitable; Evarista Robles n is the owner of such improvements,
and entitled to reimbursement therefor, and to retain the building until the same is made.

168
POSSESSION
Expenses

THE RIGHT OF A POSSESSOR IN BAD FAITH TO REMOVE IMPROVEMENTS APPLIES ONLY TO


IMPROVEMENTS FOR PURE LUXURY

141. Metropolitan Waterworks and Sewerage System v. Court of Appeals


G.R. No. L-54526; August 25, 1986
Feria, J.

FACTS:
The City of Dagupan (hereinafter referred to as the CITY) filed a complaint against the former National
Waterworks and Sewerage Authority (hereinafter referred to as the NAWASA), now the Metropolitan
Waterworks and Sewerage System (hereinafter referred to as MWSS), for recovery of the ownership and
possession of the Dagupan Waterworks System. NAWASA interposed as one of its special defenses R.A.
1383 which vested upon it the ownership, possession and control of all waterworks systems throughout the
Philippines and as one of its counterclaims the reimbursement of the expenses it had incurred for necessary
and useful improvements amounting to P255,000.00.

Judgment was rendered by the trial court in favor of the CITY on the basis of a stipulation of facts. The trial
court found NAWASA to be a possessor in bad faith and hence not entitled to the reimbursement claimed
by it. NAWASA appealed to the then Court of Appeals and argued in its lone assignment of error that the
CITY should have been held liable for the amortization of the balance of the loan secured by NAWASA for
the improvement of the Dagupan Waterworks System.

However, as already found above, these useful expenses were made in utter bad faith for they were
instituted after the complaint was filed and after numerous Supreme Court decisions were promulgated
declaring unconstitutional the taking by NAWASA of the patrimonial waterworks systems of cities,
municipalities and provinces without just compensation.

ISSUE:
Does a possessor in bad faith have the right to remove useful improvements?

HELD:
No. Under Article 499 of the Civil Code, “he who builds, plants, or sows in bad faith on the land of another,
loses what is built, planted, or sown without right to indemnity.” Additionally, under Article 546 of the Civil
Code, only a possessor in good faith shall be refunded for useful expenses with the right of retention until
reimbursed.

Finally, under Article 547 of the Civil Code, only a possessor in good faith may remove useful improvements
if this can be done without damage to the principal thing and if the person who recovers the possession
does not exercise the option of reimbursing the useful expenses. The right of a possessor in bad faith to
remove improvements applies only to improvements for pure luxury or mere pleasure, provided the thing
does not suffer any injury and the lawful possessor does not prefer to retain them by paying their value at
the time of his possession.

In this case, MWSS is a builder in bad faith so it loses whatever useful improvements it made without right
to indemnity.

169
USUFRUCT
Concept of Usufruct

SHARE OF STOCK ARE PART OF THE PROPERTY IN USFRUCT

142. Bachrach v. Seifert


G.R. No. L-2659, October 12, 1950
Ozaeta, J.

FACTS:
The will of E. M. Bachrach provided for the distribution of the considerable property which he had left. The
sixth and eighth paragraphs of the provisions of the will provide as follows:
Sixth: It is my will and do herewith bequeath and devise to my beloved wife Mary McDonald Bachrach for
life all the fruits and usufruct of the remainder of all my estate after payment of the legacies, bequests and
gifts provided for above; and she may enjoy such usufruct and use or spend such fruits as she may in any
manner wish. Eighth: It is my wish that upon the death of my beloved wife, Mary McDonald Bachrach, all
my estate, personal, real and otherwise, and all the fruits and usufruct thereof which during her life pertained
to her, shall be divided as follows:One-half thereof shall be given to such charitable hospitals in the
Philippines she may designate; in case she fails to designate, then said sum shall begiven to the Chief
Executive of these Islands who shall distribute it, share andshare alike to all charitable hospitals in the
Philippines excluding those belonging to the governments of the Philippines or of the United States;One-
half thereof shall be divided, share and share alike by and between my legal heirs, to the exclusion of my
brothers.

The estate of E. M. Bachrach, as owner of 108,000 shares of stock of the Atok-Big Wedge Mining Co., Inc.,
received from the latter 54,000 shares representing 50% stock dividend on the said 108,000 shares. Mary
McDonald Bachrach, as usufructuary or life tenant of the estate, petitioned the lower court to authorize the
Peoples Bankand Trust Company as administrator of the estate of E. M. Bachrach, to her the said 54,000
share of stock dividend by endorsing and delivering to her the corresponding certificate of stock, claiming
that said dividend, although paid out in the form of stock, is fruit or income and therefore belonged to her as
usufructuary or life tenant. Sophie Siefert and Elisa Elianoff, legal heirs of the deceased, opposed said
petition on the ground that the stock dividend in question was not income but formed part of the capital and
therefore belonged not to the usufructuary but to the remainder man. And they have appealed from the
order granting the petition and overruling their objection.

ISSUE:
Is a stock dividend fruit or income, which belongs to the usufructuary, or is itcapital or part of the corpus of
the estate, which pertains to the remainderman?

HELD:
The 108,000 shares of stock are part of the property in usufruct. The 54,000 shares of stock dividend are
civil fruits of the srcinal investment. They represent profits, and the delivery of the certificate of stock
covering said dividend is equivalent to the payment of said profits. Said shares may be sold independently
of the original shares, just as the offspring of a domestic animal may be sold independently of its mother.

170
USUFRUCT
Concept of Usufruct

USUFRUCTUARY HAS THE RIGHT TO USE AND RIGHT TO FRUITS, WHILE THE OWNER MAINTAINS
THE RIGHT TO DISPOSE

143. Hemedes v. Court of Appeals


G.R. No. 107132, October 8, 1999
Gonzaga-Reyes, J.

FACTS:
These are petitions for review on certiorari on the decision of the eleventh division of the Court of Appeals
affirming in toto the decision of Branch 24 of the Regional Trial Court of Laguna and the resolution denying
petitioner R & B Insurance Corporation's motion for reconsideration.

Jose Hemedes executed a document entitled "Donation Inter Vivos With Resolutory Conditions" whereby
he conveyed ownership over the subject land, together with all its improvements, in favor of his third wife,
Justa Kauapin, subject to the resolutory condition that upon the latter’s death or remarriage, the title to the
property donated shall revert to any of the children, or heirs, of the DONOR expressly designated by the
DONEE. Pursuant thereto, on Justa Kausapin executed a "Deed of Conveyance of Unregistered Real
Property by Reversion" conveying to Maxima Hemedes the subject property. Maxima Hemedes filed an
application for registration and an Original Certificate of Title was issued in her name with the annotation
that "Justa Kausapin shall have the usufructuary rights over the parcel of land during her lifetime or
widowhood." R & B Insurance claims that Maxima Hemedes and her husband constituted a real estate
mortgage over the subject property in its favor to serve as security for a loan. R & B Insurance extrajudicially
foreclosed the mortgage since Maxima Hemedes failed to pay the loan even after it became due. Land was
sold at a public auction with R & B Insurance as the highest bidder and the RD issued TCT No. 41985 in
the name of R & B Insurance. The annotation of usufruct in favor of Justa Kausapin was maintained.

Despite the earlier conveyance, Justa Kausapin executed a "Kasunduan" she transferred the same land to
her stepson Enrique D. Hemedes, pursuant to the resolutory condition in the deed of donation. In 1979,
Enrique Hemedes sold the property to Dominium Realty and Construction Corporation. Dominium leased
the property to Asia Brewery, Inc. who constructed two warehouses. Upon learning of Asia Brewery's
constructions upon the subject property, R & B Insurance sent it a letter informing them of its ownership of
the property.

Dominium and Enrique D. Hemedes filed a complaint with the Court of First Instance of Binan, Laguna for
the annulment of TCT No. 41985 issued in favor of R & B Insurance and/or the reconveyance to Dominium
of the subject property. The trial court rendered judgment ruling in favor of Dominium.

ISSUE:
Which of the two conveyances conveyances by Justa Kausapin, the first in favor of Maxima Hemedes and
the second in favor of Enrique D. Hemedes, effectively transferred ownership over the subject land?

HELD:
The Court upholds the deed of conveyance in favor of Maxima Hemedes. Enrique D. Hemedes and his
transferee, Dominium, did not acquire any rights over the subject property. The donation in favor of Enrique
D. Hemedes is null and void for the purported object thereof did not exist at the time of the transfer, having
already been transferred to his sister. Similarly, the sale of the subject property by Enrique D. Hemedes to
Dominium is also a nullity for the latter cannot acquire more rights than its predecessor-in-interest and is
definitely not an innocent purchaser for value since Enrique D. Hemedes did not present any certificate of
title upon which it relied.

The annotation of usufructuary rights in favor of Justa Kausapin upon Maxima Hemedes' OCT dose not
impose upon R & B Insurance the obligation to investigate the validity of its mortgagor's title. Usufruct gives
a right to enjoy the property of another with the obligation of preserving its form and substance. The
usufructuary is entitled to all the natural, industrial and civil fruits of the property and may personally enjoy
the thing in usufruct, lease it to another, or alienate his right of usufruct, even by a gratuitous title, but all the
contracts he may enter into as such usufructuary shall terminate upon the expiration of the usufruct. Clearly,
only the jus utendi and jus fruendi over the property is transferred to the usufructuary.

171
The owner of the property maintains the jus disponendi or the power to alienate, encumber, transform, and
even destroy the same. This right is embodied in the Civil Code, which provides that the owner of property
the usufruct of which is held by another, may alienate it, although he cannot alter the property's form or
substance, or do anything which may be prejudicial to the usufructuary. There is no doubt that the owner
may validly mortgage the property in favor of a third person and the law provides that, in such a case, the
usufructuary shall not be obliged to pay the debt of the mortgagor, and should the immovable be attached
or sold judicially for the payment of the debt, the owner shall be liable to the usufructuary for whatever the
latter may lose by reason thereof.

172
USUFRUCT
Rights of the Usufructuary

USUFRACTUARY HAS THE RIGHT TO THE RENT, TO CHOOSE THE TENANT, TO FIX THE AMOUNT
OF RENT AND RIGHT TO CHOOSE HERSELF AS TENANT THEREOF

144. Fabia v. David


G.R. No. L-123, December 12, 1945
Ozaeta, J.

FACTS:
This is original action institution by petitioner Josefa Fabie to annul the order of the dismissal and to require
the CFI to try and decide the case on the merits.

Josefa Fabie is the usufructuary of the income of certain houses located at Santo Cristo, Binondo, and
Ongpin, Santa Cruz, Manila, under the ninth clause of the will of the deceased Rosario Fabie y Grey. The
owner of Santo Cristo property is the respondent Juan Grey.

In June 1945 Josefa Fabie commenced an action of unlawful detainer against respondent Ngo Boo Soo
alleging that the defendant is occupying the premises located at Santo Cristo on a month-to month rental.
The defendant, without plaintiff's consent and contrary to their agreement, had subleased to another
Chinese the property, but plaintiff refused, based on the fact that the herein plaintiff very badly needs the
said house to live in. The defendant was duly notified on March 24 and April 14, 1945, to leave the said
premises, but he refused"; and she prayed for judgment of eviction and for unpaid rentals.

The defendant answered alleging that he was renting it from its owner and administrator Juan Grey. He
alleged that that plaintiff is merely the usufructuary of the income therefrom and her only right as
usufructuary of the income is to receive the whole of such income, and that she has no right or authority to
eject tenants, such right being in the owner and administrator of the house, the aforesaid Juan Grey. Juan
Grey intervened in the unlawful detainer suit, alleging in his complaint in intervention that he is the sole and
absolute owner of the premises in question; that the plaintiff Josefa Fabie is the usufructuary of the income
of said premises has the only right to receive the rents therefrom when due; and that as usufructuary she
has no right nor authority to administer the said premises nor to lease them nor to evict tenants, which right
and authority are vested in the intervenor as owner of the premises.

ISSUE:
Whether or not Josefa Fabie, a usufructuary, has the right or authority to eject tenants

HELD:
Yes. The Court finds that the usufructuary has the right to administer the property in question. All the acts
of administration — to collect the rents for herself, and to conserve the property by making all necessary
repairs and paying all the taxes, special assessments, and insurance premiums thereon — were by said
judgment vested in the usufructuary. The pretension of the respondent Juan Grey that he is the administrator
of the property with the right to choose the tenants and to dictate the conditions of the lease is contrary to
both the letter and the spirit of the said clause of the will, the stipulation of the parties, and the judgment of
the court.

To permit the owner Juan Grey to arrogate to himself the privilege to choose the tenant, to dictate the
conditions of the lease, and to sue when the lessee fails to comply therewith, would be to place the
usufructuary entirely at his mercy. It would place her in the absurd situation of having a certain indisputable
right without the power to protect, enforce, and fully enjoy it. Moreover, the Court thinks that corollary to her
right to all the rent, to choose the tenant, and to fix the amount of the rent, she necessarily has the right to
choose herself as the tenant thereof, if she wishes to; and, as she fulfills her obligation to pay the taxes and
insure and conserve the property properly, the owner has no legitimate cause to complain. As Judge Nable
of the municipal court said in his decision, "the pretension that the plaintiff, being a mere usufructuary of the
rents, cannot occupy the property, is illogical if it be taken into account that that could not have been the
intention of the testatrix."

173
USUFRUCT
Obligations of the Usufructuary

THE USUFRUCTUARY’S DESIGNATION AS ADMINISTRATOR OF PROPERTIES IS LIMITED BY HIS


REFUSAL AND/OR DEATH AND THEREFORE IT DOES NOT RUN COUNTER TO ARTICLE 870

145. In Re: Aranas v. Aranas


G.R. No. L-56249, May 29, 1987
Paras, J.

FACTS:
This is a petition for certiorari which seeks to declare the orders of respondent Judge as an exercise of a
gross abuse of discretion amounting to lack of jurisdiction, by ruling that the properties under Group C of
the testate estate of the late Fr. Teodoro Aranas are subject to remunerative legacies.

Fr. Aranas died. He executed his Last Will and Testament which was admitted to probate. In said Last Will
and Testament, Fr. Aranas stipulated the following: xxx
“C. The special administration of the remainder of the estate of the testator by Vicente Aranas, a faithful
and serviceable nephew and designating him also as recipient of 1/2 of the produce of said properties
after deducting the expenses for the administration and the other 1/2 of the produce to be given to the
Catholic Church for the eternal repose of the testator's soul.” xxx

The lower court in ruled that the "perpetual inalienability and administration of the portion of the estate of
the late Rev. Fr. Aranas, administered by Vicente Aranas, is null and void after twenty years. However, said
Order was subsequently set aside. The court ruled in its questioned order that this particular group of
properties (Group "C") is subject to “Remunerative legacy by way of usufruct xxx in favor of Vicente Aranas,
during his lifetime and shall continue an administrator of the estate, and, who, upon his death or refusal to
continue such usufruct, may be succeeded xxx”. Petitioners rely heavily on the doctrine laid down in Art.
870 of the NCC to wit: "The dispositions of the testator declaring all or part of the estate inalienable for more
than twenty years are void."

ISSUE:
Did the lower court err in not applying the provisions on Usufruct with respect to the properties referred to
as Group "C" in the Last Will and Testament?

HELD:
No. The proviso must be respected and be given effect until the death or until the refusal to act as such of
the instituted usufructuary/administrator, after which period, the property can be properly disposed of,
subject to the limitations provided in Art. 863 of the Civil Code concerning a fideicommissary substitution,
said Article says: "A fideicommissary substitution by virtue of which the fiduciary or first heir instituted is
entrusted with the obligation to preserve and to transmit to a second heir the whole or part of the inheritance,
shall be valid and shall take effect, provided such substitution does not go beyond one degree from the heir
originally instituted, and provided further, that the fiduciary or first heir and the second heir are living at the
time of the death of the testator."

Vicente Aranas therefore as a usufructuary has the right to enjoy the property of his uncle with all the
benefits which result from the normal enjoyment (or exploitation) of another's property, with the obligation
to return, at the designated time, either the same thing, or in special cases its equivalent. This right of
Vicente to enjoy the fruits of the properties is temporary and therefore not perpetual as there is a limitation
namely his death or his refusal. Likewise his designation as administrator of these properties is limited by
his refusal and/or death and therefore it does not run counter to Art. 870 of the Civil Code relied upon by
the petitioners.

174
USUFRUCT
Obligations of the Naked Owner

USUFRUCT IS CONSTITUTED NOT ONLY ON THE BUILDING BUT ON THE LAND AS WELL, THEN
THE USUFRUCT IS NOT DEEMED EXTINGUISHED BY THE DESTRUCTION OF THE BUILDING

146. Vda. de Albar v. Fabie


G.R. No. L-13361, December 29, 1959
Bautista Angelo, J.

FACTS:
Doña Rosario Fabie y Grey was the owner of a lot situated in the City of Manila with a building and
improvements thereon erected at Ongpin. On her will left by her upon her death which was duly probated
she devised the naked ownership of the whole property to Rosario Grey Vda. de Albar, Et. Al. but its usufruct
to Josefa Fabie for life.

During liberation, as a consequence of the fire that gutted the buildings in many portions of Manila, the
building on the Ongpin lot was burned, leaving only the walls and other improvements that were not
destroyed by the fire. One Au Pit, a Chinaman, offered to lease the property for a period of five years at a
monthly rental of P500.00, at the same time agreeing to construct on the lot a new building worth P30,000.00
provided the naked owners as well as the usufructuary sign the agreement of lease. By reason of the
destruction of the building on the Ongpin property, the United States War Damage Commission approved
the claim that was presented for the damage caused to the property which was paid to and received by the
naked owners. In the meantime, the usufructuary paid the real estate taxes due on the property at Ongpin
for the years 1945 to 1952, as well as the real estate taxes for the years 1953 and 1954.

The usufructuary maintains that she has the exclusive right to cede the property by lease and to receive the
full rental value by virtue of her right of usufruct while on the other hand the naked owners maintain that the
right of usufruct was extinguished when the building was destroyed, the right of the usufructuary being
limited to the legal interest on the value of the lot and the materials. Petitioners contend that the provision
of the will should be interpreted as constituting only a life usufruct on the rentals of the buildings erected on
the lands and that once these buildings are destroyed the usufruct is extinguished. Respondent, on the
other hand, contends that that provision should be interpreted as constituting a life usufruct both on the
buildings and the lands because the former cannot be separated from the latter.

ISSUE:
Is the usufruct in favor of Josefa Fabie extinguished once the buildings subject of usufructuary destroyed?

HELD:
No. It is clear that when the deceased constituted the life usufruct on the rentals of the "fincas situadas" in
Ongpin and Sto. Cristo streets, she meant to impose the encumbrance both on the building and the land on
which it is erected for indeed the building cannot exist without the land. And as the Court well said, "The
land, being an indispensable part of the rented premises cannot be considered as having no rental value
whatsoever." Moreover, in the Spanish language, the term "fincas" has a broad scope; it includes not only
building but land as well. (Diccionario Ingles- Español, por Martines Amador) Since only the building was
destroyed and the usufruct is constituted not only on the building but on the land as well, then the usufruct
is not deemed extinguished by the destruction of the building for under the law usufruct is extinguished only
by the total loss of the thing subject of the encumbrance (Article 603, old Civil Code).

It is the Court’s opinion that this case comes under Article 517 of the same Code which provides: "If the
usufruct is constituted on immovable property of which a building forms part, and the latter should be
destroyed in any manner whatsoever, the usufructuary shall have a right to make use of the land and
materials." This is a temporary measure calculated to maintain the usufruct alive until the very thing that
has been destroyed be reconstructed or replaced. The reason is obvious: since the usufruct has not been
extinguished by the destruction of the building and the usufruct is for life as in this case, it is but fair that the
usufructuary continue to enjoy the use of the land and the materials that may have been left by the fire or
to the use of the new building that may be constructed on the land.

175
USUFRUCT
Extinguishment of Usufruct

RTC HAS JURISDICTION TO INTERPRET AND APPLY STATUTES, RULES AND REGULATIONS
RELATING TO LAND REFORM AND GENERAL CIVIL LAW INCLUDING LAW ON USUFRUCT

147. Locsin v. Valenzuela


G.R. No. L-51333, May 18, 1989
Feliciano, J.

FACTS:
This is a petition for review of the decision of the CFI Negros Occidental, Branch 3 and the decision of the
Court of Agrarian Reations wherein both of these decisions dismissed the petitioners’ complaint for lack of
jurisdiction.

Petitioners Ramona R. Locsin, Teresita R. Guanzon, Celina R. Sibug, Ma. Lusia R. Perez, Editha R. Ylanan
and Ana Marie R. Benedicto were co-owners of a large tract of agricultural land known as "Hacienda Villa
Regalado" located in Barrio Panubigan, Canlaon City, Negros Occidental. A portion of this land, known as
Lot No. 2-C-A-3 and consisting of an area of 60.07464 hectares, was subject to the lifetime usufructuary
rights of respondent Helen Schon. The bulk of this lot was cultivated by the following lessees-tenants who
customarily delivered the rental to Helen Schon.

On 1972, Presidential Decree No. 27 was promulgated, decreeing the "Emancipation of Tenants." The tract
of land owned in common by the petitioners, including the portion thereof subject to Helen Schon's
usufructuary rights, fell within the scope of "Operation Land Transfer." In consequence, staff members of
the Department of Agrarian Relations Identified the tenanttillers of said land, and the necessary parcellary
map sketch was made and submitted to the Bureau of Lands Office in Dumaguete City. Petitioners through
counsel sought the opinion of the DAR as to who (petitioners or respondent Helen Schon) should be entitled
to receive the rental payments which continued to be made by the respondent tenants to Helen Schon. The
DAR District Officer rendered an opinion that the rental payments were properly considered as amortization
payments for the land and as such should pertain to the land- owners and not to the usufructuary.

Petitioners filed against the spouses Joseph and Helen Schon a civil case for collection of rentals plus
damages with prayer for preliminary injunction. Schon contended the Court of First Instance was bereft
ofjurisdiction over the subject matter of the case. That jurisdiction, the Schon spouses urged, was vested in
the Court of Agrarian Relations (CAR) instead. Petitioners filed a second complaint with the CAR but such
was dismissed by CAR ruling that it had no jurisdiction to decide the case since for all legal intents and
purposes, Helen Bennett-Schon belongs to the category of a landowner, since she is the recipient of any
and all fruit derived from the land of which the plaintiffs are the naked owners. The usufruct lasts for as long
as Helen Bennett-Schon lives. Therefore, this case actually is a dispute between two landowners — one,
the naked owners, the other, the beneficial owner — hose controversy revolves on who of them should
receive the rentals being paid by the tenants or lessees on the land in question.

ISSUE:
Which court has jurisdiction to hear the case, the RTC or CAR?

HELD:
The Regional Trial Court has jurisdiction over the case. The Regional Trial Courts have full authority and
jurisdiction to interpret and apply both the mass of statutes and rules and regulations relating to land reform
and the general civil law, including the law on usufruct. Unlike a regional trial court sitting as a probate court,
a region al trial court seized of an agrarian dispute and interpreting and applying statutes and administrative
rules and regulations concerning land reform and the sliminations of agricultural tenancy relationships,
continues to act as a court of general and plenary jurisdiction. Section 44 of b.P. Blg. 129 abolished the
Courts of Agrarian Relations and did not re-create them.

The Court note that resolution of the underlying substantive issues here raised requires examination of
both land reform statutes and related rules and regulations (and as well the practice of the relevant
administrative agency or executive department) and the Civil Code provisions on usufruct.

176
EASEMENTS
Easements

WATER RIGHTS APPURTENANT TO A PARCEL OF LAND PASS WITH THE CONVEYANCE OF THE
LAND, ALTHOUGH NOT SPECIFICALLY MENTIONED IN THE CONVEYANCE

148. Valisno v. Adriano


G.R. No. L-37409, May 23, 1988
Griño-Aquino, J.

FACTS:
This case was certified to this Court by the Court of Appeals in a resolution dated August 10, 1973, the sole
issue being a question of law and beyond its jurisdiction to decide.

Adriano siblings (Felipe and Honorata) previously owned two parcels of land. On the land of Honorata was
an existing irrigation which passes through the land of Felipe, whose property adjoins the river. When
Valisno bought the property of Honorata, he cultivated therein different fruits and crops. Felipe then enclosed
the irrigation providing water to the land of Valisno, causing the latter prejudice.

ISSUE:
Whether or not Valisno has the right to continue to enjoy the easement of aqueduct or water through the
defendant’s (Felipe) land

HELD:
Yes. The existence of the irrigation canal on defendant's land for the passage of water from the Pampanga
River to Honorata's land prior to and at the time of the sale of Honorata's land to the plaintiff was equivalent
to a title for the vendee of the land to continue using it as provided in Article 624 of the Civil Code.

The deed of sale in favor of Valisno included the "conveyance and transfer of the water rights and
improvements" appurtenant to Honorata Adriano's property. By the terms of the Deed of Absolute Sale, the
vendor Honorata Adriano Francisco sold, ceded, conveyed and transferred to Dr. Nicolas Valisno all "rights,
title, interest and participations over the parcel of land.

Water rights, such as the right to use a drainage ditch for irrigation purposes, which are appurtenant to a
parcel of land, pass with the conveyance of the land, although not specifically mentioned in the conveyance.

Hence, as an easement of waters in favor of the appellant has been established, he is entitled to enjoy it
free from obstruction, disturbance or wrongful interference (19 CJ 984), such as the appellee's act of
levelling the irrigation canal to deprive him of the use of water from the Pampanga River.

177
EASEMENTS
Easements

EASEMENT OF RIGHT OF WAY MAY NOT BE ACQUIRED THROUGH PRESCRIPTION

149. Ronquillo v. Roco


G.R. No. L-10619, February 28, 1958
Montemayor, J.

FACTS:
Involving as it does only a question of law, the present appeal from the order of the Court of First Instance,
dismissing the amended and supplemental complaint of plaintiffs was taken directly to this Court.

Petitioners’ parcel of land was connected to the Naga Market Place and Igualdad St. by an easement of a
right of way through the land of the Respondents, which they have been using for more than 20 years. On
May 1953, however, respondents built a chapel right in the middle of the road, blocking their usual path to
the marketplace. One year after, by means of force, intimidation, and threats, the owners (respondents) of
the land where the easement was situated, planted wooden posts and fenced with barbed wires the road,
closing their right of way from their house to Igualdad St. and Naga public market. The petitioners claim that
they have acquired the easement of right of way over the land of the defendants thru prescription.

ISSUE:
Whether or not the easement of a right of way may be acquired by prescription.

HELD:
No. Art. 620 of the CC provides that only continuous and apparent easements may be acquired by
prescription. The easement of a right of way cannot be considered continuous because its use is at intervals
and is dependent on the acts of man. Hence, an easement of right of way though it may be apparent is,
nevertheless, discontinuous or intermittent and, therefore, cannot be acquired through prescription, but only
by virtue of a title.

It is held that under the present law, particularly, the provisions of the Civil Code, old and new, unless and
until the same is changed or clarified, the easement of right of way may not be acquired through prescription.

Minority Opinion (including the ponente):

Easements of right of way may already be acquired by prescription, at least since the introduction into this
jurisdiction of the special law on prescription through the Old Code of Civil Procedure, Act No. 190. Said
law, particularly, Section 41 thereof, makes no distinction as to the real rights which are subject to
prescription, and there would appear to be no valid reason, at least to the writer of this opinion, why the
continued use of a path or a road or right of way by the party, specially by the public, for ten years or more,
not by mere tolerance of the owner of the land, but through adverse use of it, cannot give said party a vested
right to such right of way through prescription.

178
EASEMENTS
Easements

ALIENATION OF THE DOMINANT AND SERVIENT ESTATES TO DIFFERENT PERSONS IS NOT ONE
OF THE GROUNS FOR EXTINGUISHMENT OF THE EASEMENT

150. Tañedo v. Bernad


G.R. No. L-66520, August 30, 1988
Padilla, J.

FACTS:
This is a petition for review on certiorari of the Order issued by the respondent judge, Hon. Juanita A.
Bernad, which dismissed the complaint for legal redemption filed by the petitioner and the Order of the same
respondent judge, which denied petitioner's motion for reconsideration.

Antonio Cardenas (respondent) is the original owner of 2 parcels of land (7501-A and 7501-B). He
constructed an apartment building in Lot A and in Lot B he constructed an apartment, house, bodega and
a septic tank for common use of the occupants of the two lots. Cardenas sold Lot A and mortgaged Lot B
to Eduardo Tañedo (petitioner). He also agreed that should he decide to sell Lot B he would sell it to Tañedo.

However, Cardenas sold Lot B to Spouses Sim (respondent). Sim blocked the sewage pipe connecting the
building on Lot A to the septic tank. He also asked Tañedo to remove that portion of his building encroaching
Lot B. Tañedo filed an action for legal redemption and damages against respondents. Cardenas admitted
that he had agreed to sell the lot to petitioner and claimed by way of cross-claim against spouses Sim that
the Deed of Sale he had executed was only intended as an equitable mortgage. Respondent Judge
dismissed the complaint and the cross-claim.

ISSUE:
Whether or not the right to continue to use the septic tank ceased upon the subdivision of the land and its
subsequent sale to different owners.

HELD:
No. The alienation of the dominant and servient estates to different persons is not one of the grounds for
the extinguishment of an easement as provided in Article 631 of the New Civil Code. On the contrary, use
of the easement is continued by operation of law as provided in Art 624 because no abolishment or
extinguishment was provided in the deed of absolute sale. Nor did Cardenas stop the use of the drain pipe
and septic tanks before he sold the lots. Accordingly, the spouses Sim cannot impair, in any manner, the
use of the servitude.

179
EASEMENTS
Easements

CONVENIENCE OF THE DOMINANT ESTATE IS NOT A GAUGE FOR THE GRANT OF COMPULSORY
RIGHT OF WAY

151. Costabella Corp. v. Court of Appeals


G.R. No. 80511, January 25, 1991
Sarmiento, J.

FACTS:
The principal issue raised in this petition for review on certiorari of the decision of the Court of Appeals,
which modified the decision rendered by the Regional Trial Court, is whether or not the private respondents
had acquired an easement of right of way, in the form of a passageway, on the petitioner's property.

Petitioners owned a lot wherein they started constructing their beach hotel. Before such construction, the
private respondent, in going to and from their respective properties and the provincial road, passed through
a passageway which traversed the petitioner’s property. As a result of the construction, this passageway,
including the alternative route, was obstructed. Private respondent filed for injunction plus damages. In the
same complaint the private respondents also alleged that the petitioner had constructed a dike on the beach
fronting the latter’s property without the necessary permit, obstructing the passage of the residents and local
fishermen, and trapping debris of flotsam on the beach. The private respondent also claim that the have
acquired the right of way through prescription. They prayed for the re-opening of the “ancient road right of
way” (what they called the supposed easement in this case) and the destruction of the dike. Petitioner
answered by saying that their predecessor in interest’s act of allowing them to pass was gratuitous and in
fact, they were just tolerating the use of the private respondents. CA ruled in favor of the private
respondents.

ISSUE:
Whether or not the private respondents had acquired an easement of right of way in the form of a
passageway, on the petitioner’s property.

HELD:
No. The owner of the dominant estate may validly claim a compulsory right of way only after he has
established the existence of four requisites, to wit: (1) the (dominant) estate is surrounded by other
immovables and is without adequate outlet to a public highway; (2) after payment of the proper indemnity;
(3) the isolation was not due to the proprietor's own acts; and (4) the right of way claimed is at a point least
prejudicial to the servient estate. Additionally, the burden of proving the existence of the foregoing pre-
requisites lies on the owner of the dominant estate. The private respondent failed to prove that there is no
adequate outlet from their respective properties to a public highway.

In the case at hand, the respondent court committed a reversible error by considering the passageway in
issue as a compulsory easement which the private respondents, as owners of the "dominant" estate, may
demand from the petitioner the latter being the owner of the "servient" estate. Apparently, the CA lost sight
of the fact that the convenience of the dominant estate was never a gauge for the grant of compulsory right
of way. There must be a real necessity and not mere convenience for the dominant estate to acquire such
easement.
Hence, when there is already an existing adequate outlet from the dominant estate to a public highway,
even if the said outlet, for one reason or another, be inconvenient, the need to open up another servitude is
entirely unjustified. For to justify the imposition of an easement or right of way, "there must be a real, not a
fictitious or artificial necessity for it. Also, the private respondents made no mention of their intention to
indemnify the petitioners. But while a right of way is legally demandable, the owner of the dominant estate
is not at liberty to impose one based on arbitrary choice. Under Article 650 of the Code, it shall be established
upon two criteria: (1) at the point least prejudicial to the servient state; and (2) where the distance to a public
highway may be the shortest. According, however, to one commentator, "least prejudice" prevails over
"shortest distance."

180
EASEMENTS
Easements

IT IS THE NEEDS OF THE DOMINANT PROPERTY WHICH ULTIMATELY DETERMINE THE WIDTH OF
THE PASSAGE, AND THESE NEEDS MAY VARY FROM TIME TO TIME

152. Encarnacion v. Court of Appeals


G.R. No. 77628, March 11, 1991
Fernan, C.J.

FACTS:
Presented for resolution in the instant petition for review is the not-so-usual question of whether or not
petitioner is entitled to a widening of an already existing easement of right-of-way. Both the trial court and
the Appellate Court ruled that petitioner is not so entitled, hence the recourse to this Court.

Tomas Encarnacion is the owner of the dominant estate which is bounded on the north by the servient
estates of Eusebio de Sagun and Mamerto Masigno, on the south by a dried river and the Taal Lake. The
servient estate is bounded on the north by the National Highway.

Prior to 1960, persons going to the national highway would just cross the servient estate at no particular
point. In 1960, Sagun and Masigno enclosed their lands with a fence but provided a road path 25 meters
long and about 1 meter in width. At this time, Encarnacion started his plant nursery business on his land.
When his business flourished, it became more difficult to transfer the plants and garden soil through the use
of a pushcart so Encarnacion bought an owner-type jeep for transporting the plants. However, the jeep
could not pass through the road path so he approached Sagun and Masigno asking them if they would sell
to him 1 ½ meters of their property to add to the existing road path but the 2 refused the offer. Encarnacion
then instituted an action before the RTC to seek the issuance of a writ of easement of a right of way over
an additional width of at least 2 meters. The RTC dismissed the complaint for there is another outlet, which
is through the dried river bed. This was affirmed by the CA thus the case at bar.

ISSUE:
Whether or not Valisno has the right to continue to enjoy the easement of aqueduct or water through the
Whether or not Encarnacion is entitled to a widening of an already existing easement of right-of-way.

HELD:
Yes. Encarnacion has sufficiently established his claim. Generally, a right of way may be demanded: (1)
when there is absolutely no access to a public highway, and (2) when, even if there is one, it is difficult or
dangerous to use or is grossly insufficient.

In the case at bar, although there is a dried river bed, it traversed by a semi-concrete bridge and there is no
egress or ingress from the highway and such access is grossly inadequate. For the jeep to reach the level
of the highway, it must literally jump 4-5 meters up. And during rainy season, it is impassable due to the
floods. When a private property has no access to a public road, it has the right of easement over adjacent
servient estates as a matter of law. With the non-availability of the dried river bed as an alternative route,
the servient estates should accommodate the needs of the dominant estate. Article 651 of the Civil Code
provides that "(t)he width of the easement of right of way shall be that which is sufficient for the needs of
the dominant estate, and may accordingly be changed from time to time." This is taken to mean that under
the law, it is the needs of the dominant property which ultimately determine the width of the passage. And
these needs may vary from time to time. To grant the additional easement of right of way of 1 ½ meters and
since the easement to be established in favor of petitioner is of a continuous and permanent nature, the
indemnity shall consist of the value of the land occupied and the amount of the damage caused to the
servient estate pursuant to Article 649 of the Civil Code.

181
EASEMENTS
Easements

PRESUMPTIONS WITH RESPECT TO PARTY WALLS

153. Case v. Heirs of Tuason


G.R. No. 5044, December 1, 1909
Torres, J.

FACTS:
In a written opposition, counsel for the heirs of Pablo Tuason and Leocadia Santibañez alleged that the
parties whom he represents are owners in common of the property adjoining that of the petitioner on the
southwest. It was further alleged that the petitioner, in making the plan attached to his petition, extended his
southwest boundary line to a portion of the lot of the said heirs of Tuason and Santibañez in the form
indicated by the red line in the annexed plan; that the true dividing line between the property of the petitioner
and that of the said heirs is the walls indicated in black ink on the accompanying plan; that said walls belong
to the opponents, and that about two years ago, when the applicant made alterations in the buildings erected
on his land, he improperly caused a portion of them to rest on the wall owned by the parties whom he
represents. For which reason the opponent prayed the court to direct the applicant to amend the line marked
in his plan so that it may agree with the wall indicated on the plan which accompanied the written opposition,
and that the applicant be compelled to remove the supports that he placed for his buildings on the wall of
the representatives of the petitioner, and that he be sentenced to pay the costs.

ISSUE:
Is the wall covered by a title, a property of the opposing party?

HELD:
Yes, the wall belongs to the opposing party.

Under article 572 of the Civil Code the easement of party walls is presumed, unless there is a title or exterior
sign, or proof to the contrary, among others, in dividing walls of adjoining buildings up to the common point
of elevation. The legal presumption as to party walls is limited to the three cases dealt with in the said article
of the code, and is that of juris tantum unless the contrary appear from the little of ownership of the adjoining
properties, that is to say, that the entire wall in controversy belongs to one of the property owners, or where
there is no exterior sign to destroy such presumption and support a presumption against the party wall. (Art.
573, Civil Code.)

It cannot be presumed that the aforesaid portion was a party wall, and that it was not exclusively owned by
the defendants, inasmuch as the latter have proven by means of a good title that has not been impugned
by the petitioner, that when one of their ancestors and principals acquired the property the lot was already
inclosed by the wall on which the building was erected. It must therefore be understood that in the purchase
of the property the wall by which the land was inclosed was necessarily included. The above documentary
evidence has not been overcome by any other presented by the petitioner, by apart from the record
discloses the existence of certain unquestionable signs. These consist of constructions made by the
petitioner himself on his own property which entirely destroy any presumption that it is a party wall, and
indeed gives rise to a presumption against it.

Three openings have been made in the wall, undoubtedly to allow the passage of air and light. Two of them
are beveled on the side toward the land of the objectors, and the third had recently imbedded in the wall on
the side of the property of the opponents. These things constitute exterior signs and were recorded as the
result of personal inspection by the trial court in company with the experts of both parties. These signs
positively and conclusively prove that the said wall is not a party wall, but the exclusive property of the
defendant.

182
EASEMENTS
Easements

WINDOWS WITH DIRECT VIEWS, OR BALCONIES OR ANY SIMILAR OPENINGS PROJECTING OVER
THE ESTATE OF A NEIGHBOR, CANNOT BE MADE WITHOUT COMPLYING WITH THE DISTANCE
PROVIDED BY LAW

154. Choco v. Santamaria


G.R. No. 6076, December 29, 1911
Mapa, J.

FACTS:
The defendant, Isidro Santamaria, in the building of his house made several openings and windows in the
walls of the house on both sides overlooking the property of the plaintiffs, Severina and Flora Choco. At the
time the defendant was building his house, and the windows and openings were being made, the plaintiffs
protested, and later on and in the year 1905 made written protest and demand on the defendant. The
defendant received the written protest and referred it to his counsel, who, from the evidence, appears to
have suggested an amicable adjustment of the matter, but the adjustment was not made, and this action
was brought.

The trial court rendered a judgment in favor of the plaintiffs and against the defendant, forever prohibiting
the opening of the window stated, which must be closed, and forever prohibiting the opening of the windows
and openings marked, which must be closed or made to conform to the requirements of law with regard to
dimensions and an iron grate embedded in the wall.

ISSUE:
Whether or not lower court erred by not ordering in his judgment the final and perpetual closing of the large
window opened in the balcony of the back part of the appellee’s house and that, though the appellants’ lot
can be seen from this window, it is not contiguous to the latter’s property.

HELD:
Yes, the window must be closed because its opening is a manifest violation of the provisions of article 582
of the Civil Code.

It appears from the evidence, that the window referred to is next to the appellants’ lot. To judge from the
photographic views, it opens on the boundary line between the said lot and that of the appellee and is
situated perpendicularly above a part of the wall that belongs to the appellants. This opinion is corroborated
by the testimony of the defendant’s witness who took the said photographs, in so far as he said that "a part
of the window in question is in front of the plaintiffs’ property, and a person approaching the window may
clearly see the said lot." And certainly if it is in front of this lot, it is unquestionable that it directly overlooks
the same; but even though it did not and only a side or oblique view of the lot could be obtained from it, it
could not be kept open, since between it and the plaintiffs’ property there does not intervene the distance
required by law — that of two meters in the first case, and 60 centimeters in the second. In reality, there is
no distance at all between the said window and the plaintiffs’ lot because this window is perpendicular to
the boundary line of the said lot. Thus, because of the lack of the distance required by law, the window in
question must be closed, and consequently the judgment appealed from should be modified in this sense,
as regards this window.

Therefore, the defendant is ordered to close finally and forever the window marked, the judgment appealed
from in so far as it refers to said window being thus modified, but affirmed in all other respects.

183
EASEMENTS
Easements

SERVITUDES ARE MERELY ACCESSORIES TO THE TENEMENTS OF WHICH THEY FORM PART,
AND EVEN IF THEY ARE POSSESSED OF A SEPARATE JURIDICAL EXISTENCE, THEY CANNOT BE
ALIENATED FROM THE TENEMENT OR MORTGAGED SEPARATELY

155. Solid Manila Corp. v. Bio Hong Trading Co., Inc.


G.R. No. 90596, April 8, 1991
Sarmiento, J.

FACTS:
The petitioner, Solid Manila Corporation, is the owner of a parcel of land located in Ermita, Manila, covered
by a transfer certificate of title. The same lies in the vicinity of another parcel, registered in the name of the
private respondent corporation, Bio Hong Trading Co., Inc., under another transfer certificate of title. The
private respondent's title came from a prior owner, and in their deed of sale, the parties thereto reserved as
an easement of way. As a consequence, an annotation was entered in the private respondent's title.

The petitioner claims that ever since, it had (as well as other residents of neighboring estates) made use of
the above private alley and maintained and contributed to its upkeep, until sometime in 1983, when, and
over its protests, the private respondent constructed steel gates that precluded unhampered use. The
petitioner then commenced suit for injunction against the private respondent, to have the gates removed
and to allow full access to the easement.

ISSUE:
Whether or not there still an easement on the property in question regardless of the sale.

HELD:
Yes, the sale of the property did include the alley.

The Court rejects the petitioner's contention that the deed of sale "excluded" it, because as a mere right-of-
way, it cannot be separated from the tenement and maintain an independent existence. Servitudes are
merely accessories to the tenements of which they form part. Although they are possessed of a separate
juridical existence, as mere accessories, they cannot, however, be alienated from the tenement, or
mortgaged separately.

As the petitioner indeed hastens to point out, the deed itself stipulated that "a portion thereof [of the
tenement] measuring NINE HUNDRED FOURTEEN SQUARE METERS, more or less, had been converted
into a private alley for the benefit of the neighboring estates. . ." and precisely, the former owner, in
conveying the property, gave the private owner a discount on account of the easement. Hence, albeit the
private respondent did acquire ownership over the property –– including the disputed alley –– as a result of
the conveyance, it did not acquire the right to close that alley or otherwise put up obstructions thereon and
thus prevent the public from using it, because as a servitude, the alley is supposed to be open to the public.

The Court is furthermore of the opinion, contrary to that of the Court of Appeals, that no genuine merger
took place as a consequence of the sale in favor of the private respondent corporation. According to the
Civil Code, a merger exists when ownership of the dominant and servient estates is consolidated in the
same person. Merger then, as can be seen, requires full ownership of both estates. The servitude in
question is a personal servitude, that is to say, one constituted not in favor of a particular tenement (a real
servitude) but rather, for the benefit of the general public.

Thus, in the case at bar, the defense of merger is, clearly, not a valid defense, indeed, a sham one, because
as we said, merger is not possible, and secondly, the sale unequivocally preserved the existing easement.

184
EASEMENTS
Easements

MERE CONVENIENCE FOR THE DOMINANT ESTATE IS NOT WHAT IS REQUIRED BY LAW AS THE
BASIS FOR SETTING UP A COMPULSORY EASEMENT

156. Floro v. Llenado


G.R. No. 75723, June 2, 1995
Romero, J.

FACTS:
Simeon Floro is the owner of a piece of land known as the Floro Park Subdivision situated in Barangay
Saluysoy, Meycauayan, Bulacan. The subdivision has its own egress and ingress to and from the MacArthur
Highway by means of its Road Lot 4 and the PNR level crossing. Orlando A. Llenado, on the other hand,
was the registered owner of two (2) parcels of land known as the Llenado Homes Subdivision ("Llenado
Homes," for brevity). Prior to its purchase by Llenado from the owner Francisco de Castro, the land was
known as the Emmanuel Homes Subdivision, a duly licensed and registered housing subdivision in the
name of Soledad Ortega. Bounded on the South by the 5 to 6 meter-wide Palanas Creek, which separates
it from the Floro Park Subdivision, and on the west by ricelands belonging to Marcial Ipapo, Montaos and
Guevarra, the Llenado Homes does not have any existing road or passage to the MacArthur Highway.
However, a proposed access road traversing the idle riceland of Marcial Ipapo has been specifically
provided in the subdivision plan of Emmanuel Homes Subdivision which was duly approved by the defunct
Human Settlement Regulatory Commission (now Housing and Land Use Regulatory Board).

Sometime in February, 1983, the Llenados sought, and were granted, permission by the Floros to use Road
Lots 4 and 5 of the Floro Park Subdivision as the passageway to and from MacArthur Highway. On April 7,
1983, however, Floro barricaded Road Lot 5 with a pile of rocks, wooden posts and adobe stones, thereby
preventing its use by the Llenados.

ISSUE:
Whether or not there is a valid contract of easement of right of way when the owner of one estate voluntarily
allows the owner of an adjacent estate a passage through his property for a limited time without
compensation.

HELD:
No, private respondents Llenados cannot claim entitlement to a right of way.

It is not disputed that Floro granted the Llenados verbal permission to pass through the Floro Park
Subdivision in going to and from the MacArthur Highway. Whether such permission, as claimed by Floro,
was for the month of March only, without compensation and as a neighborly gesture for the purpose merely
of enabling the Llenados to install stone monuments on their land, or was in relation to the easement of right
of way granted in their favor, as insisted by the Llenados, the fact remains that no such contract of easement
of right of way was actually perfected between Floro and Llenado. Both Orlando and Wenifreda Llenado
testified the conditions of the easement of right of way were still to be up by Floro’s lawyer. Thus, no
compensation was agreed upon, and none was paid, for the passage through Floro’s property during the
month of March. The use of Road Lots 4 and 5 by the Llenados during the month of March was by mere
tolerance of Floro pending the negotiation of the terms and conditions of the right of way. Furthermore, for
the Llenados to be entitled to a compulsory servitude of right of way under the Civil Code, the preconditions
provided under Articles 649 and 650 thereof must be established. These preconditions are: (1) that the
dominant estate is surrounded by other immovables and has no adequate outlet to a public highway; (2)
after payment of proper indemnity; (3) that the isolation was not due to acts of the proprietor of the dominant
estate; and, (4) that the right of way claimed is at the point least prejudicial to the servient estate; and insofar
as consistent with this rule, where the distance from the dominant estate to a public highway may be the
shortest. The burden of providing the existence of the prerequisites to validly claim a compulsory right of
way lies on the owner of the dominant estate. The Court found that private respondents have failed in this
regard.

185
EASEMENTS
Easements

THE CRITERION OF LEAST PREJUDICE TO THE SERVIENT ESTATE MUST PREVAIL OVER THE
CRITERION OF SHORTEST DISTANCE

157. Quimen v. Court of Appeals


G.R. No. 112331, May 29, 1996
Bellosillo, J.

FACTS:
Petitioner Anastacia Quimen together with her brothers Sotero, Sulpicio, Antonio and sister Rufina inherited
a piece of property situated in Pandi, Bulacan. They agreed to subdivide the property equally among
themselves, as they did, with the shares of Anastacia, Sotero, Sulpicio and Rufina abutting the municipal
road. In February 1982 Yolanda purchased a lot from her uncle Antonio through her aunt Anastacia who
was then acting as his administratrix. According to Yolanda, when petitioner offered her the property for
sale she was hesitant to buy as it had no access to a public road. But Anastacia prevailed upon her to buy
the lot with the assurance that she would give her a right of way on her adjoining property for P200.00 per
square meter. Thereafter, Yolanda constructed a house on the lot she bought using as her passageway to
the public highway a portion of Anastacia's property. But when Yolanda finally offered to pay for the use of
the pathway Anastacia refused to accept the payment. In fact, she was thereafter barred by Anastacia from
passing through her property.

In February 1986 Yolanda purchased the other lot of Antonio Quimen, located directly behind the property
of her parents who provided her a pathway gratis et amore between their house, extending about nineteen
(19) meters from the lot of Yolanda behind the sari sari store of Sotero, and Anastacia's perimeter fence.
The store is made of strong materials and occupies the entire frontage of the lot. Although the pathway
leads to the municipal road it is not adequate for ingress and egress. The municipal road cannot be reached
with facility because the store itself obstructs the path so that one has to pass through the back entrance
and the facade of the store to reach the road.

ISSUE:
Whether or not right of way may be established on petitioner’s property.

HELD:
Yes. Article 650 of the New Civil Code explicitly states that the easement of right of way shall be established
at the point least prejudicial to the servient estate and, insofar as consistent with this rule, where the distance
from the dominant estate to a public highway may be the shortest. Where the easement may be established
on any of several tenements surrounding the dominant estate, the one where the way is shortest and will
cause the least damage should be chosen. However, as elsewhere stated, if these two (2) circumstances
do not concur in a single tenement, the way which will cause the least damage should be used, even if it
will not be the shortest. In applying Art. 650, respondent Court of Appeals declared that the proposed right
of way of Yolanda, which is one (1) meter wide and five (5) meters long at the extreme right of petitioner's
property, will cause the least prejudice and/or damage as compared to the suggested passage through the
property of Yolanda's father which would mean destroying the sari sari store made of strong materials.
Absent any showing that these findings and conclusion are devoid of factual support in the records, or are
so glaringly erroneous, the Court accepts and adopts them. As between a right of way that would demolish
a store of strong materials to provide egress to a public highway, and another right of way which although
longer will only require an avocado tree to be cut down, the second alternative should be preferred.

In sum, the Court found that the decision of respondent appellate court is thoroughly backed up by law and
the evidence.

186
EASEMENTS
Easements

DUTY OF AN ADJACENT OWNER NOT TO DEPRIVE ANY ADJACENT LAND OR BUILDING OF


SUFFICIENT LATERAL OR SUBJACENT SUPPORT IS AN ABSOLUTE ONE

158. De Jesus v. Howmart Corp.


CA No. 44191-R. August 28, 1974
Leuterio, J.

FACTS:
Manuel De Jesus and Luz Miranda de Jesus owned a lot which adjoined the lot owned by defendant
Howmart Corporation. Over the latter property, a six-storey reinforced concrete building was constructed. A
complaint for damages was filed by the plaintiffs De Jesus stating that defendant corporation failed to
observe the necessary care and precautions to protect the construction of the plaintiffs by depriving it of
sufficient lateral and subjacent support. As a result, the building of the plaintiffs sank in some places and
the roofing of the building was damaged with the accumulated debris piled thereon. Sketch marks show that
the buildings owned by the parties are so close to each other that they almost appeared to be attached
together.

Judgment was rendered in favour of the plaintiffs stating that it is incumbent upon Howmart Corporation to
exercise due care in making the excavation in a way that will not deprive the property of plaintiffs its natural
support to prevent it from sinking, sagging or crumbling. The position of the defendant is that they took all
the necessary preconstruction precautions so as not to damage the lateral and subjacent support of the
adjoining property.

ISSUE:
Whether or not proper precautions had been taken by the defendant in constructing the building in question
so as to prevent damages to the building of the plaintiffs.

HELD:
No. Article 684 of the New Civil Code provides, “No property shall make such excavations upon his land as
to deprive any adjacent land or building sufficient lateral or subjacent support”. A reading of Article 684
shows that the duty of an adjacent owner not to deprive any adjacent land or building of sufficient lateral or
subjacent support is an absolute one. It does not depend on the degree of care and precaution made by
the proprietor in making the excavation or building on his land. Plaintiffs’ house which adjoins the seven-
storey concrete building constructed by the defendants had sunk by about eight inches. The sinking of the
left side of the house of the plaintiffs was due to the weakening of subjacent support and to the weight of
the seven-storey concrete building constructed by the defendant, as the excavation made necessarily
disturbed the subjacent soil of the plaintiffs’ land. Defendants, having failed to provide the plaintiffs’ land
and house with sufficient lateral and subjacent support, are liable for damages.

187
EASEMENTS
Easements

A LEGAL OR COMPULSORY EASEMENT IS THAT WHICH IS CONSTITUTED BY LAW FOR PUBLIC


USE OR FOR PRIVATE INTEREST, WHEREAS A VOLUNTARY EASEMENT ON THE OTHER HAND IS
CONSTITUTED SIMPLY BY WILL OR AGREEMENT OF THE PARTIES

159. La Vista Association, Inc. v. Court of Appeals


G.R. No. 95252, September 5, 1997
Bellosillo, J.

FACTS:
Mangyan Road was originally part of a vast tract of land owned by the Tuasons in Quezon City and Marikina.
The Tuasons sold to Philippine Building Corporation (PBC) a portion of their landholdings by virtue of a
Deed of Sale with Mortgage. The deed provides that the boundary line between the property sold and the
adjoining property of the vendors shall be a road fifteen (15) meters wide, one-half of which shall be taken
from the property sold to the vendee and the other half from the portion adjoining belonging to the vendors.
Thereafter, Philippine Building Corporation sold and assigned, with the consent of the Tuasons, the subject
parcel of land to Ateneo which assumed the mortgage. The Tuasons developed a part of the estate adjoining
the portion sold to PBC a residential village known as La Vista Subdivision. The boundary between La Vista
and the portion sold to PBC was known as the Mangyan Road.

Thereafter, the Tuasons developed its 7.5-meter share of the 15-meter wide boundary. Ateneo deferred
improvement on its share and erected instead an adobe wall on the entire length of the boundary of its
property. Ateneo then offered to sell the property to the public subject to the condition that the right to use
the 15-meter roadway will be transferred to the vendee wherein La Vista became one of the bidders.
However, it lost to Solid Homes, Inc., in the bidding. Subsequently, Solid Homes, Inc., developed a
subdivision now known as Loyola Grand Villas and together they now claim to have an easement of right-
of-way along Mangyan Road however, La Vista’s President informed Solid Homes, Inc., that La Vista could
not recognize the right-of-way over Mangyan Road. La Vista, then prevented the residents of Loyola from
passing through.

ISSUE:
Whether or not an easement of right of way exist along Mangyan Road.

HELD:
Yes. A legal or compulsory easement of right-of-way should be distinguished from a voluntary easement. A
legal or compulsory easement is that which is constituted by law for public use or for private interest. By
express provisions of Arts. 649 and 650 of the New Civil Code, the owner of an estate may claim a legal or
compulsory right-of-way only after he has established the existence of four (4) requisites, namely, (a) the
estate is surrounded by other immovables and is without adequate outlet to a public highway; (b) after
payment of the proper indemnity; (c) the isolation was not due to the proprietor's own acts; and, (d) the right-
of-way claimed is at a point least prejudicial to the servient estate, and insofar as consistent with this rule,
where the distance from the dominant estate to a public highway may be the shortest. A voluntary easement
on the other hand is constituted simply by will or agreement of the parties.

From the facts of the instant case it is very apparent that the parties and their respective predecessors-in-
interest intended to establish an easement of right-of-way over Mangyan Road for their mutual benefit, both
as dominant and servient estates. The parties concerned had indeed constituted a voluntary easement of
right-of-way over Mangyan Road and, like any other contract, the same could be extinguished only by
mutual agreement or by renunciation of the owner of the dominant estate. The free ingress and egress
along Mangyan Road created by the voluntary agreement between Ateneo and Solid Homes, Inc., is thus
legally with the corresponding duty on the servient estate not to obstruct the same.

188
EASEMENTS
Easements

WHERE A PERSON IS ALLOWED TO CONSTRUCT HIS HOUSE ON THE LAND OF ANOTHER TO


FACILITATE HIS GATHERING OF FRUITS, THIS WOULD BE IN THE NATURE OF A PERSONAL
EASEMENT

160. Alcantara v. Reta, Jr.


G.R. No. 136996, December 14, 2001
Pardo, J.

FACTS:
Petitioners filed with the RTC a complaint against the Cornelio B. Reta, Jr. for the exercise of the right of
first refusal under PD No. 1517 otherwise known as "The Urban Land Reform Act,". They claimed that they
were tenants or lessees of the land owned by Reta. The land has been converted by Reta into a commercial
center and that Reta is threatening to eject them from the land. They assert that they have the right of first
refusal to purchase the land in accordance with the above-mentioned decree since they are legitimate
tenants or lessees thereof. On the other hand, Reta claimed that the land is beyond the ambit of the said
decree since it has not been proclaimed as an Urban Land Reform Zone (ULRZ) and that the applicable
law is BP 25 for failure of the plaintiffs to pay the rentals for the use of the land. In fact, petitioners filed a
petition with the National Housing Authority requesting that the land they were occupying be declared as an
ULRZ.

Respondent Reta denies that he has lease agreements with petitioners Edilberto Alcantara and Ricardo
Roble. Edilberto Alcantara, on the other hand, failed to present proof of a lease agreement other than his
testimony in court that he bought the house that he is occupying from his father-in-law. Respondent Reta
allowed petitioner Ricardo Roble to use sixty-two (62) coconut trees for P186 from where he gathered tuba.

ISSUE:
Whether or not petitioners have the right of personal easement.

HELD:
Yes. Under the law, petitioners were allowed to construct his house on the land because it would facilitate
his gathering of tuba. This would be in the nature of a personal easement under Article 614 of the Civil
Code. The arrangement between respondent Reta and petitioner Ricardo Roble allowing the latter to use
sixty-two (62) coconut trees for P186 from where he gathered tuba would show that it is a usufruct and not
a lease. Usufruct gives a right to enjoy the property of another with the obligation of preserving its form and
substance, unless the title constituting it or the law otherwise provides.

189
EASEMENTS
Easements

THE WORD PASSAGE DOES NOT CLEARLY AND UNMISTAKABLY CONVEY A MEANING THAT
INCLUDES A RIGHT TO INSTALL WATER PIPES ON THE ACCESS ROAD SINCE THE ORDINARY
MEANING OF THE WORD IS THAT IT IS THE ACT OR ACTION OF PASSING: MOVEMENT OR
TRANSFERENCE FROM ONE PLACE OR POINT TO ANOTHER

161. Prosperity Credit Resources Inc. v. Court of Appeals


G.R. No. 114170, January 15, 1999
Mendoza, J.

FACTS:
In 1984, the petitioner gave a loan to private respondent and to secure the payment of the loan, private
respondent mortgaged to petitioner seven parcels of land located at 685 Tandang Sora Avenue, Bo. Banlat,
Quezon City. The lots comprise a commercial compound with Tandang Sora Avenue as the nearest public
road. Private respondent defaulted in the payment of the loan prompting petitioner to foreclose the
mortgage. Petitioner became the highest bidder and purchaser of the seven (7) lots subject of the mortgage.
Later, private respondent negotiated with petitioner for the redemption of three lots all located on the
southern and middle portions of the compound. As the reacquisition of these three lots by private respondent
would leave the remaining four lots on the northwestern side without access to Tandang Sora Avenue,
petitioner acceded to private respondent’s request on the condition that petitioner be given a right of way
on the existing private road which forms part of the area to be redeemed by private respondent. They
executed a Memorandum of Undertaking (MOU) which provides that Lot 11, being an existing private road,
will remain open to ingress and egress for whatever kind of passage.

ISSUE:
Whether or not a passage was made in favor of the petitioner.

HELD:
No. The right of the complainant must be clear and unmistakable because, unlike an ordinary preliminary
injunction, the writ of preliminary mandatory injunction requires the performance of a particular act or acts
and thus tends to do more than maintain the status quo. In the case at bar, petitioner anchors its alleged
right to the preliminary mandatory injunction on the MOU. There is no question as to the meaning of the
terms "ingress" and "egress." They give petitioner the right to use the private road as a means of entry into
and exit from its property on the northwestern side of the compound. The question concerns the meaning
of the phrase "for whatever kind of passage." The trial court read this phrase to mean that petitioner had
the right to make excavations on the side of the access road in order to install a network of water pipes. The
word "passage" does not, however, "clearly and unmistakably" convey a meaning that includes a right to
install water pipes on the access road. The ordinary meaning of the word, as defined in Webster’s Dictionary,
is that it is "the act or action of passing: movement or transference from one place or point to another." Its
legal meaning is not different. It means, according to Black’s Law Dictionary, the "act of passing; transit;
transition." To achieve a meaning such as that which petitioner proposes requires the consideration of
evidence showing the parties’ intention in using the word which can only be done during trial on the merits.
Until such time, petitioner cannot claim to have a "clear and unmistakable" right justifying the issuance of a
writ of preliminary mandatory injunction in this case. Thus, the trial court should have observed caution and
denied petitioner’s application for the preliminary writ.

190
EASEMENTS
Easements

THE NEEDS OF THE DOMINANT ESTATE DETERMINE THE WIDTH OF THE EASEMENT

162. Villanueva v. Velasco


G.R. No. 130845, November 27, 2000
Quisumbing, J.

FACTS:
Petitioner is the registered owner of the parcel of land located in Quezon City which he bought it from Pacific
Banking Corporation. The bank had acquired it from the spouses Maximo and Justina Gabriel at a public
auction. When petitioner bought the parcel of land there was a small house on its southeastern portion. It
occupied one meter of the two-meter wide easement of right of way the Gabriel spouses granted to the
Espinolas, predecessors-in-interest of private respondents, in a Contract of Easement of Right of Way.
Unknown to petitioner, even before he bought the land, the Gabriels had constructed the aforementioned
small house that encroached upon the two-meter easement. Petitioner was also unaware that private
respondents, Julio Sebastian and Shirley Lorilla, had filed a civil case for easement, damages and with
prayer for a writ of preliminary injunction and/or restraining order against the spouses Gabriel. 4 As
successors-in-interest, Sebastian and Lorilla wanted to enforce the contract of easement. The RTC issued
a writ of preliminary mandatory injunction ordering the Gabriels to provide the right of way and to demolish
the small house encroaching on the easement. Thereafter, a Writ od Demolition was issued. Petitioner filed
a Third Party Claim with Prayer to Quash Alias Writ of Demolition maintaining that the writ of demolition
could not apply to his property since he was not a party to the civil case. However, quashed.

ISSUE:
Whether or not easement on the property binds the petitioner.

HELD:
Yes. The Court note that the subject easement of right of way originally was voluntarily constituted by
agreement between the Gabriels and the Espinolas. But as correctly observed by the CA, the easement in
the instant petition is both (1) an easement by grant or a voluntary easement, and (2) an easement by
necessity or a legal easement. A legal easement is one mandated by law, constituted for public use or for
private interest, and becomes a continuing property right. As a compulsory easement, it is inseparable from
the estate to which it belongs, as provided for in said Article 617 of the Civil Code. The essential requisites
for an easement to be compulsory are: (1) the dominant estate is surrounded by other immovables and has
no adequate outlet to a public highway; (2) proper indemnity has been paid; (3) the isolation was not due to
acts of the proprietor of the dominant estate; (4) the right of way claimed is at a point least prejudicial to the
servient estate; and (5) to the extent consistent with the foregoing rule, where the distance from the
dominant estate to a public highway may be the shortest. The RTC and CA have declared the existence of
said easement. This finding of fact of both courts below is conclusive on this Court, hence the Court sees
no need to further review, but only to re-affirm, this finding. The small house occupying one meter of the
two-meter wide easement obstructs the entry of private respondents’ cement mixer and motor vehicle. One
meter is insufficient for the needs of private respondents. It is well-settled that the needs of the dominant
estate determine the width of the easement.

191
EASEMENTS
Easements

WHERE THE LAND WAS ORIGINALLY PUBLIC LAND, AND AWARDED BY FREE PATENT WITH A
RESERVATION FOR A LEGAL EASEMENT OF A RIGHT OF WAY IN FAVOR OF THE GOVERNMENT,
JUST COMPENSATION NEED NOT BE PAID FOR THE TAKING OF A PART THEREOF FOR PUBLIC
USE AS AN EASEMENT OF RIGHT OF WAY, UNLIKE IF THE LAND WERE ORIGINALLY PRIVATE
PROPERTY

163. National Irrigation Administration v. Court of Appeals


G.R. No. 114348, September 20, 2000
Pardo, J.

FACTS:
In 1974, Mangpalus acquired a lot situated in the municipality of Alcala, Province of Cagayan, from Vicente
Manglapus by absolute sale. Said lot was issued in the name of Vicenete Manglapus by free patent and
was subsequently registered by the latter. The land grant provided, among others, a condition that the land
shall be subject to all conditions and public easements and servitudes recognized and prescribed by law
especially those mentioned in Sections 109, 110, 111, 112, 113 and 114 of Commonwealth Act No, 141, as
amended.

Sometime in 1982, the NIA entered into a contract with Villamar Development Construction. Under the
contract, NIA was to construct canals in Amulung Cagayan and Alcala, Cagayan. The NIA then entered into
a portion of Manglapus’ land and made diggings and fillings thereon. Manglapus filed with the RTC a
complaint for damages against NIA alleging that the diggings and fillings destroyed the agricultural use of
his land and that no reasonable compensation was paid for its taking. NIA contends that the TCT covering
the parcel of land contained a reservation granting a right over the land covered therein.

ISSUE:
Whether or not Manglapus is entitled with just compensation for the taking of a portion of his property for
use of an easement of right of way.

HELD:
No, Manglapus is not. The NIA is under no obligation to pay just compensation for the easement of right of
way. The TCT contained a reservation stating that the title of the land shall be: “… and subject, further to
such conditions contained in the original title as may be subsisting.” Under the Original Certificate of Title,
there was a reservation and condition that the land is subject to “all conditions and public easements and
servitudes recognized and prescribed by law especially those mentioned in Sections 109,110,111,112,113,
and 114, Commonwealth Act No. 141, as amended.” This reservation, unlike the other provisos imposed
on the grant, was not limited by any time period and thus is a subsisting condition.

In the present case, we find and declare that a legal easement of a right-of-way exists in favor of the
government. The land was originally public land, and awarded to respondent Manglapus by free patent. The
ruling would be otherwise if the land were originally private property, in which case, just compensation must
be paid for the taking ofa part thereof for public use as an easement of a right of way. Hence, Manglapus is
not entitled to just compensation for the portion of his property taken for use as easement of right of way.

192
EASEMENTS
Easements

AN EASEMENT OF A RIGHT OF WAY IS DISCONTINUOUS AND CANNOT BE ACQUIRED THROUGH


PRESCRIPTION

164. Alolino v. Flores


G.R. No.198774, April 4, 2016
Brion, J.

FACTS:
Alolino is the registered owner of two (2) contiguous parcels of land situated at No. 47 Gen. Luna Street,
Barangay Tuktukan, Taguig since 1977. On this property, Alolino constructed a two-storey house up to the
edge of said property. In. 1994, the Floreses constructed their house/sari sari store on the vacant
municipal/barrio road immediately adjoining the rear perimeter wall of Alolino’s house. Since they were
constructing on a municipal road, the respondents could not secure a building permit. The respondents'
construction deprived Alolino of the light and ventilation he had previously enjoyed and prevented his ingress
and egress to the municipal road.

Despite repeated demands from Alolino since 1995 that the respondent spouses remove their structure, the
latter refused, prompting Alolino to file a complaint on 2003. Alolino contends, among others, that he
acquired an easement of light and view by virtue of a title because the respondents constructed their house
on a barrio road and that he has a right of way over the lot occupied by the respondents because it is a
barrio road.

The respondent counters that they had occupied the land since 1955 and Alolino has not acquired an
easement of light and view or an easement of right of way, by either prescription or title and that it his fault
as he constructed his house without observing the required setback by law, and that the Sanggunian had
already withdrawn the subject barrio road from public use and reclassified it as a residential lot.

ISSUE:
Whether or not Alolino acquire an easement of light and view and an easement of right of way over the lot.

HELD:
No, Alolino did not. An easement of light and view can be acquired through prescription counting from the
time when the owner of the dominant estate formally prohibits the adjoining lot owner from blocking the view
of a window located within the dominant estate.

Notably, Alolino had not made (and could not have made) a formal prohibition upon the respondents prior
to their construction in 1994; Alolino could not have acquired an easement of light and view through
prescription.

Likewise, Alolino did not acquire an easement of right of way.

Article 649 creates a legal easement in favor of an owner or any person entitled to use any immovable,
which is landlocked by other immovables pertaining to other persons without an adequate access to a public
highway. Article 652 creates a legal easement in favor of an isolated piece of land acquired by sale,
exchange, partition, or donation when it is surrounded by other estates of the vendor, exchanger, co-owner,
or donor. Article 653 grants the same right of way. in favor of the vendor, exchanger, co-owner, or donor
when his property is the one that becomes isolated. Article 656 grants the owner of an estate, after payment
of indemnity, a right of way to carry materials through the estate of another when it.is indispensable for the
construction or repair of a building in his estate. Finally, Article 657 governs right of way easements for the
passage of livestock.

None of these provisions are applicable to Alolino's property with respect to the barrio road where the
respondents' house stands on.

193
EASEMENTS
Easements

AN OWNER, BY VIRTUE OF HIS SURFACE RIGHT, MAY MAKE EXCAVATIONS ON HIS LAND, BUT
HIS RIGHT IS SUBJECT TO THE LIMITATION THAT HE SHALL NOT DEPRIVE ANY ADJACENT LAND
OR BUILDING OF SUFFICIENT LATERAL OR SUBJACENT SUPPORT

165. Castro v. Monsod


G.R. No. 183719, February 2, 2011
Nachura, J.

FACTS:
Margarita Castro (Castro), petitioner, is the registered owner of a parcel of land located on Manuela Homes,
Las Piñas City. Respondent, Napoleon Monsod, is the owner of the property adjoining the lot of petitioner,
located on Moonwalk Village, Las Piñas City. On February 29, 2000, Monsod caused the annotation of an
adverse claim against sixty-five (65) sq.m. of the property of Castro. The adverse claim was filed without
any claim of ownership over the property as Monsod was merely asserting the existing legal easement of
lateral and subjacent support at the rear portion of his estate to prevent the property from collapsing, since
his property is located at an elevated plateau of fifteen (15) feet, more or less, above the level of Castro’s
property. Castro avers that when she bought the property from Manuela Homes in 1994, there was no
annotation or existence of any easement over the property. Monsod neither asked permission nor talked to
her with regard to the use of 65 sq.m. of her property as easement.

Monsod claims that when he bought the property in 1983, the land elevation of Moonwalk Village was almost
on the same level as Manuela Homes. However, the developer of Manuela Homes, excavated and
transferred portions of the elevated land to the lower portions of Manuela Homes. Thus, Manuela Homes
became lower than Moonwalk Village. Before the said excavation, respondent personally complained to the
developer and was assured that, as provided by the National Building Code, an embankment will be retained
at the boundary of Manuela Homes and Moonwalk Village, which is more or less fifteen (15) feet higher
than Manuela Homes. Manuela Homes retained the embankment consisting of soil and rocks. Monsod had
the open space riprapped with stones as reinforcement against any potential soil erosion, earthquake, and
possible digging by any person. Monsod asserted that the affidavit of adverse claim was for the annotation
of the lateral and subjacent easement of his property over the property of Castro, in view of the latter’s
manifest determination to remove the embankment left by the developer of Manuela Homes.

ISSUE:
Whether or not easement of lateral an subjacent support exist on the subject adjacent properties.

HELD:
Yes, an easement of lateral and subjacent support exists on the subject adjacent properties. Article 437 of
the Civil Code provides that the owner of a parcel of land is the owner of its surface and of everything under
it, and he can construct thereon any works, or make any plantations and excavations which he may deem
proper. However, such right of the owner is not absolute and is subject to the following limitations: (1)
servitudes or easements, (2) special laws, (3) ordinances, (4) reasonable requirements of aerial navigation,
and (5) rights of third persons. Article 684 of the Civil Code provides that no proprietor shall make such
excavations upon his land as to deprive any adjacent land or building of sufficient lateral or subjacent
support. An owner, by virtue of his surface right, may make excavations on his land, but his right is subject
to the limitation that he shall not deprive any adjacent land or building of sufficient lateral or subjacent
support. Between two adjacent landowners, each has an absolute property right to have his land laterally
supported by the soil of his neighbor, and if either, in excavating on his own premises, he so disturbs the
lateral support of his neighbor’s land as to cause it, or, in its natural state, by the pressure of its own weight,
to fall away or slide from its position, the one so excavating is liable.

194
EASEMENTS
Easements

LEAST PREJUDICE IS ABOUT THE SUFFERING OF THE SERVIENT ESTATE. ITS VALUE IS NOT
DETERMINED SOLELY BY THE PRICE OF THE PROPERTY, BUT ALSO BY THE VALUE OF THE
OWNER’S FOREGONE OPPORTUNITY FOR USE, RESULTING FROM THE LIMITATIONS IMPOSED
BY THE EASEMENT

166. Reyes v. Valentin


G.R. No. 194488, February 11, 2015
Leonen, J.

FACTS:
On March 28, 2006, petitioner Alicia B. Reyes (Reyes), filed a Complaint for easement of right of way against
respondents, Spouses Francisco S. Valentin and Anatalia Ramos (Sps Valentin).
Reyes alleged that she was the registered owner of a 450-square-meter parcel of land which is surrounded
by respondents’ 1,500-square-meter property, and that the latter’s property was the only adequate outlet
from her property to the highway. A 113-square-meter portion of respondents' property was the "point least
prejudicial to the respondents.” The easement sought was the vacant portion near the boundary of
respondents' other lot. The trial court found that petitioner's proposed right of way was not the least onerous
to the servient estate of respondents. It noted that the proposed right of way would pass through
improvements, such as respondents' garage, garden, and grotto. The trial court also noted the existence of
an irrigation canal that limited access to the public road. However, the trial court pointed out that ‘other than
the existing irrigation canal, no permanent improvements/structures can be seen standing on the subject
rice land." Moreover, the nearby landowner was able to construct a bridge to connect a property to the
public road. Hence, "the way through the irrigation canal would appear to be the shortest and easiest way
to reach the barangay road."

Petitioner argued that the Regional Trial Court and the Court of Appeals failed to consider that it was not
her property that was adjacent to the irrigation canal but her sister's. Her property was surrounded by other
estates belonging to other persons. Hence, she had to pass through other properties before reaching the
irrigation canal.

ISSUE:
Whether or not petitioner have a compulsory right of way over the respondent’s property.

HELD:
No, petitioner does not. Mere convenience for the dominant estate is not what is required by law as the
basis of setting up a compulsory easement. An easement of right of way is a real right. When an easement
of right of way is granted to another person, the rights of the property's owner are limited. An owner may
not exercise some of his or her property rights for the benefit of the person who was granted the easement
of right of way. The following requisites need to be established before a person becomes entitled to demand
the compulsory easement of right of way:

(1) An immovable is surrounded by other immovables belonging to other persons, and is without adequate
outlet to a public highway;
(2) Payment of proper indemnity by the owner of the surrounded immovable;
(3) The isolation of the immovable is not due to its owner's acts; and
(4) The proposed easement of right of way is established at the point least prejudicial to the servient estate,
and insofar as consistent with this rule, where the distance of the dominant estate to a public highway may
be the shortest.

In the case at hand, petitioner failed to establish that there was no adequate outlet to the public highway
and that the proposed easement was the least prejudicial to respondents' estate. As way the RTC and
affirmed by the CA, there is an adequate exit to a public highway.

195
EASEMENTS
Easements

EASEMENT OF RIGHT OF WAY SHALL BE ESTABLISHED AT THE POINT LEAST PREJUDICIAL TO


THE SERVIENT ESTATE, AND, INSOFAR AS CONSISTENT WITH THIS RULE, WHERE THE
DISTANCE FROM THE DOMINANT ESTATE TO A PUBLIC HIGHWAY MAY BE SHORTEST

167. Calimoso v. Roullo


G.R. No. 198594, January 25, 2016
Brion, J.

FACTS:
In his Complaint for Easement of Right of Way, the respondent mainly alleged: that he is the owner of a lot
situated in Iloilo City; that his lot is isolated by several surrounding estates, including lot owned by petitioners
Helen, Marilyn, and Liby, all surnamed Calimoso (the Calimosos); that he needs a right-of-way in order to
have access to a public road; and that the shortest and most convenient access to the nearest public
road, i.e., Fajardo Subdivision Road, passes through the petitioners’ lot.

The Calimosos objected to the establishment of the easement because it would cause substantial damage
to the two (2) houses already standing on their property. They alleged that the respondent has other right-
of-way alternatives, such as the existing wooden bridge over Sipac Creek bounding the respondent’s lot on
the northeast; that the bridge, if made concrete, could provide ingress or egress to the Fajardo Subdivision
Road.

ISSUE:
Whether or not requisites for the establishment of a legal easement of right of way was satisfied.

HELD:
No, he did not. Article 650 of the Civil Code provides that the easement of right-of-way shall be
established at the point least prejudicial to the servient estate, and, insofar as consistent with this rule, where
the distance from the dominant estate to a public highway may be the shortest. Under this guideline,
whenever there are several tenements surrounding the dominant estate, the right-of-way must be
established on the tenement where the distance to the public road or highway is shortest and where the
least damage would be caused. If these two criteria (shortest distance and least damage) do not concur in
a single tenement, we have held in the past that the least prejudice criterion must prevail over the
shortest distance criterion.

In this case, the establishment of a right-of-way through the petitioners’ lot would cause the destruction of
the wire fence and a house on the petitioners’ property. Although this right-of-way has the shortest distance
to a public road, it is not the least prejudicial considering the destruction pointed out, and that an option to
traverse two vacant lots without causing any damage, albeit longer, is available. We have held that "mere
convenience for the dominant estate is not what is required by law as the basis of setting up a compulsory
easement;" that "a longer way may be adopted to avoid injury to the servient estate, such as when there
are constructions or walls which can be avoided by a round-about way."

196
EASEMENTS
Easements

AN EASEMENT OF RIGHT OF WAY MAY BE DEMANDED BY THE OWNER OF AN IMMOVABLE OR


BY ANY PERSON WHO BY VIRTUE OF A REAL RIGHT MAY CULTIVATE OR USE THE SAME

168. Andres v. Sta. Lucia Realty & Development, Inc.


G.R. No. 201405, August 24, 2015
Del Castillo, J.

FACTS:
Petitioners and Liza filed a Complaint for Easement of Right-of-Way against respondent before the RTC.
They alleged that they are co-owners and possessors for more than 50 years of three parcels of
unregistered agricultural land in Pag-asa, Binangonan, Rizal. A few years back, however, respondent
acquired the lands surrounding the subject property, developed the same into a residential subdivision
known as the Binangonan Metropolis East, and built a concrete perimeter fence around it such that
petitioners and Liza were denied access from subject property to the nearest public road and vice versa.
They thus prayed for a right-of-way within Binangonan Metropolis East in order for them to have access to
Col. Guido Street, a public road.

Respondent denied knowledge of any property adjoining its subdivision owned by petitioners and Liza. It
pointed out that petitioners and Liza failed to sufficiently allege in their complaint the existence of the
requisites for the grant of an easement of right-of-way.

ISSUE:
Whether or not petitioner can demand easement of right of way from respondent.

HELD:
No. Under Article 649 of the Civil Code, an easement of right-of-way may be demanded by the owner of an
immovable or by any person who by virtue of a real right may cultivate or use the same.

"Prescription is one of the modes of acquiring ownership under the Civil Code." However, it was clarified in
the Heirs of Mario Malabanan v. Republic of the Philippines, that only lands of the public domain
subsequently classified or declared as no longer intended for public use or for the development of national
wealth, or removed from the sphere of public dominion and are considered converted into patrimonial lands
or lands of private ownership, may be alienated or disposed through any of the modes of acquiring
ownership under the Civil Code. Otherwise, Article 1113 of the Civil Code, which provides that property of
the State not patrimonial in character shall not be the subject of prescription, applies.

In the absence of such proof of declaration in this case, petitioners' claim of ownership over the subject
property based on prescription necessarily crumbles. Conversely, they cannot demand an easement of
right-of-way from respondent for lack of personality.

197
EASEMENTS
Easements

DRIED UP RIVER IS NOT PART OF LEGAL EASEMENT AND BELONG TO THE STATE

169. Republic v. Santos III


G.R. No. 160453, November 12, 2012
Bersamin, J.

FACTS:
Respondent Arcadio Ivan A. Santos III applied on March 7, 1997 for the registration of Lot 4998-B in the
RTC in Parañaque City alleging continuous and adverse possession of more than ten years. The property
was bounded in the Northeast by Lot 4079 belonging to respondent Arcadio C. Santos, Jr., in the Southeast
by the Paraque River, in the Southwest by an abandoned road, and in the Northwest by Lot 4998-A also
owned by Arcadio Ivan.

On May 21, 1998, Arcadio Ivan amended his application for land registration to include Arcadio, Jr. as his
co-applicant because of the latter’s co-ownership of the property. He alleged that the property had been
formed through accretion and had been in their joint open, notorious, public, continuous and adverse
possession for more than 30 years. The City of Parañaque opposed the application for land registration,
stating among others that the property was within the legal easement of 20 meters from the river bank and
that assuming that the property was not covered by the legal easement, title to the property could not be
registered in favor of the applicants for the reason that the property was an orchard that had dried up and
had not resulted from accretion.

ISSUE:
Whether or not respondent acquire the subject parcel of land through the process of accretion.

HELD:
No. Accretion is the process whereby the soil is deposited along the banks of rivers. The deposit of soil, to
be considered accretion, must be: (a) gradual and imperceptible; (b) made through the effects of the current
of the water; and (c) taking place on land adjacent to the banks of rivers.

The RTC and the CA grossly erred in treating the dried-up river bed as an accretion that became
respondent’s property pursuant to Article 457 of the Civil Code. That land was definitely not an accretion.
The process of drying up of a river to form dry land involved the recession of the water level from the river
banks, and the dried-up land did not equate to accretion, which was the gradual and imperceptible
deposition of soil on the river banks through the effects of the current. In accretion, the water level did not
recede and was more or less maintained. Hence, respondents as the riparian owners had no legal right to
claim ownership of Lot 4998-B.

Considering that the clear and categorical language of Article 457 of the Civil Code has confined the
provision only to accretion, we should apply the provision as its clear and categorical language tells us to.
Axiomatic it is, indeed, that where the language of the law is clear and categorical, there is no room for
interpretation; there is only room for application. The first and fundamental duty of courts is then to apply
the law.

198
EASEMENTS
Easements

A ROAD OF RIGHT OF WAY (RROW) CAN BE CONSIDERED AS A PROPERTY OF PUBLIC


DOMINION, WHICH IS OUTSIDE THE COMMERCE OF MAN

170. Hi-Lon Manufacturing, Inc. v. Commssion on Audit


G.R. No. 210669, August 1, 2017
Peralta, J.

FACTS:
Sometime in 1978, the government, through the then Ministry of Public Works and Highways (now DPWH),
converted to a road right-of-way (RROW) a 29,690 sq. m. portion of the 89,070 sq. m. parcel of land (subject
property) located in Mayapa, Calamba, Laguna, for the Manila South Expressway Extension Project.

Later on, PPIC acquired the subject property. PPIC then mortgaged the subject property with the DBP, a
government financing institution, which later acquired the property in a foreclosure proceeding. On April 16,
1995, TGPI executed a Deed of Absolute Sale in favor of HI-LON over the entire subject property. HI-LON
registered the Deed with the Register of Deeds of Calamba, Laguna. On December 21, 2001, a Deed of
Sale was executed between HI-LON and the Republic of the Philippines, represented by Lope S. Adriano,
URPO-PMO Director, by authority of the DPWH Secretary, covering the subject property converted to
RROW. Based on the auditor's observations, the Director of the HI-LON determined as just compensation
which should have been paid for the conversion of the RROW the amount of P9,937,596.20, representing
the difference between the partial payment of P10,461,338.00 to HI-LON and the amount of P532.741.80.
The COA determined that HI-LON or its predecessor-in-interest TGPI does not own the RROW in question,
as it has been the property of the Republic of the Philippines since its acquisition by the DBP up to the
present, the COA concluded that the proper valuation of the claim for just compensation is irrelevant as HI-
LON is not entitled thereto in the first place.

ISSUE:
Whether or not Hi-Lon is entitled to just compensation for the conversion of the RROW.

HELD:
NO. Concededly, the 29,690 sq. m. portion of the subject property is not just an ordinary asset, but is being
used as a RROW for the Manila South Expressway Extension Project, a road devoted for a public use since
it was taken in 1978. Under the Philippine Highway Act of 1953, "right-of-way" is defined as the land secured
and reserved to the public for highway purposes, whereas "highway" includes rights-of-way, bridges, ferries,
drainage structures, signs, guard rails, and protective structures in connection with highways. Article 420 of
the New Civil Code considers as property of public dominion those intended for public use, such as roads,
canals, torrents, ports and bridges constructed by the state, banks, shores, roadsteads, and others of similar
character.

Being of similar character as roads for public use, a road right-of-way (RROW) can be considered as a
property of public dominion, which is outside the commerce of man, and cannot be leased, donated, sold,
or be the object of a contract, except insofar as they may be the object of repairs or improvements and other
incidental matters. However, this RROW must be differentiated from the concept of easement of right of
way under Article 649 of the same Code, which merely gives the holder of the easement an incorporeal
interest on the property but grants no title thereto, inasmuch as the owner of the servient estate retains
ownership of the portion on which the easement is established, and may use the same in such a manner
as not to affect the exercise of the easement.

199
EASEMENTS
Easements

IT IS WELL SETTLED THAT THE CRITERION OF LEAST PREJUDICE TO THE SERVIENT ESTATE
MUST PREVAIL OVER THE CRITERION OF SHORTEST DISTANCE ALTHOUGH THIS IS A MATTER
OF JUDICIAL APPRECIATION

171. Spouses Williams v. Zerda


G.R. No. 207146, March 15, 2017
Mendoza, J.

FACTS:
Zerda filed a complaint against Spouses Williams for easement of right of way. Immediately behind Zerda’s
land (the dominant estate) was a swampy mangrove area owned by the Republic of the Philippines. On
both sides were lands registered under the name of Woodridge Properties, Inc. in the name of one Luis
Dilag. In front of Zerda’s property was the land owned by petitioners-Spouses Williams, where the national
highway ran along. Zerda alleged that his lot was without adequate outlet to a public highway except by
passing through Williams’ property. However, Spouses Williams countered that the complaint should be
dismissed. They claimed that they were in negotiation with Agripina Sierra the former owner of the dominant
estate, for its sale to them but the sale did not materialize due to the intervention of Zerda. Further, they
contended that they have introduced visible improvements in their property.

ISSUE:
Whether or not Zerda is entitled to an easement of right of way.

HELD:
Yes. The conferment of the legal easement of right of way is governed by Articles 649 and 650 of the Civil
Code. All the requisites of an easement of right of way are present in this case.

First, the dominant estate is surrounded by other immovables and has no adequate outlet to a public
highway (Art. 649, par. 1). There is no dispute that Zerda’s property was surrounded by other immovables
owned by different individuals, including Spouses Williams.

Second, there is payment of proper indemnity (Art. 649, par. 1). Zerda stated in his letter to the Williams
that he was willing to pay a reasonable value or to swap a portion of his property in exchange of providing
him with a right of way.

Third, the isolation is not due to the acts of the proprietor of the dominant estate (Art. 649, last par.). The
isolation of the dominant estate was not due to the respondent Zerda’s own acts. The property he purchased
was already surrounded by other immovables leaving him no adequate ingress or egress to a public
highway.

Fourth, the right of way claimed is at the point least prejudicial to the servient estate; and insofar as
consistent with this rule, where the distance from the dominant estate to a public highway may be the
shortest (Art. 650). The Court finds that the right of way sought by the respondent is at the point least
prejudicial to the servient estate and it is the shortest distance to the national highway as evidenced by the
Sketch Plan showing that the requested right of way was alongside the perimeter of Spouses Williams'
property.

200
NUISANCE
Nuisance

A PROPERTY OWNER WHO MERELY RECREATES NATURE SUCH AS AN ARTIFICIAL POOL OF


WATER WITHOUT ADDING ADDITIONAL DANGERS CANNOT BE HELD LIABLE FOR CREATING AN
ATTRACTIVE NUISANCE

172. Hidalgo Enterprises Inc. v. Balandan


G.R. No. L-3422, June 13, 1952
Bengzon, J.

FACTS:
This is an appeal of the decision of the Court of Appeals requiring Hidalgo Enterprises, Inc. to pay Guillermo
Balandan and his wife, damages in the sum of P2,000 for the death of their son Mario.

Hidalgo Enterprises, Inc. owned an ice-plant factory in Laguna. Two tanks full of water, nine feet deep, were
installed in its premises, for cooling purposes of its engine. The tanks were not provided with any kind of
fence or top covers. The edges of the tanks were a foot high from the surface of the ground.

Anyone could easily enter the said factory with its wide gate entrance, which was continually open. There
was no guard assigned on the gate. At about noon of April 16, 1948, Guillermo’s son, Mario Balandan
(“Mario”), a boy barely 8 years old, while playing with and accompanied by other boys his age, entered the
factory premises through the gate to take a bath in one of said tanks. While bathing, Mario sank to the
bottom of the tank and drowned.

The lower courts damages to the Balandan spouses in the sum of P2,000 for the death of their son on the
theory that the tank was an attractive nuisance and for petitioner’s negligence in not adopting the necessary
precautions to avoid accidents to persons entering its premises

ISSUE:
Is the water tank considered an attractive nuisance?

HELD:
No. The doctrine of attractive nuisance states that “One who maintains on his premises dangerous
instrumentalities or appliances of a character likely to attract children in play, and who fails to exercise
ordinary care to prevent children from playing therewith or resorting thereto, is liable to a child of tender
years who is injured thereby, even if the child is technically a trespasser in the premises.

American Jurisprudence shows us that the attractive nuisance doctrine generally is not applicable to bodies
of water, artificial as well as natural, in the absence of some unusual condition or artificial feature other than
the mere water and its location. Nature has created streams, lakes and pools which attract children. Lurking
in their waters is always the danger of drowning. Against this danger, children are early instructed so that
they are sufficiently presumed to know the danger; and if the owner of private property creates an artificial
pool on his own property, merely duplicating the work of nature without adding any new danger, he cannot
be held liable for having created an "attractive nuisance." Thus, the petitioner cannot be held liable for
Mario’s death.

201
NUISANCE
Nuisance

UNLESS A NUISANCE IS A NUISANCE PER SE, IT MAY NOT BE SUMMARILY ABATED; AN OWNER
WHO MAKES IMPROVEMENTS ON HIS OWN PROPERTY WHICH ENDANGERS NEIGHBORS IS
RESPONSIBLE FOR MAKING THE NECESSARY CONSTRUCTIONS TO NULLIFY SAID DANGER

173. Rana v. Wong


G.R. Nos. 192861-62, June 30, 2014
Perlas-Bernabe, J.

FACTS:
This is an appeal of the decision of the CA which affirmed the RTC which, among others, declared the
improvements made by petitioners as nuisances that could be summarily abated.

Teresita Lee Wong (“Wong”) and the Spouses Shirley and Ruben Ang Ong (“Sps. Ong”) are co-owners pro-
indiviso of a residential land situated in Peace Valley Subdivision, Lahug, Cebu City (“Wong-Ong property”)
adjacent to an existing road. On the opposite side of this road and across the Wong-Ong property, is the lot
of the Spouses Wilson and Rosario Uy (“Sps. Uy”) and the Spouses Reynaldo and Linda Rana (“Sps.
Rana”). The said lots (“Uy property” & “Rana property”) follow a rolling terrain with the Rana property
standing about two (2) meters higher than and overlooking the Uy property. The Wong-Ong property, on
the other hand, is at the same level with the subject road.

Sometime in 1997, the Sps. Rana elevated and cemented a portion of the road that runs between the Rana
and Wong-Ong properties in order to level the said portion with their gate in order to ease the ingress-egress
of their vehicle (“elevation work”). This affected the “usual and normal manner” of ingress-egress of the
vehicles to the Wong-Ong property, effectively denying them their enjoyment of their right of way. The Sps.
Rana also backfilled a portion of the perimeter fence separating the Rana and Uy properties without erecting
a retaining wall (“backfilling”). This created a risk of weakening the perimeter fence of the Sps. Uy due to
the seeping of rain water from the Rana property. Thus, Wong, the Sps. Ong, and the Sps. Uy
(“respondents-complainants”) filed a Complaint for Abatement of Nuisance with Damages against the Sps.
Rana before the RTC.

While the case was pending, the respondents-complainants moved for the summary abatement of the
elevation work which the RTC granted “in order to develop the area and make use of the right of way which
is located between the [Wong-Ong and Rana properties].” Despite the limited tenor of the Order, the
respondents-complainants proceeded to demolish the elevation work in a manner which hampered the Sps.
Rana’s ingress and egress to their own residence and resulted in trapping the vehicle of the Sps. Rana
inside their garage.

ISSUES:
1. May the elevation work be summarily abated as nuisance per se?
2. What is the liability of the Sps. Rana with respect to their act of backfilling?

HELD:
1. No, the elevation cannot be summarily abated as it is not a nuisance per se.

Jurisprudence classifies nuisances in relation to their legal susceptibility to summary abatement. In this
regard, a nuisance may either be: (a) a nuisance per se or (b) a nuisance per accidens. There is a nuisance
per se when the nuisance affects the immediate safety of persons and property. Thus, it may be summarily
abated under the undefined law of necessity. The existence of a nuisance per accidens, on the other hand,
depends upon certain conditions and circumstances. Because its existence is question of fact, a nuisance
per accidens cannot be abated without due hearing.

With respect to the elevation work, the Court finds that the same is not a nuisance per se. By its nature, it
is not injurious to the health or comfort of the community. It was built primarily to facilitate the ingress and
egress of Sps. Rana from their house which was admittedly located on a higher elevation than the subject
road and the adjoining Uy and Wong-Ong properties. Since the subject portion is not a nuisance per se, it
cannot be summarily abated. Its summary demolition was thus not unwarranted.

202
2. The Sps. Rana must erect a retaining wall to support their backfilling in order to avoid endangering the
occupants of the Uy property.

Records show that the perimeter fence of the Uy property was not designed to act as a retaining wall but
merely to withhold windload and its own load. Both the RTC and the CA found the subject backfilling to have
added pressure on the fence, consequently endangering the safety of the occupants of the Uy property,
especially considering the higher elevation of the Rana property. With these findings, the Court thus agrees
with the courts a quo that there is a need for Linda Rana to construct a retaining wall which would bear the
weight and pressure of the filling materials introduced on their property.

203
NUISANCE
Nuisance

PREVAILING JURISPRUDENCE HOLDS THAT UNLESS A NUISANCE IS A NUISANCE PER SE, IT


MAY NOT BE SUMMARILY ABATED

174. Cruz v. Pandacan Hiker’s Club


G.R. No. 1888213, January 11, 2016
Peralta, J.

FACTS:
This is a petition for review on certiorari under Rule 45 of the Rules of Court seeking to annul and set aside
the Decision of the Court of Appeals which reversed and set aside an earlier decision of the Office of the
Ombudsman dismissing the complaint filed against petitioners.

Petitioner Natividad C. Cruz was the Punong Barangay Barangay 848, Zone 92, City of Manila. On
November 10, 2006, around five o'clock in the afternoon, and along Central Street, Pandacan, Manila, within
the vicinity of her barangay, she allegedly confronted persons playing basketball with the following
statements: “Bakit nakabukas ang (basketball) court? Wala kayong karapatang maglaro sa court na 'to,
barangay namin ito! xxx xxx xxx Wala kayong magagawa. Ako ang chairman dito. Mga walanghiya kayo,
patay gutom! Hindi ako natatakot! Kaya kong panagutan lahat!” She then allegedly gave an order to
Barangay Tanod Benjamin dela Cruz to destroy the basketball ring by cutting it up with a hacksaw which
Dela Cruz promptly complied with. The acts of Cruz and Dela Cruz prompted the filing of a Complaint.

Petitioners maintain that they acted merely with the intention to regain free passage of people and vehicles
over the street and restore the peace, health, and sanitation of those affected by the basketball court, thus
citing the General Welfare clause under the Local Government Code. Cruz, in particular, asserts that she
merely abated a public nuisance which she claimed was within her power as barangay chief executive to
perform and was part of her duty to maintain peace and order.

ISSUE:
May a barangay official order the summary abatement of a basketball ring?

HELD:
No. The petitioners’ actions, though well-intentioned, were improper and done in excess of their authority.

The destructive acts of petitioners find no legal sanction. Although petitioners claim to have merely
performed an abatement of a public nuisance, the same was done summarily while failing to follow the
proper procedure therefor and for which, petitioners must be held administratively liable.

The basketball ring is not a nuisance per se which is susceptible of summary abatement because it does
not pose an immediate effect upon the safety of persons and property. A basketball ring, by itself, poses no
immediate harm or danger to anyone but is merely an object of recreation. Neither is it, by its nature,
injurious to rights of property, of health or of comfort of the community and, thus, it may not be abated as a
nuisance without the benefit of a judicial hearing.

In any case, although it has been held that a nuisance may be abated via an ordinance, petitioners could
cite no barangay nor city ordinance that would have justified their summary abatement through the exercise
of police powers found in the general welfare clause of the Local Government Code. No barangay nor city
ordinance was violated; neither was there one which specifically declared the said basketball ring as a
nuisance per se that may be summarily abated. Clearly, the complete destruction of the basketball ring by
the petitioners is justified neither by law or ordinance nor even by equity or necessity, which makes the act
illegal and petitioners liable.

204
NUISANCE
Nuisance

FOR AN OBJECT TO BE LEGALLY CONSIDERED AS A NUISANCE PER ACCIDENS, THE PARTY


SEEKING ITS ABATEMENT MUST PROVE THAT IT IS A NUISANCE BY EVIDENCE

175. North Greenhills Assocation, Inc. v. Morales


G.R. No. 222821, August 9, 2017
Mendoza, J.

FACTS:
This is an appeal with application for provisional remedies where petitioner North Greenhills Association,
Inc. (NGA) seeks the review of Decision and Resolution of the Court of Appeals which affirmed the Decision
and Resolution of the Office of the President.

Morales is a resident of North Greenhills Subdivision in San Juan City. His house is located alongside Club
Filipino Avenue and adjacent to McKinley Park, an open space/playground area owned and operated by
NGA. He also has a personal access door, which he built through a wall separating his house from the park.
This access door, when unlocked, opens directly into the park. NGA constructed a pavilion or kiosk
occupying the side of the park adjacent to the residence of Morales. Part of the design was a public restroom
which was constructed alongside the concrete wall separating the house of Atty. Morales from the park.
Thus, Morales filed a complaint before the HLURB seeking the demolition of the pavilion and the restroom

The HLURB Arbiter rendered a Decision ordering the removal of the pavilion and the relocation of the
common toilet in a place where it will not be a nuisance to any resident. The Office of the President and the
CA affirmed the HLURB. The CA for its part, characterized the restrooms as nuisance per accidens
considering that the structure posed sanitary issues which could adversely affect not only Morales, but also
his entire household. NGA however questions the finding as Morales did not submit any evidence at all that
restroom has caused physical annoyance or discomfort to him.

ISSUE:
May an object be declared as nuisance per accidens without evidence?

HELD:
No. A nuisance per accidens is one which depends upon certain conditions and circumstances, and its
existence being a question of fact, it cannot be abated without due hearing thereon in a tribunal authorized
to decide whether such a thing does in law constitute a nuisance. As such, a finding that something is a
nuisance per accidens requires a proper appreciation of evidence.

The CA was not even sure that the restroom has caused such annoyance to Morales or his family. Its
declaration that the restroom is a nuisance per accidens had no basis in evidence. There is nothing in the
records which discloses that Morales had introduced any evidence, testimonial or documentary, to prove
that the restroom annoyed his senses, that a foul odor emanated from it, or that it posed sanitary issues
detrimental to his family's health. No certification by the City Health Officer was even submitted to the
HLURB to attest on such matters.

It was improper on the part of the CA to assume those negative effects because modern day restrooms,
even those for the use of the public, are clean, safe and emitting no odor as these are regularly maintained.
For said reason, it was an error on the part of the CA to rule that the restroom was a nuisance per accidens
and to sustain the order that it should be relocated.

205
NUISANCE
Nuisance

THE ATTRACTIVE NUISANCE DOCTRINE WILL NOT APPLY WHEN THE CHILD INVOLVED IS OF
SUFFICIENT MATURITY OR CAPACITY, DETERMINED ON A CASE TO CASE BASIS, SUCH THAT
THE CHILD CAN BE DEEMED TO BE THE PROXIMATE CAUSE OF HIS INJURY

176. Taylor v. Manila Electric Railroad and Light Co.


G.R. No. L-4977, March 22, 1910
Carson, J.

FACTS:
This is an action to recover damages for the loss of an eye and other injuries, instituted by David Taylor, a
minor, with assistance from his father.

The defendant is a foreign corporation engaged in the operation of a street railway and an electric light
system in the city of Manila. The plaintiff was 15 years old at the time he was injured. Plaintiff along with a
friend, went to the premises of the defendant company to visit one of its employees named Murphy. The
boys, impelled apparently by youthful curiosity and unusual interest in machinery, spent some time in
wandering about the company's premises. At some point, they found some twenty or thirty brass fulminating
caps scattered on the ground. The boys took these caps home and experimented with the caps. They pried
it open with a knife and finding a yellow substance within, brought a match near the substance which
resulted to an explosion. Plaintiff testified that no measures seems to have been adopted by the defendant
company to prohibit or prevent visitors from entering and walking about its premises unattended, when they
felt disposed so to do.

ISSUE:
Is the defendant company liable for the injury sustained by the plaintiff?

HELD:
No. The attractive nuisance doctrine admits exceptions when the child involved is of sufficient age and
maturity such that fault may be attributed solely to his own negligence and thus be deemed to have been
the proximate cause of his own injury.

In case of adults, the rule is that to entitle him to recover damages for an injury resulting from the fault or
negligence of another he must himself have been free from fault. As for children, the general rule is that this
does not apply because children are presumed to not know any better. Unlike adults, the care and caution
required of a child is according to his maturity and capacity, and this is to be determined in each case by
the circumstances of the case.

In this case, the plaintiff at the time of the accident was a well-grown youth of 15, more mature both mentally
and physically than the average boy of his age; he had been to sea as a cabin boy; was able to earn P2.50
a day as a mechanical draftsman thirty days after the injury was incurred; and the record discloses
throughout that he was exceptionally well qualified to take care of himself. The evidence of record leaves
no room for doubt that, despite his denials on the witness stand, he well knew the explosive character of
the fulminating cap. He cannot be said to have been free from fault therefore, when he willfully and
deliberately cut open the detonating cap, and placed a match to the contents, knowing that his action would
result in an explosion. It may be that he may not have known the precise nature of the explosion which he
might expect from igniting the contents of the cap, but he nevertheless willfully, recklessly, and knowingly
produced the explosion.

The law fixes no arbitrary age at which a minor can be said to have the necessary capacity to understand
and appreciate the nature and consequences of his own acts, so as to make it negligence on his part to fail
to exercise due care and precaution in the commission of such acts. It must thus be determined on a case
to case basis. The plaintiff in this case though is proven to have had sufficient capacity and understanding
to be sensible of the danger to which he exposed himself when he put the match to the contents of the cap.
Thus, while it may be true that the injuries would not have been incurred but for the negligence act of the
defendant in leaving the caps exposed on its premises, nevertheless plaintiff's own act was the proximate
and principal cause of the accident which inflicted the injury. Volenti non fit injuria.

206
DIFFERENT MODES OF ACQUIRING OWNERSHIP
Theory of Mode and Title

OWNERSHIP MUST BE ESTABLISHED TO HAVE BEEN ACQUIRED THROUGH ONE OF THE


RECOGNIZED MODES OF ACQUIRING OWNERSHIP

177. Acap v. Court of Appeals


G.R. No. 118114, December 7, 1995
Padilla, J.

FACTS:
This is a petition for review on certiorari of the decision of the CA, holding that private respondent Edy de
los Reyes had acquired ownership of the land based on a document entitled "Declaration of Heirship and
Waiver of Rights", and ordering the dispossession of petitioner as leasehold tenant of the land for failure to
pay rentals.

After both spouses Santiago and Lorenza Vasquez died, their only son Felixberto inherited the lot. Felixberto
executed a duly notarized document entitled "Declaration of Heirship and Deed of Absolute Sale" in favor
of Cosme Pido. When Pido died intestate, his surviving heirs executed a notarized document denominated
as "Declaration of Heirship and Waiver of Rights of Lot No. 1130 Hinigaran Cadastre". On the basis of this
Declaration of Heirship and Waiver of Rights, allegedly made in favor or private respondent De Los Reyes,
he filed a complaint for recovery of possession and damages against petitioner Eodoro Acap, alleging in
the main that as his leasehold tenant, petitioner Acap refused and failed to pay the agreed annual rental.
Acap however pointed out that the Declaration of Heirship and Waiver of Rights is not one of the recognized
modes of acquiring ownership under the Civil Code.

ISSUE:
Is the Declaration of Heirship and Waiver of Rights in this case a mode of acquiring ownership?

HELD:
No, the Declaration of Heirship and Waiver of Rights relied upon by respondent was not mode of acquiring
ownership.

An asserted right or claim to ownership over a thing arising from a juridical act, however justified, is not per
se sufficient to give rise to ownership over the res. That right or title must be completed by fulfilling certain
conditions imposed by law. Ownership and real rights are acquired only pursuant to a legal mode or process.
In other words, possession of title is not sufficient to claim ownership. The thing must have been acquired
by some mode.

Hence, while title is the juridical justification of ownership, mode is the actual process of acquisition or
transfer of ownership over a thing in question. Under Article 712 of the Civil Code, the modes of acquiring
ownership are generally classified into two (2) classes, namely, the original mode (i.e., through occupation,
acquisitive prescription, law, or intellectual creation) and the derivative mode (i.e., through succession mortis
causa or tradition as a result of certain contracts, such as sale, barter, donation, assignment or mutuum).
The declaration of heirship and waiver of rights in this case was not a contract. The declaration only makes
known that successional rights had been waived in favor of co-heirs.

De Los Reyes was not a co-heir and thus, being a stranger to the succession of Cosme Pido, he cannot
conclusively claim ownership over the subject lot on the sole basis of the waiver document which neither
recites the elements of either a sale, or a donation, or any other derivative mode of acquiring ownership.

207
DIFFERENT MODES OF ACQUIRING OWNERSHIP
Donation

A DONATION MADE TO A CORPORATION BY ESTOPPEL IS VALID AND CURED BY ITS


SUBSEQUENT INCORPORATION

178. The Missionary Sisters of Our Lady of Fatima (Peach Sisters of Laguna) v. Alzona
G.R. No. 224307, August 6, 2018
Reyes, Jr., J.

FACTS:
This is a petition for review on certiorari under Rule 45 of the Rules of Court seeking to annul and set aside
the Decision of the Court of Appeals ruling that petitioner, being an unregistered corporation, cannot
exercise the powers, rights, and privileges expressly granted by the Corporation Code, including the right
to enter into a contract of Donation.

The late Purificacion Y. Alzona, impelled by her unmaterialized desire to be nun, decided to devote the rest
of her life in helping others. Purificacion called Mother Concepcion, petitioner Missionary Sisters of Our
Lady of Fatima Superior General and handed her a handwritten letter stating that she is donating her house
and lot to petitioner through Mother Concepcion. Then, Purificacion executed a Deed of Donation Inter Vivos
(Deed) in favor of the petitioner, treating it as a corporation.

Amando Alzona, the donor’s brother, sought to annul the Deed executed between Purificacion and the
petitioner, on the ground that the petitioner’s certificate of registration as a corporation sole was only issued
two days after the donation had been made, which fact is admitted.

ISSUE:
Was the donation to a non-existent corporation validly made?

HELD:
Yes, this is because the donation was made to a corporation by estoppel. According to Section 21 of the
Corporation Code, one who assumes an obligation to an ostensible corporation as such cannot resist
performance thereof on the ground that there was in fact no corporation.

First, Purificacion dealt with the petitioner as if it were a corporation. This is evident from the fact that
Purificacion executed two (2) documents - handwritten letter and Deed conveying her properties in favor of
the petitioner.

Second, Purificacion acquired benefits from the act even if it was a deonation. To recall, the subject
properties were given by Purificacion, as a token of appreciation for the services rendered to her during her
illness. The subject deed partakes of the nature of a remuneratory or compensatory donation, having been
made for the purpose of rewarding the donee for past services, which services do not amount to a
demandable debt. In donations made to a person for services rendered to the donor, the donor's will is
moved by acts which directly benefit him. The motivating cause is gratitude, acknowledgment of a favor, a
desire to compensate.

We must also take into consideration that Purificacion performed the donation twice, once defectively by a
private document and then by public instrument. With the execution of two (2) documents of conveyance in
favor of the petitioner, it is clear that what Purificacion intended was for the sisters comprising the petitioner
to have ownership of her properties to aid them in the pursuit of their charitable activities, as a token of
appreciation for the services they rendered to her during her illness.

The subsequent incorporation of the petitioner corporation cured whatever defect existed at the time of the
acceptance of the donation on its behalf by Mother Concepcion.

208
DIFFERENT MODES OF ACQUIRING OWNERSHIP
Donation

ACTIONS FOR REVERSION OF PROPERTY DONATED BY ONEROUS DONATION WHICH PROVIDE


FOR AN AUTOMATIC REVERSION CLAUSE ARE NOT GOVERNED ARTICLE 764 AND THUS DOES
NOT PRESCRIBE IN 4 YEARS BUT IN 10 YEARS

179. De Luna v. Abrigo


G.R. No. L-57455, January 18, 1990
Medialdea, J.

FACTS:
This is a petition for review on certiorari under Rule 45 of the Order of respondent Judge Sofronio F. Abrigo,
dismissing the complaint for the cancellation of the donation alleging that the terms and conditions of the
donation were not complied with by the foundation.

Prudencio de Luna donated a portion of his lot to the Luzonian University Foundation, Inc. (“Foundation”).
According to its Deed, the donation was subject to the condition that the donee Foundation shall construct
at its own expense a chapel, a nursery and kindergarten school, to be named after St. Veronica, and
provided for the automatic reversion to the donor of the donated property in case of violation or non-
compliance.

Petitioners, who claim to be the children and only heirs of the late Prudencio de Luna filed a complaint
alleging that the terms and conditions of the donation had not been complied with by the foundation. Among
others, it prayed for the cancellation of the donation and the reversion of the donated land to the heirs. In
its answer, the Foundation invoked the affirmative defense of prescription of action and prayed for the
dismissal of the complaint. The RTC ruled in favor of the Foundation holding that the action prescribed in 4
years.

Petitioners come before the Court arguing that the lower court erred in treating the complaint as one for
judicial decree of revocation of the which admittedly prescribes in four (4) years. This is because the
complaint must be deemed to be an action to enforce a written contract which prescribes in ten (10) years
as provided in Article 1144.

ISSUE:
Is an onerous donation governed by the rules on prescription pertaining to contracts?

HELD:
Yes, Article 733 of the Civil Code expressly provides that onerous donations are governed by the rules on
contracts.

While Article 764 provides that actions for the revocation of a donation must be brought within four (4) years
from the non-compliance of the conditions of the donation, the provision does not apply to onerous contracts
that provide for an automatic reversion clause. In this case, the Deed provides for the automatic reversion
of the donated area in case of non-compliance with its conditions. This stipulation is a valid as it is not
contrary to law, moral, public order or policy (CIVIL CODE, Art. 1306). The stipulation is in the character of
a resolutory condition which cancels and rescinds the contract without need for judicial action. Where one
of the parties contests or denies the rescission, judicial action is not for the purposes of rescinding the
contract but for the purposes of determining whether the extrajudicial rescission had been proper. Thus,
there is no need for judicial action for revocation under Article 764 in order to revoke the donation subject
to this case.

Being a written contract, the action to enforce its terms prescribes in 10 years as provided under Article
1144[1].

209
DIFFERENT MODES OF ACQUIRING OWNERSHIP
Donation

THE NATURE OF A DONATION IS DETERMINED BY THE TERMS OF THE DONATION AND NOT BY
ITS DENOMINATION OR CAPTION

180. Reyes v. Mosqueda


G.R. No. L-45262, July 23, 1990
Gutierrez, Jr., J.

FACTS:
The heirs of Dr. Emilio Pascual, who died intestate, commenced special proceedings in the then Court of
First Instance of Pampanga for the administration of his estate. Ursula Pascual, his sister, filed a motion to
exclude some properties from the inventory of Pascual's estate on the basis of a document titled as
"Donation Mortis Causa" made in her favor in 1966.

The petitioners, however, claim that the donation mortis causa, being of later effect, must yield to a deed of
donation of real property inter vivos over the same subject properties made in favor of Ofelia D. Parungao,
a minor with her mother, Rosario Duncil, accepting the gift and donation for and in her behalf in 1969. The
petitioners argue that the “Donation Mortis Causa” was void as it did not comply with the formalities of wills.
The trial court however ruled that the donation was valid as it was in the nature of a donation inter vivos.
Thus, the later donation made to Parungao was invalid.

ISSUE:
Did the trial court err in holding that the deed entitled “Donation Mortis Causa” is in fact a donation inter
vivos?

HELD:
No. The trial court did not err in holding that the deed entitled “Donation Mortis Causa” is in fact a donation
inter vivos

It has been held that dispositions in a deed of donation do not depend on the title or term used in the deed
of donation. It is the body of the document which should be considered in ascertaining the intention of the
donor. For a donation to be a donation mortis causa, the following characteristics should be present: (1) It
conveys no title before the death of the transferor or the transferor retains ownership over the property; (2)
Before his death, the transfer should be revocable by the transferor at will; (3)The transfer is void should
the transferor survive the transferee.

These requisites are not present with respect to Ursula’s deed. The transfer of ownership was immediate
and independent of the death of the donor. The provision stating that the donor has reserved sufficient
properties for himself to maintain him for life confirms the intention of the donor to give naked ownership
immediately after execution of the deed of donation. Thus, the donation is in fact in the nature of a donation
inter vivos.

It follows that, at the time of donation to Parungao, the donor was no longer the owner of the subject
properties and could not have donated the same. In any event, the trial court also expressed serious doubts
as to the authenticity of the second deed.

210
DIFFERENT MODES OF ACQUIRING OWNERSHIP
Donation

ONLY THE PARTY WITH KNOWLEDGE OF THE ILLEGALITY OF THE CAUSE OF A DONATION IS
BARRED FROM RAISING THE ILLEGALITY AS A CAUSE OR DEFENSE

181. Liguez v. Court of Appeals


G.R. No. L-11240, December 18, 1957
Reyes, J.B.L., J.

FACTS:
This is an Appeal under Rule 45 from a decision of the Court of Appeals, dismissing petitioner Conchita
Liguez’s complaint for recovery of land against the widow and heirs of the late Salvador P. Lopez on the
basis of a deed of donation made in her favor.

The defense interposed that the donation was null and void for having an illicit cause or consideration. It is
alleged that the donation was made in consideration of petitioner entering into marital relations with Salvador
who at the time was a married man. It was proven by evidence that Salvador had confessed to his love for
Conchita, who was 16 years old at the time, to the instrumental witnesses, with the remark that her parents
would not allow Salvador to live with her unless he first donated the land in question.

Conchita however argues that the in pari delicto applies such that the courts should just leave the parties
as they are. The trial court and CA both dismissed the complaint on the ground of illegality of cause and
because the property donated was conjugal.

ISSUE:
May a donation based on illegal cause be given effect?

HELD:
Yes, a donation with illegal cause may be given legal effect.

Without question, the donation was proven and established to have been based on an illegal cause.
However, the lower courts erred in holding that the in pari delicto rule applies as the rule presupposes equal
guilt. The facts however are more suggestive of seduction than of immoral bargaining. First, Salvador was
a man advanced in years and mature experience, while Conchita was only 16 when the donation was made.
Second, the CA did not find that she was fully aware of the terms of the bargain entered into by her parents.
Third, her acceptance of the deed does not imply knowledge of conditions and terms not set forth therein.
Lastly, witnesses testified that it was Conchita’s parents who insisted on the donation.

That being said, only Salvador was barred from raising illegality as a cause for revoking the donation. Since
he is barred from raising said cause of action, so are his heirs and assigns barred from raising illegality as
basis to revoke the donation made in favor of Conchita. Thus, the donation must be upheld as valid and
effectual as the deed itself is valid on its face.

However, since the property is conjugal, only that half interest on the land which pertained to Salvador was
donated to Conchita.

211
DIFFERENT MODES OF ACQUIRING OWNERSHIP
Donation

DONATIONS OF A PARCEL OF LAND WITHOUT COMPLYING WITH THE FORMALITIES REQUIRED


BY LAW ARE INVALID

182. Tan Queto v. Court of Appeals


G.R. No. L-35638, February 27, 1987
Paras, J.

FACTS:
In a Motion for Reconsideration, petitioner Tan Queto asks for the reversal of the decision of the Court with
respect to finding that the disputed lot is paraphernal and that petitioner Tan Queto is a builder in bad faith.

Resituta Tacalinar Guanaco de Pombuena (Restituta) obtained the disputed lot either as a donation or by
way of purchase, with P50.00 as consideration therefore, from her mother. This sale or donation was
consummated while Restituta was already married to her husband Juan Pombuena (Juan). Juan filed for
himself and his supposed co-owner Restituta an application for a Torrens Title over the land, and both were
eventually pronounced owners of the said lot.

Pershing Tan Queto (Tan Queto) entered into a contract of lease over the lot with Restituta, who obtained
her husband’s consent. The lease contract expired after 10 years, but Tan Queto refused to vacate.
Restituta thus filed an unlawful detainer case against Tan Queto.

The spouses won in the unlawful detainer case before the MTC, but on appeal to the CFI, the entire case
was dismissed due to a barter entered into by Tan Queto and Juan. In the said barter, Tan Queto became
the owner of the disputed lot, while the spouses became owners of the parcel of land previously owned by
Tan Queto.

Restituta thereafter filed a petition for reconveyance of title over the disputed lot, for annulment of the barter,
and recovery of the land with damages on the ground that the land was part of her paraphernal properties.
Thus, Juan had no right to barter it. Both the CFI and CA agreed.

ISSUE:
Was the disputed lot part of the paraphernal property of Restituta?

HELD:
No, the lot is deemed to be part of the conjugal property of Resituta and Juan.

It was not proven that Restituta acquired the property by donation. An oral donation inter vivos of land is
void and without effect. Nor was there a donation mortis causa as the formalities therefor were not complied
with. Nor can it be sustained that Restituta’s mother had contractually conveyed the same to her as part of
Restituta’s hereditary share of the estate because contractual transmission of future inheritance is
forbidden.

What was proven was that the land was acquired by the spouses by tradition as a consequence of a contract
of sale with P50.00 as the price. The lot is therefore conjugal, having been acquired by the spouses through
onerous title during the coverture of the marriage.

212
DIFFERENT MODES OF ACQUIRING OWNERSHIP
Donation

THE PURPOSE OF THE FORMAL REQUIREMENT IS TO INSURE THAT THE ACCEPTANCE OF THE
DONATION IS DULY COMMUNICATED TO THE DONOR AND IT MAY BE DISPENSED BY OTHER
FORMS OF COMMUNICATION TO THE DONOR

183. Pajarillo v. Intermediate Appellate Court


G.R. No. 72908, August 11, 1989
Cruz, J.

FACTS:
The mother was Juana Balane de Suterio, who had a brother named Felipe Balane and a sister named
Perfecta Balane de Cordero. Perfecta died in 1945 leaving inter alia a tract of land consisting of about 28
hectares and covered by TCT No. 4671 in the Registry of Deeds of Quezon Province. On May 20, 1946,
Juana and Felipe executed a public instrument entitled "Extra-judicial Settlement of the Estate of the
Deceased Perfecta Balane de Cordero." In this instrument, they donated the said parcel of land to Salud,
daughter of Juana. These instruments were never registered nor was title transferred in Salud's name
although she says she immediately took possession of the land.

Sometime in 1951, she transferred the possession of the land to her mother. During the period they were
occupying the land, Claudio, her brother, paid the realty taxes thereon. Juana purportedly executed a deed
of absolute sale conveying the land to Claudio for the declared consideration of P12,000.00. Two years
later, Claudio had the land registered in as name and was issued TCT No. 32050 in the land records of
Quezon Province. Claudio died in 1961 and his mother in 1963. On June 30, 1965, the Salud and her
husband filed a complaint for the reconveyance of the property on the ground that the deed of sale in favor
of Claudio was fictitious and its registration in his name was null and void. Claudio’s heirs on the other hand
argue that the donation to Salud was legally inefficacious and defective as the acceptance made in a
separate instrument was not noted on the instrument of donation as required by law. The RTC ruled in favor
of Salud, hence this appeal.

ISSUE:
Does non-compliance with the formality required by law render a donation invalid?

HELD:
No. Strict compliance with the formalities required by law should not be enjoined by the Court if it would
tend to work injustice.

It is true that the third paragraph of Art. 633 (now Article 749) provides that “If the acceptance is made, by
separate public instrument, authentic notice thereof shall be given the donor, and this proceeding shall be
noted in both instruments.” A strict interpretation of Article 633 (now Article 749) can lead to no other
conclusion than the annulment of the donation for being defective in form as urged by the petitioners. This
would be in keeping with the unmistakable language of the above-quoted provision. However, we find that
under the circumstances of the present case, a literal adherence to the requirement of the law might result
not in justice to the parties but conversely a distortion of their intentions. It is also a policy of the Court to
avoid such an interpretation.

The purpose of the formal requirement is to insure that the acceptance of the donation is duly communicated
to the donor. In the case at bar, it is not even suggested that Juana was unaware of the acceptance for she
in fact confirmed it later and requested that the donated land be not registered during her lifetime by
Salud. Given this significant evidence, the Court cannot in conscience declare the donation ineffective solely
because there is no notation in the extrajudicial settlement of the donee's acceptance. That would be placing
too much stress on mere form over substance. It would also disregard the clear reality of the acceptance of
the donation as manifested in the separate instrument dated June 20,1946, and as later acknowledged by
Juana.

213
DIFFERENT MODES OF ACQUIRING OWNERSHIP
Donation

THE PERSON SEEKING THE REVOCATION HAS THE BURDEN OF PROOF THAT THE LEGITIME OF
A CHILD ADOPTED SUBSEQUENT TO A DONATION HAS BEEN IMPAIRED BY SUCH DONATION IN
ORDER FOR IT TO BE REVOKED

184. Cruz v. Court of Appeals


G.R. No. L-58671, November 22, 1985
Plana, J.

FACTS:
This is an appeal from the Decision of the CA which reversed the Decision of the CFI to dismiss the
petitioner’s complaint for revocation of donation.

Eduvigis J. Cruz, a childless widow, donated a 235.5 sqm. residential lot in San Isidro, Taytay Rizal together
with the two-door apartment erected thereon to her grandnieces, private respondents herein, in a deed of
donation entitled "Kasulatan Sa Kaloobpala". The property was accordingly transferred to the names of
private respondents. In 1974, Eduvigis Cruz judicially adopted Cresencia Ocreto, a minor, after which she
extrajudicially tried to revoke the donation, but the donees resisted, alleging that the property in question
was co-owned by Eduvigis Cruz and her brother and that Eduvigis Cruz owns another property, hence the
donation did not impair the presumptive legitime of the adopted child.

ISSUE:
Should the donation be revoked by the subsequent adoption made by the donor?

HELD:
No, the donation should not be revoked.

In case of the subsequent adoption of a minor by one who had previously donated some or all of his
properties to another, the donor may sue for the annulment or reduction of the donation within four years
from the date of adoption if the donation impairs the legitime of the adopted, taking into account the whole
estate of the donor at the time of the adoption of the child (Civil Code, Articles 760, 761 and 763).

The burden of proof is on the plaintiff-donor, who must allege and establish the requirements prescribed by
law, on the basis of which annulment or reduction of the donation can be adjudged. Unfortunately, in the
case at bar, the complaint for annulment of donation does not allege that the subject donation impairs the
legitime of the adopted child. Indeed it contains no indication at all of the total assets of the donor. Nor is
there proof of impairment of legitime.

214
DIFFERENT MODES OF ACQUIRING OWNERSHIP
Donation

A JUDICIAL ACTION IS PROPER ONLY WHEN THERE IS ABSENCE OF A SPECIAL PROVISION


AUTHORIZING EXTRAJUDICIAL CANCELLATION

185-A. The Roman Catholic Archbishop of Manila v. Court of Appeals


G.R. No. 77425, June 19, 1991
Regalado, J.

FACTS:
On August 23, 1930, the spouses Eusebio de Castro and Martina Rieta, now both deceased, had executed
a deed of donation in favor of the Roman Catholic Archbishop of Manila covering a parcel of land located
at Kawit, Cavite, containing an area of 964 square meters, more or less. The deed of donation allegedly
provides that the donee shall not dispose or sell the property within a period of one hundred (100) years
from the execution of the deed of donation, otherwise a violation of such condition would render null and
void the deed of donation ipso facto and the property would revert to the estate of the donors.

It is further alleged that on or about June 30, 1980, and while still within the prohibitive period to dispose of
the property, petitioner Roman Catholic Bishop of Imus, in whose administration all properties within the
province of Cavite owned by the Archdiocese of Manila was allegedly transferred on April 26, 1962,
executed a deed of absolute sale of the property subject of the donation in favor of petitioners Florencio and
Soledad C. Ignao in consideration of the sum of P114,000.00. As a consequence of the sale, Transfer
Certificate of Title No. 115990 was issued by the Register of Deeds of Cavite on November 15, 1980 in the
name of said petitioner spouses. Hence, private respondents as plaintiffs, filed a complaint for nullification
of deed of donation, rescission of contract, and reconveyance of real property with damages against the
herein petitioners. Petitioners argued that the action has already prescribed.

ISSUE:
Should an action to revoke a conditional donation with an automatic rescission clause be brought within
four (4) years?

HELD:
No. Although it is true that under Article 764 of the Civil Code an action for the revocation of a donation must
be brought within four (4) years from the non-compliance of the conditions of the donation, the same is not
applicable in the case at bar. The deed of donation involved herein expressly provides for automatic
reversion of the property donated in case of violation of the condition therein, hence a judicial declaration
revoking the same is not necessary.

In support of its aforesaid position, respondent court relied on the rule that a judicial action for rescission of
a contract is not necessary where the contract provides that it may be revoked and cancelled for violation
of any of its terms and conditions. It called attention to the holding that there is nothing in the law that
prohibits the parties from entering into an agreement that a violation of the terms of the contract would cause
its cancellation even without court intervention, and that it is not always necessary for the injured party to
resort to court for rescission of the contract. It reiterated the doctrine that a judicial action is proper only
when there is absence of a special provision granting the power of cancellation.

215
DIFFERENT MODES OF ACQUIRING OWNERSHIP
Donation

A CONDITION THAT A DONATED PROPERTY CANNOT BE DISPOSED FOR 100 YEARS IS


CONTRARY TO PUBLIC POLICY AS IT CONSTITUTES AN UNDUE RESTRICTION ON THE RIGHTS
ARISING FROM OWNERSHIP

185-B. The Roman Catholic Archbishop of Manila v. Court of Appeals


G.R. No. 77425, June 19, 1991
Regalado, J.

FACTS:
On August 23, 1930, the spouses Eusebio de Castro and Martina Rieta, now both deceased, had executed
a deed of donation in favor of the Roman Catholic Archbishop of Manila covering a parcel of land located
at Kawit, Cavite, containing an area of 964 square meters, more or less. The deed of donation allegedly
provides that the donee shall not dispose or sell the property within a period of one hundred (100) years
from the execution of the deed of donation, otherwise a violation of such condition would render null and
void the deed of donation ipso facto and the property would revert to the estate of the donors.

It is further alleged that on or about June 30, 1980, and while still within the prohibitive period to dispose of
the property, petitioner Roman Catholic Bishop of Imus, in whose administration all properties within the
province of Cavite owned by the Archdiocese of Manila was allegedly transferred on April 26, 1962,
executed a deed of absolute sale of the property subject of the donation in favor of petitioners Florencio and
Soledad C. Ignao in consideration of the sum of P114,000.00. As a consequence of the sale, Transfer
Certificate of Title No. 115990 was issued by the Register of Deeds of Cavite on November 15, 1980 in the
name of said petitioner spouses. Hence, private respondents as plaintiffs, filed a complaint for nullification
of deed of donation, rescission of contract, and reconveyance of real property with damages against the
herein petitioners.

ISSUE:
May the donor impose a restriction on alienation or disposition of the donated property for 100 years?

HELD:
No. The donor cannot impose a restriction on alienation or disposition of the donated property for 100 years
as it is an undue restriction on the donee’s ownership rights.

Once a donation (inter vivos) is made, the donee becomes the absolute owner of the property donated.
Although the donor may impose certain conditions in the deed of donation, the same must not be contrary
to law, morals, good customs, public order, or public policy. A prohibition on the disposition of the donated
property for 100 years is an unreasonable restriction on an indispensable attribute of ownership. A condition
prohibiting disposition of the donated property cannot be perpetual or be for an unreasonable amount of
time

In this case, the condition imposed should be declared as an illegal or impossible condition within the
contemplation of Article 727 of the Civil Code and is thus deemed to have not been imposed. It follows that
the donation is deemed absolute and without condition. It follows that private respondents have no cause
of action in the first place.

216
DIFFERENT MODES OF ACQUIRING OWNERSHIP
Donation

CRIMES AGAINST CHASTITY, HONOR, PROPERTY, LIBERTY, AND SECURITY OF THE DONOR
WHICH ALSO OFFEND THE PERSON OF THE DONOR ARE INCLUDED IN THOSE WHICH SHOW
INGRATITUDE AND CONSTITUTE CAUSES FOR REVOCATION

186. Eduarte v. Court of Appeals


G.R. No. 105944, February 9, 1996
Francisco, J.

FACTS:
Pedro Calapine was the registered owner of a parcel of land located in San Cristobal, San Pablo City. On
April 26, 1984, he executed a deed entitled "Pagbibigay-Pala (Donacion InterVivos)" ceding one-half portion
thereof to his niece Helen S. Doria (“Helen”). On July 26, 1984, another deed identically entitled was
purportedly executed by Pedro Calapine ceding unto Helen the whole of the parcel of land covered by OCT
No. P-2129, on the basis of which said original certificate was cancelled and in lieu thereof Transfer
Certificate of Title No. T-23205 was issued in her name. Consequently, Helen sold to the petitioners herein
a part of the allegedly donated parcel of land.

Upon knowledge of sale and claiming that his signature to the second deed of donation was a forgery and
that Helen was unworthy of his liberality, Pedro Calapine brought suit against Helen, the Calauan Christian
Reformed Church, Inc. and the Spouses Romulo and Sally Eduarte to revoke the donation made in favor of
Helen S. Doria to declare null and void the deeds of donation and sale that she had been executed in favor
of the Calauan Christian Reformed Church, Inc. and the spouses Romulo and Sally Eduarte. Herein
petitioners argued that falsification of public documents made by Helen is not a crime against the person of
the donor and so should not be considered as an act of ingratitude.

ISSUE:
Is the offense pertaining to act of ingratitude as a ground for revocation of donation limited to crimes against
person of the donor?

HELD:
No. Crimes which are not classified as crimes against persons under the Revised Penal Code may also be
acts of ingratitude and form the basis of revocation of a donation.

Article 765(1) of the New Civil Code provides that a donation may also be revoked at the instance of the
donor, by reason of ingratitude, “if the donee should commit some offense against the person, the honor or
the property of the donor, or of his wife or children under his parental authority.”

All crimes which offend the donor show ingratitude and are causes for revocation. Any attempt to categorize
the offenses according to their classification under the Revised Penal Code is unwarranted considering that
crimes like illegal detention, threats, and coercion are considered as crimes against the person of the donor
despite the fact that they are classified as crimes against personal liberty and security under the Revised
Penal Code.

Therefore, the crime of falsification committed by Helen is sufficient cause for revocation on the ground of
ingratitude.

217
DIFFERENT MODES OF ACQUIRING OWNERSHIP
Donation

THE LACK OF AN ACKNOWLEDGMENT BY THE DONEE BEFORE THE NOTARY PUBLIC DOES NOT
RENDER THE DONATION NULL AND VOID

187. Quilala v. Alcantara


G.R. No. 132681, December 3, 2001
Ynares-Santiago, J.

FACTS:
On February 20, 1981, Catalina Quilala (“Catalina”) executed a "Donation of Real Property Inter Vivos" in
favor of Violeta Quilala (“Violeta”) over a parcel of land located in Sta. Cruz, Manila. The "Donation of Real
Property Inter Vivos" consists of two pages. The first page contains the deed of donation itself, and is signed
on the bottom portion by Catalina as donor, Violeta as donee, and two instrumental witnesses. The second
page contains the Acknowledgment, which states merely that Catalina personally appeared before the
notary public and acknowledged that the donation was her free and voluntary act and deed. There appears
on the left-hand margin of the second page the signatures of Catalina and one of the witnesses, and on the
right-hand margin the signatures of Violeta and the other witness.

On November 7, 1983, Catalina died. Violeta likewise died on May 22, 1984. Petitioner Ricky Quilala
(“Ricky”) alleges that he is the surviving son of Violeta. Meanwhile, respondents Gliceria Alcantara, Leonora
Alcantara, Ines Reyes and Juan Reyes (“respondents”) claiming to be Catalina's only surviving relatives
within the fourth civil degree of consanguinity, executed a deed of extrajudicial settlement of estate, dividing
and adjudicating unto themselves the above-described property. On September 13, 1984, respondents
instituted against Ricky, an action for the declaration of nullity of the donation inter vivos on the ground that
the deed of donation was only acknowledged before a notary public only by the donor, Catalina.
Consequently, there was no acceptance by Violeta of the donation in a public instrument, thus rendering
the donation null and void.

ISSUE:
Does the lack of an acknowledgment by the donee before the notary public render the donation null and
void?

HELD:
No. The lack of an acknowledgment by the donee before the notary public does not render the donation null
and void.

The instrument should be treated in its entirety. It cannot be considered a private document in part and a
public document in another part. The fact that it was acknowledged before a notary public converts the deed
of donation in its entirety a public instrument. The fact that the donee was not mentioned by the notary public
in the acknowledgment is of no moment. The donee’s acceptance, which is explicitly set forth on the first
page of the notarized deed of donation, was therefore also made in a public instrument.

In any case, it is only the conveyance that should be acknowledged as a free and voluntary act.

218
DIFFERENT MODES OF ACQUIRING OWNERSHIP
Donation

A DONATION INTER VIVOS, ONCE ACCEPTED, TRANSFERS THE OWNERSHIP OF THE OBJECT OF
THE DONATION TO THE DONEE AND CANNOT BE REVOKED BY MERE DENIAL OF THE DEED OF
DONATION BY THE DONOR

188. Hemedes v. Court of Appeals


G.R. No. 107132, October 8, 1999
Gonzaga-Reyes, J.

FACTS:
On March 22, 1947, Jose Hemedes, father of Maxima Hemedes and Enrique D. Hemedes, executed a
document entitled “Donation Inter Vivos With Resolutory Conditions” whereby he conveyed ownership over
an unregistered parcel of land in Cabuyao, Laguna, together with all its improvements, in favor of his third
wife, Justa Kauapin, subject to the resolutory condition that upon the her death or remarriage, the title to
the property donated shall revert to any of the children, or their heirs, of the donor who shall be “expressly
designated by the DONEE in a public document”.

Pursuant to the said condition, Justa Kausapin executed a “Deed of Conveyance of Unregistered Real
Property by Reversion” conveying to Maxima Hemedes, the subject property. Despite the earlier
conveyance of the subject land in favor of Maxima Hemedes, Justa Kausapin executed a “Kasunduan”
whereby she transferred the same land to her stepson Enrique D. Hemedes. Maxima argues that the second
document should not be given any credence since she is obviously a biased witness as it has been shown
that she is dependent upon Enrique for her daily subsistence, and she was most probably influenced by him
to execute the “Kasunduan” in his favor.

ISSUE:
Was the Kasunduan made in favor of Enrique Hemedes valid?

HELD:
No. The Kasunduan made in favor of Enrique Hemedes is null and void because the purported object thereof
did not exist at the time of execution, having already been transferred to his sister.

Enrique failed to produce clear, strong, and convincing evidence to overcome the positive value of the “Deed
of Conveyance of Unregistered Real Property by Reversion” – a notarized document. The mere denial of
its execution by the donor will not suffice for the purpose. Thus, the “Deed of Conveyance of Unregistered
Real Property by Reversion” made in favor of Maxima must be deemed to have transferred the property to
Maxima. Consequently, when Justa Kausapin sought to transfer to her stepson exactly what she had earlier
transferred to Maxima, she was no longer the owner thereof. Therefore, Enrique, and his transferee
Dominium, acquired no rights over the subject property.

The declarations of real property by Enrique, his payment of realty taxes, and his being designated as owner
of the subject property in the cadastral survey of Cabuyao, Laguna and in the records of the Ministry of
Agrarian Reform office in Calamba, Laguna cannot defeat a certificate of title, which is an absolute and
indefeasible evidence of ownership of the property in favor of the person whose name appears therein.
Particularly, with regard to tax declarations and tax receipts, this Court has held on several occasions that
the same do not by themselves conclusively prove title to land.

219
DIFFERENT MODES OF ACQUIRING OWNERSHIP
Donation

FOR PRESUMPTION OF FRAUD TO APPLY, IT MUST BE ESTABLISHED THAT THE DONOR DID NOT
LEAVE ADEQUATE PROPERTIES WHICH CREDITORS MIGHT HAVE RECOURSE FOR THE
COLLECTION OF THEIR CREDITS EXISTING BEFORE THE EXECUTION OF THE DONATION

189. Siguan v. Lim


G.R. No. 134685, November 19, 1999
Davide, Jr., C.J.

FACTS:
In 1992, Rosa Lim was convicted for violation of Batas Pambansa Blg. 22. On appeal, however, Lim was
acquitted, but was held civilly liable for actual damages, plus legal interest. On July 1991, a Deed of
Donation conveying the parcels of land and purportedly executed by Lim on 10 August 1989 in favor of her
children, Linde, Ingrid and Neil, was registered with the Office of the Register of Deeds of Cebu City.

On 23 June 1993, Maria Antonia Siguan filed an accion pauliana against Lim and her children to rescind
the questioned Deed of Donation and to declare as null and void the new transfer certificates of title issued
for the lots covered by the questioned Deed. Siguan claimed that Lim, through a Deed of Donation,
fraudulently transferred all her real property to her children in bad faith and in fraud of creditors, including
her, conspired and confederated with her children in antedating the questioned Deed of Donation, to her
and other creditors' prejudice, and left no sufficient properties to pay her obligations at the time of the
fraudulent conveyance. On the other hand, Lim denied any liability to petitioner. She claimed that her
convictions in criminal cases were erroneous, which was the reason why she appealed said decision to the
Court of Appeals. As regards the questioned Deed of Donation, she maintained that it was not antedated
but was made in good faith at a time when she had sufficient property. Finally, she alleged that the Deed of
Donation was registered only on 2 July 1991 because she was seriously ill.

ISSUE:
May the Deed of Donation executed by respondent Lim in favor of her children be rescinded for being in
fraud of her alleged creditor, Siguan?

HELD:
No.

Article 1381 of the Civil Code enumerates the contracts which are rescissible, and among them are "those
contracts undertaken in fraud of creditors when the latter cannot in any other manner collect the claims due
them." Article 1387 of the same Code provides: "All contracts by virtue of which the debtor alienates property
by gratuitous title are presumed to have been entered into in fraud of creditors when the donor did not
reserve sufficient property to pay all debts contracted before the donation. Likewise, Article 759 of the same
Code states that the donation is always presumed to be in fraud of creditors when at the time thereof the
donor did not reserve sufficient property to pay his debts prior to the donation. For this presumption of fraud
to apply, it must be established that the donor did not leave adequate properties which creditors might have
recourse for the collection of their credits existing before the execution of the donation.

In the instant case, the alleged debt of Lim in favor of Siguan was incurred in August 1990, while the deed
of donation was purportedly executed on 10 August 1989. Siguan cannot, therefore, be said to have been
prejudiced or defrauded by such alienation. The Court is not convinced with the allegation of the petitioner
that the questioned deed was antedated to make it appear that it was made prior to petitioner's credit.
Notably, that deed is a public document, it having been acknowledged before a notary public. As such, it is
evidence of the fact which gave rise to its execution and of its date, pursuant to Section 23, Rule 132 of the
Rules of Court. Accordingly, since the requirements for the rescission of a gratuitous contract are not present
in this case, petitioner's action must fail.

220
DIFFERENT MODES OF ACQUIRING OWNERSHIP
Donation

THE USURPATION OF THE DONOR’S PROPERTY IS AN ACT OF INGRATITUDE AND DOES NOT
NEED TO BE PROVEN BY CRIMINAL CONVICTION

190-A. Noceda v. Court of Appeals


G.R. No. 119730, September 2, 1999
Gonzaga-Reyes, J.

FACTS:
Aurora Directo (“Directo”), Rodolfo Noceda (“Noceda”), and Maria Arbizo, the daughter, grandson, and
widow, respectively, of the late Celestino Arbizo (“Celestino”), who died in 1956, extrajudicially settled a
parcel of land owned by Celestino. Directo’s share was 11,426 square meters, Noceda got 13,294 square
meters, and the remaining 41,810 square meters went to Maria Arbizo. On the same day, Directo donated
625 square meters of her share to Noceda, who is her nephew being the son of her deceased sister,
Carolina.

Sometime in 1981, Noceda constructed his house on the land donated to him by Directo. Directo fenced
the portion allotted to her in the extrajudicial settlement, excluding the donated portion, and constructed
thereon three huts. But in 1985, Noceda removed the fence earlier constructed by Directo, occupied the
three huts (3) and fenced the entire land of plaintiff Directo without her consent. Directo demanded from
Noceda to vacate her land, but the latter refused. Hence, Directo filed the present suit, a complaint for the
recovery of possession and ownership and rescission/annulment of donation, against defendant Noceda
before the lower court.

She also sought to revoke the donation she made to Noceda on the ground of ingratitude.

ISSUE:
Is the usurpation of the donor’s property an “offense against the person of the donor”?

HELD:
Yes, a crime against the donor is sufficient to constitute an act of ingratitude, including crimes againt the
property of the donor, and is grounds for revoking a donation made.

It was established that Noceda occupied not only the portion donated to him by Directo but he also fenced
the whole area of the lot which belongs to Directo. Thus petitioners act of occupying the portion pertaining
to private respondent Directo without the latter’s knowledge and consent is an act of usurpation which is an
offense against the property of the donor and considered as an act of ingratitude of a donee against the
donor. The law does not require conviction of the donee; it is enough that the offense be proved in the action
for revocation.

221
DIFFERENT MODES OF ACQUIRING OWNERSHIP
Donation

THE BURDEN OF PROVING THAT THE ACTION TO REVOKED HAS PRESCRIBED IS ON THE DONEE
RESISTING THE REVOCATION

190-B. Noceda v. Court of Appeals


G.R. No. 119730, September 2, 1999
Gonzaga-Reyes, J.

FACTS:
Aurora Directo (“Directo”), Rodolfo Noceda (“Noceda”), and Maria Arbizo, the daughter, grandson, and
widow, respectively, of the late Celestino Arbizo (“Celestino”), who died in 1956, extrajudicially settled a
parcel of land owned by Celestino. Directo’s share was 11,426 square meters, Noceda got 13,294 square
meters, and the remaining 41,810 square meters went to Maria Arbizo. On the same day, Directo donated
625 square meters of her share to Noceda, who is her nephew being the son of her deceased sister,
Carolina.

Sometime in 1981, Noceda constructed his house on the land donated to him by Directo. Directo fenced
the portion allotted to her in the extrajudicial settlement, excluding the donated portion, and constructed
thereon three huts. But in 1985, Noceda removed the fence earlier constructed by Directo, occupied the
three huts (3) and fenced the entire land of plaintiff Directo without her consent. Directo demanded from
Noceda to vacate her land, but the latter refused. Hence, Directo filed the present suit, a complaint for the
recovery of possession and ownership and rescission/annulment of donation, against defendant Noceda
before the lower court.

She also sought to revoke the donation she made to Noceda. On this point, Noceda contends that the right
to enforce the revocation had already prescribed since Noceda’s alleged usurpation of the property occurred
in the first week of September 1985 while the complaint for revocation was filed on September 16, 1986,
thus more than one (1) year later.

ISSUE:
Has the right of action to revoke already prescribed?

HELD:
No. As provided in Article 769, the action to revoke by reason of ingratitude prescribes within one (1) year
to be counted from the time
(a) the donor had knowledge of the fact; and
(b) provided that it was possible for him to bring the action.

It is incumbent upon donee to show proof of the concurrence of these two conditions in order that the donor’s
right of action can be considered to have already prescribed.

Here, Noceda reckoned the one-year prescriptive period from the occurrence of the alleged usurpation of
the property of Directo in the first week of September 1985, and not from the time Directo obtained
knowledge of the usurpation. Even assuming she had knowledge of the act of ingratitude as of the first week
of September 1985, Noceda also failed to prove that at the time, it was possible for Directo to institute an
action for revocation of her donation. There being no competent proof to prove his allegation of prescription,
the action must be deemed to have not prescribed.

222
DIFFERENT MODES OF ACQUIRING OWNERSHIP
Donation

THE DONEE BECOMES THE ABSOLUTE OWNER OF THE PROPERTY DONATED ONCE A
DONATION IS ACCEPTED

191. Heirs of Velasquez v. Court of Appeals


G.R. No. 126996, February 15, 2000
Gonzaga-Reyes, J.

FACTS:
Spouses Leoncia de Guzman and Cornelio Aquino died intestate sometime in 1945 and 1947, respectively
and were childless. Leoncia de Guzman was survived by her sisters Anatalia de Guzman (mother of the
respondents) and Tranquilina de Guzman (grandmother of the petitioners). During the existence of their
marriage, spouses Aquino were able to acquire real properties.

In 1989, the heirs of Anatalia, all surnamed Meneses filed a complaint for annulment, partition and damages
against the heirs of Cesario Velasquez, son of Tranquilina, for the latters’ refusal to partition all of the
conjugal properties of the Spouses Aquino. The complaint alleged that Leoncia de Guzman, before her
death, called a conference among them, telling them that all their properties which are conjugal in nature
shall be divided equally between Anatalia and Tranquilina and not to believe the documents purportedly
signed by her because she did not sign them.

The petitioners, however, adduced uncontroverted and ancient documents showing that during the lifetime
of the Aquino spouses, they already donated by several deeds, four of the six parcels of land subject of the
complaint to the petitioners’ predecessors-in-interest.

ISSUE:
Can the action for partition prosper?

HELD:
No.

A donation as a mode of acquiring ownership results in an effective transfer of title over the property from
the donor to the donee and the donation is perfected from the moment the donor knows of the acceptance
by the donee. And once a donation is accepted, the donee becomes the absolute owner of the property
donated.

The donation of the first parcel made by the Aquino spouses to petitioners Jose and Anastacia Velasquez
who were then nineteen (19) and ten (10) years old respectively was accepted through their father Cesario
Velasquez, and the acceptance was incorporated in the body of the same deed of donation and made part
of it, and was signed by the donor and the acceptor. Legally speaking there was delivery and acceptance
of the deed, and the donation existed perfectly. These donations inter vivos may be revoked only for the
reasons provided in Articles 760, 764 and 765 of the Civil Code.

The donation propter nuptias in favor of Cesario Velasquez and Camila de Guzman over the third and sixth
parcels including a portion of the second parcel became the properties of the spouses Velasquez since
1919. The deed of donation propter nuptias can be revoked by the non-performance of the marriage and
the other causes mentioned in article 86 of the Family Code. The alleged reason for the repudiation of the
deed, i.e, that the Aquino spouses did not intend to give away all their properties since Anatalia (Leoncia’s
sister) had several children to support is not one of the grounds for revocation of donation either inter vivos
or propter nuptias, although the donation might be inofficious.

In view of the foregoing, we conclude that this action of partition cannot be maintained. The properties
sought to be partitioned by private respondents have already been delivered to petitioners and therefore no
longer part of the hereditary estate which could be partitioned. After finding that no co-ownership exists
between private respondents and petitioners, we find no reason to discuss the other arguments raised by
the petitioners in support of their petition.

223
DIFFERENT MODES OF ACQUIRING OWNERSHIP
Donation

NON-REGISTRATION OF A DEED OF DONATION SHALL NOT AFFECT ITS VALIDITY; THE


NECESSITY OF REGISTRATION COMES INTO PLAY WHEN THE RIGHTS OF THIRD PERSONS ARE
AFFECTED

192. Gonzales v. Court of Appeals


G.R. No. 110335, June 18, 2001
Melo, J.

FACTS:
Marina Gonzales died intestate and appointed as administratrix of her estate was petitioner Lilia Gonzales.
Prior to the partition of said estate, Ignacio Gonzales, Marina’s spouse, executed a Deed of Donation,
conveying his share of the conjugal property in favor of his 14 grandchildren. The said donation was not
registered.

When Presidential Decree No. 27 took effect on October 21, 1972, the landholdings of the spouses
Gonzales were placed under Operation Land Transfer by virtue of said decree, and the private respondents,
farmers and tenants of the Gonzales spouses who had been cultivating the parcels of land even before
World War II either personally or through their predecessors-in-interest, were accordingly issued the
corresponding Certificates of Land Transfer and Emancipation Patents.

Administratrix Lilia Gonzales filed an application for retention with the then Ministry of Agrarian Reform,
requesting that their property be excluded from the coverage of Operation Land Transfer. After initial
investigation, the Hearing Officer recommended the denial of said application for retention and this action
was affirmed by the then Assistant Secretary of Agrarian Reform. A reinvestigation was conducted, resulting
in the Department of Agrarian Reform (DAR) resolution, recommending that the land subject of the Ignacio’s
Deed of donation be exempt from Operation Land Transfer which was granted by the DAR Secretary
Benjamin Leong. Thus, the Certificates of Land Transfer issued in favor of private respondents were
cancelled, hence this complaint by respondents.

ISSUE:
Should the property subject of the deed of donation, which was not registered when P.D. No. 27 took effect,
be excluded from the Operation Land Transfer?

HELD:
No, the property should not be excluded from the Option Land Transfer despite the fact that it was not
registered

Article 749 of the Civil Code provides inter alia that "in order that the donation of an immovable may be
valid, it must be made in a public document, specifying therein the property donated and the value of the
charges which the donee must satisfy." Corollarily, Article 709 of the same Code explicitly states that "the
titles of ownership, or other rights over immovable property, which are not duly inscribed or annotated in the
Registry of property shall not prejudice third persons."

From the foregoing provisions, it may be inferred that as between the parties to a donation of an immovable
property, all that is required is for said donation to be contained in a public document. Registration is not
necessary for it to be considered valid and effective. However, in order to bind third persons, the donation
must be registered in the Registry of Property (now Registry of Land Titles and Deeds). Although the non-
registration of a deed of donation shall not affect its validity, the necessity of registration comes into play
when the rights of third persons are affected, as in the case at bar.

224
DIFFERENT MODES OF ACQUIRING OWNERSHIP
Donation

ACTION FOR REDUCTION OF DONATION ON THE GROUND OF IMPAIRMENT OF LEGITIME


PRESCRIBES IN 10 YEARS

193. Imperial v. Court of Appeals


G.R. No. 112483, October 8, 1999
Gonzaga-Reyes, J.

FACTS:
Petitioner seeks to set aside the Decision of the Court of Appeals in C.A.-G.R. CV No. 31976, affirming the
Decision of the Regional Trial Court of Legazpi City, which found the donation made by Leoncio Imperial in
favor of petitioner, Eloy Imperial, inofficious.

Leoncio Imperial was the registered owner of the subject land. In a contract designated as one of “Absolute
Sale”, he sold the same to his natural son, Eloy. Despite the contract’s designation, it was admitted by the
parties to be in fact a donation. Two years after, Leoncio filed a complaint for annulment of the said Deed
of Absolute Sale on the ground that he was deceived by petitioner herein into signing the said document.
The dispute, however, was resolved through a compromise agreement, under which terms, Leoncio
recognized the legality and validity of the rights of petitioner to the land donated. On January 8, 1962,
Leoncio subsequently died, leaving only two heirs — Eloy and an adopted son, Victor Imperial. Sometime
in 1996, the heirs of Victor filed a complaint for the nullification of the Deed of Absolute Sale on the ground
of the same being inofficious, allegedly impairing the legitime of Victor.

Eloy however argues that Leoncio had conveyed sufficient property to Victor to cover his legitime and in
any case, since it took 24 years for the respondents to file this action, the same has already prescribed.

ISSUE:
Is the action already barred by prescription?

HELD:
Yes, the action is barred by prescription.

The Civil Code specifies the following instances of reduction or revocation of donations: (1) four years, in
cases of subsequent birth, appearance, recognition or adoption of a child; (2) four years, for non-compliance
with conditions of the donation; and (3) at any time during the lifetime of the donor and his relatives entitled
to support, for failure of the donor to reserve property for his or their support. Interestingly, donations as in
the instant case, the reduction of which hinges upon the allegation of impairment of legitime, are not
controlled by a particular prescriptive period, for which reason we must resort to the ordinary rules of
prescription.

Under Article 1144 of the Civil Code, actions upon an obligation created by law must be brought within ten
years from the time the right of action accrues. Thus, the ten-year prescriptive period applies to the
obligation to reduce inofficious donations, required under Article 771 of the Civil Code, to the extent that
they impair the legitime of compulsory heirs.

From when shall the ten-year period be reckoned? The case of Mateo vs. Lagua (29 SCRA 864), which
involved the reduction for inofficiousness of a donation propter nuptias, recognized that the cause of action
to enforce a legitime accrues upon the death of the donor-decedent. Clearly so, since it is only then that the
net estate may be ascertained and on which basis, the legitimes may be determined.

Thus, the action prescribed in January 8, 1972. It took private respondents 24 years since the death of
Leoncio to initiate this case. The action, therefore, has long prescribed.

225
DIFFERENT MODES OF ACQUIRING OWNERSHIP
Donation

WHERE A DONATION IS SUBJECT TO A CONDITION, THE CONDITION IS NOT VIOLATED WHEN


THE PROPERTY IS USED IN A MANNER THAT IS CONSISTENT OR IS FOR THE FURTHERANCE OF
THE OBJECTIVE OF THE DONATION

194-A. Republic v. Silim


G.R. No. 140487, April 2, 2001
Kapunan, J.

FACTS:
This is an appeal seeking the reversal of the Decision of the Court of Appeals, which declared null and void
the donation made by respondents of a parcel of land in favor of the Bureau of Public Schools, Municipality
of Malangas, Zamboanga del Sur.

On 17 December 1971, respondents, the Spouses Leon Silim and Ildefonsa Mangubat, donated a 5,600
sqm. parcel of land in favor of the Bureau of Public Schools, Municipality of Malangas, Zamboanga del Sur
(BPS). In the Deed of Donation, respondents imposed the condition that the said property should "be used
exclusively and forever for school purposes only." This donation was accepted by Gregorio Buendia, the
District Supervisor of BPS, through an Affidavit of Acceptance and/or Confirmation of Donation. Through a
fund-raising campaign organized by the Parent-Teachers Association of Brgy. Kauswagan, a school
building was constructed on the donated land.

Later on, public funds for a second school building for the school district, which was planned to be erected
on the donated land, could not be released since the government required that the building should be built
a one (1) hectare parcel of land. To remedy this predicament, District Supervisor Buendia was authorized
to officially transact for the exchange of the donated land for another which is suitable and compliant with
the specifications of the government. Thus, District Supervisor Buendia and Teresita Palma entered into a
Deed of Exchange whereby the donated lot was bartered for a bigger lot owned by the latter. Consequently,
the new school buildings were constructed on the new school site and the school building previously erected
on the donated lot was dismantled and transferred to the new location.

Thus, upon discovering that the donated land had been sold as patrimonial property, the respondent-donors
filed a Complaint to revoke the conditional donation and reversion of property. The trial court dismissed the
complaint for lack of merit, stating that there was no breach or violation of the condition imposed in the
subject Deed of Donation by the donee. It said that the exchange is proper since the lot was still effectively
used exclusively for school purposes and for the expansion and improvement of the school facilities within
the community. The Deed of Exchange is but a continuity of the desired purpose of the donation made by
plaintiff Leon Silim.

On appeal, the Court of Appeals reversed the decision of the trial court and declared that, among other
grounds, the donation null and void because the condition imposed on the donation was violated. Hence,
this petition.

ISSUE:
Did the use of the lot to barter for a bigger lot that is to be used for the same purpose a valid disposition
which does not violate the condition in the donation?

HELD:
Yes, the condition for the donation was not in any way violated when the lot donated was exchanged with
another one.

The purpose for the donation remains the same, which is for the establishment of a school. The exclusivity
of the purpose was not altered or affected. In fact, the exchange of the lot for a much bigger one was in
furtherance and enhancement of the purpose of the donation. The acquisition of the bigger lot paved the
way for the release of funds for the construction of Bagong Lipunan school building which could not be
accommodated by the limited area of the donated lot.

Thus, the Court of Appeals erred and must be reversed.

226
DIFFERENT MODES OF ACQUIRING OWNERSHIP
Donation

WHERE A DONATION IS SUBJECT TO A CONDITION, THE CONDITION IS NOT VIOLATED WHEN


THE PROPERTY IS USED IN A MANNER THAT IS CONSISTENT OR IS FOR THE FURTHERANCE OF
THE OBJECTIVE OF THE DONATION

ACTUAL KNOWLEDGE FULFILLS THE FORMAL REQUIREMENT OF COMMUNICATING


ACCEPTANCE TO DONOR

194. Republic v. Silim


G.R. No. 140487, April 2, 2001
Kapunan, J.

FACTS:
This is an appeal seeking the reversal of the Decision of the Court of Appeals, which declared null and void
the donation made by respondents of a parcel of land in favor of the Bureau of Public Schools, Municipality
of Malangas, Zamboanga del Sur.

On 17 December 1971, respondents, the Spouses Leon Silim and Ildefonsa Mangubat, donated a 5,600
sqm. parcel of land in favor of the Bureau of Public Schools, Municipality of Malangas, Zamboanga del Sur
(BPS). In the Deed of Donation, respondents imposed the condition that the said property should "be used
exclusively and forever for school purposes only." This donation was accepted by Gregorio Buendia, the
District Supervisor of BPS, through an Affidavit of Acceptance and/or Confirmation of Donation.

It was initially used for a school building. Later on, the need to expand the school arosed. However, because
the donated lot was only about ½-hectare and government regulations required that schools be built on a
1-hectare land, the lot was used to barter for a larger parcel of land on which new school buildings were
erected.

Thus, upon discovering that the donated land had been sold as patrimonial property, the respondent-donors
filed a Complaint to revoke the conditional donation and reversion of property. The trial court dismissed the
complaint for lack of merit, stating that there was no breach or violation of the condition imposed in the
subject Deed of Donation by the done, since the lot was still effectively used exclusively for school purposes
and for the expansion and improvement of the school facilities within the community. On appeal, the Court
of Appeals reversed the decision of the trial court. When the case was elevated on appeal to the Supreme
Court, the respondents raised a new defense that the deed of donations had not been properly accepted.

ISSUE:
Is the donation void for lack of a proper acceptance?

HELD:
No. The donation was properly accepted by the petitioner.

First, it must be noted that the respondents raise this matter for the first time on appeal. In any case, if there
was really a defect, why did the respondents take 10 years to question it? They must at least be held in
estoppel.

Second, contrary to the arguments of the respondents, it was already settled in Pajarillo v. IAC that the
requirement that the acceptance made in a separate instrument be noted on the original deed is not
indispensable when it is proven that the donor was made aware of such acceptance in another manner. In
this case, a school building was immediately constructed after the donation was executed. Respondents
had knowledge of the existence of the school building put up on the donated lot through the efforts of the
Parents-Teachers Association of Barangay Kauswagan. This knowledge already fulfilled the legal
requirement that the acceptance of the donation by the donee be communicated to the donor.

Finally, BPS District Supervisor Gregorio Buendia did not need a special power of attorney from the Republic
of the Philippines to accept the donation because he is expressly authorized to do so under Section 47 of
the 1987 Administrative Code which provides that “Contracts or conveyances may be executed for and in
behalf of the Government or of any of its branches, subdivisions, agencies, or instrumentalities, whenever
demanded by the exigency or exigencies of the service and as long as the same are not prohibited by law.

227
DIFFERENT MODES OF ACQUIRING OWNERSHIP
Donation

A VALID DONATION, ONCE ACCEPTED, BECOMES IRREVOCABLE EXCEPT UPON LAWFUL


CAUSE AND IN PROPER JUDICIAL PROCEEDINGS

195. Spouses Gestopa v. Court of Appeals


G.R. No. 111904, October 5, 2000
Quisumbing, J.

FACTS:
The spouses Diego and Catalina Danlag were the owners of six parcels of unregistered land. In 1965 and
1966, they executed three deeds of “donation mortis causa” in favor of private respondent Mercedes
Danlag-Pilapil. All of the deeds contained the reservation of the rights of the donors (1) to amend, cancel or
revoke the donation during their lifetime, and (2) to sell, mortgage, or encumber the donated properties
during the donors' lifetime, if deemed necessary.

On January 16, 1973, Diego Danlag, with the consent of his wife, Catalina Danlag, executed a deed of
donation inter vivos covering the same parcels of land plus two other parcels, again in favor of Mercedes.
This deed contained two conditions, that (1) the Danlag spouses shall continue to enjoy the fruits of the land
during their lifetime, and that (2) the donee cannot sell or dispose of the land during the lifetime of the said
spouses, without their prior consent and approval. Mercedes caused the transfer of the parcels' tax
declaration to her name and paid the taxes on them.

On June 28, 1979 and August 21, 1979, Diego and Catalina Danlag sold two of donated parcels to
petitioners Mr. and Mrs. Agripino Gestopa. On September 29, 1979, the Danlags executed a deed of
revocation recovering the six parcels of land subject of the deed of donation inter vivos.

Thus, on March 1, 1983, Mercedes Pilapil filed a petition, for quieting of title against the Gestopas and the
Danlags. In their opposition, the Gestopas and the Danlags averred that the deed of donation inter vivos
was null and void because it was obtained by Mercedes through machinations and undue influence. Even
assuming it was validly executed, the intention was for the donation to take effect upon the death of the
donor and thus revocable without cause. They also cite Mercedes' alleged vehemence in prohibiting them
from gathering coconut trees and her filing of instant petition for quieting of title as acts of ingratitude.

The trial court ruled for the petitioners but was reversed by the CA ruling that the reservation by the donor
of lifetime usufruct indicated that the donation was one that is inter vivos and thus could not be revoked
without cause. Hence, this petition.

ISSUE:
Is the donation made in 1973 revocable?

HELD:
No. Being a valid donation inter vivos, it cannot be revoked without cause. The conditions placed on the
donation did not make it a donation mortis causa.

In ascertaining the intention of the donor, all of the deed's provisions must be read together. The
establishment of a life-time usufruct in favor of the donor and the imposition of a restriction on the disposition
of the naked ownership during the lifetime of the donor-usufructuaries does not militate against the transfer
of ownership of the land to Mercedes in a donation inter vivos. These conditions show that the donor
intended to transfer the naked ownership over the properties despite reserving sufficient properties for his
maintenance in accordance with his standing in society.

A valid donation, once accepted, becomes irrevocable, except on account of officiousness, failure by the
donee to comply with the charges imposed in the donation, or ingratitude. The donor-spouses did not invoke
any of these reasons in the deed of revocation.

While alleged, there is no showing that Mercedes prohibited the donors from gathering coconuts. Even
assuming that Mercedes prevented the donor from gathering coconuts, this could hardly be considered an
act of ingratitude under Article 765 of the Civil Code. Nor does the filing of the petition for quieting of title
constitute ingratitude as she is merely asserting what she believed was her right under the law.

228
Also, records do not show that the donor-spouses instituted any judicial action to revoke the donation in
accordance with Article 769 of the Civil Code. Consequently, the supposed revocation on September 29,
1979, had no legal effect.

229
DIFFERENT MODES OF ACQUIRING OWNERSHIP
Donation

WHEN THE SUBJECT OF DONATION IS PURCHASE MONEY, ARTICLE 748 OF THE NEW CIVIL
CODE APPLIES SUCH THAT A DONATION OF MONEY EXCEEDING FIVE THOUSAND PESOS MUST
APPEAR IN A WRITTEN INSTRUMENT TO BE VALID

196. Carinan v. Spouses Cueto


G.R. No. 198636, October 8, 2014
Reyes, J.

FACTS:
Sometime before 2005, petitioner Esperanza Carinan and her husband entered into a Deed of Assignment
with a certain Roberto Ventura whereby the Sps. Carinan undertook to payment the remaining balance on
the purchase price of a property in Binan, Laguna conditionally sold by the GSIS to Ventura.

By 2005 however, the Sps. Carinan also failed to make several payments and was on the verge of default.
Thus, Esperanza sought financial assistance from her brother Gavino Cueto, herein respondent. Thus,
Gavino paid the obligations of Esperanza out of the conjugal funds of the Sps. Cueto amounting to almost
P1.3 Million. The Sps. Cueto alleged that Gavino and Esperanza agreed that in exchange for Gavino’s help,
she would execute a deed of sale with pacto de retro in favor of the Sps. Cueto to guarantee her repayment
of the substantial financial assistance. As a result of Gavino’s help, the title to the property was transferred
from the GSIS to Esperana Carinan. The certificate of title was then delivered to the Sps. Cueto.

Sometime in 2006, the Sps. Cueto demanded from Esperanza the execution of the agreed upon deed of
sale. Esperanza refused however and so the Sps. Cueto filed a complaint before the RTC. In her defense,
Esperanza alleged that Gavino paid her outstanding balance with the GSIS out of sheer generosity and pity
upon her. She denied having borrowed the respondents’ money because given her financial standing, she
knew that she could not afford to pay it back.

Both the trial court and the CA ruled in favor of the Sps. Cueto holding that the payment of Esperanza’s
obligations by the Sps. Cueto could not have been gratuitous. The CA also noted the fact that Esperanza
has voluntarily delivered the certificate of title to Gavino. Hence, this petition.

ISSUE:
Did the Sps. Cueto donate to Esperanza?

HELD:
No, the existence of a donation is not proven.

There was a clear agreement that Esperanza would return the amounts which the Sps. Cueto spent for the
acquisition, transfer, and renovation of the subject property. The Sps. Cueto then reasonably expected to
get their money back from Esperanza. Esperanza’s claim that the expenses and payments in her behalf
were purely gratuitous remained unsupported by records.

In order to sufficiently substantiate her claim that the money paid by the respondents was actually a
donation, Esperanza should have also submitted in court a copy of a written deed of donation or a contract
evincing such agreement. A donation must comply with the mandatory formal requirements set forth by law
for its validity. When the subject of donation is purchase money, Article 748 of the NCC is applicable.
Accordingly, a donation of money that exceeds P5,000.00, as well as its acceptance, should be in writing.
Otherwise, the donation is void for non-compliance with the formal requisites prescribed by law.

230
DIFFERENT MODES OF ACQUIRING OWNERSHIP
Donation

ARTICLE 765 OF THE CIVIL CODE ON THE GROUNDS OF REVOKING A DONATION BY REASON OF
INGRATITUDE APPLIES TO THE GRATUITOUS PORTION OF A DONATION THAT IS PARTLY
ONEROUS AND PARTLY GRATUITOUS

197. Calanasan v. Spouses Dolorito


G.R. No. 171937, November 25, 2013
Brion, J.

FACTS:
The petitioner, Cerila J. Calanasan (“Cerila”), was the one who took care of her orphaned niece, herein
respondent Evelyn C. Dolorita (“Evelyn”). In 1982, when Evelyn was already married to Virgilio Dolorita
(“Virgilio”), Cerila donated to Evelyn an encumbered parcel of land under the condition that Evelyn redeem
the land from its mortgagee and that Cerila would be granted a lifetime usufruct over the same. Evelyn
accepted the donation on the same deed and complied with the conditions of the donation.

On August 15, 2002, however, Cerila, assisted by her sister Teodora J. Calanasan (“Teodora”) filed a
petition to revoke the donation on the ground of ingratitude against the respondent Sps. Dolorita. During the
pendency of the trial, Cerila died. After the prosecution rested its case, the respondents filed a demurrer to
evidence arguing that the alleged acts of ingratitude were those committed by Virgilo against Teodora.
Hence, the acts proven do not constitute grounds for revocation.

The trial court granted the demurrer and dismissed the case. The CA affirmed on appeal while also holding
that because the donation was onerous, it was governed by the law on contracts and is not covered by
Article 765. Hence this appeal.

ISSUE:
May the donation made in favor of Evelyn be revoked?

HELD:
No. It may not be revoked.

First, the Court must correct the Court of Appeals in that the donation is not entirely onerous. The amount
paid by Evelyn to redeem the property (P15,000.00) is far less than the value of the property. Thus, it is only
onerous to the extent of P15,000.00 but is gratuitous as to the rest. Consequently, the CA erred in ruling
that Article 765 is not applicable. The donation may still be revoked on the ground of ingratitude as to the
gratuitous portion.

In any event, the trial court is correct in holding that there was no ground for revocation as the ungrateful
acts were committed not by the donee but by her husband. Also, the allegedly ungrateful acts were
perpetrated not against the person of the donor but to her sister. These twin considerations place the case
out of the purview of Article 765 of the New Civil Code.

231
DIFFERENT MODES OF ACQUIRING OWNERSHIP
Donation

AN ABSOLUTE SALE DISGUISED AS A DONATION IS A RELATIVELY SIMULATED CONTRACT

198. Victoria v. Pidlaoan


G.R. No. 196470, April 20, 2016
Brion, J.

FACTS:
The petitioners Rosario Victoria (Rosario) and Elma lived together since 1978 until Rosario left for Saudi
Arabia. In 1984, Elma bought a parcel of land with an area of 201 square meters in Lucena City and was
issued TCT no. T-50282. When Rosario came home, she caused the construction of a house on the lot but
she left again after the house was built.

Elma allegedly mortgaged the house and lot to a certain Thi Hong Villanueva in 1989. When the properties
were about to be foreclosed, Elma allegedly asked for help from her sister-in-law, Eufemia Pidlaoan
(Eufemia), to redeem the property. On her part, Eufemia called her daughter abroad, Normita, to lend money
to Elma. Normita agreed and provided the loan.

Elma then allegedly began attempting the house and the lot. When she failed to find a buyer, she then
offered to Eufemia and Normita, who agreed. On March 21, 1993, Elma executed a deed of sale entitled
"Panananto ng Pagkatanggap ng Kahustuhang Bayad". The deed contained a provision that obliged Elma
shall eject the person who erected the house and deliver the lot to Normita. The document was signed by
Elma, Normita, and two witnesses but it was not notarized.

When Elma and Normita were about to have the document notarized, the notary public advised them to
enter into a donation instead to avoid capital gains tax. Thus, on the next day, Elma executed a deed of
donation in Normita's favor and had it notarized. TCT No. T-50282 was cancelled and TCT No. T-70990
was issued in Normita's name. Since then, Normita had been paying the real property taxes over the lot
although Elma continued to occupy the house. Rosario found out about the donation when she returned to
the country a year or two after the transaction.

In 1997 Elma and Rosario filed a complaint against Eufemia and Normita seeking the cancellation of TCT
No. T-70990 on the ground that the lot was co-owned. The complaint also prayed for the reformation of the
deed of donation since the intent of the parties was to enter into an equitable mortgage with the donation
being simulated only to evade taxes. Eufemia and Normita admitted that the donation was simulated but
insisted that the original intent of the parties was to enter into a contract of sale.

The RTC ruled in favor of Elma and Rosario. However, the CA reversed the RTC and dismissed the case
holding that the donation was not simulated. Hence, this petition.

ISSUE:
What was the contract between the parties?

HELD:
There is a relatively simulated contract of sale.

There are two kinds of simulation of contracts: absolute and relative. In absolute simulation, there is no
intention to be bound to a contract at all. In relative simulation however, there is contractual intent except
that the parties merely concealed their true intent.

In this case, the facts showed that Eufemia and Normita intended to enter into a sale that would transfer the
ownership of the subject matter of their contract but disguised it as a donation. The contents of the
"Panananto ng Pagkatanggap ng Kahustuhang Bayad" evidence an unconditional sale of property between
Elma and Normita. Contrary to the petitioner’s arguments, there is no evidence of any intention that the
transfer of the lot was merely intended to secure a debt or to grant a right to repurchase. Hence, there is no
equitable mortgage on the property as contemplated in Article 1602 of the Civil Code. Elma sold the entire
property to Normita.

232
DIFFERENT MODES OF ACQUIRING OWNERSHIP
Donation

A RIGHT GRATUITOUSLY DISPOSED MUST BE PROPRIETARY

199. Heirs of Gozo v. Philippine Union Mission Corporation of the Seventh Day Adventist Church
G.R. No. 195990, August 5, 2015
Perez, J.

FACTS:
This is a Petition for Review on Certiorari filed by the heirs of Rafael Gozo seeking to reverse and set aside
the Decision of the Court of Appeals, which reversed the RTC Decision declaring the contract of donation
void.

Petitioners claim that they are the heirs of the Spouses Rafael and Concepcion Gozo (Sps. Gozo) who,
before their death, were the original owners of a parcel of land located in Sitio Simpak, Brgy. Lala,
Municipality of Kolambugan, Lanao del Norte. The respondents, PUMCO-SDA, on the other hand, claim
that they own a 5,000 sqm portion of the property. The assertion is based on a Deed of Donation dated 28
February 1937 made in favor of respondent. Respondents took possession of the subject property by
introducing improvements thereon through the construction of a church building, and later on, an elementary
school.

The Sps. Gozo were not the registered owners of the property at the time they executed the Deed of
Donation in 1937. However, they were the lawful possessors thereof. It was only on 5 October 1953 that
the Original Certificate of Title (OCT) No. P-642 covering the entire property was issued in the name of the
Sps. Gozo pursuant to a Homestead Patent granted by the President of the Philippines on 22 August 1953.
Upon the death of Rafael Gozo, the heirs caused the extrajudicial partition of the property which gave rise
to the need to survey the property. Concepcion Gozo included the portion occupied by PUMCO-SDA in the
survey. PUMCO-SDA therefore notified Concepcion of its claim to a 5,000 sqm portion of the property in
view of the Deed of Donation.

On June 19, 2000, petitioners filed an action for Declaration of Nullity of Document, Recovery of Possession
and Ownership with Damages against PUMCO-SDA, claiming, among others, that the deed of donation
was invalid for lack of acceptance which is an essential requisite for a valid contract of donation. On the
other hand, PUMCO-SDA argued that the question of validity of the donation was already barred by laches.

ISSUE:
Is the Deed of Donation executed on February 28, 1937 valid?

HELD:
No, the Deed of Donation is not valid but not because of a lack of acceptance but because the Sps. Gozo
were not the owners of the property at the time of the donation.

At the time of the donation, the subject property was part of the inalienable public domain. It was only almost
after two decades later or on 5 October 1953 that the State ceded its right over the land in favor of the Sps.
Gozo. Thus, prior to such conferment of title, the Sps. Gozo possessed no right to dispose the land which,
by all intents and purposes, belongs to the State. It is an established principle that no one can give what
one does not have, nemo dat quod non habet. It is true that gratuitous disposal in donation may consist of
a thing or a right but the term right must be understood in a “proprietary” sense over which the possessor
has jus disponendi. This is because in true donations there results a consequent impoverishment of the
donor or diminution of his assets. Hence, the donation of the subject property which took place before
October 5, 1953 is null and void.

233
DIFFERENT MODES OF ACQUIRING OWNERSHIP
Donation

TITLE TO DONATED IMMOVABLE PROPERTY PASSES ONLY UPON ACCEPTANCE IN A PUBLIC


INSTRUMENT AND NOTIFICATION OF THE DONOR OF THE SAME

200. Sumipat v. Banga


G.R. No. 155810, August 13, 2004
Tinga, J.

FACTS:
This is a Petition for Review on Certiorari of the CA Decision which reversed and set aside the RTC Decision
and partially annulled the Deed of Absolute Transfer and/or Quitclaim subject of the case.

Spouses Placida Taboro and Lauro Sumipat acquired three parcels of land. The couple was childless but
Lauro had five illegitimate children (Lydia, Laurito, Alicia, Alejandro, and Lirafe) with Pedra Dacola. Lauro
Sumipat executed a document denominated as a “Deed of Absolute Transfer and/or Quitclaim over Real
Properties” in favor of his five illegitimate children covering three parcels of land. The signature of Placida
appears on the document.

It appears that when the document was executed, Lauro Sumipat was already very sick and bedridden and
Lydia requested Lauro and Placida to affix their signatures on the document which she had brought. After
Lauro’s death, Placida and his five illegitimate children jointly administered the properties, 50% of produce
of which went to Placida. As Placida’s share dwindled until she no longer received any and upon learning
that the titles to the properties were already transferred to the five illegitimate children, she filed a complaint
for declaration of nullity of titles, contracts, partition, and recovery of ownership. The RTC ruled in favor of
the five illegitimate children, holding that by virtue of the assailed document the properties were absolutely
transferred to the defendants.

The petitioners claim that Placida freely consented to the execution of the deed and that they did not commit
fraudulent acts in connection with its execution. Respondents, on the other hand, claim that Placida was
deceived and misled into affixing her signature on the deed.

ISSUE:
Did the questioned deed validly transfer title to the respondents?

HELD:
No, there was no transfer of title over the immovable properties. A perusal of the deed reveals that it is
actually an onerous donation where Lauro Sumipat purportedly imposed upon the petitioners the condition
that he and his wife, Placida, shall be entitled to one-half (1/2) of all the fruits or produce of the parcels of
land for their subsistence and support. Since the deed involves the donation of 3 parcels of immovable
property, the donation should have been made in a public instrument. Its acceptance shall likewise be made
in a public instrument, whether in the same instrument as the donation or in a separate instrument. Finally,
the donor should have been notified of the acceptance. Where the formal requirements set forth in Article
749 of the Civil Code are not met, the donation shall be void.

In this case, the donees’ acceptances of the donation are not manifested either in the deed itself or in a
separate public instrument. Hence, there is no donation to speak of.

234
DIFFERENT MODES OF ACQUIRING OWNERSHIP
Donation

DONATION OF REAL PROPERTY VOID WITHOUT OBSERVANCE OF THE FORMALITIES IN ARTICLE


749 OF THE CIVIL CODE

201. Heirs of Mariano v. City of Naga


G.R. No. 197743, March 12, 2018
Tijam, J.

FACTS:
This is a Petition for Review on Certiorari, appealing the Decision of the CA which annulled the RTC
Decision ruling against the existence of the Deed of the Donation.

In 1954, the officers of homeowners’ association of City Heights Subdivision, offered to donate 5 hectares
of land to the City of Naga on which the new city hall of Naga may be built on the condition that the contract
be awarded to the Subdivision. The area is a portion of the land registered in the names of Macario Mariano
and Jose A. Gimenez, who signed the offer together with their spouses.

On August 11, 1954, the Municipal Board adopted a resolution accepting the Subdivision’s offer of donation
and its proposed contract and authorizing the City Mayor to execute the Deed of Donation on the City’s
behalf. A deed of donation was allegedly executed on August 16, 1954 by Mariano and Gimenez. By virtue
thereof, the City entered the property and began construction of the government center. It also declared the
five-hectare property in its name for tax purposes. Subsequently, other government agencies and
instrumentalities entered the same property and built their offices thereon.

However, the heirs of Mariano (petitioners) aver that the plan to donate five hectares to the City did not
materialize as the contract to build the City Hall was not awarded to the Subdivision but to another contractor
(Francisco Sabaria). This caused Mariano and officers of the Subdivision to meet with the Mayor to demand
the return of the five-hectare lot as the condition for the donation was not complied with. Mayor Imperial
purportedly assured them that the City would instead buy the property from them. Despite demand, the city
never paid for the lot before Mariano passed away.

Thus, the administrator of the estate made a new demand upon the City to vacate and return the property.
When the City did not comply, petitioners filed a complaint for unlawful detainer against the City. In its
defense, the City asserted ownership over the subject property by virtue of the alleged donation made in
1954 by the landowners in its favor. It presented a Deed of Donation dated August 16, 1954 as evidence.

ISSUE:
Is the Deed of Donation dated August 16, 1954 valid?

HELD:
No, it is invalid for lack of compliance with the formalities required for validity.

Where the law requires that a contract be in some form to be valid, such requirement is absolute and
indispensable; its non-observance renders the contract void and of no effect. One such law is Article 749 of
the Civil Code of the Philippines. Thus, donation of real property, which is a solemn contract, is void without
observance of the formalities specified in Article 749.

The Deed of Donation cannot be considered a public instrument because of its defective notarization. First,
the deed shows that the City Mayor affixed his signature thereon on August 21, 1954, or four days after it
was notarized, thus he could not have acknowledged the same before the notary public on August 16, 1954.

Second, the acknowledgement was not made be either the donors or the donee but by Eusebio M. Lopez
and the other officers of the Subdivision. Verily, the notary public could not have certified to knowing the
parties to the donation, or to their execution of the instrument, or to the voluntariness of their act. These
glaring defects are fatal to the validity of the alleged donation. Not being a public document, the purported
Deed of Donation is void.

235
DIFFERENT MODES OF ACQUIRING OWNERSHIP
Prescription

TRIAL COURTS MAY DISMISS AN ACTION ON THE GROUND OF PRESCRIPTION ONLY WHEN
PLEADINGS SHOW IT IS TIME-BARRED

202. James v. Eurem Realty Development Corp.


G.R. No. 190650, October 14, 2013
Reyes, J.

FACTS:
This is a petition for review of the CA Decision which dismissed the appeal from the RTC’s Resolution
holding that the action filed by petitioner for the declaration of nullity of title and ownership of real property
with damages had already prescribed.

On September 17, 2003, the heirs of Gorgonio James filed a Civil Case (Civil Case 5877) against Eurem
Realty Development Corporation, alleging among others that the title of respondent’s predecessor-in-
interest, Eufracio Lopez, is void ab initio as it was derived from the void title of Primitivo James (Gorgonio’s
brother). The title of Primitivo was declared void by a CA decision in CA-G.R. No. 50208-R (Civil Case 1447)
which attained finality.

Petitioners also alleged that Eufracio Lopez acted in bad faith in assigning the property to Eurem Realty
knowing fully that he had no right or interest over said property. Eurem Realty, on the other hand, argued
that prescription has already set in, since the petitioners filed the complaint on September 17, 2003 or more
than 30 years after its predecessor-in-interest Lopez bought the property from Primitivo way back in April
25, 1972.

ISSUE:
Is the action filed by petitioners barred by prescription?

HELD:
No, the period for filing of Civil Case 5877 has not yet prescribed.

The Court notes that the RTC’s dismissal was triggered by the defenses raised by the respondent in its
answer. There was yet to be a trial on the merits but the RTC merely relied on the averments in the complaint
and answer and forthwith dismissed the case. On this point, the Court has already ruled that the “affirmative
defense of prescription does not automatically warrant the dismissal of a complaint.

While trial courts have authority and discretion to dismiss an action on the ground of prescription, it may
only do so when the parties’ pleadings or other facts on record show it to be indeed time-barred. If the issue
of prescription is one involving evidentiary matters requiring a full-blown trial on the merits, it cannot be
determined in a motion to dismiss.

In any case, Civil Case 5877 is one for the declaration of nullity of TCT No. T-10713 in the name of the
respondent and for the declaration of the petitioners’ absolute ownership over said property. As basis for
their claim, the petitioner claimed that the respondent’s title over the property is void ab initio. An action to
declare the nullity of a void title does not prescribe.

236
DIFFERENT MODES OF ACQUIRING OWNERSHIP
Prescription

ALIENABLE AND DISPOSABLE LANDS OF THE PUBLIC DOMAIN ARE NOT PRIVATE PROPERTY
AND ARE THUS NOT SUSCEPTIBLE BY ACQUISITIVE PRESCRIPTION

203. Republic v. Tan


G.R. No. 199537, February 10, 2016
Brion, J.

FACTS:
This is an appeal by the Republic from the decision of the CA affirming the lower court’s decree granting
respondent’s application for original registration of title.

Respondent filed an application for original registration of title over Lot No. 4080, Cad. 545-D (new) situated
in Casili, Consolacion, Cebu (the subject lot). She alleged that she was the absolute owner in fee simple of
said lot and that she and her predecessors in interest have had been in peaceful, open, continuous,
exclusive, and notorious possession of the subject lot in the concept of an owner for over thirty (30) years.
She also proved that the subject lot was declared alienable and disposable on September 1, 1965, pursuant
to Forestry Administrative Order No. 4-1063.

The trial court approved the application on the ground that respondent had acquired the subject lot by
acquisitive prescription. The CA affirmed citing the recent case of Malabanan v. Republic (2013). The
Solicitor-General appeals the rulings of the lower court arguing that Malabanan had been incorrectly applied.

ISSUE:
May lands declared by the State as alienable and disposable susceptible to acquisitive prescription?

HELD:
No. Only private property can be acquired by prescription.

Property of public dominion is outside the commerce of man. It cannot be the object of prescription because
prescription does not run against the State in its sovereign capacity. As ruled by the Court in Malabanan,
this is true even if land is declared to be alienable and disposable as such declaration does not ipso facto
convert the land into patrimonial property. There must be an express declaration by the State that the public
dominion property is no longer intended for public service or the development of the national wealth or that
the property has been converted into patrimonial property. When this happens, the property is withdrawn
from public dominion and becomes property of private ownership, albeit still owned by the State. The
property is now brought within the commerce of man and becomes susceptible to the concepts of legal
possession and prescription. Without such express declaration, the property, even if classified as alienable
or disposable, remains property of the public dominion, pursuant to Article 420(2), and thus incapable of
acquisition by prescription.

Thus, in the present case, even though the subject lot has been declared alienable and disposable, the
property has not been withdrawn from public use or public service. Without this, prescription cannot begin
to run because the property has not yet been converted into patrimonial property of the State. It remains
outside the commerce of man. The fact that respondent and her predecessors-in interest had been in open
continuous, public, and notorious possession of the subject lot in the concept of an owner for over thirty (30)
years did not have any legal effect. In the eyes of the law, the respondent has never acquired legal
possession of the property and her physical possession thereof, no matter how long, can never ripen into
ownership.

237

You might also like